Vous êtes sur la page 1sur 228

NOUVEAU PROGRAMME 2019

Livre
du professeur
Sous la direction de Joël Malaval
Michel Bachimont
Jean-Luc Bousseyroux
Bernard Chrétien
Pierre-Antoine Desrousseaux
Fabrice Destruhaut
Anne Keller
Jean-Marc Lécole
Isabelle Lericque
Annie Plantiveau
Frédéric Puigredo
Joël Ternoy
Mickaël Védrine
Myriam Vialaneix

172909_PremPages_001-004.indd 1 30/07/2019 15:43:03


Le photocopillage, c’est l’usage abusif et collectif de la photocopie sans autorisation des auteurs
et des éditeurs.
Largement répandu dans les établissements d’enseignement, le photocopillage menace l’avenir
du livre, car il met en danger son avenir économique. Il prive les auteurs d’une juste rémunération.
En dehors de l’usage privé du copiste, toute reproduction totale ou partielle de cet ouvrage
est interdite. »

© Nathan 2019
25 avenue Pierre de Coubertin, 75013 PARIS
ISBN : 978-209-172909-1

172909_PremPages_001-004.indd 2 30/07/2019 15:43:03


Sommaire

ALGÈBRE
1 Suites numériques . . . . . . . . ........................................................... 5
2 Comportement d’une suite ...................................................... 23
3 Second degré . . . . . . . . . . . . . . . . ........................................................... 37

ANALYSE
4 Dérivation . . . . . . . . . . . . . . . . . . . . . ........................................................... 57
5 Applications de la dérivation .................................................... 69
6 Fonction exponentielle . ........................................................... 89
7 Trigonométrie . . . . . . . . . . . . . . . ........................................................... 107
8 Fonctions sinus et cosinus . ...................................................... 127

GÉOMÉTRIE
9 Produit scalaire et calcul vectoriel ........................................... 143
10 Applications du produit scalaire ............................................... 161

STATISTIQUES ET PROBABILITÉS
11 Probabilités conditionnelles et indépendance . . ........................ 183

12 Variables aléatoires .. . . . . ........................................................... 201

13 Simulation d’échantillons . . ....................................................... 217

Sommaire  ★  3

172909_PremPages_001-004.indd 3 30/07/2019 15:43:03


172909_PremPages_001-004.indd 4 30/07/2019 15:43:03
1 Suites numériques

3  a) •  a1 = 0 , 25 × 64.

Découvrir a2 = 0 , 25 × a1 = 0 , 252 × 64.


On multiplie 64, deux fois par 0,25 pour obtenir
a2 .
•  a3 = 0 , 25 × a2 = 0 , 253 × 64.
On multiplie 64, trois fois par 0,25 pour obtenir
1  Évolutions successives a3 .
à accroissements constants •  a4 = 0 , 25 × a3 = 0 , 254 × 64 = a0 × 0 , 254 .
b) Il semble que pour tout nombre n de ,
1   
an = a0 × 0 , 25n.
Rangée 1 2 3 4 5
c) Pour 10 carrés, le plus petit a pour aire, en m2 :
Nombre de bâtonnets
3 7 11 15 19 a9 = a0 × 0 , 259 = 64 × 0 , 259.
sur la rangée
1m2 = 10 4 cm2 , donc l’aire du plus petit carré
2  a) Pour obtenir un+1, on ajoute 4 à un . est 64 ´ 0 , 259 ´ 10 4 cm2 c’est-à-dire environ
En effet, à la (n + 1) -ième rangée, on a le même 2 cm2.
nombre de bâtonnets qu’à la n-ième rangée plus
2 bâtonnets de chaque côté.
b) Pour tout nombre n de , un+1 = un + 4.
c) Pour calculer u100 , il faudrait connaître u99 ,
donc u98 , u97 , …
Acquérir des automatismes
3  a) 
n 1 2 3 4 5
4n 4 8 12 16 20
un 3 7 11 15 19 3 •  b5 = 5 × 5 − 4 = 25 − 4 = 21
Il semble que pour tout nombre n de  avec •  b100 = 5 × 100 − 4 = 496
n > 1, un = 4 n − 1.
b)  u8 = 4 × 8 − 1 = 32 − 1 = 31. 4 • w0 = - 2 • w1 = 1 • w2 = 0 • w3 = - 5
• w4 = - 14 • w5 = - 27 • w6 = - 44 • w7 = - 65
2  Évolutions successives • w8 = - 90 • w9 = - 119 • w10 = - 152
à taux constant
5 •  t3 = 3 × 32 − 33 × 3 + 72 = 27 − 99 + 72 = 0
1    •  t 8 = 3 × 82 − 33 × 8 + 72 = 192 − 264 + 72 = 0
Carré n° 1 n° 2 n° 3 n° 4
Aire (en cm2) 64 16 4 1 Ainsi, t3 = t 8 .

2  a) On obtient an+1 en multipliant an par : 6 a)  k1 = 5k0 − 7 = 5 × (−5) − 7


75 25 1 k1 = −25 − 7 = −32
1− = = = 0 ,25.
100 100 4 b)  k2 = 5k1 − 7 = 5(−32) − 7 = −167
b) Pour tout nombre n de , an+1 = 0 , 25 × an . k3 = 5k2 − 7 = 5(−167) − 7 = −842

Chapitre 1  ★  Suites numériques 5

172909_Chap01_000-000.indd 5 25/07/2019 17:43:22


7   22 a1 = 3   a2 = −1   a3 = 3   a4 = −1
n 0 1 2 3 4 et ainsi de suite.
zn 2 4 16 256 65 536 Il semble que pour tout n impair, an = 3 et pour tout
n pair, an = −1.
10 a)  w100 = w 0 + 100r = 1 + 100 × (−2) = −199
b) Pour tout nombre n de , 23 u1 = 4 × 1 − 1 = 3
w n = w 0 + nr = 1 + (−2)n = 1 − 2n v1 = 4v 0 − 1 = 4 × (−1) − 1 = −5
Donc u1 > v1 et l’affirmation (2) est vraie.
11 k100 = k5 + (100 − 5)r = −50 + 95 × 0 ,5 = −2,5
24 Par exemple, u0 = 5 et pour tout nombre n de
12 a)  t 6 = t2 × q 6−2 4
= −2,5 × 3 = −202,5 , un+1 = un2 − 1.
b) Pour tout nombre n de ,
t n = t2 × q n−2 = −2,5 × 3n−2 25 •  u0 = −2 • 
u1 = −0 ,5 u2 = 0
3n 1 •  u3 = 0 ,25 •  u4 = 0 ,4
Remarque : 3n−2 = 2 = × 3n
3 9
26 •  u0 = 1 • 
u1 = 3 u2 = 5
2,5 25 5
donc t n = − × 3n = − × 3n = − × 3n •  u3 = 7 •  u4 = 9 = 3
9 90 18
13 m10 = m15 × q10−15 = 7 × 0 ,1−5 = 700000 1 5 7
27 •  u0 = u1 =
•  u2 =
2 6 6
16 a)  Sn = u2 + (u2 − 15 , )+
, ) + (u2 − 2 × 15 3 11
•  u3 = •  u4 =
+ (u2 − (n − 2) × 15
, ) 2 6
Sn = u
2 + u + … + u
2 2 − 15
, × (1 + 2 + … + (n − 2)))
(n−2+1) fois 28 a) •  u0 = 1 •  u1 = 2  • 
u2 = 5  • 
u3 = 10
Sn = (n − 1)u2 − 15, × (1 + 2 +  + (n − 2)) b) 
(n − 2)(n − 1) u3 = 10
Sn = 0 ,5(n − 1) − 15
, ×
2
Sn = 0 ,5(n − 1) − 0 ,75(n − 2)(n − 1)
Sn = (n − 1)[0 ,5 − 0 ,75(n − 2)]
Sn = (n − 1)(−0 ,75n + 2)
Sn = −0 ,75n2 + 2,75n − 2 u2 = 5
b) Avec la calculatrice, on lit que Sn = −77 pour
n = 12.
u1 = 2
17 S = v1 + 2 v1 + 22 v1 +  + 27 v1
u0 = 1
S = v1(1 + 2 + 22 +  + 27 )
1 − 28 O 1 2 3
S = −1× = 1 − 28 = −255
1− 2 c)  un = 50 équivaut à n2 + 1 = 50 c’est-à-dire
n2 = 49 soit n = 7 ou n = −7.
18 T = w2 + 0 ,8w2 + 0 ,82 w 2 +  + 0 ,85 w2
Or, n Î , donc un = 50 pour n = 7.
T = w 2 (1 + 0 ,8 + 0 ,82 +  + 0 ,85 )
1 − 0 ,8 6 29  
T = 10 × = 50(1 − 0 ,86 )
1 − 0 ,8
T = 36 ,892 8

19 Pour tout nombre n de , 2n + 1 est un nombre


impair. Donc l’affirmation (2) est exacte.

20 Pour tout nombre n de , cn = n2 .

21 Par exemple, pour tout nombre n de , (fenêtre : 0 < X < 11, pas 1 et
v n = n2 − 3n + 1 -160 < Y < 0 , pas 10)

172909_Chap01_000-000.indd 6 25/07/2019 17:44:50


30 a)  u0 = 4 et pour tout nombre n de , 34  
un+1 = un2 − 2un .
b)  u1 = u02 − 2un = 4 2 − 2 × 4 = 16 − 8 = 8
u2 = u12 − 2u1 = 82 − 2 × 8 = 48
u3 = u22 − 2u2 = 482 − 2 × 48 = 2 208
c) Voici la valeur de u6 affichée dans la console :

(fenêtre : 0 < X < 5, pas 


1 et
Remarque : la calculatrice affiche : 0 < Y < 2, pas 1)
u6 ≈ 5, 628 827 661× 1026
35 u1 = u0 + 6 soit u0 = u1 − 6 = 4 − 6.
1 1
31 a)  v1 = − v 0 + 1 = − × 2 + 1 = 0 L’affirmation (3) est exacte.
2 2
1 1 36 a) Oui : 2 − (−1) = 5 − 2 = 8 − 5 = 11 − 8 = 3
v 2 = − v1 + 1 = − × 0 + 1 = 1
2 2 b) Non : 4 − 2 = 2 mais 8 − 4 = 4.
1 1 1 c) Non : −8 − (−10) = 2 mais 0 − (−8) = 8.
v3 = − v2 + 1 = − ×1 + 1 =
2 2 2 d) Oui :
b)  3 − 7 = −1 − 3 = −5 − (−1) = −9 − (−5) = −4
V¬2
Pour i allant de 1 à n 37 • v1 = 3 • v2 = 5,5 • v3 = 8 • v4 = 10,5
V ← −0 , 5 V + 1
Fin Pour 38 • w1 = - 10 • w2 = - 20 • w3 = - 30 • w4 = - 40
c) 
39 t5 − t 4 = 20 − 8 = 12
Donc la suite arithmétique (t n ) a pour raison 12.

40 h6 = h4 + (6 − 4)r
Voici la valeur de v10 affichée dans la console : 0 = 8 + 2r c’est-à-dire r = −4.
Donc la suite arithmétique (hn ) a pour raison -4.

41 • Succursale A :
Ainsi, v10 » 0 ,668. 34 − 35 = 33 − 34 = −1 mais 31 − 33 = −2.
On ne peut pas modéliser le chiffre d’affaires de A
1 1 1
32 k1 = = = avec une suite arithmétique.
1 + k0 1 + 2 3
• Succursale B :
1 1 1 3
k2 = = = = 37 − 35 = 39 − 37 = 41 − 39 = 2.
1 + k1 1 + 1 4 4
On peut modéliser le chiffre d’affaires de B durant ces
3 3
4 mois avec une suite arithmétique de raison 2.
1 1 1 4
k3 = = = = Si l’évolution se poursuit ainsi, le chiffre d’affaires de
1 + k2 1 + 3 7 7
B, le mois n (avec n Î , n >1), sera 35 + 2(n − 1)
4 4
c’est-à-dire 33 + 2n.

33 •  u3 = −3u2 + 1 = −3 × (−4) + 1 = 13 42 • Pays A :


1
•  u2 = −3u1 + 1 c’est-à-dire u1 = − (u2 − 1). 0 ,60 − 0 ,57 = 0 ,63 − 0 ,60 = 0 ,66 − 0 ,63 = 0 ,03.
3
1 5 Donc on peut modéliser l’évolution du prix du timbre-
Ainsi, u1 = − (−4 − 1) = . poste entre 2015 et 2018 dans ce pays A, avec une
3 3
1 suite arithmétique de raison 0,03.
• De la même façon, u0 = − (u1 − 1).
3 Si l’évolution se poursuit ainsi, le prix du timbre-poste,
1  5  1 2 2 l’année 2015 + n (avec n Î  ) sera 0 ,57 + 0 ,03n,
Ainsi, u0 = −  − 1 = − × = − .
3  3  3 3 9 en euro.

Chapitre 1  ★  Suites numériques 7

172909_Chap01_000-000.indd 7 25/07/2019 17:45:52


• Pays B : 50 Pour tout nombre n de , un = −7 + 5n.
0 ,53 − 0 ,51 = 0 ,02 mais 0 ,57 − 0 ,53 = 0 ,04.
Donc on ne peut pas modéliser l’évolution du prix du
timbre-poste dans ce pays B avec une suite arithmé-
tique.
• Pays C :
0 ,58 − 0 ,60 = 0 ,56 − 0 ,58 = 0 ,54 − 0 ,56 = −0 ,02.
Donc on peut modéliser l’évolution du prix du timbre-
poste entre 2015 et 2018 dans ce pays C, avec une (fenêtre : 0 < X < 10 , pas 1 et
suite arithmétique. -10 < Y < 45, pas 5)
Si l’évolution se poursuit ainsi, le prix du timbre-poste,
l’année 2015 + n (avec n Î  ) sera 0 ,60 - 0 ,02n, 51 a) 
en euros.

43 u8 = u0 + 8r = 1 + 8 × (−10) = −79

44 u50 = u2 + (50 − 2)r b)  t10 = −118


, et t15 = −19 ,3
u50 = 3 + 48 × 0 ,2 = 12,6
15 × 16
52 S = = 15 × 8 = 120
45 •  u3 = u2 + r donc r = u3 − u2 2
soit r = 5 − (−4) = 9.
53 Le nombre de sons émis par l’horloge en 24 h
•  u0 = u2 + (0 − 2)r = −4 − 2 × 9 = −22
est :
12 × 13
46 •  u8 = u6 + 2r soit 12 = 4 + 2r 2 × (1 + 2 + 3 +  + 12) = 2 × = 156
2
c’est-à-dire 2r = 8. Ainsi, r = 4.
•  u5 = u6 + (5 − 6)r = 4 − 4 = 0 54 F = 3 × (1 + 2 + 3 + 4 +  + 9 + 10)
10 × 11
47 •  u10 = u2 + (10 − 2)r soit 1 = 7 + 8r F = 3× = 3 × 5 × 11 = 15 × 11 = 165
2
3
c’est-à-dire 8r = −6. Ainsi, r = − . L’affirmation (2) est exacte.
4
 3  17
•  u0 = u2 + (0 − 2)r = 7 − 2 × −  = 55 S = u0 + (u0 + 5) + (u0 + 2 × 5) + 
 4  2
+ (u0 + 10 × 5)
48 Pour tout nombre n de , S = u0 + u0 + … + u0 + 5 × (1 + 2 + … + 10)

bn = b1 + (n − 1)r 11 termes

3 1 1 5 S = 11u0 + 5 × (1 + 2 +  + 10)
bn = − (n − 1) = − n +
4 2 2 4 10 × 11
S = 11× 2 + 5 ×
2
49 a) Pour tout nombre n de , S = 11× (2 + 25) = 11× 27 = 297.
an = a0 + nr
an = 7 − 2n 56 T = v 0 + (v 0 − 3) + (v 0 − 2 × 3) + 
b)  + (v 0 − 30 × 3)
a0 T = v 0 + v 0 + … + v 0 − 3 × (1 + 2 + … + 30)

31 termes

a1 T = 31v 0 − 3(1 + 2 +  + 30)


30 × 31
T = 31× 7 − 3 ×
a2 2
T = 31× (7 − 45) = 31× (−38) = −1178.

a1 = 1  1  1
57 E = w 0 + w 0 +  + w 0 + 2 ×  + 
 2   2
O 1 2 3
 1
+ w 0 + 15 × 
 2
8

172909_Chap01_000-000.indd 8 25/07/2019 17:46:53


1 64 •  v1 = 1  • 
v 2 = 5  • 
v 3 = 25  • 
v 4 = 125
E = 16 w 0 + × (1 + 2 +  + 15)
2
1 15 × 16 65 t5 = qt 4 c’est-à-dire 7,5 = 5q
E = 16 × (−1) + × 7,5
2 2 soit q = , .
= 15
 15  11 5

E = 16 × −1 +  = 16 × = 44.
 4 4
44 Î  donc Émilie a raison. 66 h6 = q2h4 c’est-à-dire 54 = 6q2 soit q2 = 9.
Ainsi, q = 3 ou q = −3.
58 Sn = a0 + (a0 + 15 , ) +  + (a0 + 15
, n)
1200 960 768
, × (1 + 2 +  + n)
Sn = (n + 1)a0 + 15 67 a)  = = = 0 ,8
1500 1200 960
Or, a0 = a2 + (0 − 2)r = −1 − 2 × 15
, = − 4.
On constate donc que les taux d’évolution d’un mois
n(n + 1)
Donc, Sn = −4(n + 1) + 15 , × au mois suivant sont constants. Il s’agit d’une baisse
2
Sn = (n + 1)(−4 + 0 ,75n) de 20 %.
b) On note hn la hauteur d’eau, en mm, dans le puits
59 a) De 2 à 4, il y a 3 nombres (4 − 2 + 1 = 3). le n-ième mois après le 1er juin. (hn ) est la suite géo-
De 2 à 10, il y a 9 nombres (10 − 2 + 1 = 9). métrique de raison 0,8 telle que h0 = 1500.
De 2 à 100, il y a 99 nombres (100 − 2 + 1 = 99). Pour tout nombre n de , hn = 1500 × 0 , 8n.
b)  F = h2 + (h2 + 5) + (h2 + 2 × 5) +  c)  h12 = 1500 × 0 , 812 et h12 » 103.
Le 1er juin de l’année suivante, il y aura environ
+ (h2 + 98 × 5)
103 mm d’eau dans le puits si l’évolution continue
F = 99 h2 + 5 × (1 + 2 +  + 98)
98 × 99 ainsi.
F = 99 × 3 + 5 ×
2
 1 8 24 1 1
F = 99 × (3 + 5 × 49) = 99 × 248 = 24 552 68 u8 = u0 q 8 = 16 ×   = 8 = 4 =
 2  2 2 16
60 a) Pour tout nombre n de , pn+1 = pn + 180.
b) La suite ( pn ) est arithmétique de raison 180, donc 32 1 1
69 u0 = u6 q 0−6 = 9 × (−3)−6 = 6
= 4 =
pour tout nombre n de , pn = 1000 + 180n. 3 3 81
On note S le nombre de puces que prévoit de fabri- 70 u10 = qu9 soit 18 = q × (−6)
quer l’usine de 2019 à 2025. 18
S = p0 + p1 +  + p6 c’est-à-dire q = = −3.
−6
S = p0 + ( p0 + 180) +  + ( p0 + 6 × 180)
S = 7 p0 + 180 × (1 + 2 +  + 6) 71 a)  v7 = v 4 q3 soit −32 = −4 q3
6 ×7 c’est-à-dire q3 = 8. Ainsi q = 2.
S = 7 × 1000 + 180 ×
2 b)  v10 = v 4 q 6 = −4 × 26 = −256
S = 7 × (1000 + 3 × 180) = 7 × 1540 = 10 780
L’usine prévoit de fabriquer 10 780 puces de 2019 à 72 a)  k6 = k 4 q2 soit 510 = 2 040q2
2025. 510 1
c’est-à-dire q2 = = .
2 040 4
5 1 1
61 u1 = qu0 c’est-à-dire 5 = 4u0 soit u0 = . Or, q < 0, donc q = − =−
4 4 2
L’affirmation (1) est exacte.
b) Pour tout nombre n de ,
8 16 30  1 n−4
62 a) Non : = = 2 mais = 1875
, kn = k 4 q n−4 = 2 040 × − 
4 8 16  2 
10 1 0 ,1 0 ,01  1 n  1 −4
b) Oui : = = = = 0 ,1 Remarque : kn = 2 040 × −  − 
100 10 1 0 ,1  2   2 
0 20
c) Non : = 0 mais =2  1 n  1 n
−10 10 c’est-à-dire kn = 2 040 × 16−  = 32 640 × −  .
 2   2 
4 −8 16 −32
d) Oui : = = = = −2  1 9−4
 1 5
−2 4 −8 16 c)  k9 = 2 040 × −  = 2 040 × − 
 2   2 
63 Il avait recouvert la moitié de l’étang au bout de 2 040
k9 = − = −63,75
39 jours. 32
Chapitre 1  ★  Suites numériques 9

172909_Chap01_000-000.indd 9 25/07/2019 17:48:20


73 • On note (un ) cette suite géométrique telle
1 − (−10)6 1
que u21 = 3145 728 et u22 = 6 291456. 78 F = = × (1 − 106 ) = −90 909
1 − (−10) 11
u
Sa raison est : q = 22 = 2. L’affirmation (1) est exacte.
u21
•  u8 = u21q 8−21 = u21q−13 79 S = u0 + 4u0 + 4 2 u0 +  + 4 9 u0 + 410 u0
3145 728 3145 728
u8 = 3145 728 × 2−13 = = S = u0 (1 + 4 + 4 2 +  + 4 9 + 410 )
213 8 192
Ainsi, u8 = 384. 1 − 411 1 − 411 2
S = −2 × = −2 × = × (1 − 411)
•  u15 = u8 q7 1− 4 −3 3
S = −2 796 202
u15 = 384 × 27 = 49 152
80 T = v 5 + (−3)v 5 + (−3)2 v 5 +  + (−3)15 v 5
74 a) Pour tout nombre n de ,
T = v 5 (1 + (−3) + (−3)2 +  + (−3)15 )
 1 n
gn = g0 q n = 10 ×   1 − (−3)16 1 − 316
 2  T= = = −10 761680
b)  1 − (−3) 4
10
3  3 2  3 7
81 M = w 0 + w 0 +   w 0 +  +   w 0
4  4   4 
 3 3
2
3  7

M = w 0 1 + +   +  +   
 
4 4  4  

5  3 8
1 −     3 8 
 4  
M = w0 = 4w 0 1 −   
3   4  
1−
2,5 4
2
1,25  3 −2  4 2 16 80
Or, w 0 = w2   = 5 ×   = 5 × =
0,625  4   3  9 9

80   3   320   3  
O 1 2 3 4 8  8
Donc, M = 4 × × 1 −    = × 1 −   
9   4   9   4  
75  
82 a) 
k 1 2 3 4
S 1 2,5 4,75 8,125 13,187 5
On obtient S = 13,187 5 à la fin de l’algorithme.
b) Cet algorithme donne la somme des (n + 1) pre-
miers termes de la suite géométrique (un ) de raison
1,5 telle que u0 = 1.
(fenêtre : 0 < X < 9 , pas 1 et
c) 
-40 < Y < 55, pas 5)

76 a) Ce programme affiche les termes de u1 à u10


1
de la suite géométrique (un ) de raison telle que
2
u0 = −1000 000. On obtient :
b) Le dernier nombre affiché par ce programme est
-976 , 562 5 (c’est-à-dire u10 ).
En fait, -1953,125 est la valeur de u9 (avant dernier
nombre affiché).
83  
9 9 9
1− 3 1− 3 3 −1 S¬3
77 S = = =
1− 3 −2 2 Pour k allant de 1 à 10
L’affirmation (1) est exacte.
S ← S + 2, 5k
Fin Pour

10

172909_Chap01_000-000.indd 10 25/07/2019 17:49:05


84 1. B 2. C 3. D 4. D 5. A 6. C

85 1. B. D 2. B. C 3. A. C 4. B. C. D S’entraîner


86 1. Vrai. En effet, u100 = u200 − 100r et
u300 = u200 + 100r . Donc u100 + u300 = 2 × u200 .
2. Vrai. En effet, pour tout nombre n de  avec n >1, 93 a) 
w
w n−1 = n et w n+1 = qw n . i 1 2 3
q
U 2 000 2 200 2 250 2 262,5
Donc w n−1 × w n+1 = w n2 soit w n = w n−1 × w n+1 .
3. Vrai. En effet, pour tout nombre n de , À la fin de l’algorithme, on obtient U = 2 262, 5 ; c’est
(−2)n le solde du compte de Pierre au 1er avril 2018.
hn = h1q n−1 = −4 × (−2)n−1 = −4 × b)  v 0 = 2 000 et pour tout nombre n de ,
−2
hn = 2 × (−2)n v n+1 = 0 ,25v n + 1700.
On tabule cette suite avec la calculatrice et on observe
que h17 = −262144. 94 a) 
i 1 2 3 4
87 a) Il manque la donnée de la valeur d’un terme, U 200 190 182 175,6 170,48
par exemple v 0 ou v1 ou … À la fin de l’algorithme, on obtient U = 170 , 48. Ainsi,
b)  v1 = 102
, v 0 − 200 = 102
, × 5 000 − 200 = 4 900 U » 170 ; cela représente le nombre d’adhérents au
c)  v 2 = 102
, v1 − 200 = 102
, × 4 900 − 200 = 4 798 club de judo de cette ville en 2019.
v 3 = 102
, v 2 − 200 = 102
, × 4 798 − 200 = 4 693, 96 b)  v 0 = 200 et pour tout nombre n de ,
v n+1 = 0 ,8v n + 30.
88 u6 = u3 + 3r c’est-à-dire 42 = 15 + 3r
soit 3r = 27. Ainsi, r = 9. 96 • Mathématisation
On note an et bn les soldes respectifs des comptes
89 a)  k7 = k5q2 c’est-à-dire −8 = −0 ,5q2 de Gaylor avec l’option 1 et l’option 2.
soit q2 = 16. Ainsi, q = 4 ou q = −4. Pour tout nombre n de ,
b)  k 4 = k1q3 c’est-à-dire 108 = 0 ,5q3 soit q3 = 216. 2
an = a0 + 1000 × n = 1000 + 20n
Pour tout nombre réel k, l’équation x3 = 4 a une 100
n
seule solution dans . En tabulant la fonction cube  1, 8 
bn = b0 1 +  = 1000 × 1018
, n
avec le pas 1, on observe que 63 = 216. Donc q = 6.  100 
• Résolution du problème
90 • L’algorithme 1 affiche les termes de u1 à u10 On tabule les suites (an ) et (bn ) avec la calculatrice.
de la suite arithmétique de raison -10 telle que
u0 = −2.
• L’algorithme 3 affiche les termes de u1 à u12 de la
1
suite géométrique de raison telle que u0 = 10.
4
91 a)  S = u2 + (u2 + 0 ,8) +  + (u2 + 8 × 0 ,8)
S = 9u2 + 0 ,8 × (1 + 2 +  + 8) On constate que pour n <12, an > bn et que pour
8×9 n = 13, a13 < b13 .
S = 9 × 4 + 0 ,8 ×
2 À partir de n = 13, le solde du compte de Gaylor
S = 64 ,8 augmente de 20 € par an avec l’option 1, alors qu’il
b)  S = v 2 + 0 ,8v 2 + 0 ,82 v 2 +  + 0 ,88 v 2 augmente d’au moins 22  € par an (car
S = v 2 (1 + 0 ,8 + 0 ,82 +  + 0 ,88 ) 1261× 1, 8 % ≈ 22). Donc, pour tout n >13, an < bn .
1 − 0 ,8 9 • Conclusion
S = 4× = 20 × (1 − 0 ,89 ) Après 13 ans de placement, l’option 2 devient plus
1 − 0 ,8
intéressante.

97 On note an et bn les nombres de timbres res-


pectifs de Yasmine et Carole au bout de n mois.

Chapitre 1  ★  Suites numériques 11

172909_Chap01_000-000.indd 11 25/07/2019 17:50:30


Pour tout nombre n de , C et D n’appartiennent pas à la courbe de f, donc non
an = 1000 − 30n plus à la représentation graphique de (v n ).
 3, 5 
n E et F appartiennent à la courbe de f mais pas à la
bn = 1000 × 1 −  = 1000 × 0 ,965n représentation graphique de (v n ) ( -10 et 4,6 ne
 100 
On tabule ces deux suites avec la calculatrice. sont pas des nombres entiers naturels).

 1 
102 a)  1200 × 1 + + 800 = 2 012
 100 
Donc il y avait bien 2 012 € sur le compte d’Inès au
2 janvier 2001.
b) Pour tout nombre n de ,
On constate que pour n < 9, an > bn et que pour  1 
Sn+1 = Sn 1 + + 800
n = 10, a10 < b10 .  100 
À partir de n = 10, le nombre de timbres de Yasmine Sn+1 = 101
, Sn + 800
diminue de 30 par mois, alors que celui de Carole c) Avec la calculatrice, on tabule la suite (Sn ) définie
diminue d’au plus 24 par mois par S0 = 1200 et pour tout nombre n de ,
(car 700 ,28 × 3, 5 % ≈ 24 ). Donc, pour tout n >10, Sn+1 = 101
, Sn + 800.
an < bn . On constate que S20 » 19 079.
• Conclusion Au 2 janvier 2020, le solde du compte d’Inès s’élève à
Après 10 mois de ventes dans ces conditions, le environ 19 079 €.
nombre de timbres restants à Carole devient plus
important que celui de Yasmine. 103 a) On exécute Factorielle (5) et on obtient :

98 a) Zoé a remarqué que :


−4 ,2 − (−5) = −3,4 − (−4 ,2) = (−2,6) − (−3,4)
= (−18
, ) − (−2,6) = −1 − (−18, ) = 0 ,8
b) Pour tout nombre n de ,
un+1 − un = 0 ,8(n + 1) − 5 − (0 ,8n − 5)
un+1 − un = 0 ,8n + 0 ,8 − 5 − 0 ,8n + 5 b) La ligne 5 du programme indique que pour tout
un+1 − un = 0 ,8 nombre n de  avec n >1,
Donc la suite (un ) est arithmétique de raison 0,8. Factorielle(n) = n × Factorielle(n − 1)
c’est-à-dire un = n × un−1.
15
, 4 ,5 13,5 40 ,5 1215 ,
99 a)  = = = = =3 Ainsi, (un ) est la suite définie par u0 = 1 et pour tout
0 ,5 15 , 4 ,5 13,5 40 ,5
nombre n de , un+1 = (n + 1)un .
b) Pour tout nombre n de ,
c) Pour n >1, n! est le produit des nombres entiers
3n+1
un+1 = naturels de 1 à n.
2
En effet,
3 × 3n
un+1 = un = n un−1 = n(n − 1)un−2 = n(n − 1)(n − 2)un−3
2
un+1 = 3un et de proche en proche :
Donc la suite (un ) est géométrique de raison 3. un = n(n − 1)(n − 2)× 2 × 1
Donc pour tout n >1, n ! = n(n − 1)× 2 × 1.
100 a)  v 0 = 1   v1 = 4   v 2 = 12 Remarque : le fait que 0 ! = 1 est une convention.
v1 − v 0 = 4 − 1 = 3 et v 2 − v1 = 12 − 4 = 8 donc la
104 a)  u0 = 0 et pour tout nombre n de ,
suite (v n ) n’est pas arithmétique.
un+1 = un + 2n.
v 4 v 12
b)  1 = = 4 et 2 = = 3 donc la suite (v n ) b)  u0 = 2 et pour tout nombre n de , un+1 = un2 − 1.
v0 1 v1 4
n’est pas géométrique. 105 u0 = 0
1 1
101 a) f est la fonction définie sur [0 ; + ∞[ par u1 = = =1
1 − u0 1 − 0
f( x) = x2 + 2 x − 3. 1
u2 = mais 1 − u1 = 1 − 1 = 0 et on ne peut pas
b) A et B appartiennent à la courbe de f et à la repré- 1 − u1
sentation graphique de (v n ). diviser par 0.
12

172909_Chap01_000-000.indd 12 25/07/2019 17:51:53


Donc les données de u0 = 0 et de cette relation de un
v n+1 − v n = =1
récurrence ne définissent pas une suite sur . un
106 •  v1 + v 2 +  + v 8 est égal à Donc, (v n ) est une suite arithmétique de raison 1.
v1 + (v1 + r ) +  + (v1 + 7r ) c) • Pour tout nombre n de ,
c’est-à-dire 8v1 + r(1 + 2 +  + 7) vn = v0 + n ×1 = v0 + n
7×8 1 1
soit 8v1 + r . Or, v 0 = = , donc pour tout nombre n de ,
2 u0 2
Ainsi, 8v1 + 28r = 92 c’est-à-dire 2v1 + 7r = 23. 1
v n = + n.
• Or, v 4 = v1 + 3r c’est-à-dire v1 + 3r = 9. 2
• On résout le système • Pour tout nombre n de ,
2v1 + 7r = 23 1 1 1 2
un = = = =
 vn 1 2n + 1 2n + 1
v1 + 3r = 9 n+
2 2
On multiplie par -2 la 2e équation et on ajoute
membre à membre avec la 1re équation. On obtient : 109 a) On passe du motif n° n au motif n°  (n + 1) en
2v1 + 7r − 2v1 − 6r = 23 − 2 × 9 ajoutant 3 carrés. Ainsi, pour tout nombre n de ,
r=5 Cn+1 − Cn = 3.
On remplace r par 5 dans la 2e équation du système ; Donc la suite (Cn ) est arithmétique de raison 3.
il vient : b) Pour tout nombre n de ,
v1 + 3 × 5 = 9 c’est-à-dire v1 = −6. Cn = C1 + 3(n + 1) = 1 + 3(n − 1)
• Conclusion : v1 = −6 et r = 5. c’est-à-dire Cn = 3n − 2.
Donc, C1000 = 3 × 1000 − 2 = 2 998.
107 Pour tout nombre n de ,
v n+1 − v n = 5un+1 − 1 − (5un − 1) 110 Pour calculer l’aire ! de b
v n+1 − v n = 5(un+1 − un ). chaque trapèze, on utilise ici
(un ) est une suite arithmétique de raison 6, donc la formule suivante (avec les h
v n+1 − v n = 5 × 6 notations ci-contre) :
v n+1 − v n = 30 (b + B)h B
Ainsi, (v n ) est une suite arithmétique de raison 30. !=
2
Remarque : on peut établir cette formule en décom-
u0 2 2
108 a) •  u1 = = = posant ce trapèze en un rectangle et un triangle rec-
u0 + 1 2 + 1 3
tangle.
2  
u1 2 3 2 1 + 1 × 1 + 1 × 1
u2 = = 3 = × =  2  5
u1 + 1 2 a)  a0 = =
+1 3 5 5 2 4
3 3 1 
2  + × 2 + 1 × 1
 2 2  7
u2 2 5 2 a1 = =
u3 = = 5 = × = 2 4
u2 + 1 2 + 1 5 7 7  
1
2 + × 3 + 1 × 1
5  
2 9
2 a2 = =
u3 2 7 2 2 4
u4 = = 7 = × = b) Pour tout nombre n de ,
u3 + 1 2
+1 7 9 9 1 
7  n + 1 + 1 (n + 1) + 1 × 1
 2 2 
1 3 1 5 an =
•  v1 = = •  v2 = = 2
u1 2 u2 2
5 2n + 5
1 7 1 9 n+
•  v 3 = = •  v4 = = 2 = 2 2n + 5
u3 2 u4 2 an = =
2 2 4
b) Pour tout nombre n de , c) Pour tout nombre n de ,
1 1 2(n + 1) + 5 2n + 5
v n+1 − v n = − an+1 − an = −
un+1 un 4 4
2n + 7 − 2n − 5 1
un + 1 1 an+1 − an = =
v n+1 − v n = − 4 2
un un

Chapitre 1  ★  Suites numériques 13

172909_Chap01_000-000.indd 13 25/07/2019 17:53:07


1 Donc la suite (mn ) est géométrique de raison 0,835.
Donc, la suite (an ) est arithmétique de raison .
2 b) Pour tout nombre n de ,
d) Pour tout nombre n de ,
mn = m0 q n = m0 × 0 , 835n
 1  1  1
Sn = a0 + a0 +  + a0 + 2 ×  +  + a0 + n ×  1 1
c)  mn < m0 équivaut à m0 ´ 0 ,835n < m0 c’est-à-
 2   2 
 2 2 1 2
1 dire 0 ,835n < .
Sn = (n + 1)a0 + (1 + 2 +  + n) 2
2
5 1 n(n + 1) Avec la calculatrice, on tabule la suite (0 ,835n ) et on
Sn = (n + 1) +
4 2 2 observe que 0 ,8353 » 0 ,58 et 0 ,8354 » 0 ,49.
1  5 1  Donc au bout de 4 jours, la masse de radon a diminué
Sn = (n + 1) + n
2  2 2  de moitié de sa masse initiale.
1
Sn = (n + 1)(n + 5) 
4 50 
114 a)  p1 = 1200 × 1 + = 1800
1 3 5  100 
Sn = n2 + n +
4 2 4  50 
p2 = 1800 × 1 + = 2 700
Sn est l’aire du trapèze rectangle OABC avec  100 
 1  b) Pour tout nombre n de ,
A(n + 1 ; 0), Bn + 1 ; (n + 1) + 1 et C(0 ; 1).
 2   50 
pn+1 = 1 + p = 15 , pn
111 1. a) On note (un ) la suite des nombres entiers  100  n
pairs ; c’est la suite arithmétique de raison 2 telle que Donc la suite ( pn ) est géométrique de raison 1,5.
u1 = 2. c) Pour tout nombre n de ,
Pour tout nombre n de , un = 2. pn = p0 q n = 1200 × 1, 5n
S = 2 + 4 +  + 100 Ainsi, p24 = 1200 × 1, 524
S = u1 + u2 +  + u50 soit p24 » 20 200 935
S = u1 + (u1 + 2) +  + (u1 + 49 × 2)
S = 50 u1 + 2 × (1 + 2 +  + 49) 115 a) Pour obtenir l’aire d’un carré, on multiplie par
49 × 50  3 2
S = 50 × 2 + 2 ×   l’aire du carré précédent. Autrement dit, pour
2  4 
9
S = 100 + 49 × 50 = 2 550 tout nombre n de , un+1 = un .
16
b) On note (v n ) la suite des nombres entiers impairs ; 9
c’est la suite arithmétique de raison 2 telle que La suite (un ) est géométrique de raison .
16
v 0 = 1. 9  9 n
Pour tout nombre n de , v n = 2n + 1. b)  S = u0 + u0 +  +   u0
16 16 
T = 1 + 3 +  + 99  9  9 
2
 9 n 
T = v 0 + v1 +  + v 49 S = u0 1 + +   +  +   
 16 16  16 

T = v 0 + (v 0 + 2) +  + (v 0 + 49 × 2)
n+1
T = 50 v 0 + 2 × (1 + 2 +  + 49) 9
1 −  
49 × 50 16 
T = 50 × 1 + 2 × S = u0 ×
2 9
1−
T = 50 + 49 × 50 = 2 500 16
2. On note ∑ = 1 + 2 +  + 100 Or, u0 = 12 = 1, donc :
1re méthode 16   9  
n+1

∑ = S + T = 2 550 + 2 500 = 5 050 S = × 1 −    .


7  16  
2e méthode  
100 × 101
∑= = 50 × 101 = 5 050  2 
116 a)   1 = 500 × 1 + = 510
2  100 
112 Pour tout nombre n de ,  2 
un+1 = a × b n+1 = a × b × b n = bun .  2 = 510 × 1 + = 520 ,2
 100 
Donc la suite (un ) est géométrique de raison b. b) Pour tout nombre n de ,
 2 
113 a) Pour tout nombre n de ,  n+1 =  n 1 + , × n
= 102
 100 
mn+1 = mn − 0 ,165 mn
c) La suite ( n ) est géométrique de raison 1,02.
mn+1 = 0 , 835 mn
14

172909_Chap01_000-000.indd 14 25/07/2019 17:54:24


d) On note S le montant total des loyers durant T = w 0 (1 + 10−1 +  + 10−19 )
9 années de location.
1 − 10−18
S = 12 × ( 0 +  1 + … +  8 ) T = 10−1 ×
1 − 10−1
S = 12 × ( 0 + 102,  0 + … + 102, 8  0) 1
2 T = (1 − 10−18 ) = 0,111 111 111 111
S = 12  0 (1 + 102
, + 102 , , 8)
+ … + 102 9
, 9
1 − 102
S = 12 × 500 × 118 a)  u3 = 0 ,999 = 9 × 10−1 + 9 × 10−2 + 9 × 10−3
1 − 102
,
9 9 9
, 9
1 − 102 u3 = + +
S = 6 000 × 10 102 103
−0 ,02 b) Pour tout nombre n de ,
, 9 − 1)
S = 300 000(102 ……
un = 0 , 99 9
soit S » 58 527,77 €. n fois

e) Avec la calculatrice, on tabule la suite ( n ) et on un = 9 × 10−1 + 9 × 10−2 + … + 9 × 10−n


observe que : On note (an ) la suite géométrique de raison 10-1
 23 » 788 et  24 » 804 telle que a1 = 9 × 10−1.
Donc, après 24 ans de location dans ces conditions, le Pour tout nombre n de ,
loyer dépassera 800 € par mois. an = a1 q n−1 = 9 × 10−1 × (10−1)n−1

117 a) On note (un ) la suite géométrique de rai- an = 9 × 10−1 × 10−n+1


son 4 telle que u0 = 4. an = 9 × 10−n
Pour tout nombre n de , un = 4 × 4 n = 4 n+1. Ainsi, un = a1 + a2 +  + an
Avec la calculatrice, on observe que u9 = 1048 576. c) 1re méthode
Donc R = u0 + u1 +  + u9 Pour tout nombre n de  avec n >1,
R = u0 + 4u0 +  + 4 9 u0 un = a1 + 10−1a1 +  + (10−1)n−1a1
R = u0 (1 + 4 + 4 2 +  + 4 9 ) un = a1[1 + 10−1 + 10−2 +  + (10−1)n−1]
1 − 410 4 1 − (10−1)n
R = 4× = × (410 − 1) un = 9 × 10−1 ×
1− 4 3 1 − 10−1
R = 1398 100
1 − 10−n
b) On note (v n ) la suite géométrique de raison -
1 un = 9 × 10−1 ×
2 9 × 10−1
1 1
telle que v 0 = . un = 1 − 10−n = 1 − n
2 n 10
1  1 1
Pour tout nombre n de , v n = −  = . d) 2e méthode
2  2  (−2)n+1
1 Pour tout nombre n de  avec n >1,
On observe que v7 = −
256 0 , 99……9 + 0 , 00……01 = 1
Donc S = v 0 + v1 +  + v7 n fois n− 1 zéros
−n
1  1 7 c’est-à-dire un + 10 =1
S = v 0 − v 0 +  + −  v 0 1
2  2  soit un = 1 − 10−n
= 1− .
 1  1
2
 1 7  10n
S = v 0 1 − + −  +  + −  
 2  2   2 
 119 On note an la quantité d’eau, en L, dans la
8
 1  citerne au jour n (avec n Î  ) :
1 − − 
1   1 
8
1  2  2
S= × = × 1 −    a0 = × 1500 = 1000 et pour tout nombre n de ,
2  1 3   2   3 
1 − −   
an+1 = an 1 −
5 
= 0 ,95an .
 2   100 
85
S= La suite (an ) est géométrique de raison 0,95.
256
Pour tout nombre n de  :
c) On note (w n ) la suite géométrique de raison 10-1
an = 1000 × 0 ,95n.
telle que w 0 = 10−1.
Ainsi, a10 » 598 ,7 donc après 10 jours de sécheresse,
Pour tout nombre n de , w n = 10−1(10−1)n = 10−(n+1).
il reste environ 598,7 L d’eau dans la citerne.
Donc T = w 0 + w1 +  + w19
Pour arroser ses arbres au 10e jour, le jardinier aura
T = w 0 + 10−1w 0 +  + 10−19 w 0
besoin de 650 L d’eau ( 65 ´ 10  L).

Chapitre 1  ★  Suites numériques 15

172909_Chap01_000-000.indd 15 25/07/2019 17:55:37


Donc le jardinier n’aura pas suffisamment d’eau dans 124 a) 
sa citerne.

120 • P est fausse. En effet, u0 = −16 et −16 < 0.


• Q est vraie. En effet, u101 = 10 185.
• R est fausse. En effet, n2 − 16 = 425 équivaut à
n2 = 441, c’est-à-dire n = 21 (car n Î  ). Donc
u21 = 425.
La suite (un ) semble périodique de période 3 :
121 Par définition, dire que (un ) est une suite géo-
u0 = u3 = u6 =  = 1
métrique, signifie que : il existe un nombre réel q, tel
u1 = u4 = u7 =  = 2
que pour tout nombre n de , un+1 = qun .
u2 = u5 = u8 =  = 0
C’est donc la proposition Q qui définit une suite géo-
b) On note (v n ) la suite définie sur  par :
métrique.
Donc P ne définit pas une suite géométrique parce ïïì1 si n s’écrit sous la forme 3k
que le nombre q dépend de n. Par exemple, la propo- v n = ïí 2 si n s’écrit sous la forme 3k + 1 (avec k Î  )
ïï
sition est vraie pour la suite (un ) définie par ïî 0 si n s’écrit sous la forme 3k + 2
un = n + 1. En effet, pour tout nombre n de , 1er cas : n = 3k
n+2 Donc v n = 1.
un+1 = qun avec q = . 3 5 3 5
n +1 − v n2 + v n + 1 = − + + 1 = 2 = v n+1 car n + 1
Mais cette suite (un ) n’est pas géométrique : 2 2 2 2
u2 u s’écrit sous la forme 3k + 1.
¹ 1.
u1 u0 2e cas : n = 3k + 1
Donc v n = 2.
122 • non P : « Il existe un nombre n de  tel que 3 5
− v n2 + v n + 1 = −6 + 5 + 1 = 0 = v n+1 car n + 1
un+1 − un ≠ 2.  » 2 2
• non Q : « Pour tout nombre n de , un ¹ 0.  » s’écrit sous la forme 3k + 2.
3e cas : n = 3k + 2
Donc v n = 0.
3 5
− v n2 + v n + 1 = 0 + 0 + 1 = 1 = v n+1 car n + 1
2 2
Organiser son raisonnement s’écrit sous la forme 3(k + 1).
Conclusion : la suite (v n ) est telle que v 0 = 1 et
pour tout nombre n de ,
3 5
v n+1 = − v n2 + v n + 1.
2 2
123 a) On suppose qu’il existe une suite arithmé- Donc, pour tout nombre n de , v n = un et la suite
tique de raison r dont les trois premiers termes sont (un ) est périodique de période 3.
u0 , u1 = 5 et u2 . c)  2 020 = 2 019 + 1 = 3 × 673 + 1
Alors, u0 = 5 − r et u2 = 5 + r . Ainsi, 2 020 est de la forme 3k + 1.
Le triangle est rectangle donc d’après le théorème de Donc u2 020 = 2.
Pythagore :
125 a) Voici un tableau de suivi des variables.
(5 + r )2 = 52 + (5 − r )2
c’est-à-dire 25 + 10r + r 2 = 25 + 25 − 10r + r 2 N 15 46 23 70 35 106
soit 20r − 25 = 0. N%2==0 Faux Vrai Faux Vrai Faux Vrai
25 Syracuse(N) 46 23 70 35 106 53
Ainsi, r = = 125
, et :
20 N 53 160 80 40 20 10 5
•  u0 = 3,75  •  u1 = 5  •  u2 = 6 ,25 N%2==0 Faux Vrai Vrai Vrai Vrai Vrai Faux
Donc il existe bien une suite arithmétique dont les Syracuse(N) 160 80 40 20 10 5 16
trois premiers termes sont les mesures des longueurs
N 16 8 4 2
u0 , u1 = 5 et u2 des côtés d’un triangle rectangle.
N%2==0 Vrai Vrai Vrai Vrai
Syracuse(N) 8 4 2 1

16

172909_Chap01_000-000.indd 16 25/07/2019 17:56:56


On obtient la suite de nombres : Donc (v n ) est une suite géométrique de raison 1,04.
15 ; 46 ; 23 ; 70 ; 35 ; 106 ; 53 ; 160 ; 80 ; 40 ; 20 ; 10 ; 5 ; Son premier terme est
16 ; 8 ; 4 ; 2 ; 1. v 0 = u0 + 325 = 300 + 325 = 625.
b) • Pour N = 3, on obtient : c) • Pour tout nombre n de ,
3 ; 10 ; 5 ; 16 ; 8 ; 4 ; 2 ; 1 , n
v n = v 0 × 104
• Pour N = 34, on obtient : Or, v 0 = 625 donc v n = 625 × 104 , n.
34 ; 17 ; 52 ; 26 ; 13 ; 40 ; 20 ; 10 ; 5 ; 16 ; 8 ; 4 ; 2 ; 1 • Pour tout nombre n de ,
• Pour N = 75, on obtient : un = v n − 325 = 625 × 104, n − 325
75 ; 226 ; 113 ; 340 ; 170 ; 85 ; 256 ; 128 ; 64 ; 32 ; 16 ; d) On observe que u9 » 564 ,6 et u10 » 600 ,2.
8 ; 4 ; 2 ; 1 Après 10 semaines, les chardons auront envahi plus
Remarque : on dit que l’ensemble des étapes de la de 600 m2.
suite est le vol et que le nombre d’étapes est la durée
du vol. On peut vérifier par exemple qu’avec 128 1. a) Pour tout nombre n de ,
N = 77 671, la durée du vol est 231. un = v n − α.
c) Collatz a émis la conjecture que pour n’importe La suite (un ) est géométrique si, et seulement si, il
quel nombre entier naturel N>1, on finit toujours existe un nombre réel q tel que pour tout nombre n
par obtenir 1. de ,
un+1 = qun
126 Un édifice de n étages, contient
v n+1 − α = q (v n − α)
2 + 4 + 6 +  + 2n cartes dressées
1
et 1 + 2 +  + (n − 1) cartes horizontales. v n + 2 − α = qv n − q α
3
Le nombre S total de cartes est : 1 
 − qv n = α − q α − 2
S = 2 + 4 + 6 +  + 2n + 1 + 2 +  + (n − 1)  3 
S = 2(1 + 2 + 3 +  + n) + 1 + 2 +  + (n − 1) 1
S = 3(1 + 2 + 3 +  + n) − n − q = 0 et α − q α − 2 = 0
3
n(n + 1) 1 1
S = 3× −n q = et α = α + 2
2 3 3
3 3  n(3n + 1) 3n2 + n 1
S = n n + − 1 = = q = et α = 3
 2 2  2 2 3
5 × 52 = 260 , donc on cherche n pour que S = 260 Donc la suite (un ) définie sur  par un = v n − 3 est
c’est-à-dire pour que 3n2 + n = 520 soit 1
géométrique de raison .
3n2 + n − 520 = 0. 3
b) Pour tout nombre n de ,
∆ = 12 − 4 × 3 × (−520) = 6 241 = 792
−1 − 79 40 −1 + 79  1n
n′ = =− ou n′′ = = 13 un = u0  
 3 
6 3 6
Or, n est un nombre entier naturel, donc n = 13. Or, u0 = v 0 − 3 = 5 − 3 = 2, donc pour tout nombre
Ainsi, Raphaël pourra construire un château de cartes n de ,
de 13 étages sur ce modèle.  1n
un = 2 ×   .
 3 
127 a)  u0 = 300.
L’aire du jardin envahie la (n + 1) -ième semaine est Par conséquent, pour tout nombre n de ,
4  1n
obtenue en multipliant par 1 + l’aire du jardin v n = un + 3 = 2 ×   + 3
100  3 
envahie la n-ième semaine et en ajoutant 13 m2 dus à 2. En procédant comme ci-dessus, on obtient q = −3
la dissémination des graines. et α = −3α + 2
Donc, pour tout nombre n de , 1
Ainsi, q = −3 et α = .
un+1 = 104
, un + 13 2
1
b) Pour tout nombre n de , La suite (un ) définie sur  par un = w n − est géo-
2
v n+1 = un+1 + 325 métrique de raison -3.
v n+1 = 104
, un + 13 + 325 Donc, pour tout nombre n de ,
v n+1 = 104
, un + 338 un = u0 (−3)n
v n+1 = 104
, (un + 325) 1 1 3
Or, u0 = w 0 − = −1 − = − .
v n+1 = 104
, vn 2 2 2

Chapitre 1  ★  Suites numériques 17

172909_Chap01_000-000.indd 17 25/07/2019 17:58:17


Donc pour tout nombre n de , 131 • Pour tout nombre n de  avec n >1, l’aire !n
3 du domaine jaune est :
un = − × (−3)n
2 ! n = un2 − un2−1
Par conséquent, pour tout nombre n de , ! n = (1 + 2 +  + n)2 − (1 + 2 +  + (n − 1))2
1 3 1
w n = un + = − × (−3)n +  n(n + 1)  2  (n − 1)n  2
2 2 2 !n =   − 
Remarque : de façon générale, si (un ) est une suite  2   2 
1
telle que pour nombre n de , un+1 = aun + b avec a ! n = [n2 (n + 1)2 − (n − 1)2 n2 ]
4
et b nombres réels, a ¹ 0 et a ¹ 1, alors la suite (v n )
n2
définie sur  par v n = un − α, où a est la solution de ! n = [(n + 1)2 − (n − 1)2 ]
4
l’équation x = a x + b, est géométrique de raison a. 1 2
! n = n (n + 1 + n − 1)(n + 1 − n + 1)
4
129 a) Pour tout nombre n de  avec n >1, le 1
! n = n2 × 2n × 2
triangle OA nA n+1 est rectangle isocèle en A n+1. 4
En effet, les deux droites d’équations respectives ! n = n3
y = x et y = −x sont les bissectrices de « l’angle n3
droit du repère d’origine O », donc un
A 
n OA n+1 = OA n A n+1 = 45°.
D’où OA n+1 = A nA n+1 et OA n = A n−1A n .
2 2
A nA n+1 = sin(45°) × OA n = OA n = A n−1A n . un - 1
2 2
b) Pour tout nombre n de  avec n >1, on note  la
distance A n-1A n . La suite ( n ) est donc géométrique
2 2
de raison et de premier terme  1 = sin(45°) = .
2 2

On pose Sn =  1 +  2 +  3 + … +  n .
 2
 2 n−1
 2  2 
Sn =  11 + +   + … +   . 6
 2  2   2   33
 3
 2 n 23
1 −   1
  n  13
2  2   2 
Sn = × = ( 2 + 1) 1 −    0 u1 u2 u3 … un - 1 un
2 2   2  
1−   Géométriquement, la somme 13 + 23 +  + n3
2
représente ci-dessous, l’aire du carré de côté un .
130 On note un la distance, en m, entre les sacs et le Ainsi, pour tout nombre n de ,
n-ième arbre (avec 1< n < 20 ). 13 + 23 +  + n3 = un2 = (1 + 2 +  + n)2
Pour tout nombre n de  avec 1< n < 20 ,
un+1 − un = 4 132 On note Tn le nombre de transistors dans les
et donc, un = u1 + 4(n − 1) tous nouveaux microprocesseurs de l’année
c’est-à-dire un = 15 + 4(n − 1) 1970 + n (avec n Î  ).
soit un = 4 n + 11 D’après l’énoncé, T0 = 2 300 et pour tout nombre n
On note S = u1 + u2 +  + u20 et D la distance par- de , Tn+1 = 1,40 × Tn .
courue par le jardinier. On souhaite déterminer le premier rang n tel que
D = 2S Tn >15 ´ 109.
D = 2(u1 + (u1 + 4) +  + (u1 + 19 × 4)) On peut procéder avec un tableur ou une calculatrice.
D = 2 × [20 u1 + 4(1 + 2 +  + 19)] On trouve n > 47.
 19 × 20  Selon la loi de Moore, les scientifiques auraient pu
D = 2 × 20 × 15 + 4 × 
 2  construire des ordinateurs quantiques à partir de
D = 2 × (300 + 4 × 190) 2017 !
D = 2 × (300 + 760) = 2120 Il n’en est rien car, comme dit dans le manuel, la
Au total, le jardinier aura parcouru 2 120 m. miniaturisation des composants demande mainte-
nant plus de temps.
18

172909_Chap01_000-000.indd 18 25/07/2019 17:59:27


133 1. Voici comment résoudre ce casse-tête en c) 
7 déplacements ; on note le n° du disque déplacé et
au-dessous la tige sur lequel il est disposé.
1 2 1 3 1 2 1
C B B C A C C
Remarque : dans la suite des numéros de disques on
peut observer certaines symétries
1 2 1 3 1 2 1
On dit que c’est l’algorithme du miroir itéré qui régit
les déplacements des disques.
d) On exécute Fibonacci(20) et on obtient :
Déplacer un 1er disque
Déplacer un autre disque
Déplacer à nouveau le 1er disque
Répéter
2. a) On transporte les n - 1 disques les plus petits
en B, puis on transporte le disque le plus grand en C
et enfin, on transporte les n -1 disques les plus
petits de B en C.
Ainsi, pour tout nombre n de  avec n >1,
Tn = 2 Tn−1 + 1
b) Pour tout nombre n de  avec n >1,
un = Tn + 1
un = 2 Tn−1 + 1 + 1
un = 2 Tn−1 + 2
un = 2(Tn−1 + 1)
un = 2un−1
Donc la suite (un ) est géométrique de raison 2.
c) Pour tout nombre n de  avec n >1,
2. 
un = u1 × 2n−1
Mois 1 2 3 4 5 6
Or, u1 = T1 + 1 = 1 + 1 = 2, donc pour tout nombre n
de  avec n >1,
un = 2 × 2n−1 = 2n
Par conséquent, pour tout nombre n de  avec n >1,
Tn = un − 1 = 2n − 1
d)  T10 = 210 − 1 = 1023
On peut transporter une tour de 10 disques de A en C
en 1 023 coups au minimum.

134 1. a)  F2 = F1 + F0 = 1 + 0 = 1
F3 = F2 + F1 = 1 + 1 = 2
F4 = F3 + F2 = 2 + 1 = 3 Nombres
b)  X ¬ 0 de couples 1 1 2 3 5 8
Y ¬1 e
Au 7  mois, parmi les 8 couples existants, seuls 5 sont
Afficher X, Y adultes (trame grise) et vont procréer. Il y aura donc
Pour i allant de 2 à n 13 couples (8 + 5 = 13).
Z←X+Y Plus généralement, le nombre de couples Fn+2 au
X¬Y mois n + 2 est égal au nombre de couples adultes, à
Y¬Z savoir ceux présents au mois n + 1, Fn+1, plus ceux
Afficher Z nés au mois n (et qui vont donc devenir adultes au
Fin Pour mois n + 2 ), Fn .

Chapitre 1  ★  Suites numériques 19

172909_Chap01_000-000.indd 19 25/07/2019 18:00:07


Ainsi, Fn+2 = Fn+1 + Fn 
X = x + 1
Or, F1 = 1 et F2 = 1. soit  x −1
Donc l’évolution de cette population de lapins est  4
X = 1
modélisée par la suite de Fibonacci.

F12 = 144 , donc il y aura 144 couples de lapins au X = x + 1
bout d’un an.  x −1
 2
X = 1
135 Avec n nombre de  et n >1, la n-ième ligne  x +1
comporte 2n - 1 cases et se termine par n2 . X = x − 1 (car X ¹ 1 )


Le carré immédiatement supérieur à 2 020 est 452 X = −1
(soit 2 025). 2 020 est donc sur la ligne débutant par
x +1
44 2 + 1 (soit 1 937) et finissant par 2 025 et compor- L’équation = −1 c’est-à-dire x + 1 = −x + 1 a
x −1
tant 89 cases. Donc 2 020 figure sur la 45e ligne. pour solution x = 0.
2 020 − 1937 = 83, donc 2 020 occupe le 84e rang Donc l’équation initiale a pour unique solution 0 et
de cette 45e ligne. 6 = {0}.

136 Si u0 , u1, u2 sont les trois premiers termes  5 


138 1.  u1 = (3 000 + 80)1 − = 2 926
d’une suite arithmétique de raison r, alors u0 = u1 − r  100 
et u2 = u1 + r . 2. Pour tout nombre n de ,
 5 
Si u0 , u1, u2 sont les trois premiers termes d’une suite un+1 = (un + 80)1 −
 100 
u
géométrique de raison q, alors u0 = 1 et u2 = qu1.
q un+1 = 0 ,95 un + 80 × 0 ,95
On cherche donc r et q tels que
un+1 = 0 ,95 un + 76
 u1
u1 − r = q 3. a) Pour tout nombre n de ,
 v n+1 = un+1 − 1520

u1 + r = qu1 v n+1 = 0 ,95 un + 76 − 1520
Par addition membre à membre de ces équations, il v n+1 = 0 ,95 un − 1444
 1 1
vient 2u1 = u1q +  c’est-à-dire q + = 2 soit v n+1 = 0 ,95(un − 1520)
 q  q v n+1 = 0 ,95 v n
q 2 − 2q + 1 = 0. Donc la suite (v n ) est géométrique de raison 0,95.
De (q − 1)2 = 0 , on déduit q = 1. v 0 = u0 − 1520 = 3 000 − 1520 = 1480
En remplaçant q par 1 dans la 2e équation du sys- b) Pour tout nombre n de ,
tème, il vient u1 + r = u1 soit r = 0. v n = v 0 × 0 ,95n
Conclusion : u0 , u1, u2 ne sont les trois premiers Or, v 0 = 1480 donc, pour tout nombre n de ,
termes d’une suite à la fois arithmétique et géomé- v n = 1480 × 0 ,95n
trique, que dans le cas où u0 = u1 = u2 . Par conséquent, pour tout nombre n de ,
Une suite arithmétique de raison 0 et une suite géo- un = v n + 1520 = 1480 × 0 ,95n + 1520
métrique de raison 1 sont des suites constantes. 4. 
n¬0
137 Résoudre cette équation équivaut à résoudre le u ¬ 3 000
système : Tant que u > 2 000
 x +1 n ← n+1
X = x − 1 u ← 1480 × 0,95n + 1520


1 + X + X2 + X3 = 0 Fin Tant que

 x +1 Remarque : avec un programme en langage Python ou


X = x − 1 avec la calculatrice, on constate que le nombre de céta-
c’est-à-dire  cés dans la réserve sera inférieur à 2 000 pour n = 22.
1 − X 4
 =0
 1 − X
(en effet, X ¹ 1 car x + 1 ≠ x − 1)

20

172909_Chap01_000-000.indd 20 25/07/2019 18:01:26


Pour inspirer la même quantité d’air à El Alto qu’à son

Exploiter ses compétences


lieu de résidence habituel, Lina doit effectuer environ
19 inspirations par minute au repos.

141 • On note un le coût de forage de la n-ième


dizaine de mètres avec n nombre de  et n >1.
139 Chaque rangée comprend trois sièges de plus Pour tout nombre n de  avec n >1,
que la précédente. un+1 = un + 200 000
Pour tout nombre n de  avec 1< n < 30 , on note Sn un+1 − un = 200 000
le nombre de sièges sur la rangée numérotée n. Donc, la suite (un ) est arithmétique de raison
(Sn ) est une suite arithmétique de raison 3 telle que 200 000.
S1 = 21. Pour tout nombre n de  avec n >1,
On note S le nombre total de sièges dans les ran- un = u1 + 200 000(n − 1)
gées 1 à 15. un = 100 000 + 200 000n − 200 000
S = S1 + S2 +  + S15 un = 200 000n − 100 000
S = S1 + (S1 + 3) +  + (S1 + 14 × 3) • Le coût S, en euro, pour un forage de n dizaines de
S = 15 S1 + 3 × (1 + 2 +  + 14) mètres est :
14 × 15 S = u1 + u2 +  + un
S = 15 × 21 + 3 ×
2 S = u1 + (u1 + 200 000) +  + (u1 + 200 000(n − 1))
S = 15 × 21 + 3 × 7 × 15 = 2 × 15 × 21 = 630 S = nu1 + 200 000 × (1 + 2 +  + (n − 1))
On note S¢ le nombre total de sièges dans les ran- (n − 1) × n
gées 16 à 30. S = 100 000n + 200 000 ×
2
S′ = S16 + S17 +  + S30 S = 100 000n + 100 000n(n − 1)
S′ = S16 + (S16 + 3) +  + (S16 + 14 × 3) S = 100 000n2
S′ = 15 S16 + 3 × (1 + 2 +  + 14) • On cherche donc le plus grand nombre entier n tel
Or, S16 = S1 + 15 × 3 = 21 + 15 × 3 = 66. que 100 000n2 < 14 400 000 c’est-à-dire tel que
14 × 15 n2 < 144. Donc n = 12.
Donc, S′ = 15 × 66 + 3 ×
2 Conclusion : avec ce budget, la compagnie pétrolière
S′ = 15 × 66 + 3 × 7 × 15 = 1305
peut effectuer un forage de 120 m.
La recette totale pour une salle qui affiche complet
est donc donnée par : 142 ( xn ) est la suite définie par x0 = 0 ,63 et pour
630 630 1305 1305
50 × + 40 × 2 × + 40 × + 30 × 2 × . tout nombre n de ,
3 3 3 3
xn+1 = r xn (1 − xn )
On trouve une recette de 70 800 €.
où r est un nombre réel de l’intervalle [0 ; 4 ].
140 • L’altitude de son lieu de résidence habituel est On envisage plusieurs valeurs de r et on représente
100 m. graphiquement la suite ( xn ) correspondante
 125 ,  (fenêtre : 0 < X < 20 , pas 1 et 0 < Y < 1, pas 0,1)
1013 × 1 − ≈ 1000
 100  1er cas : r ∈ [0 ; 1]
La pression atmosphérique de son lieu de résidence La population s’éteint.
habituel est environ 1 000 hPa. Ici, r = 0 ,8.
• L’altitude à El Alto est 41´ 100 m.
41
 125 , 
1300 × 1 −  ≈ 776
 100 
La pression atmosphérique à El Alto est donc d’envi-
ron 776 hPa.
•  P × N = constante donc :
776 × N ≈ 1000 × 15
1000 × 15
soit N ≈
776
et N » 19

Chapitre 1  ★  Suites numériques 21

172909_Chap01_000-000.indd 21 25/07/2019 18:02:27


2e cas : r ∈ [1 ; 3] Ici, r = 3,1. Les valeurs de xn oscillent entre deux
L’effectif de la population se stabilise. valeurs, environ 0,6 et 0,8 ; l’effectif de la population
oscille vers 60 loups ou 80 loups.

Ici, r = 13, . Les valeurs xn se stabilisent vers 0,25 et


l’effectif de la population vers 25 loups. Ici, r = 3,5. Les valeurs de xn oscillent entre quatre
valeurs, environ 0,4 ; 0,5 ; 0,8 et 0,85. L’effectif de la
population oscille vers 40 ou 50 ou 80 ou 85 loups.
4e cas : r ∈ [3,57 ; 4 ]
Le chaos s’installe.

Ici, r = 2,5. Après avoir oscillé au début, les valeurs


de xn se stabilisent vers 0,6 et l’effectif de la popula-
tion vers 60 loups.
3e cas : r ∈ [3 ; 3,57]
L’effectif de la population oscille entre plusieurs Ici, r = 4. Aucune oscillation n’est visible.
valeurs.

22

172909_Chap01_000-000.indd 22 25/07/2019 18:02:48


2 Comportement
d’une suite

Découvrir Acquérir des automatismes

1 Sens de variation d'une suite 3 a)  un+1 − un = 2n − 9 , 2n − 9 > 0 pour tout


9
n > , la suite (un ) est croissante à partir du rang 5.
1   2
b) Pour tout nombre n de , v n = f (n) où f est la
Modèle A Modèle B Modèle C
fonction définie sur  par f( x) = −3 x2 − 3 x + 1 .
Pour tout nombre réel x, f ′( x) = −6 x − 3. D’où le
tableau de variations de la fonction f.
1
x −∞ - +∞
Population 2
croissante à Population décroissante Population oscillante, Signe de f ¢(x) + -
taux avec apport annuel ni croissante ni 7
d'évolution constant décroissante Variations de f 4
constant
 1 
2  Le modèle A ne parait pas durablement réaliste La fonction f est décroissante sur − ; + ∞  donc la
 2 
du fait de l’augmentation constante de la popu- suite (v n ) est décroissante.
lation.
un+1 u
4 a)  = 0 , 5 donc n+1 < 1,
3  a)  pn+1 − pn = 0 , 035 pn expression positive un un
p c’est-à-dire un+1 < un .
b)  n+1 = 1, 035 > 1
pn Donc la suite (un ) est décroissante.
c) On prouve ainsi que la suite ( pn ) du modèle A v v
b)  n+1 = 3 donc n+1 > 1 c’est-à-dire v n+1 > v n .
est croissante. vn vn
Donc la suite (v n ) est croissante.
2 Notion de limite infinie w w
d'une suite c)  n+1 = 1, 02 donc n+1 > 1
wn wn
c’est-à-dire w n+1 > w n .
1  a)  c5 = 15, c6 = 21
Donc la suite (w n ) est croissante.
n(n + 1)
b)  cn = , c50 = 1275 .
2
2  a) Il semble que quand n prend des valeurs de
plus en plus grandes, cn prend des valeurs de
plus en plus grandes.
b)  c141 > 10000 , c 4 600 > 106

Chapitre 2  ★  Comportement d’une suite 23

172909_Chap02_000-000.indd 23 25/07/2019 18:07:22


7 a)  1 1
14 a)  w n+1 − w n = −
n + 2 n +1
n + 1 − (n + 2)
=
(n + 1)(n + 2)
−1
5
= .
10 + 4 (n + 1)(n + 2)
b)  4 − 3n < −105 équivaut à n > soit
3 b) Pour tout n de , (n + 1)(n + 2) > 0 donc
n > 33334 , 67.
−1
v n < −105 pour tout entier supérieur ou égal à < 0 , la suite (w n ) est décroissante.
(n + 1)(n + 2)
33 335.
1020 + 4 2 2
4 − 3n < −1020 équivaut à n > soit 15 a)  t n+1 − t n = 1 − − 1+
3 n+4 n+3
n > 3, 33 × 1019.
v n < −105 pour tout entier strictement supérieur à 2n + 8 − 2n − 6
=
3, 33 ´ 1019. (n + 4)(n + 3)
c) Il semble que la suite (v n ) a pour limite −∞. 2
= .
(n + 4)(n + 3)
8 a)  b) Pour tout n de , (n + 4)(n + 3) > 0 donc
2
> 0 , la suite (t n ) est croissante.
(n + 4)(n + 3)

16 v n+1 − v n = −2v n2 < 0, la suite (v n ) est décrois-


b) Comme w1 = 3, 2 < w n < 2, 01 équivaut à sante.
1
2 + < 2, 000 1 soit n > 1 000.
n n +1 n
c) Il semble que la suite (w n ) a pour limite 2. 17 t n+1 − t n = −
2n + 3 2n + 1
1
9 (un ) semble croissante, (v n ) semble décrois- = > 0,
(2n + 3)(2n + 1)
sante, (w n ) semble croissante à partir du rang 2.
la suite (t n ) est croissante.
10 Julia se trompe, un+1 − un = 5 > 0, la suite (un )
18 un+1 − un = 4 − (n + 2)2 − 4 + (n + 1)2
est croissante.
= − 2n − 3 < 0 ,
11 Paolo se trompe, v n+1 − v n = −2 < 0, la suite la suite (un ) est décroissante.
(v n ) est décroissante.
19 a)  Sn+1 − Sn = n + 1,
12 a)  un+1 − un = −3 < 0, la suite (un ) est b) Comme n > 1, n + 1 > 0 , la suite (Sn ) est crois-
décroissante. sante.
1
b)  un+1 − un = > 0, la suite (un ) est croissante.
5 20 un+1 − un = n2 − 10 > 0 si et seulement si
c)  un+1 − un = 2n + 1 > 0, la suite (un ) est crois- n2 > 10 , c’est-à-dire n > 10 soit n > 3,16.
sante. La suite (un ) est croissante à partir du rang 4.
d)  un+1 − un = 2n + 1 < 0 pour n > 1, la suite (un )
est décroissante. 21 a)  en+1 − en = 40 000 × 0 , 89n+1 + 13 000
−40 000 × 0 , 89n − 13 000
13 a)  v n+1 − v n = n + 4, la suite (v n ) est crois-
= 40 000 × (0 , 89 − 1)
sante. = −0 ,11× 40 000
b)  v n+1 − v n = −2n2 < 0, la suite (v n ) est décrois- b)  en+1 − en < 0, la suite (en ) est décroissante.
sante. L’empreinte carbone de l’entreprise va diminuer.

24

172909_Chap02_000-000.indd 24 25/07/2019 18:08:56


22 La différence de deux termes impairs consécu- 29 a) La suite (un ) définie par un = 4 n − 7 est
tifs est croissante.
u2k +3 − u2k +1 = 3 + (−2)2k +3 − 3 − (−2)2k +1 b) La suite (un ) définie par un = −n + 5 est décrois-
= (−2)2k +1 × ((−2)2 − 1) sante.

= 3 × (−2)2k +1. 30 a) x −∞ -1 +∞
2 k +1
Comme (−2) < 0 , les termes impairs de la suite
(un ) sont décroissants. f (x)
0
Par conséquent la suite (un ) n’est ni croissante ni
décroissante. La fonction f définie par f( x) = ( x + 1)2 est croissante
sur [0 ; + ∞[ , donc la suite (un ) est croissante.
23 La fonction f est croissante sur l’intervalle b) x −∞ 0 +∞
[0 ; + ∞[ , donc la suite (un ) est croissante.
f (x)
24 La suite (un ) est croissante à partir du rang 2.

10
25 a) x 0 +∞ La fonction f définie par f( x) = 2 − est décrois-
x
+∞ sante sur [0 ; + ∞[ , donc la suite (un ) est décrois-
f (x)
0 sante.

b) La suite (un ) définie sur  par un = n2 a le même 31 La fonction f définie par f( x) = x2 − 200 x est
sens de variation que la fonction f définie sur décroissante sur ]− ∞ ; 100 ], puis croissante sur
[0 ; + ∞[ par f( x) = x2 , elle est croissante. [100 ; + ∞[ , donc la suite (un ) est croissante à partir
du rang 100.
26 a) x 0 +∞ Laura se trompe.

f (x) 32 w n est de la forme f (n), avec f la fonction défi-


2x
nie sur ]0 ; + ∞[ par f( x) = 1 + .
1 x +1
b) La suite (un ) définie sur  par un = a le même
n 2
sens de variation que la fonction f définie sur f ′( x) = > 0.
( x + 1)2
1
[0 ; + ∞[ par f( x) = , elle est décroissante. La fonction f est croissante sur [0 ; + ∞[ , donc la
x
suite (w n ) est croissante.
27 a) La fonction f définie sur  par f( x) = 2 − 3 x
permet de définir la suite (an ). 33 v n est de la forme f (n), avec f la fonction défi-
nie sur ]0 ; + ∞[ par f( x) = 0 , 2 x2 + x − 3.
b) f est décroissante sur [0 ; + ∞[ , la suite (an ) est
donc décroissante. f ′( x) = 0 , 4 x + 1 > 0 sur ]0 ; + ∞[.
La fonction f est croissante sur [0 ; + ∞[ , donc la
28 a) La fonction f définie sur  par f( x) = 2 − x2 suite (v n ) est croissante.
permet de définir la suite (bn ).
34 a) 
b)  f ′( x) = −2 x , f ′( x) < 0 pour tout x ∈ [0 ; + ∞[.
x 0 +∞

f (x)  

On en déduit que la suite (bn ) définie sur  par b)  w n est de la forme f (n), avec f la fonction définie
un = 2 − n2 a le même sens de variation que la fonc- sur ]0 ; + ∞[ par  :
tion f définie sur [0 ; + ∞[ par f( x) = 2 − x2 , elle est 1
f( x) = 2 + .
décroissante. x+3

Chapitre 2  ★  Comportement d’une suite 25

172909_Chap02_000-000.indd 25 25/07/2019 18:10:09


1 9 n +2
f ′( x) = − < 0 sur [ 0 ; + ∞[.
( x + 3)2 u n+1 9 n +2 7n 9
41 n+1 = 7 n+1 = n+1 × n+1 = > 1, la suite
La fonction f est décroissante sur [ 0 ; + ∞[ , donc la un 9 7 9 7
suite (w n ) est décroissante. 7n
(un ) est croissante.
35 hn est de la forme f (n), avec f la fonction défi-
nie sur [2 ; 13] par f( x) = −0 , 2 x2 + 2, 5 x − 1, 6. 42 a) Pour tout n de , 3n > 0 et n + 2 > 0, d’où
f ′( x) = −0 , 4 x + 2, 5. v n > 0 pour tout n de .
La fonction f est croissante sur [2 ; 6 , 25] puis décrois- b) Pour tout n de , 2n + 3 et n + 3 sont stricte-
sante sur [6 , 25 ; 13], donc la suite (hn ) est décrois- ment positif, leur quotient l’est aussi donc :
sante à partir du rang 6. v n+1 v
− 1 > 0 et n+1 > 1.
À 8 h on est en marée descendante. vn vn
La suite (v n ) est croissante.
36 (un ) semble être une suite géométrique
décroissante.  1 n
43 un = 130  .
 4 
v 2
37 Rayan a raison n+1 = < 1 et v 0 > 0 la suite
vn 3  1 n+1
(v n ) est décroissante. 130 
un+1  4  1
= n
= < 1.
un+1 un  1 4
38 a)  = 2 > 1, la suite (un ) est croissante. 130 
 4 
un
Lisa a raison, la suite est décroissante.
un+1 2
b)  = < 1, la suite (un ) est décroissante.
un 2 v
44 a)  n+1 = 0 , 35 < 1, la suite (v n ) est décrois-
u vn
c)  n+1 = 2 − 1 < 1, la suite (un ) est décroissante. sante.
un mn+1 − mn = v n+1 − v n < 0 la suite (mn ) est décrois-
39 Pour tout n de , 2n > 0 , comme 5 > 0 , tous sante.
les termes de la suite (v n ) sont positifs.
45 1. a) 
v 5 × 2n+1
b)  n+1 = = 2, la suite (v n ) est la suite géo-
vn 5 × 2n
métrique de raison 2 et de premier terme v 0 = 5.
v
c)  n+1 = 2 > 1, la suite (v n ) est donc croissante.
vn

40 a) 

mn+1 0 , 6 × 1, 22n+1
La suite (un ) semble décroissante. b)  = = 1, 22 > 1, la suite (mn ) est
mn 0 , 6 × 1, 22n
2n+1 croissante.
un+1 n +2 2n+1 7n+1 2 2. a)  m48 » 8383 g, m72 » 990966 g.
b)  = 7 n = n +2 × n = .
un 2 7 2 7 b) La masse de la culture dépasse 20 kg au bout de
n+1
7 53 h.
2 3. Le modèle choisi par les étudiants se traduit par
c)  < 1, donc un+1 < un , la suite (un ) est décrois-
7 une augmentation infinie de masse.
sante.

26

172909_Chap02_000-000.indd 26 25/07/2019 18:11:22


46 Méthode 1 suite (w n ). 1
51 a)  Sn+1 − Sn = > 0 pour tout n Î . La
n+1
Méthode 2 suite (v n ). 2
Méthode 3 suite (un ). suite (Sn ) est donc croissante.
1 1 1
b)  Sn = + +… + n ,
47 a)  2 4 2
2 2 2 1 1 1 
2 × Sn = + + … + n = 1 +  + +… + n−1 
2 4 2 
2 4 2 
1
= 1 + Sn − n .
2
1
On obtient 2Sn − Sn = Sn = 1 − n .
2
c)  S0 = 1 − 1 = 0.
La suite (un ) semble croissante à partir du rang 2. Pour tout n Î , Sn < 1 et comme la suite (Sn ) est
2n+1 croissante.
u n −1
b)  n+1 − 1 = n +n 1 − 1 = . Pour tout n Î , 0 < Sn < 1.
un 2 n +1
n v n+1 un+1 − 1 2un − 2
52 a)  = = = 2, (v n ) est la
n −1 u vn un − 1 un − 1
Pour n > 1, > 0, donc n+1 > 1.
n +1 un suite géométrique de raison 2 et de premier terme
La suite (un ) est croissante à partir du rang 2. v 0 = u0 − 1 = 2.
b)  v n = 2 × 2n = 2n+1. Donc un = 2n+1 + 1.
v n+1
48 a)  = 0 , 7 < 1, la suite (v n ) est donc c)  un+1 − un = 2n+2 + 1 − 2n+1 − 1 = 2n+1(2 − 1)
vn
décroissante. = 2n+1 > 0 ,
b)  un+1 − un = 8 − v n+1 − (8 − v n ) = v n − v n+1 > 0 la suite (un ) est croissante.
car (v n ) est décroissante.
53 a)  u0 = 1 équivaut à A × 30 + B = 1 soit
La suite (un ) est donc croissante.
A + B = 1, u1 = 3 × 1 + 2 = 5, on a donc 3A + B = 5.
49 a)  u0 = 6 , u1 = 4 , u3 = 3, u4 = 4. On obtient A = 2 et B = −1.
b)  un+1 = un − 3 + n + 1 On a donc un = 2 × 3n − 1.
c)  un+1 − un = −3 + n + 1, −3 + n + 1 > 0 si et seu- b)  un+1 − un = 2 × 3n+1 − 1 − 2 × 3n + 1
lement si n > 2, la suite (un ) est croissante à partir = 2 × 3n (3 − 1) = 4 × 3n > 0
du rang 3. La suite (un ) est donc croissante.

50 a)  54 a)  u1 = 650 , u2 = 785.


b) La chaine garde 90 % de ses abonnés soit un coef-
ficient de 0,9. Elle gagne 200 abonnés, d’où une aug-
mentation de 200 soit un+1 = 0 , 9un + 200. .
c)  v n+1 = un+2 − un+1
= 0 , 9un+1 + 200 − un+1
= −0 ,1un+1 + 200
La suite (an ) semble croissante. c)  = −0 ,1(0 , 9un + 200) + 200
b) Considérons la fonction f définie sur [ 0 ; + ∞[ par = −0 , 09un + 180
3x − 1 = 0 , 9(−0 ,1un + 200)
f( x) = .
4x + 5 v n = un+1 − un = 0 , 9un + 200 − un = −0 ,1un + 200.
19 v
Sa dérivée f ′( x) = est strictement positive.
(4 x + 5)2 D’où n+1 = 0 , 9.
vn
La fonction f est donc croissante sur [ 0 ; + ∞[ la suite
(v n ) est la suite géométrique de raison 0,9 et de pre-
(an ) est croissante.
mier terme v 0 = 150.
d) Pour tout n Î , v n > 0, donc un+1 − un > 0  : la
suite (un ) est croissante. La chaîne locale peut donc
se féliciter de ses résultats.

Chapitre 2  ★  Comportement d’une suite 27

172909_Chap02_000-000.indd 27 25/07/2019 18:13:02


55 Les deux suites semblent tendre vers 0. 60 a) Chaque fraction est égale à l’aire du rectangle
coloré dans laquelle elle est écrite.
56 a) La suite (un ) semble tendre vers -2.
b) La somme des aires est égale à l’aire du carré de
b) La suite (un ) semble tendre vers 5.
côté 1.
57 1 1 1 1
Quand n tend vers +∞, + + +… + n +…
a) 2 4 8 2
semble tendre vers 1.

61 1. a) La suite (un ) qui modélise l’évolution du


nombre d’insecte est une suite géométrique de rai-
20
son 1 − = 0 , 8. Le nombre initial d’insectes est
100
15 000.
On a donc un = 15000 × 0 , 8n.
La suite (un ) semble tendre vers 2. b) C’est l’écran 2 qui affiche les premiers points de la
b)
représentation graphique de (un ).
2. a)

Le nombre d’insectes sera inférieur à 5 000


au bout de 5 ans.

La suite (v n ) semble tendre vers 5. Le nombre d’insectes sera inférieur à 1 000


c) au bout de 13 ans.

Le nombre d’insectes sera inférieur à 100


au bout de 23 ans.
b) La suite (un ) semble tendre vers 0. La population
d’insecte va s’éteindre.

La suite (w n ) semble tendre vers 5. 62 a) Le coefficient qui permet de modéliser l’évo-
9
lution de population est 1 − = 0 , 91. On a donc
58 a)  v 0 = 3 × 0 , 4 0 = 3, v1 = 3 × 0 , 4 = 1, 2, 100
v n = 1250 × 0 , 91n.
v 2 = 3 × 0 , 4 2 = 0 , 48 ,
b) En 2020 il y aura 1250 ´ 0 , 914 soit environ 857
v 3 = 3 × 0 , 4 3 = 0 ,192, v 4 = 3 × 0 , 4 4 = 0 , 0768.
gazelles.
b) 
c) La population de gazelles sera inférieure à 100 indi-
vidus en 2047.
d) La suite (v n ) semble tendre vers 0. La population
0 < v n < 10−3 0 < v n < 10−6
de gazelles va s’éteindre.
pour n > 8. pour n > 16.
c) La suite (v n ) semble tendre vers 0. 63 a) La limite de la suite 1 semble être +∞,
b) la limite de la suite 1 semble être −∞.
1 1 1 1
59 a)  u2 = 2
= , u3 = 2 = ,
2 4 3 9 64 a) La limite de la suite semble être +∞,
1 1 1 1 b) la limite de la suite semble être −∞.
u4 = 2 = , u5 = 2 = .
4 16 5 25
b)  65 a) La limite de la suite (un ) semble être +∞,
b) La limite de la suite (v n ) semble être 5,
c) La limite de la suite (un ) semble être −∞.
0 < un < 10−2 0 < un < 10−4
pour tout entier n > 10. pour tout entier n > 100.
c) La suite (un ) semble tendre vers 0.
28

172909_Chap02_000-000.indd 28 25/07/2019 18:13:52


70 a)  un = 10 × 1, 25n.
66 a)      
b)


b)  un < −100 pour n > 11, un < −1000 pour
n > 32.
c) La suite (un ) semble tendre vers −∞.

67 a)      

c) Le nombre de bactéries est supérieur à 100 000 au


bout de 52 heures.
b)  v n > 500 pour n > 11, v n > 5000 pour n > 32. d) La suite (un ) semble tendre vers +∞.
c) La suite (v n ) semble tendre vers +∞.
71 a)  pn = 150000 × 1, 05n.
68 a)  un = 1 + 3, 5n b)
999
b) 1 + 3, 5n > 1000 équivaut à n > soit n > 285.
3, 5
106 − 1
1 + 3, 5n > 106 équivaut à n >
3, 5
soit n > 28714.
c) La suite (un ) semble tendre vers +∞.

 3 n
69 a)  un = 0 ,1×  
 2 
b)

un > 1000 pour n > 22.

un > 106 pour n > 39.


c) La suite (un ) semble tendre vers +∞.
c) La suite ( pn ) semble tendre vers +∞.
La population de la ville semble croitre indéfiniment.

72 1. 3               2. 1               3. 4               4. 3

Chapitre 2  ★  Comportement d’une suite 29

172909_Chap02_000-000.indd 29 25/07/2019 18:14:35


73 1. 3               2. 1               3. 3

74 1. Affirmation fausse, n > 1, donc n2 > 1 et S'entraîner


1 1
2
> 0. On a donc 2 + 2 > 2.
n n 1
2. Affirmation vraie, n > 1, donc n2 > 1 et 2 < 1.
1 n
On a donc 2 + 2 < 3. 78 a) 
n
3. Affirmation fausse,
1 1 n2 − (n + 1)2
un+1 − un = 2 + − 2 − =
(n + 1)2 n2 n2
−2n − 1
= . b) seuil 500 : seuil (500) = 35 ; seuil (106 ) = 76 ; seuil
n2
−2n − 1 (1010 ) = 127
Comme n > 1, < 0 , la suite (un ) est décrois- La suite (un ) semble avoir pour limite +∞.
sante. n2
4. Affirmation vraie, n > 1000, équivaut à n2 > 106 79 a) 
1 1
soit 2 < 10−6. On sait que 2 + 2 > 2, on peut
n n
1
donc affirmer que 2 < 2 + 2 < 2 + 10−6.
n
5. Affirmation fausse,
1 b) seuil (100) = 8 ; seuil (1000) = 24 ;
comme 2 < 2 + 2 < 2 + 10−6 pour n > 1000,
n seuil (106 ) = 709
on peut affirmer qu’à partir du rang 1 000, c) La suite (un ) semble avoir pour limite +∞.
un ∈ ]2 − 10−6 ; 2 + 10−6 [.
La suite (un ) a pour limite 2. ,n
81 a)  pn = 270 × 11
b) 
75 1. a) 

b) 


La suite (un ) semble croissante à partir du rang 2.
2.  f ′( x) = 2 x − 4, f ′( x) > 0 pour x > 2.
x 0 2 +∞ Il y aura plus de 100 000 pies à partir de l’année 38.
f ¢(x) - +
82 b)  t n > 100 pour n > 8, t n > 1000 pour
6
n > 19, t n > 10000 pour n > 57, t n > 106 pour
f (x)
n > 568.
2
c) La suite (t n ) semble avoir pour limite +∞.
La suite (un ) a le même sens de variation que la fonc-
tion f, elle est croissante à partir du rang 2. 83 1. a)  un = u0 + nr .
b)  a = n et b = u0
76 1. (un ) figure 1, (v n ) figure 2 et (w n ) figure 3. c) Si a > 0, la fonction f est croissante sur [ 0 ; + ∞[
2. a) 1, 99 < v n < 2 pour n > 300 et la suite (un ) est croissante. Si a < 0, la fonction f
b)  v n < −1010 pour n > 18 est décroissante sur [ 0 ; + ∞[ et la suite (un ) est
c)  v n > 106 . pour n > 995. décroissante.
3. La suite (un ) semble tendre vers +∞. La suite 5 5
2. a)  f( x) = x − 1, > 0 , la fonction f est crois-
(v n ) semble tendre vers 2 2 2
La suite (w n ) semble tendre vers −∞. sante, la suite (un ) est croissante.
b)  f( x) = (1 − 2 )x + 7,1 − 2 < 0 , la fonction f est
décroissante, la suite (un ) est décroissante.
30

172909_Chap02_000-000.indd 30 25/07/2019 18:16:09


un+1 Comme 2n > 0 et n2 (n + 1)2 > 0 , le signe de
84 1.  = q. Si 0 < q < 1, la suite (un ) est
un un+1 − un dépend du signe de n2 - 2n - 1.
décroissante, si q > 1, (un ) est croissante. un+1 − un > 0 pour tout entier supérieur à 1 + 2
2. v n+1 − v n = v 0 q n (q − 1). soit pour n > 3.
a) Si v 0 > 0 , le signe de v n+1 − v n est le même que
celui de q n (q − 1), (v n ) et (q n ) ont même sens de 88 1. 
variation.
b) Si v 0 < 0 , le signe de v n+1 − v n est le signe
contraire de celui de q n (q − 1), (v n ) et (q n ) ont des
sens de variation contraire.
3. a)  2 > 0 et 3 > 1, la suite (un ) est croissante.
b)  −5 < 0 et 0 , 75 < 1, la suite (un ) est croissante.
c)  6 > 0 et 0 , 5 < 1, la suite (un ) est décroissante. (un ) semble croissante.
d)  −3 < 0 et 4 , 5 > 1, la suite (un ) est décroissante. (v n ) semble décroissante.
(un + 3v n ) (un + 2v n )
85 a) 80  % correspond à un coefficient de 0,8, les w n+1 −
4 3 (v n − un )
2. a)  = =
nouveaux adhérents sont au nombre de 20  : on a wn v n − un 12(v n − un )
donc un+1 = 0 , 8un + 20. 1
v 0 , 8un − 80 = .
b)  n+1 = = 0 , 8. 12
1
vn un − 100 (w n ) est la suite géométrique de raison et de
12
La suite (v n ) est la suite géométrique de raison 0,8 et premier terme 1.
de premier terme v 0 = 50.  1 n
v n = 50 × 0 , 8n , 50 > 0 et 0 , 8 < 1, la suite (v n ) est b)  w n =   .
12 
décroissante.  u + 2v n   u + 3v n 
un = 50 × 0 , 8n + 100 , un+1 − un = v n+1 − v n < 0, la 3. t n+1 = 3 n 
 + 8 n 
 3   4 
suite (un ) est décroissante.
= 3un + 8v n = t n .
4 1 La suite (t n ) est constante et pour tout n Î ,
86 1. a)  p1 = =
32 8 t n = 19.
b) Gagner au moins une fois sur n parties est l’événe- 4. a)  un et v n sont solution du système :
ment contraire de perdre n fois.
 3un + 8v n = 19
 7 n 
pn = 1 − ( p1)n = 1 −   .   n .
 8  −un + v n =  1 
 12 
2. a) n ¬ 1 b) On obtient
n n
7 n = 23 19 8  1  19 3 1
p¬ Soit un = −   et v n = +   .
8 11 1112  11 1112 
Tant que 1 - p < 0,95 n
8  1   1
n ← n +1 b)  un+1 − un =   1 −  > 0. La suite (un ) est
 7 n 1112   12 
p ←   croissante. De même v n+1 − v n < 0. La suite (v n ) est
 8 
décroissante.
Fin de tant que
Afficher n
89 a) • p1 = (1 + r )p0 + µ
donc 670 = (1 + r ) × 500 + µ soit 500r + µ = 170.
87 a) La suite (un ) semble croissante à partir du
• p2 = (1 + r )p1 + µ donc 874 = (1 + r ) × 670 + µ 

rang 3.
b) Étudions le signe de soit 670r + µ = 204.
On obtient r = 0 , 2 et µ = 70.
un+1 − un =
2n+1 2n
− 2 =
2 n+1
(
× n2 − 2n (n + 1)2 ) b) Pour tout n Î ,
2 2 2
(n + 1) n n (n + 1)
v n+1 = pn+1 + 350 = 1, 2 pn + 420 ,
2n (n2 − 2n − 1)
= . v n+1 = 1, 2( pn + 350) = 1, 2v n . 
Donc (v n ) est géomé-
n2 (n + 1)2
trique de raison 1,2 avec v 0 = 850.

Chapitre 2  ★  Comportement d’une suite 31

172909_Chap02_000-000.indd 31 25/07/2019 18:18:13


c) Pour tout n Î , v n = 850 × 1, 2n 93 a) 
et pn = 850 × 1, 2n − 350.
d) Pour tout n Î ,
pn+1 − pn = 850 × 1, 2n (1, 2 − 1) = 850 × 1, 2n × 0 , 2 b)
donc pn+1 − pn > 0.
La suite ( pn ) est croissante.

90 a)  v n+1 − v n = (n + 1)un+1 − nun


= nun + 4 − nun = 4.
La suite (v n ) est arithmétique de raison 4 et de pre- u = 2.0009765625, n = 5 u = 2.0000009536743164, n = 10
mier terme v1 = 0. c) La suite (un ) semble tendre vers 2.
v 4
b)  v n = v 0 + (n − 1) × 4 = 4 n − 4. un = n = 4 − .
4 n n 94 a) 
un+1 − un = > 0.
n(n + 1)
La suite (un ) est croissante.

91 a) 

La suite (un ) semble tendre vers −∞.


b)

b) 

on a n = 4 on a n = 7
c) La suite (un ) semble tendre vers −∞ et confirmer
les conjectures.

95 a) L’évolution du salaire est modélisée par la


suite géométrique de premier terme 15 000 et de rai-
c) La suite semble tendre vers 140. Le niveau de la son 1,015.
b) n¬0
mer va atteindre 136 m.
u ¬ 15000
Tant que u < s
92 a)  bn+1 = v n+1 − 8 = 0 , 5v n + 4 − 8
n ← n +1
= 0 , 5(v n − 8) = 0 , 5bn
u ¬ 1, 015u
(bn ) est la suite géométrique de raison 0,5 et de pre-
Fin de tant que
mier terme -5.
Afficher n
b)  bn = −5 × 0 , 5n
c)  c) 

La valeur affichée en sortie pour n est 13. Le salaire


d)  v n = bn + 8, la suite (v n ) semble rendre vers 8. annuel dépassera 18 000 € la treizième année.
32

172909_Chap02_000-000.indd 32 25/07/2019 18:18:57


96 a) L’évolution du salaire est modélisée par la e) Vrai, u100 > 106 et (un ) est croissante. Donc pour
suite arithmétique de premier terme 16 000 et de rai- tout n > 100, on a un > 106.
son 145.
b) n¬0
100 a) Il existe un entier naturel n tel que un > un+1.
u ¬ 16000
Tant que u < s b) Pour tout nombre entier naturel n, un < 100.
n ← n +1 c) Il existe un nombre réel A, tel que pour tout nombre
u ← u + 145 entier naturel n, un < A.
Fin de tant que
Afficher n
c) 

Organiser son raisonnement

La valeur affichée en sortie pour n est 13. Le salaire 101 La suite qui modélise le nombre de joueurs au
annuel dépassera 18 000 € la treizième année. bout de n semaines est définie par u0 = 7500 et
pour tout entier naturel n par un+1 = 0 , 8un + 300.
97 a) 
En utilisant un tableur, on conjecture que (un ) est
décroissante et tend vers 1 500. On considère la suite
(v n ) définie par v n = un − 1500.
v n+1 − v n = 0 , 8(un − 1500) = 0 , 8v n .
La suite (v n ) est définie par v n = 6 000 × (0 , 8)n .
On a donc un = 6 000 × (0 , 8)n + 1500.
1 − 1, 5n+1 Pour tout entier n, un > 1500 Le jeu sera maintenu
b)  Sn = u0 = −2(1 − 1, 5n+1)
1 − 1, 5 sur le site
c) Comme (1, 5n+1) semble tendre vers +∞, il
semble que (Sn ) tende aussi vers +∞. 102 a) La suite est géométrique de raison q, avec
0 < q < 1. Elle est donc décroissante.
98 a) Vrai, la suite (un ) est décroissante et tend
b) On utilise un programme en langage Python pour
vers 0.
déterminer en quelle année les sacs seront interdits :
b) Faux, u0 = 500 > 100
on obtient n = 10.
c) Faux un = 500 × 0 , 5n ≠ 0.
d) Vrai, à partir d’un certain rang, tous les termes de Les sacs seront interdits en 2029.
la suite (un ) seront inférieurs à 5.
e) Vrai, la suite (un ) est décroissante de premier
terme égal à 500.
f) Faux, la suite (un ) est décroissante de premier
terme égal à 500.

99 a) Faux, u47 = 103828 > 500.


b) Vrai, u2155 > 1010
103 a) La suite est géométrique de raison q, avec
c) Vrai, pour tout nombre n de , n3 > 0 donc
0 < q < 1. Elle est donc décroissante.
un > 5 .
d) Faux, un = f (n) avec f définie sur [ 0 ; + ∞[ par b) L’assurance lui rembourse 450 × 0 , 8 4 = 184 , 32  €.
f( x) = x3 + 5. c) On utilise un programme en langage Python pour
f ′( x) = 3 x2 > 0 , donc f est croissante sur [ 0 ; + ∞[. déterminer en quelle année le remboursement sera
Ainsi (un ) est croissante. inférieur à 80 €. On obtient n = 8. Ce sera en 2023.

Chapitre 2  ★  Comportement d’une suite 33

172909_Chap02_000-000.indd 33 25/07/2019 18:19:58


• La pression atmosphérique est égale à 700 mbar,
approximativement à 3 600 m.
• La pression atmosphérique est égale à 500 mbar,
approximativement à 6 900 m.
b) On conjecture que la limite de ( pn ) est égale à 0.

106 a)  S0 = 1, S1 = 11
, , S2 = 111
, , S3 = 1111
, et
104 1. a)  (un ) : est définie par u0 = 200 000 et S4 = 11111
, .
un+1 = 0 , 987un . b)  Sn est la somme des n + 1 premiers termes de la
(v n ) : est définie par v 0 = 150 000 et v n+1 = 1, 015v n . suite géométrique de premier terme 1 et de raison
b)  1
.
10
 1 n+1
1 −  
10   1  
n+1
10 
Sn = = 1 −   .
1−
1 9  10  
10

 1 n+1
  tend vers 0 quand n tend vers l’infini, la limite
10 
10
de la suite (Sn ) est .
9

3 3
107 a)  T0 = , T1 = .
4 16
b) Notons pn la partie de Tn non coloriée.

3
On a pn+1 = pn .
4
3
c) La population de la ville A semble décroitre, celle La suite ( pn ) est géométrique de raison et de pre-
de la ville B semble croitre. n 4
3 3  3 
mier terme donc pn = ×  .
2. La population de la ville B, dépassera celle de la 4 4  4 
ville A en 2015 + 11, soit en 2026.
3   3  
n
La partie colorée du triangle Tn est × 1 −    .
105 1. pn+1 = 0 , 99 pn avec p0 = 1000. 4   4  
 
2. a) n ¬ 0  3 n
Le plus petit entier n tel que 1 −   > 0 , 99 est 17.
p ¬ 1000  4 
Tant que p > A
c) On conjecture que la limite de la suite des aires des
n ← n +1
3
p ¬ 0 , 99 p triangles Tn est .
4
Fin de tant que
Afficher n´100
108 a) Une perte de 20 % du chiffre d’affaire se tra-
b)  duit par un coefficient égal à 0,8, les clients qui
s’abonnent entrainent une augmentation de 18 :
un+1 = 0 , 8un + 18 et u0 = 65.
b) La recette ne dépassera pas 4 420 € en 2018
(u5 = 4 254 , 016).
3. a) 
• 
La pression atmosphérique est égale à c) La recette mensuelle de la société tend vers 4 680 €
800 mbar, approximativement à 2 300 m. du fait de la stabilisation du nombre de clients à 90.
34

172909_Chap02_000-000.indd 34 25/07/2019 18:20:49


109 On modélise l’évolution des populations avec u1 3 u1 3 u
u4 − u3 = + − − = − 1 > 0 si u1 < 0
un tableur. 6 2 2 2 3
 u1 3 
 + 
u4  6 2  u 11
u5 = +1= +1= 1 + ,
4 4 24 8
u1 11 u1 3 u1 1
u5 − u4 = + − − = − − > 0 si u1 < −1
24 8 6 2 8 8
Comme la suite cn est géométrique de raison 0,98, sa  u1 11
 + 
limite est zéro. u5  24 8  u 51
Les deux autres suites tendent vers 55 000. u6 = +1= +1= 1 + ,
5 5 120 40
u 51 u1 11 u 1
1 u6 − u5 = 1 + − − =− 1 − >0
110 a) Pour tout entier naturel n, an+1 = an . 120 40 24 8 30 8
4
C’est donc la suite géométrique de premier terme 15
si u1 < −
1 4 u1 51
a1 = 9 et de raison . +
4 u6 u 97
Sn est la somme des n premiers termes de cette suite. u7 = + 1 = 120 40 + 1 = 1 + , u7 − u6
6 6 720 80
 1 n 1 1
1 −     1 n  =− − > 0 si u1 < −9
 4   144u1 16
Sn = 9 × = 121 −   .
1   4   On conjecture que la suite (un ) décroit à partir du
1−
4 rang 2, si u1 < 1 .
b) Pour tout entier n > 6, 11, 999 < Sn < 2.
Il semble que plus u1 est petit plus la plage de crois-
c) La limite de la suite (Sn ) semble être 12.
sance de la suite (un ) est grande.
111 a) En utilisant une feuille de calcul, on peut
112 a) 
observer les variations de la suite (un ) pour quelques
valeurs de u1 .
Les valeurs maximales de un sont surlignées.

En utilisant un tableur, on peut modéliser la suite des


chiffres d’affaires par :
cn = 0 , 4 n2 + 4 ,1n + 32, 8.
b) La suite (cn ) semble tendre vers +∞. Il est peu
probable qu’un chiffre d’affaire devienne infiniment
grand.

113 Pour tout n de * :


1
v n − un = n + 1 − n = .
u2 = u1 + 1, u2 − u1 = 1 , la suite (un ) est croissante n +1+ n
du rang 1 au rang 2 quelle que soit la valeur de u1 On conjecture que la suite : (v n − un ) tends vers 0.
u u +1 u 3 Donc v n - un peut devenir aussi proche de 0 que l’on
u3 = 2 + 1 = 1 +1= 1 + ,
2 2 2 2 veut.
u 3 u 1
u3 − u2 = 1 + − u1 − 1 = − 1 + > 0 si u1 < 1,
2 2 2 2
114 La suite des aires non colorée est définie sur *
la suite (un ) est croissante du rang 2 au rang 3 si 25π
u1 < 1 par un = .
2n
Et ainsi de suite On conjecture que cette suite tend vers 0. La suite des
 u1 3  aires du domaine bleu est définie sur * par
 + 
u3  2 2  u 3 25π 25π 25p
u4 = +1= +1= 1 + , an = − tend vers .
3 3 6 2 2 2n 2

Chapitre 2  ★  Comportement d’une suite 35

172909_Chap02_000-000.indd 35 25/07/2019 18:21:42


115 1. u1 = 165, u2 = 176 117 Le nombre d’arbres est modélisé par la suite
 p0 = 48900
2. a) On choisit l’algorithme 2, on atteint 220 par ( pn ) : 
valeurs inférieures.  p n+1 = 0 , 95 pn + 3000

b) L’algorithme affiche 13. La population devrait se stabiliser aux environs de
3. On tabule la suite (un ) dans la calculatrice. Elle ne 60 000  arbres.
semble pas atteindre 250. Les organisateurs n’auront
pas à refuser des inscriptions. 118 La quantité de médicament dans le sang est
q0 = 10
modélisé par la suite (qn ) : 
qn+1 = 0,8qn + 1
La quantité de médicament dans le sang se stabilise
au bout de 18 heures à environ 5 mL;

Exploiter ses compétences 119

116 La tornade est classée F3 sur l’échelle de Fujita


et a causé des dégâts considérables.
En tabulant la suite géométrique de premier terme
318 et de raison 0,98 on trouve que sa durée a été de L’effectif se stabilisera à partir de 2029 autour de
30 minutes. 1 600  élèves.

36

172909_Chap02_000-000.indd 36 25/07/2019 18:21:49


3 Second degré

−0 , 3( x − 4)2 + 4 , 8 = 2, 82
Découvrir −0 , 3( x − 4)2 = −4 , 8 + 2, 82
−0 , 3( x − 4)2 = −1, 98
−1, 98
( x − 4)2 =
−0 , 3
( x − 4)2 = 6 , 6
1 Équation du second degré
et forme canonique x − 4 = 6 , 6 ou x − 4 = − 6 , 6
x = 4 + 6 , 6 ou x = 4 − 6 , 6
1  f( x) = x(−0 , 3 x + 2, 4)
On résout f( x) = 0 {
6 = 4 + 6, 6 ; 4 − 6, 6 . }
x = 0 ou −0 , 3 x + 2, 4 = 0 b) À 0,9m au-dessus de l’eau, la largeur du pont

x = 0 ou x =
−2 , 4 ( )
est égale à 4 + 13 − 4 − 13 = 2 13 m,
−0 , 3 soit environ 7, 21 m > 3, 90 m.
x = 0 ou ou x = 8 À 2,92 m au-dessus du niveau de l’eau, la largeur
La largeur du pont est égale à 8 m. de l’arche est égale à :

2  a) On développe l’expression donnée :


( )
4 + 6 , 6 − 4 − 6 , 6 = 2 6 , 6 m, soit environ
5,14 m > 3 m.
Pour tout x ∈ [0 ; 8 ] : Le bateau pourra donc passer sous l’arche de ce
−0 , 3( x − 4)2 + 4 , 8 pont.
= −0 , 3( x2 − 8 x + 16) + 4 , 8
= −0 , 3 x 2 + 2 , 4 x − 4 , 8 + 4 , 8
= −0 , 3 x 2 + 2 , 4 x 2 Inéquation du second degré
= f( x)
b) Pour tout x ∈ [0 ; 8 ], −0 , 3( x − 4)2 < 0 donc : 1  La longueur de la ligne est 60 m :
−0 , 3( x − 4)2 + 4 , 8 < 4 , 8 L + 2 = 60
La hauteur maximale de l’arche est 4,8 m. La zone de baignade doit avoir une superficie
supérieure à 400 m² : L > 400.
3  a) La forme qui permet de résoudre les équa-
tions est la forme canonique : 2  D’après la 1re équation, on obtient : L = 60 − 2.
−0 , 3( x − 4)2 + 4 , 8 = 0 , 9 On remplace alors L dans l’inéquation :
(60 − 2) > 400
−0 , 3( x − 4)2 = 0 , 9 − 4 , 8
60  - 2 2 - 400 > 0
−0 , 3( x − 4)2 = −3, 9
−3, 9 3  a) On utilise la forme factorisée :
( x − 4)2 =
−0 , 3 −2( − 20)( − 10) > 0
( x − 4)2 = 13  − 20 = 0 ou  − 10 = 0
x − 4 = 13 ou x − 4 = − 13  = 20 ou  = 10
x = 4 + 13 ou x = 4 − 13 On dresse le tableau de signe correspondant à
{
6 = 4 + 13 ; 4 − 13 . } l’inéquation :

Chapitre 3  ★  Second degré 37

172909_Chap03_000-000.indd 37 25/07/2019 18:24:35


< 0 10 20 60 ∆ < 0 donc l’équation n’a pas de solution.
< - 20 - - 0 + L’ensemble des solutions est 6 = ∅
< - 10 - 0 + + c) Ici  a = 0 , 25, b = −0 , 4 et c = 0 ,16
∆ = (−0 , 4)2 − 4 × 0 , 25 × 0 ,16 = 0
-2 - - -
∆ = 0 donc l’équation a une unique solution :
-2(< - 20)(< - 10) - 0 + 0 - 0, 4
6 = [20 ; 60 ]. x0 = = 0, 8
2 × 0 , 25
b) La zone de baignade de Marie Pierre peut L’ensemble des solutions est 6 = {0 , 8}.
avoir pour dimensions :
10 <  < 20 et 20 < L < 40. 8 a) Ici  a = −1, b = 5 et c = 14
∆ = 52 − 4 × (−1) × (14) = 81
∆ > 0 donc l’équation a deux solutions distinctes :
−5 − 81 −5 + 81
Acquérir des automatismes
x1 = = 7 et x2 = = −2.
2 × (−1) 2 × (−1)
L’ensemble des solutions est 6 = {−2 ; 7}.
b) Ici  a = 9, b = 6 et c = 1
∆ = 62 − 4 × 9 × 1 = 0
∆ = 0 donc l’équation a une unique solution :
3 a) −x2+120 = 0 équivaut à −6 1
x0 =
( 120 − x )( )
120 + x = 0 2×9
=− .
3
 1
c’est-à-dire x = 120 = 2 30 L’ensemble des solutions est 6 = − .
 3 
ou x = − 120 = −2 30 c) Ici  a = 0 , 4 , b = 2 et c = 5
{
L’ensemble des solutions est 6 = −2 30 ; 2 30 . } ∆ = 22 − 4 × 0 , 4 × 5 = −4
1 2 1 ∆ < 0 donc l’équation n’a pas de solution.
b)  x + 3 x + 9 = 0 équivaut à ( x2 + 12 x + 36) L’ensemble des solutions est 6 = ∅
4 4
1
= 0 qui est équivalente à ( x + 6)2 = 0 , 9 1 est une solution évidente de l’équation.
4
c’est-à-dire : x + 6 = 0 soit x = −6. −6
L’autre solution vérifie x ′ × 1 = .
L’ensemble des solutions est 6 = {−6}. 2
Donc x ′ = −3
c)  3 x2 + 15 x = 0 équivaut à 3 x( x + 5) = 0 , L’ensemble des solutions est 6 = {−3 ; 1}
c’est-à-dire x = 0 ou x + 5 = 0
L’ensemble des solutions est 6 = {0 ; − 5} 10 2 est une solution évidente de l’équation.
6
d)  4 x2 − 4 x + 1 = 0 équivaut à (2 x − 1)2 = 0 , L’autre solution vérifie x ′ × 2 = − .
3 2
c’est-à-dire 2 x − 1 = 0. Donc x ′ = − .
 1 2
L’ensemble des solutions est 6 =  .  3 
 2  L’ensemble des solutions est 6 = − ; 2.
 2 
4 a)  g(t ) = −3(t 2 + 2t − 6) 11 -1 est une solution évidente de l’équation.
En utilisant la méthode de la complétion du carré : 1 1
L’autre solution vérifie x ′ × (−1) = − . Donc x ′ = .
g(t ) = −3[(t + 1)2 − 12 − 6 ] 2 2
 1 
g(t ) = −3(t + 1)2 + 21 L’ensemble des solutions est 6 =  ; − 1.
b) Pour tout t Î , −3(t + 1)2 < 0 donc g (t ) < 21  2 
2
g admet 21 pour maximum atteint pour t = −1. 14 a)  ∆ = 7 − 4 × (−3) × 26 = 361
∆ > 0 donc la fonction polynôme f a deux racines :
7 a) Ici  a = −2, b = 11 et c = −12 −7 − 361 13 −7 + 361
x1 = = et x2 = = − 2.
∆ = 112 − 4 × (−2) × (−12) = 25 2 × (−3) 3 2 × (−3)
∆ > 0 donc l’équation a deux solutions distinctes : Donc le tableau de signes de f ( x ) :
−11 − 25 −11 + 25 3
x1 = = 4 et x2 = = x −∞ -2
13
+∞
2 × (−2) 2 × (−2) 2 3
 3  f ( x) 0 0
L’ensemble des solutions est 6 =  ; 4 - + -
 2  2
b) Ici  a = 1, b = 1 et c = 1 b)  ∆ = (−2) − 4 × 1× 9 = −32
∆ = 12 − 4 × 1× 1 = −3 ∆ < 0 donc la fonction polynôme g n’a pas de racine.
38

172909_Chap03_000-000.indd 38 25/07/2019 18:26:05


D’où le tableau de signes de g( x) :  2  3  3 2
c)   x +  x − 1 = x2 − x + x −
 
3  2  2 3
x −∞ +∞
g( x) + 3 2 2
= x −
2 3
c)  ∆ = 24 2 − 4 × (−9) × (−16) = 0
∆ = 0 donc la fonction polynôme h a une seule  2  2  2 2
d)   x +  x −  = x2 −  
racine :  5  5  5 
−24 4
x0 = = 4
2 × (−9) 3 = x2 −
25
Donc le tableau de signes de h( x) :
Les expressions a), c) et d) correspondent à des
4
x −∞ +∞ expressions du second degré.
3
h( x) - 0 -
23 f2 ( x) = 3 x2 + 7 + 9 x2 − 24 x + 16
 13   4  f2 ( x) = 12 x2 − 24 x + 23
15 a)  6 =  − 2 ;  b)  6 = ∅ c)  6 =   f3 ( x) = 1 − 10 x + 25 x2 − 25 x2
 3   3 
f3 ( x) = −10 x + 1
16 f( x) = (2 x)2 + 2 × 2 x × 1 + 12 − 4 x2 + 6
Maya a tort  : la fonction f3 n’est pas une fonction
f( x) = 4 x + 7
polynôme de degré 2.
Louisa a tort : ce n’est pas une fonction polynôme de
degré 2.
23
3 Forme factorisée Forme développée
17 a)  a = , b = −1, c = 4
2
(2 x + 3)( x + 1) 2 x2 + 7 x + 3
b)  a = −4, b = 0, c = 5
c)  a = −1, b = 3, c = 0 ( x + 3)(2 x + 1) −2 x 2 + 5 x − 3
d)  a = 1, b = 2, c = −3 (2 x − 1)(3 − x) 2 x2 + 5 x + 3
(3 − 2 x)( x − 1) −2 x 2 + 7 x − 3
18 f( x) = −2 x − 2 x × 3 x + 4
f( x) = −6 x2 − 2 x + 4 25 a)  f( x) = 2 x2 − 10 x − 28
Laura a raison  : f est une fonction polynôme de b) 1re méthode :
degré 2. c −28
Le produit est = = −14
a 2
19 On développe f : b −10
La somme est − = − =5
f( x) = (2 x)2 − 2 × 2 x + 1 − x2 e a 2
2 méthode :
f( x) = 3 x2 − 4 x + 1
Les racines de f sont les solutions de :
On factorise f :
( x − 7)(2 x + 4) = 0
f( x) = (2 x − 1 − x)(2 x − 1 + x)
f( x) = ( x − 1)(3 x − 1) c’est-à-dire : x − 7 = 0 ou 2 x + 4 = 0
Ce sont les formes 1 et 3. c’est-à-dire : x = 7 ou x = −2
La somme des racines est : 7 + (−2) = 5
20 Tous les polynômes de la forme : Le produit des racines est : 7 × (−2) = −14
a( x − 4)( x + 3) avec a ¹ 0. Wesley a raison.

5 9 5 9 26 a)  f( x) = ( x − 5)( x − 4)
21 −(−1)2 + × (−1) + = −1 − + = 0.
4 4 4 4 b)  ( x − 5)( x − 4) = 0
-1 est une racine.
x − 5 = 0 ou x − 4 = 0
2 x = 5 ou x = 4
1 1 1 1  2 1 1
22 a)   x +  − =  x + x +  −
2  2 2 2 4 2 6 = {4 ; 5}.
1 2 1 3
= x + x−
2 2 8 27 a)  f( x) = x[2( x + 1) − ( x − 4)]
b)  x2 − ( x2 + 2 x + 1) = −2 x − 1 f( x) = x( x + 6)

Chapitre 3  ★  Second degré 39

172909_Chap03_000-000.indd 39 25/07/2019 18:27:24


b)  f( x) = x2 + 6 x 37 a)  f( x) = 2 x2 − 2 x + 3
c) On utilise la forme factorisée : f( x) = 2( x2 − x) + 3
x = 0 ou x + 6 = 0  1  1 
2 2
f( x) = 2  x −  −    + 3
x = 0 ou x = −6  2   2  
 
6 = {0 ; − 6}. 2
 1  1

f( x) = 2 x −  − 2 × + 3
 2 4
28    2 5
1
x −∞ -5 7 +∞ f( x) = 2 x −  +
 2 2
x+5 - 0 + + 2
x -7 - - 0 + b)  g( x) = 3 x + 6 x + 12
f( x) + 0 - 0 + g( x) = 3( x2 + 2 x) + 12
g( x) = 3 ( x + 1)2 − 12  + 12
 
29 2
g( x) = 3( x + 1) + 9
1 1
x −∞ - +∞ c)  h(t ) = −5t 2 − 20t + 20
4 2
h(t ) = −5(t 2 + 4t ) + 20
1
x- - - 0 + h(t ) = −5[(t + 2)2 − 22 ] + 20
2
1 h(t ) = −5(t + 2)2 + 40
x+ - 0 + +
4
-3 - - - 38
f( x) - 0 + 0 - Fonction Forme canonique

1 −2 x 2 − 4 x + 3 −2( x + 1)2
30 > 0 donc g est du signe de (4t − 1)(3t − 2)
2 -2 x 2 - 8 x - 5 −2( x + 2)2 + 3
1 2
t −∞ +∞ -2 x 2 - 4 x - 2 −2( x + 1)2 + 5
4 3
4t - 1 - 0 + + 39 a)  ∆ = (−5)2 − 4 × 2 × (−3) = 49
3t - 2 - - 0 + 49 > 0 , donc l’équation a deux solutions :
g(t ) + 0 - 0 + 5 − 49 1 5 + 49
x1 = = − ou x2 = =3
2×2 2 2×2
31 (2) en développant le membre de droite on  1 
6 = − ; 3.
démontre l’égalité.  2 
1
32 a) forme (3) b)  ∆ = (−2)2 − 4 × 3 × = 0.
3
L’équation a une solution :
33 a) formes (4) et (2) 2 1
x0 = =
2×3 3
34 a)  ∆ = 22 − 4 × 2 × (−1) = 12  1
12 > 0 : donc l’équation a deux solutions. 6 =  .
 3 
b)  ∆ = 32 − 4 × (−1) × (−4) = −7. c) L’équation peut s’écrire :
−7 < 0 donc l’équation n’a pas de solution. x2 + 2 x − 35 = 0
4
c)  ∆ = (−4)2 − 4 × 3 × = 0 donc l’équation a une ∆ = (2)2 − 4 × 1× (−35) = 144.
3 144 > 0 , donc l’équation a deux solutions :
solution.
1 −2 − 144 −2 + 144
35 ∆ = 4 2 − 4 × × 8 = 0 donc l’équation a une x1 = = −7 ou x2 = =5
2 2 ×1 2 ×1
unique solution. 6 = {−7 ; 5}.
Octave a tort. d)  ∆ = 12 − 4 × 1× 9 = −35. ∆ < 0, donc l’équation
 5 n’a pas de solution. 6 = ∅ .
36 f( x) = 4  x2 + 2 x − 
 4
 2 2 5 40 a)  ∆ = 12 − 4 × (−3) × 4 = 49
f( x) = 4 ( x + 1) − 1 −  49 > 0 : l’équation a deux solutions :
 4 
 9  −1 − 49 4 −1 + 49
f( x) = 4 ( x + 1)2 −  x1 = = ou x2 = = −1
 4  2 × (−3) 3 2 × (−3)
40

172909_Chap03_000-000.indd 40 25/07/2019 18:28:54


1 + 13 1 − 13  c)  3t 2 − 12t + 40 = 55 cela revient à
6 =  ; .
 6
 6  3t 2 − 12t − 15 = 0
b)  ∆ = 4 2 − 4 × (−4) × 15 = 256 ∆ = (−12)2 − 4 × 3 × (−15) = 324
256 > 0 , donc l’équation a deux solutions : 324 > 0 , donc l’équation a deux solutions :
−4 − 256 5 −4 + 256 3
x1 = = ou x2 = =− 12 − 324
2 × (−4) 2 2 × (−4) 2 t1 = < 0 (impossible t représente un
2×3
 5 3  12 + 324
6 =  ; − . temps) ou t2 = = 5.
 2 2  2×3
c)  ∆ = 52 − 4 × 7 × 1 = −3 Le chauffage aura fonctionné entre 2  h et 5  h soit
−3 < 0, donc l’équation n’a pas de solution. pendant 3 h.
6 = ∅.
4 4
d)  ∆ = 2, 52 − 4 × 0 , 5 × (−7) = 20 , 25 45 a)  − (t 2 − 30t + 225) + × 324
3 3
20 , 25 > 0 , donc l’équation a deux solutions : 4 2
= − t + 40t − 300 + 432
−2, 5 − 20 , 25 3
x1 = = −7
2 × 0, 5 4 2
= − t + 40t + 132
−2, 5 + 20 , 25 3
ou x2 = =2 = N(t )
2 × 0, 5 4
6 = {−7 ; 2}. Pour tout t > 0, (t −15)2 > 0 donc − (t − 15)2 < 0
3
et on en déduit que :
41 Nassim n’a pas simplifié ses solutions. 4
N(t ) < × 324
3
2
42 (2 + 3 ) = 7 + 4 3 Le nombre maximal de bactéries est 432.
4
2
∆ = (2 − 3 ) − 4 × (−2 3 ) = 7 + 4 3 b)  − t 2 + 40t + 132 = 0
3
 4
2− 3 +2+ 3 ∆ = 402 − 4 × −  × 132 = 2 304
x1 = =2  3 
2 −40 − 2 304
x2 =
2− 3 − 2+ 3 ( ) t1 =
 4
= 33
=− 3 2 × − 
2  3 
−40 + 2 304
43 a)  3 x − 0 , 3 x2 = 0 équivaut à x(3 − 0 , 3 x) = 0. t2 = = −8 < 0 : impossible.
 4
Ce qui donne : x = 0 ou 3 − 0 , 3 x = 0 , c’est-à-dire 2 × − 
 3 
x = 0 ou x = 10
Il faudra 33 min pour que les bactéries aient toutes
La largeur de l’arche au sol est 10 m.
disparues.
b)  f( x) = −0 , 3( x2 − 10 x)
f( x) = −0 , 3( x − 5)2 + 0 , 3 × 52 46 Expressions (1) et (3)
f( x) = −0 , 3( x − 5)2 + 7, 5
b 4
On en déduit pour tout x ∈ [0 ; 10 ] , f( x) < 7, 5 . 47 a) Somme : − = −
La hauteur maximale de l’arche est 7,5 m. c a 5
Produit : = −2
a
44 a)  θ(0) = 3 × 02 − 12 × 0 + 40 = 40 b 2 c
b) Somme : − = Produit : = −5
La température au début de l’observation est égale à a 7 a
40 °C. b 1 c 1
c) Somme : − = Produit : =
b)  θ(t ) = 3(t 2 − 4t ) + 40 a 3 a 3
θ(t ) = 3[(t − 2)2 − 22 ] + 40 b c
d) Somme : − = 20 Produit : = 14
θ(t ) = 3(t − 2)2 + 28 a a
Pour tout t, (t − 2)2 > 0 et donc 3(t − 2)2 > 0
On en déduit que pour tout t : 48 On calcule le discriminant :
3(t − 2)2 + 28 > 28 ∆ = 12 − 4 × 3 × 2 = −23
La température minimale est 28 °C. La fonction n’a donc pas de racine et n’admet pas de
Le système de chauffage se déclenche lorsque forme factorisée.
t − 2 = 0 , c’est-à-dire t = 2 , après 2 h. Marius a raison.

Chapitre 3  ★  Second degré 41

172909_Chap03_000-000.indd 41 25/07/2019 18:30:10


49 a)  x = 1 est une solution évidente. 52 a)  3 × (−1)2 − 1 − 2 = 0
5
Le produit des deux racines est donné par Donc -1 est une racine de g.
-1 c 2
x ′ × 1 = −5 donc x ′ = −5 b) Le produit est donné par : = −
a 3
6 = {1 ; − 5} 2 2
c)  x ′ × (−1) = − donc x ′ =
b)  f( x) = −( x − 1)( x + 5) 3 3

50 a)  ∆ = (−1)2 − 4 × 1× 1 = −3 53 a)  f(5) = 3 × 52 − 13 × 5 − 10


f ne se factorise pas. = 3 × 25 − 65 − 10
b)  ∆ = (−4)2 − 4 × (−1) × (−4) = 0 =0
b 13
−4 b) La somme est donnée par − =
La racine double de g est − = −2 a 3
2 × (−1) c) On en déduit que la seconde racine est :
2
g( x) = −( x + 2) 13 2
x′ = −5 = −
 1 2 1 9 3 3
c)  ∆ =   − 4 × × (−2) =  2
 2  4 4 f( x) = 3( x − 5) x + 
 3
h a deux racines : 2
 5
1 9 1 9 54 −6 x + 
− − − +  6
x1 = 2 4 = −4 ou x2 = 2 4 = 2.  2 5 25 
1 1 = −6 x + x + 
2× 2×  3 36 
4 4
1 25
( x) = ( x + 4)( x − 2) = −6 x2 − 10 x −
4 6
d)  ∆ = 72 − 4 × (−1) × (−6) = 25 = h( x)
La forme canonique de Marie est correcte.
k a deux racines :
Recherche des racines de h :
−7 − 25 −7 + 25 5
x1 = = 6 ou x2 = =1 h( x) = 0 équivaut à x + = 0 d’après la forme
2 × (−1) 2 × (−1) 6 5
k( x) = −( x − 1)( x − 6) canonique. h a donc une unique racine x = − et sa
6
forme factorisée est donc bien égale à sa forme cano-
51 a)  ∆ = 72 − 4 × (−1) × (−18) = −23 nique. La réponse de Fannie est donc correcte aussi.
f ne se factorise pas
55 Le produit de deux nombres inverses est 1, or
b)  ∆ = (−20)2 − 4 × 25 × 4 = 0
les produit des deux racines de l’équation (1) et de la
g a une unique racine : 1
−20 2 (3) sont respectivement et -1, on peut donc éli-
x0 = − = . 9
2 × 25 5 miner ces équations de nos recherches.
2
 2 Pour l’équation (2) :
g ( x ) = 25 x − 
 5 ∆ = (−3)2 − 4 × 7 × 7 = −187
c)  ∆ = 32 − 4 × (−4) × 1 = 25 L’équation n’a pas de solution.
La solution est l’équation (4).
h a deux racines :
−3 − 25 56 L’équation admet deux solutions inverses l’une
x1 = = 1 ou
2 × (−4) c
de l’autre puisque leur produit donné par est 1. De
−3 + 25 −1 b 100 a
x2 = = plus, leur somme est égale à − = > 0. Ainsi les
2 × (−4) 4 a 7
deux racines sont de même signe puisque leur pro-
 1  duit est positif et leur somme étant positive, ils sont
h( x) = −4 ( x − 1) x + 
 4  donc positifs.
d)  ∆ = 32 − 4 × 2 × (−2) = 25 Armelle a raison.
k a deux racines :
57
−3 − 25 −3 + 25
x1 = = −4 ou x2 = =1 x −∞ -1 3 +∞
2 2
k( x) = (2 x + 4)( x − 1) signe + 0 - 0 +

42

172909_Chap03_000-000.indd 42 25/07/2019 18:31:03


58 a) Somme de deux nombres positifs (carré et 1) L’expression a deux racines :
donc positif sur  . −13 − 49 −13 + 49 3
a1 = = 5 et a2 = =
b) Somme de deux nombres négatifs (opposé d’un −4 −4 2
carré et -1 ) donc négatif sur  . 3 5
a −∞ +∞
2
59 Les inéquations sont la (2) et la (3). signe - 0 + 0 -
3 
60 a)  6 =  ; 5 .
 2 
x −∞ -2 7 +∞
b) L’inéquation peut s’écrire :
signe - 0 + 0 -
x2 + 2,1x − 3, 52 < 0
b) 
On calcule le discriminant :
x −∞ -8 1 +∞
∆ = 2,12 − 4 × 1× (−3, 52) = 18 , 49 > 0
signe - 0 + 0 -
L’expression a deux racines :
c)  −2,1 − 18 , 49
x −∞
x1 = = −3, 2 et
2 11 +∞ 2
signe + 0 - 0 + −2,1 + 18 , 49
x2 = ,
= 11
d)  2
3 1 x −∞ -3, 2 11
, +∞
x −∞ - - +∞
4 2 signe 0 - 0
+ +
signe + 0 - 0 +
6 = [−3, 2 ; 11
, ].
61 a)  c) L’inéquation peut s’écrire :
x −∞ -1 3 +∞ 10 x2 + 2 x + 0 ,1 > 0
signe + 0 - 0 + On calcule le discriminant :
6 =]− ∞ ; − 1[∪]3 ; + ∞[. ∆ = 22 − 4 × 10 × 0 ,1 = 0
b) On calcule le discriminant : L’expression a une racine :
∆ = (−7)2 − 4 × 1× 12 = 1 > 0 −2
x1 = = −0 , 1
L’expression a deux racines : 20
7− 1 7+ 1 x −∞ -0 ,1 +∞
t1 = = 3 et t2 = =4 signe + 0 +
2 2
−∞
6 =]− ∞ ; − 0 ,1[∪]− 0 ,1 ; + ∞[.
t 3 4 +∞
d) L’inéquation peut s’écrire :
signe + 0 - 0 + 1 13
6 =]− ∞ ; 3[∪]4 ; + ∞[. t2 − t + <0
2 16
c) On calcule le discriminant : On calcule le discriminant :
∆ = 72 − 4 × (−1) × (−6) = 25 > 0  1 2 13
L’expression a deux racines : ∆ = −  − 4 × 1× = −3 < 0
 2  16
−7 − 25 −7 + 25
x1 = = 6 et x2 = =1 t −∞ +∞
−2 −2
signe +
x −∞ 1 6 +∞
6 = ∅.
signe - 0 + 0 -
6 = [1 ; 6 ]. 63 a) On calcule le discriminant :
d) On calcule le discriminant : ∆ = (−3)2 − 4 × 2 × (−2) = 25 > 0
∆ = (−5)2 − 4 × 3 × 3 = −11 < 0 L’expression a deux racines :
u −∞ +∞ 3 − 25 1 3 + 25
signe +
x1 = = − et x2 = =2
4 2 4
6 = ∅. 1
x −∞ - 2 +∞
2
62 a) L’inéquation peut s’écrire :
signe + 0 - 0 +
−2a2 + 13a − 15 > 0
On calcule le discriminant :  1
6 =  − ∞ ; −  ∪ [2 ; + ∞[.
∆ = (13)2 − 4 × (−2) × (−15) = 49 > 0  2 

Chapitre 3  ★  Second degré 43

172909_Chap03_000-000.indd 43 25/07/2019 18:31:52


b) L’inéquation peut s’écrire : x(5 x − 6) < 0 1 5
x −∞ - +∞
6 2 3
x −∞ 0 +∞
5 signe + 0 - 0 +
signe + 0 - 0 +  1 5
6 = − ; 
 6  2 3 
6 =  0 ; .
 5 
c) On calcule le discriminant : 68 a) Forme factorisée 
∆ = 302 − 4 × (−3) × (−75) = 0 x −∞ -5 4 +∞
L’expression a une racine : signe + 0 - 0 +
−30
x1 = =5 6 = [−5 ; 4 ]
−6 b) Forme développée
x −∞ 5 +∞ f( x) > −60 est équivalente à 3 x2 + 3 x > 0 soit
signe - 0 - 3 x( x + 1) > 0
6=∅
x −∞ -1 0 +∞
d) On calcule le discriminant :
∆ = 62 − 4 × (−1) × (−9) = 0 signe + 0 - 0 +
L’expression a une racine : 6 =]− ∞ ; − 1] ∪ [0 ; + ∞[
−6 c) Forme canonique
x1 = =3 2
−2  1
Pour tout réel x,  x +  > 0 on en déduit donc
x −∞ 3 +∞  2
que pour tout réel x :
signe - 0 - 2
 1 243 243
6= 3 x +  − >−
 2  4 4
64 On calcule le discriminant : 6=
∆ = (−12)2 − 4 × 2 × 18 = 0
69 a) On remarque que x = 1 est une racine évi-
L’expression a une racine :
dente de B. La seconde racine vérifie :
12
x= =3 −10
4 x′ ×1 = = 5 donc x ′ = 5
−2
x −∞ 3 +∞ On obtient alors le tableau de signes :
signe + 0 + x 0 1 5 10
6 = {3} donc Hector a raison. signe - 0 + 0 -
b)  6 =]1 ; 5[
65 On calcule le discriminant :
c) L’entreprise réalise un bénéfice lorsqu’elle produit
∆ = (−3)2 − 4 × 2 × 4 = −32 < 0
entre 11 et 49 objets.
L’expression est partout du signe de a = 2 .
Donc pour tout réel x, 2 x2 − 3 x + 4 > 0 70 a)  3, 2I2 + 5I − 20 000 > 0
Tous les nombres sont solutions, ce qui correspond à 3, 2I2 + 5I − 20 000 = 0
l’affichage du logiciel x = x. ∆ = 256 025
−5 − 35 209
66 Une inéquation qui n’a qu’une seule solution est I1 = <0
6, 4
une inéquation telle que ∆ = 0 et le signe de a est
−5 + 35 209
opposé à celui recherché. I2 = ≈ 8, 3
6, 4
Par exemple : −x2 + 2 x − 1 > 0
I 0 I2 +∞
67 On calcule le discriminant : 2
3, 2I + 5I − 20 000 - +
∆ = (−3, 5)2 − 4 × 3 × (−2, 5) = 42, 25 > 0 6 = [ I2 ; + ∞[ : la plus petite valeur entière est
L’équation a donc deux solutions : 79 cm.
3, 5 − 42, 25 1 b)  3, 2I2 + 5I − 30 000 = 0
x1 = = − ou
6 2 ∆ = 384 025
3, 5 + 42, 25 5 −5 − ∆
x2 = = I3 = <0
6 3 6, 4
44

172909_Chap03_000-000.indd 44 25/07/2019 18:32:42


−5 + ∆ 77 a)  ∆ = 22 − 4 × 1× (−15) = 64
I4 = ≈ 96 , 0
6, 4 ∆ > 0 l’équation a deux solutions :
x 0 I4 +∞ −2 − 64 −2 + 64
x1 = = −5 ou x2 = =3
3, 2I2 + 5I − 30 000 - 2 ×1 2 ×1
6 = {−5 ; 3}.
6 = [0 ; I 4 ] : la plus petite valeur entière est 96 cm.
b) 
c) Les étirements à exercer sont entre 79cm et 96 cm.
x −∞ -5 3 +∞
71  1. A 2. B 3. C 4. D 5. B signe + 0 - 0 +

72 1. B et C 78 a)  ∆ = (−40)2 − 4 × 25 × 16 = 0


2. A et B l’équation a une unique solution :
40 4
3. A, B et C x1 = =
2 × 25 5
1  4 
73 1.  ∆ = 22 − 4 × 2 × = 0 6 =  .
 1 2  5 
f −  = 0 donc l’affirmation est fausse. b) 
 2 
4
2. ∆ = b2 − 4 × (−4) x −∞ +∞
5
∆ = b2 + 16 signe 0
+ +
Un carré est toujours positif donc ∆ > 0 : l’affirma-
 4 4 
tion est vraie. c)  6 =  − ∞ ;  ∪  ; + ∞  .
 5   5 
3. ∆ = b2 − 4 × 4 × (−1) = b2 + 16
Un carré est toujours positif donc ∆ > 0, l’équation a
donc deux solutions.
c −1
Le produit de ces solutions est égal à = <0:
a 4
les solutions sont donc de signes contraires.
L’affirmation est fausse.
S'entraîner
74 a)  f(1) = 3 × 12 + 6 × 1 − 9 = 0 donc 1 est une
racine de f
b 6
b) La somme des racines est égale à − = − = −2
a 3 80 a)  ∆ = 24 2 − 4 × 9 × 16
c)  1 + x ′ = −2 donc x ′ = −3
∆=0
la seconde racine de f est -3. 24 4
L’équation a une unique solution = .
d)  f( x) = 3( x − 1)( x + 3) 2×9 3

75 a) On développe l’expression donnée :


2( x − 2)2 + 1 = 2( x2 − 4 x + 4) + 1
= 2 x2 − 8 x + 8 + 1 Le programme affiche une valeur approchée de la
= 2 x2 − 8 x + 9 solution.
= f( x) b)  ∆ = 3, 4 2 − 4 × 0 , 5 × 2
b) Pour tout réel x, ( x − 2)2 > 0 donc on en déduit ∆ = 9 , 56
que 2( x − 2)2 + 1 > 1 > 0 L’équation a deux solutions
f est donc strictement positive sur . −3, 4 + 9 , 56
= −3, 4 + 9 , 56
2 × 0, 5
76 a)  a = 2, b = −3, c = −2
b)  ∆ = (−3)2 − 4 × 2 × (−2) = 25 −3, 4 − 9 , 56
= −3, 4 − 9 , 56
c)  ∆ > 0 l’équation a deux solutions : 2 × 0, 5
3 − 25 1 3 + 25
x1 = = − ou x2 = =2
2×2 2 2×2
 1  Le programme affiche une valeur approchée des
6 = − ; 2.
 2  deux solutions et du discriminant.

Chapitre 3  ★  Second degré 45

172909_Chap03_000-000.indd 45 25/07/2019 18:33:32


81 84 a) 

Pour f( x) = 2 x2 + 3 x + 4 on obtient :
Les courbes semblent avoir deux points d’intersec-
tion d’abscisses x1 » 0 , 4 et x2 » 11, 7
b) En raison de la racine carrée, l’équation n’a de sens
que si x > 0 .
1 1
Soit f( x) = 2( x + 0 , 75)2 + 2, 875 De plus, pour x réel positif, x + est positif.
4 2
Deux nombres positifs sont égaux si et seulement si
83 a)  leurs carrés sont égaux.
2
De plus, x étant positif, x = x
L’équation est, pour tout x ∈ [0 ; + ∞[ , équivalente
2
1 1
à :  x +  = x
 4 2
c) L’équation précédente peut s’écrire :
1 2 3 1
x − x+ =0
16 4 4
On calcule le discriminant :
 3 2 1 1 8 1
∆ = −  − 4 × × = =
 4  16 4 16 2
Les courbes # f et # g semblent avoir deux points ∆ > 0 donc l’équation a deux solutions :
d’intersection. Le premier a pour abscisse x1 ≈ −0 , 85 3 1 3−2 2

et le second x2 » 2, 35 4 2 4
x1 = = = 6−4 2
b) Les abscisses des points d’intersection des courbes 1 1

# f et # g sont les solutions dans  * de l’équation 16 8
3 1 3+2 2
f ( x) = g( x) c’est-à-dire : +
1 3 1 x2 = 4 2 = 4 =6+4 2
x− = 1 1
2 4 x 2×
1 3 16 8
Dans  * cette équation équivaut à  x −  x = 1 Les deux solutions trouvées sont positives donc les
 2 4
1 3
c’est-à-dire x2 − x − 1 = 0. abscisses des points d’intersection de # f et # g sont
2 4 6 - 4 2 et 6 + 4 2 .
 3 2 1 41
∆ = −  − 4 × × (−1) =
 4  2 16 85 1. a)  ∆ = b2 − 4 ac
∆ > 0 donc l’équation a deux solutions : a et c sont de signes contraires donc leur produit est
3 41 négatif et par conséquent -4 ac > 0

4 16 3 − 41 D est donc la somme de deux nombres positifs donc
x1 = = et
1 4 est positif : l’équation aura donc au moins une solution.

2 b) Pour que l’équation ait deux solutions réelles dis-
3 41 tinctes, il faut que D soit strictement positif.
+
x2 = 4 16 = 3 + 41 Or dans ce cas, ∆ = 0 si et seulement si b = c = 0.
1 4 Ainsi pour que l’équation ait deux solutions distinctes,

2 il suffit que b ou c soit non nul.
Donc # f et # g ont deux points d’intersections 2. x2 + 2 x + 1 = 0 est équivalente à ( x + 1)2 = 0.
d’abscisses x1 et x2 . L’équation admet donc une unique solution x = −1
On vérifie que x1 ≈ −0 , 85 et x2 » 2, 35 mais a et c sont tous les deux positifs.
46

172909_Chap03_000-000.indd 46 25/07/2019 18:34:13


86 1. a) Les nombres u et v sont les solutions de N(t ) = −5(t − 5)2 + 875
l’équation : Le nombre maximal de bactéries observables est 875
( x − u)( x − v ) = 0. après 5 min.
On développe le membre de gauche :
x2 − (u + v )x + uv = 0. 91 E(α) = −0 , 2(α 2 − 64α) + 1800
Or S = u + v et P = uv, on en déduit que u et v sont E(α) = −0 , 2(α − 32)2 + 0 , 2 × 322 + 1800
les solutions de : E(α) = −0 , 2(α − 32)2 + 2 004 , 8
x 2 − S x + P = 0. Pour tout 0 < a < 90 , −0 , 2(α − 32)2 < 0
b) u et v sont solutions de x2 − Sx + P = 0 , On en déduit que, pour tout 0 < a < 90 :
on en déduit que : −0 , 2(α − 32)2 + 2 004 , 8 < 2 004 , 8.
( x − u)( x − v ) = x2 − Sx + P. La quantité maximale d’énergie est atteinte pour une
Par identification des coefficients, on obtient : inclinaison de 32 °.
S = u + v et P = uv.
v2
c) u et v sont solutions de x2 − Sx + P = 0 si, et seu- 92 a) On résout : + 2v = 175
14
lement si, S = u + v et P = uv. v2
c’est-à-dire : + 2v − 175 = 0.
2. a) Les nombres cherchés sont solutions de l’équa- 14
tion : x2 − 6 x + 1 = 0. On calcule le discriminant :
On calcule le discriminant : 1
∆ = 22 − 4 × × (−175) = 54
∆ = (−6)2 − 4 × 1× 1 = 32. 14
Les solutions sont : Les solutions sont :
−2 − 54
6 − 32 6 + 32 v1 = = −14 − 21 6 < 0
x1 = et x2 = . 1
2 2 2×
14
b) Les longueurs du rectangle sont les solutions de
−2 + 54
l’équation : x2 − 24 x + 25 = 0. et v 2 = = −14 + 21 6
1
On calcule le discriminant : 2×
14
∆ = (−24)2 − 4 × 1× 25 = 476 Le véhicule roulait à 21 6 − 14 m ⋅ s−1 à l’instant du
Les solutions sont : freinage, soit environ 37, 4 m ⋅ s−1.
24 − 476 24 + 476
x1 = et x2 = . b)  37, 4 m ⋅ s−1 = 134 , 8 km ⋅ h−1.
2 2
Il ne respectait pas la limitation de vitesse.
87 a)  x12 + x22 = ( x1 + x2 )2 − 2 x1x2
93 Soit L et  les dimensions du terrain de Guil-
= 22 − 2 × (−11) = 26 laume. D’après les données on sait que :
1 1 x12 + x22 26 26 2(L + ) = 17 et L = 17
b)  + = = =
x12 x22 ( x1x2 )2 (−11)2 121 On en déduit donc que L et  sont les solutions de
l’équation :
88 f s’écrit sous la forme : f ( x) = a( x + 4)( x − 5). x2 − 8 , 5 x + 17 = 0.
De plus f(3) = 8 ce qui est équivalent à : On calcule le discriminant :
4 ∆ = (−8 , 5)2 − 4 × 1× 17 = 4 , 25.
a × 7 × (−2) = 8 , soit a = − .
7 Les solutions sont :
La fonction f cherchée est : 8 , 5 − 4 , 25 8 , 5 + 4 , 25
4 x1 = et x2 =
f( x) = − ( x + 4)( x − 5). 2 2
7 Le terrain de Guillaume a pour dimensions :
8 , 5 - 4 , 25 8 , 5 + 4 , 25
89 D’après leurs formes canoniques, f admet pour maxi- m, soit environ 3,2m et m
2 2
mum 109 lorsque x = 56 et g admet pour maximum
soit environ 5,3m.
436 001 561
= 109 , 000 25 lorsque x = = 56 ,1 .
4 000 10 94 a) On résout :
Donc Rafaël a tort. X2 − 3X = 0
X(X − 3) = 0
90 N(t ) = −5(t 2 − 10t ) + 1000 X = 0 ou X = 3 soit : x2 = 0 ou x2 = 3
N(t ) = −5(t − 5)2 − 5 × 52 + 1000
{
6= 0; − 3; 3 . }
Chapitre 3  ★  Second degré 47

172909_Chap03_000-000.indd 47 25/07/2019 18:35:22


b) On résout : Soit 2 x2 − 98 x − 1176 = 0
1 On calcule le discriminant :
− X2 + 3X − 13, 5 = 0.
6 ∆ = 982 − 4 × 2 × (−1176) = 19 012
On calcule le discriminant : L’équation a deux solutions :
−1
∆ = 32 − 4 × × (−13, 5) = 0 98 − 19 012 98 + 19 012
6 x1 = ou x2 =
−3 4 4
L’équation a une solution unique : X 0 = =9
 1 On remplace alors pour trouver les valeurs de y :
soit x2 = 9. 2− 
 6  98 − 19 012 98 + 19 012
6 = {−3 ; 3}. y = 49 − =
c) On résout : 4 4
98 + 19 012 98 − 19 012
3X2 + 9X − 12 = 0. ou y = 49 − =
4 4
On calcule le discriminant : Le système a deux couples solutions :
∆ = 92 − 4 × 3 × (−12) = 225  98 − 19 012 98 + 19 012 
6 =  ;  ;
L’équation a deux solutions :  4 4 
 
−9 − 225 −9 + 225  98 + 19 012 98 − 19 012 
x1 = = −4 ou x2 = = 1.  ; 
6 6  4 4 
 
Soit x2 = −4 (impossible) ou x2 = 1
6 = {−1 ; 1}
97 L’équation 1 donne : x = y + 10 soit en substi-
d) On résout :
tuant dans l’équation 2 :
2X2 + 40X + 128 = 0.
( y + 10) y = −16
On calcule le discriminant :
soit y 2 + 10 y + 16 = 0
∆ = 402 − 4 × 2 × 128 = 576
L’équation a deux solutions : On calcule le discriminant :
−40 − 576 ∆ = 102 − 4 × 1× 16 = 36
x1 = = −16 ou L’équation a deux solutions :
4
−40 + 576 −10 − 36 −10 + 36
x2 = = −4. y1 = = −8 ou y2 = = −2
4 2 2
On remplace pour trouver les valeurs de x corres-
Soit x2 = −16 ou x2 = −4 (impossible)
pondantes :
6 = ∅.
x1 = −8 + 10 = 2 et x2 = −2 + 10 = 8
95 a) On calcule le discriminant : Le système a deux couples solutions :
∆ = (−18)2 − 4 × 81× 1 = 0 6 = {(2 ; − 8) ; (8 ; − 2)}.
L’équation a une unique solution :
18 1 98 La seconde équation du système peut s’écrire :
x0 = =
2 × 81 9 x+ y
= −0 , 3 soit en utilisant l’équation 1 :
b) On pose X = x2 . xy
Cela revient à résoudre : 81X2 − 18X + 1 = 0. x + y = −0 , 3 × (−2, 5)
1 1 On en déduit : y = 0 , 75 − x
D’après la question a) : X = , soit x2 = .
9 9 L’équation 1, s’écrit alors : x(0 , 75 − x) = −2, 5
 1 1
6 = − ; . soit −x2 + 0 , 75 x + 2, 5 = 0
 3 3 
c) On pose Y = x2 . On calcule le discriminant :
Cela revient à résoudre : 81Y 4 − 18 Y 2 + 1 = 0. ∆ = 0 , 752 − 4 × (−1) × 2, 5 = 10 , 562 5
1 1 L’équation a deux solutions :
D’après la question b) : Y = − ou Y = −0 , 75 − 10 , 562 5
3 3 x1 = =2
2 1 2 1 −2
Soit x = − (impossible) ou x = .
3 3 −0 , 75 + 10 , 562 5
ou x2 = = −1, 25
 3 

3 −2
6 = − ; .
 3 3  Ce qui donne : y1 = 0 , 75 − 2 = −1, 25
 
96 L’équation 1 donne : y = 49 − x. et y2 = 0 , 75 + 1, 25 = 2
En substituant dans l’équation 2, on obtient : Le système a deux couples solutions :
x2 + (49 − x)2 = 1225 6 = {(2 ; − 1, 25) ; (−1, 25 ; 2)}.
48

172909_Chap03_000-000.indd 48 25/07/2019 19:23:34


99 En utilisant la formule de la vitesse moyenne, L’équation a deux solutions :
d 5 256 , 25 − 4 968 , 25
v = , les données de l’exercice se traduisent par : x1 = = 144 ou
t 2
 195 5 256 , 25 + 4 968 , 25
 t1 = v x2 = = 5112, 25
 1 2

 195 Soit a2 = 144 donc a = 12 puisque a > 0 et alors
t1 − 1 =
 v1 + 4 858
b= = 71, 5
En substituant t1 dans la seconde équation, on obtient : 12
195 195 Soit a2 = 5112, 25 donc a = 71, 5 puisque a > 0 et
−1= , 858
v1 v1 + 4 alors b = = 12
71, 5
soit −v12 − 4v1 + 780 = 0
Le périmètre du triangle est :
On calcule le discriminant : 71, 5 + 12 + 72, 5 = 166 m
∆ = (−4)2 − 4 × (−1) × 780 = 3126
L’équation a deux solutions : 102 M est un point de la droite d donc M a pour
4 − 3126 4 + 3126 coordonnées ( x ; 2 x + 4)
v1 = ou v1 = < 0 : impossible
−2 −2 De plus, on veut que OM = 5 soit :
3126 − 4
La vitesse du premier cycliste est km ⋅ h−1, x2 + (2 x + 4)2 = 5
2 ce qui équivaut à :
soit environ 26 km ⋅ h−1 et celle du second cycliste
x2 + (2 x + 4)2 = 5
3126 + 4
est km ⋅ h−1, soit environ 30 km ⋅ h−1. 2
c’est-à-dire : 5 x + 16 x + 11 = 0
2
∆ = 162 − 4 × 5 × 11 = 36
100 1. a) Après découpage, l’aire du carré de côté ∆ > 0 donc l’équation a deux solutions :
x + 5 est égale à l’aire du rectangle de départ ADFE −16 − 36 −16 + 36
(39) à laquelle s’ajoute l’aire du carré de côté 5. x1 = = −2, 2 et x2 = = −1
2×5 2×5
soit : ( x + 5)2 − 25 = 39 Les points M ont donc pour coordonnées
b)  ( x + 5)2 = 64 équivaut à x+5= 8 ou (−2, 2 ; − 0 , 4) et (−1 ; 2).
x + 5 = −8 soit x = 3 ou x = −13 < 0 car x est
 x 2  4 − x 2
103 a)  D( x) = 4π − π  − π
une longueur.  2   2 
2. a) On calcule le discriminant :  x2  x2 
∆ = 102 − 4 × 1× (−39) = 256 D( x) = π  4 − − 4 − 2 x + 
 4  4 
L’équation a deux solutions :
 x 
2

x1 =
−10 − 256
= −13 ou x2 =
−10 + 256
=3 D( x) = π2 x − 
2 2  4 
b) On retrouve bien les mêmes solutions. π
D( x) = (−x2 + 4 x).
2
101 On note a et b les longueurs des côtés adjacents b) Par la méthode de la complétion du carré :
à l’angle droit. On sait que a > 0 et b > 0 π
D( x) = − ( x − 2)2 + 2π.
D’après les données, on obtient le système : 2
 ab = 429 × 2 L’aire de la partie bleue est maximale lorsque M est le
 2 milieu de [AB] et Dmax = 2π.
a + b2 = 72, 52
 π 3
858 c) On doit avoir : π < (−x2 + 4 x) < π
L’équation 1 donne  : b = ce qui en substituant 2 2
a c’est-à-dire x2 − 4 x + 2 < 0 et −x2 + 4 x − 3 < 0
dans l’équation 2 :
736 164 avec 0 < x < 4.
a2 + = 5 256 , 25
a2 • x2 − 4 x + 2 = 0
Soit : a 4 − 5 256 , 25a2 + 736 164 = 0 4+2 2
∆ = 8; x1 = = 2 + 2 et x2 = 2 − 2 .
On pose X = a2 2
x 0 2- 2 2+ 2 4
Cela revient à résoudre :
X2 − 5 256 , 25X + 736 164 = 0 x2 − 4 x + 2 + - +
2
On calcule le discriminant : • −x + 4 x − 3 = 0.
∆ = (−5 256 , 25)2 − 4 × 736 164 = 24 683 508 , 062 5 ∆ = 4 ; x3 = 1  et  x4 = 3.

Chapitre 3  ★  Second degré 49

172909_Chap03_000-000.indd 49 25/07/2019 18:37:38


x 0 1 3 4 108 P1 est vraie
- - P2 est vraie
x2 − 4 x + 2 +
Réciproque de P1 : Si ∆ < 0 , alors pour tout nombre
Conclusion : Charlotte doit placer le point M tel que :
réel x, f( x) > 0
x ∈ 2 − 2 ; 1 ∪ 3 ; 2 + 2  .
    La réciproque est fausse, f( x) peut être strictement
négatif.
104 a) Soit p le prix de vente à la tonne.
Réciproque de P2 : Si ∆ > 0 , alors ac < 0
Comme il y a équilibre pour 25 tonnes,
La réciproque est fausse.
25 p = 252 + 632 × 25 + 1 075 donc p = 700 €.
b)  B(q) = 700q − C(q) = −q2 + 68q − 1075
c) L’activité est rentable lorsque B(q) > 0.
On cherche les racines de B(q). ∆ = 324, il y a donc

Organiser son raisonnement


deux racines : 25 (déjà connue), et 43.
Comme le coefficient de x2 est négatif, B(q) est posi-
tif entre les racines. L’activité est rentable pour une
production comprise entre 25 et 43 tonnes de papier.

105 a)  x ∈ [0 , 8 ; 12] 109 On note h et c les côtés du triangle.


b) L’aire de la partie restante est : h + c = 4 , 9 et h2 + c 2 = 3, 52
(12 − x)(30 − x) > 280 ce qui est équivalent à  : Cela revient à résoudre l’équation :
360 − 12 x − 30 x + x2 > 280 , h2 + (4 , 9 − h)2 = 3, 52
soit x2 − 42 x + 80 > 0 ce qui est équivalent à : 2h2 − 9 , 8h + 11, 76 = 0
c)  ∆ = (−42)2 − 4 × 1× 80 = 1444 ∆ = 1, 96
∆ > 0 donc l’équation a deux solutions : Les solutions sont :
42 − 1444 42 + 1444 9 , 8 − 1, 4 9 , 8 + 1, 4
x1 = = 2 et x2 = = 40 h1 = = 2,1 ou h2 = = 2, 8
2 2 4 4
x −∞ 2 40 +∞ Les côtés du triangle sont 2,1 et 2, 8.
signe + 0 - 0 + h + c = 4 , 9 et 3, 52 + c 2 = h2
Ainsi comme x ∈ [0 , 8 ; 12], on en déduit que les lar- Cela revient à résoudre l’équation :
geurs possibles du chemin sont comprises entre 3, 52 + (4 , 9 − h)2 = h2
0,8 m et 2 m. 36 , 26 − 9 , 8h = 0
h = 3, 7
106 1. L’inéquation est équivalente à : Les côtés du triangle sont 3,7 et 1,2.
x2 - 1000 x - 8 000 > 0
∆ = (−1000)2 − 4 × 1× (−8 000) > 0 110 a) Méthode d’Abdel : il a calculé le discriminant
Par conséquent, l’inéquation admet des solutions. de f pour déterminer les racines et en déduire ensuite
2. a) Cet algorithme permet de déterminer et d’affi- la forme factorisée.
cher la plus petite valeur positive dont l’image par h Avantage  : cette méthode convient pour toutes les
dépasse le nombre saisi pour M. fonctions
b) On obtient 1008 : 1008 est le plus petit entier Inconvénient : les calculs ne sont pas « astucieux ».
positif tel que h(1008) > 8 000. Bélinda : elle a trouvé une racine évidente de f puis a
trouvé la forme factorisée à partir de cette racine.
107 a) Faux : sur [1 ; + ∞[ , f( x) < 0 Avantage : les calculs peuvent tous être faits à la main,
b) Faux : les nombres de l’intervalle [−2 ; 0 ] ont une et on en déduit rapidement la factorisation
image positive. Inconvénient  : il n’est pas toujours facile de trouver
c) Vrai  : Tous les nombres de l’intervalle [0 ; 1] ont une racine évidente.
une image positive. Charlotte  : elle a écrit f sous forme canonique, et a
d) Faux  : Les nombres de l’intervalle [1 ; + ∞ [ ont alors trouvé la forme factorisée de f puisque ce sont
aussi une image négative. les mêmes.
e) Faux  : ] − 1 ; 1[ ⊂ ] − 2 ; 1[ donc leur image est un Avantage  : la méthode est rapide, car f a la même
nombre positif. forme canonique et factorisée.
50

172909_Chap03_000-000.indd 50 25/07/2019 18:38:34


Inconvénient : il faut ensuite trouver comment facto- 113 On calcule le discriminant :
riser la forme canonique dans le cas général. ∆ = (m − 1)2 − 4 m × (−1)
Dominic  : il a commencé à remarquer que les coeffi- ∆ = m 2 − 2m + 1 + 4 m
cients sont des multiples de 5, puis reconnait une iden- ∆ = m 2 + 2m + 1
tité remarquable ce qui lui permet de déterminer la ∆ = (m + 1)2
factorisation, sans passer par la recherche des racines. Si m = -1, alors D = 0, l’équation a une unique solution.
Avantage : il obtient directement la forme factorisée Si m ≠ −1, alors ∆ > 0 , l’équation a deux solutions.
Inconvénient : on ne peut l’utiliser que si la factorisa-
tion se fait à partir d’une identité remarquable. 114 On note n le nombre de personnes du groupe
b) La stratégie d’Abdel et celle de Charlotte auraient d’amis et le gain par personne.
pu être utilisées mais pas les deux autres car les deux L’énoncé se traduit par le système :
racines de f sont irrationnelles et la factorisation ne se nx = 2 × 106

fait pas à l’aide d’une identité remarquable. 
(n − 5)( x + 20 000) = 2 × 106

111 On note x la largeur de la croix rouge. On obtient l’équation :
L’aire de la partie rouge est donnée par : −5 x2 − 105 x + 4 × 1010
=0
2 x + 3 x − x2 = 5 x − x2 x
L’aire du drapeau est : 3 × 2 = 6 x ¹ 0, ce qui équivaut à −5 x2 − 105 x + 4 × 1010 = 0
L’aire de la croix rouge et la partie blanche ont la On calcule le discriminant :
6 ∆ = (−105 )2 − 4 × (−5) × (4 × 1010 )
même aire, si 5 x − x2 = ce qui donne :
2 ∆ = 1010 × 81
−x2 + 5 x − 3 = 0. Les solutions sont :
On calcule le discriminant : 105 − 81× 1010
∆ = 52 − 4 × (−1) × (−3) = 13 x1 = = 8 × 10 4 ou
2 × (−5)
L’équation a deux solutions :
105 + 81× 1010
−5 − 13 5 + 13 x2 = = −105 < 0
x1 = = ou −10
−2 2
Le nombre d’amis du groupe est donné par :
−5 + 13 5 − 13
x2 = = 2 × 106 200
−2 2 4
= = 25.
8 × 10 8
Or x1 » 4 , 3 : impossible, car les dimensions du dra- Il y avait 25 amis dans le groupe et chacun a gagné
peau sont 2 m et 3 m. 80 000 €.
La largeur de la croix rouge doit donc être égale à
5 - 13 115 On note L et  respectivement la longueur et la
m.
2 largeur du rectangle.
112 On note x la longueur, en cm, du côté du carré Les données se traduisent par :
 k  k
le plus petit. Les deux autres côtés ont pour longueur L +  = 2 ce qui est équivalent à L = 2 − 
 
x + 2 et x + 4 .  
 L = k  L = k
L’aire se calcule en faisant :
x2 + ( x + 2)2 + ( x + 4)2 = 515, En substituant dans la seconde équation :

équation qui est équivalente à : 3 x2 + 12 x − 495 = 0  L = k −   k
L = −
 2  2
On calcule le discriminant :  soit, 
∆ = 122 − 4 × 3 × (−495) = 6 084  k   2 k
 −   = k − +  − k = 0
L’équation a deux solutions :  2   2
−12 − 6 084 On résout la seconde équation, en calculant le discri-
x1 = = −15 ou minant :
2×3
−12 + 6 084  k 2 k 2 − 16k
x2 = = 11 ∆ =   − 4 × (−1) × (−k ) =
 2  4
6
x représente une longueur donc x > 0 . Pour que cette équation ait des solutions, il faut que
Ainsi la longueur du plus petit carré est 11  cm et la D soit positif.
longueur du polygone est donnée par : k 2 − 16k k (k − 16)
=
11 + 13 + 15 = 39 m. 4 4

Chapitre 3  ★  Second degré 51

172909_Chap03_000-000.indd 51 25/07/2019 18:39:14


k 0 16 +∞ 118 On note x la longueur en cm du segment
Signe - 0
consacrée au carré. Cela correspond à son périmètre.
+
On en déduit donc que son aire est donnée par
Ainsi, l’équation aura des solutions si k > 16.  x 2 2
  = x
 4  16
116 a) g(2) = 2 × 23 − 7 × 22 + 2 + 10 La longueur consacrée au disque est 100 - x.
g(2) = 2 × 8 − 7 × 4 + 12 Elle correspond au périmètre du cercle, soit :
g(2) = 0 100 − x = 2πr où r est le rayon du cercle.
Donc 2 est une racine évidente de g. L’aire du disque est donnée par :
2
b) On développe et on identifie les coefficients : 100 − x 2 (100 − x )
π ×   =
( x − 2)(a x2 + b x + c )  2π  4π
= a x3 + (b − 2a)x2 + (c − 2b)x − 2c a) Lorsque la figure évolue de gauche à droite, x aug-
Par identification, on obtient : mente de 0 à 100, donc l’aire du carré augmente.
 a=2 Lorsque la figure évolue de gauche à droite, x augmente
 a = 2 
de 0 à 100 on en déduit que 100 - x diminue de 100 à 0.
  b = −7 + 2 × 2
b − 2a = −7 ,        On peut donc conclure que l’aire du disque diminue.
 c = 1 + 2 × (−3)
 c − 2b = 1  b) 
  10
 −2c = 10  c= x2 10 000 − 200 x + x2 (π + 4)x2 + 40 000 − 800 x
 −2 + =
16 4π 16π
 a = 2 π + 4  2 800  40 000
 = x − x +
On obtient : b = −3 16π  π + 4  16π
 2 2
c = −5 π + 4  400  π + 4  400  2 500
=  x −  − ×
 
 +
Et donc g( x) = ( x − 2)(2 x2 − 3 x − 5) 16π  π + 4 16π  π + 4  π
2
c) On utilise la forme factorisée : π + 4  400   10 000 2 500
= x −  − +
x − 2 = 0 ou 2 x2 − 3 x − 5 = 0 16π  π + 4  π(π + 4) π
2
On calcule le discriminant pour l’équation du second π + 4  400  2 500
= x −  +
degré : ∆ = (−3)2 − 4 × 2 × (−5) = 49 16π  π + 4  π+4
∆ > 0 , l’équation a donc deux solutions : D’après la forme canonique, la somme des deux aires
admet un minimum et on en déduit qu’elle diminue
3 − 49 3 + 49 5 400
x1 = = −1 ou x2 = = jusqu’à x = puis qu’elle augmente.
2×2 2×2 2 π+4
 5  180
L’équation a trois solutions : 6 = −1 ; ; 2. 119 nx = 180, donc n = .
 2  
 x
( x − 2)(n + 3) = 180
117 a) 180 
( x − 2) + 3 = 180
 x 
( x − 2)(180 + 3 x) = 180 x
3 x2 − 6 x − 360 = 0
x2 − 2 x − 120 = 0
∆ = 480 x1 = −10 et x2 = 12.
b)  f( x) = 3( x2 + 2 x) + 5 Le montant de son remplacement est 12 € par mois
f( x) = 3( x + 1)2 − 3 + 5 pendant 15 mois.
f ( x) = 3( x + 1)2 + 2
c)  f( x) = 8 s’écrit 3( x + 1)2 + 2 = 8 est équivalent à 120 C’est une équation produit. Chaque facteur est
8−2 de la forme :
( x + 1)2 = , c’est-à-dire : ( x + 1)2 = 2. x2 − nx + 10n = 0 avec 1 < n < 40
3
d)  ( x + 1)2 = 2 est équivalent à x + 1 = 2 ou Le discriminant de cette équation est de la forme :
x +1= − 2 ∆ = (−n)2 − 4 × 10n
x = 2 − 1 ou x = − 2 − 1. ∆ = n(n − 40)
On établit le signe de D :
n 1 40
- 0

52

172909_Chap03_000-000.indd 52 25/07/2019 18:40:18


Ainsi les 39 premiers discriminants sont strictement On calcule le discriminant :
négatifs et les équations correspondantes n’ont pas ∆ = 482 − 4 × 12 × (−144) = 9 216 l’équation a
de solution. deux solutions :
Pour la dernière, x2 − 40 x + 400 = 0 , ∆ = 0 : −48 − 9 216 −48 + 9 216
40 x1 = ou x2 =
l’équation a une unique solution : = 20 2 × 12 2 × 12
2 x1 = −6 ou x2 = 2
L’équation donnée au départ a donc une unique solu-
x est une longueur donc c’est un nombre positif, par
tion : 6 = {20}.
conséquent, la longueur de l’arête du grand cube est
121 1. a) Toutes les puissances de 1 sont égales à 1, 4 + 2 = 6 m.
donc P(1) = 1n − 1 = 0.
123
b)  Q( x) = x n−1 + x n−2 +  + x + 1 1+ 5
φ=
En développant ( x − 1)Q( x) , on obtient : 2
x n − 1 = P( x). φ2 = φ + 1 φ 6 = 8φ + 5
2. a)  h(a) = an − an = 0
φ3 = 2φ + 1 φ7 = 13φ + 8
b)  h( x) = ( x − a)( x n−1 + a x n−2 +  + an−1x + an )
3. a)  f( x) = x3 − 23 φ4 = 3φ + 2 φ8 = 21φ + 13
f ( x) = ( x − 2)( x2 + 2 x + 4) φ5 = 5φ + 3
g( x) = x 4 − 34 φ21 = (φ7 )3 + (13φ + 8)2 (13φ + 8)
g( x) = ( x − 3)( x3 + 3 x2 + 9 x + 27) On reconnait les termes de la suite de Fibonacci.
Pour factoriser g( x) on aurait pu remarquer que
φ2019 = F2019 + F2018
( x2 )2 − (9)2 avec l’identité remarquable, on aurait
obtenu : 124 Les données se traduisent par :
g( x) = ( x2 − 9)( x2 + 9). x3 + 56 = ( x + 2)3
On applique la même méthode pour factoriser En développant le membre de droite de l’équation,
x2 - 9 et on obtient : on obtient :
g( x) = ( x − 3)( x + 3)( x2 + 9). x3 + 56 = x3 + 2 x2 + 4 x + 8 qui est équivalente à 
b) On utilise les formes factorisées : 2 x2 + 4 x − 48 = 0
f( x) = 0 s’écrit ( x − 2)( x2 + 2 x + 4) = 0 On calcule le discriminant :
x − 2 = 0 ou x2 + 2 x + 4 = 0 ∆ = 4 2 − 4 × 2 × (−48) = 400 l’équation a deux
La première équation donne x = 2. solutions :
Pour la seconde équation, on calcule le discriminant : −4 − 400 −4 + 400
∆ = 4 − 4 × 4 = −12 : la seconde équation n’a pas x1 = ou x2 =
2×2 2×2
de solution. x1 = −6 ou x 2 = 4
L’équation f( x) = 0 a une unique solution x = 2. x est une longueur donc la longueur de l’arête est 4 cm.
g( x) = 0 s’écrit ( x − 3)( x + 3)( x2 + 9) = 0.
x − 3 = 0 ou x + 3 = 0 ou x2 + 9 = 0 , soit x = 3 125 Partie A
x = −3 ou x2 = −9. L’état de saturation est atteint pour 3 h de travail.
La dernière équation n’a pas de solution. Partie B
L’équation g( x) = 0 a donc deux solutions : 6 = {−3 ; 3} Saisir t
200 200
122 On note x la longueur en m du petit cube. y ←− t+
9 3
Les données se traduisent par l’équation : Si y > 0
(4 + x)3 − x3 = 208 Afficher « souhait »
On développe (4 + x)3 en remarquant que Sinon
(4 + x)3 = (4 + x)(4 + x)2 Afficher « rejet »
= (4 + x)(16 + 8 x + x2 ) Fin Si
= 64 + 32 x + 4 x2 + 16 x + 8 x2 + x3 Partie C
= x3 + 12 x2 + 48 x + 64 a) On doit résoudre h( x) = 100.
3
(4 + x) − x3 = 208 est équivalente à : 1 10
− x2 + x − 100 = 0
12 x2 + 48 x − 144 = 0 36 3

Chapitre 3  ★  Second degré 53

172909_Chap03_000-000.indd 53 25/07/2019 18:41:32


On calcule le discriminant : 1200
h<
10 2  1 675
∆ =   − 4 × −  × (−100) = 0 16
 3   36  La hauteur maximale de la piscine est m, soit
9
10 environ 1,77 m.

L’équation a une solution  x = 3 = 60
 1  127 On note n la plus petite dimension de l’aquarium.
2 × − 
 36  Les autres dimensions sont n, n + 1 et n + 2.
La « saturation » est atteinte à 60  min.
Les données du document 2 permettent d’obtenir
b) On doit résoudre h( x) > 60.
l’équation :
1 10
− x2 + x − 60 > 0 n(n + 1)(n + 2) = 4 × (4 n + 4(n + 1) + 4(n + 2))
36 3
On calcule le discriminant : Ce qui est équivalent à l’équation :
10 2  1 40 n3 + 3n2 − 46n − 48 = 0
∆ =   − 4 × −  × (−60) = On remarque que -1 est une racine évidente, on
 3   36  9
peut donc factoriser le membre de gauche par
Les deux racines sont :
(n + 1), l’équation est donc équivalente à :
10 40
− − (n + 1)(n2 + 2n − 48) = 0
x1 = 3 9 = 60 + 12 10 > 60
C’est une équation produit :
 1 
2 × − 
 36  n + 1 = 0 ou n2 + 2n − 48 = 0
10 40 n = −1 : impossible puisque n > 0
− + On calcule le discriminant de l’équation du second
Ou x2 = 3 9 = 60 − 12 10
 1  degré :
2 × − 
 36  ∆ = 22 − 4 × 1× (−48) = 196
On dresse le tableau de signes : Les deux solutions sont :
x 0 60 - 12 10 60 −2 − 196 −2 + 196
Signe - 0
n= = −8 ou n = =6
+ 2 2
La personne est dite heureuse à partir de n représentant une longueur, la seule solution est
60 - 12 10 min, soit environ 22 min. n= 6.
Les dimensions de l’aquarium sont 6 m, 7 m et 8 m.
Cherchons maintenant le coût de fabrication :
Chaque arête est présente 4 fois, donc le coût pour le
Exploiter ses compétences ruban est : 4 × (6 + 7 + 8) × 0 , 2 = 16 , 8
Le prix du ruban adhésif est 16,8 €.
Cherchons l’aire de la surface vitrée :
Les faces de l’aquarium sont rectangulaires deux à
deux identiques :
126 Notons h la hauteur d’eau en m.
2 × (6 × 7 + 7 × 8 + 8 × 6) = 292 dm2
Déterminons le volume d’eau de la piscine.
292 dm2 = 2, 92 m2
Volume d’eau dans la partie de la piscine à fond plan
(parallélépipède rectangle) : Coût de la surface vitrée : 50 × 2, 92 = 146 €.
12 × 12, 5 × h = 150h m3 Le coût total de l’aquarium est :
La partie à fond incliné est un prisme droit, calculons 146 + 16 , 8 = 162,8 €.
son volume :
h × 12, 5 128 Déterminons la distance d entre deux rangées.
! AFB × 12 = × 12 = 75h m3
2 Le nombre de pommiers de la parcelle est donné
Volume d’eau de la piscine : par l’expression :
150h + 75h = 225h m3 140  28 
 + 1 + 1
Le montant à payer pour le remplissage de la piscine  d 
 d 
est donné par : 225h × 3 = 675h Ce qui permet d’obtenir l’équation :
Les contraintes liées au coût donnent : 140  28 
 + 1 + 1 = 369
675h < 1200  d 
 d 
54

172909_Chap03_000-000.indd 54 25/07/2019 18:42:15


En développant le membre de gauche : 202 + 632 × 20 + 1000 = 20 p
3 920 168 14 040 = 20 p
+ + 1 = 369
d2 d
Ce qui donne l’équation : 14 040
p= = 702 €.
−368d 2 + 168d + 3 920 20
=0
d2 Les coûts sont la somme des coûts fixes et des coûts
d ¹ 0, donc l’équation est équivalente à : variables :
−368d 2 + 168d + 3 920 = 0 C = CF + CV
On calcule le discriminant : C = 650 + 200 + 150 + q2 + 632q
∆ = 1682 − 4 × (−368) × 3 920 = 2 4082 C = q2 + 632q + 1000
Les deux solutions sont : Le bénéfice est alors donné par :
−168 − 2 4082 702q − (q2 + 632q + 1000) = −q2 + 70q − 1000
d= = 3, 5 ou Déterminons la quantité à produire pour que la pro-
2 × (−368)
duction soit rentable.
−168 + 2 4082 70 ∆ = 702 − 4 × 1000 = 900
d= =−
2 × (−368) 23 Les deux racines sont données par :
Or d étant une distance est un nombre positif. −70 − 900 −70 + 900
q1 = = 50 ou q2 = = 20
On peut donc conclure que la distance entre deux −2 −2
rangées est 3,5 m. On peut alors dresser le tableau de signes :
Le ramassage se fera donc à l’aide d’un tracteur de q 0 20 50 +∞
taille moyenne. Le coût étant de 75  € pour 1  h de Signe - 0 + 0 -
ramassage. La production est rentable pour une quantité comprise
Déterminons le nombre de rangées : strictement entre 20 et 50 tonnes de pâte à papier.
24
+ 1 = 9. Cherchons le bénéfice maximal :
3, 5
Il faudra donc prévoir 9 h pour le ramassage de la par- −q2 + 70q − 1000 = −(q2 − 70q) − 1000
celle, soit un coût égal à 9 × 75 = 675 €. = −(q − 35)2 + 352 − 1000
= −(q − 35)2 + 225
129 Déterminons le prix de vente d’une tonne de Comme (q − 35)2 > 0 pour tout nombre q, on en
pâte à papier. déduit que :
L’activité est à l’équilibre lorsque l’entreprise produit −(q − 35)2 < 0 et −(q − 35)2 + 225 < 225.
et vend 20 tonnes et on note p le prix de vente en Le bénéfice est maximal est donc 225  € et il corres-
euros d’une tonne de pâte à papier. pond à une production de 35 tonnes.

Chapitre 3  ★  Second degré 55

172909_Chap03_000-000.indd 55 25/07/2019 18:42:48


172909_Chap03_000-000.indd 56 25/07/2019 18:42:48
4 Dérivation

2 Un coût marginal


Découvrir
1  a) Le coût de production de q litres est C(q),
celui de q + 1 litres est C(q + 1), donc
Cm (q) = C(q + 1) − C(q).
b)  Cm (q) est le taux de variation de la fonction C
1 Vitesse moyenne entre q et q + 1.
et vitesse instantanée 3 97 951
c)  Cm (50) = × 502 − × 50 +
1000 1000 1000
1  a) La vitesse moyenne (en m ⋅ s−1 ) de la bille
Cm (50) = 3, 601.
entre les instants :
d(1, 2) − d(1) Lorsque l‘entreprise produit 50 litres d’huile
•  t = 1 et t = 1, 2 est = 10 , 78 d’olive, le coût induit par la production d’un litre
0, 2
supplémentaire est environ 3,60 €.
d(1) − d(0 , 9)
•  t = 0 , 9 et t = 1 est = 9 , 31
0 ,1 2  Géométriquement, la distance NP est égale au
b) Entre les instants t = 1 et t = 1 + h (avec coût marginal Cm (50).
1 + h > 0 et h ¹ 0 ), la distance parcourue par la
bille est :
• si h > 0, d(1 + h) − d(1)
• si h < 0, d(1) − d(1 + h).
La vitesse moyenne est :

Acquérir des automatismes


d(1 + h) − d(1)
• si h > 0,
h
d(1) − d(1 + h)
• si h < 0,
−h
c’est-à-dire dans les deux situations :
d(1 + h) − d(1) 3 a) Pour tout nombre réel h ¹ 0,
.
h f (2 + h) − f (2) 2(2 + h)2 + 3 − (2 × 22 + 3)
2 =
c)  d(1 + h) − d(1) = 4 , 9(1 + h) − 4 , 9 = 4 , 9h(2 + h) h h
h h h
f (2 + h) − f (2) 2(4 h + h2 ) 2h(4 + h)
d(1 + h) − d(1) = =
= 4 , 9(2 + h). h h h
h
f (2 + h) − f (2)
2  Lorsqu’on donne à h des valeurs de plus en plus = 2(4 + h).
h
d(1 + h) − d(1)
proches de 0, se rapproche de 9,8. b) # est la courbe représentative de f, A et M sont les
h
points de # d’abscisses 2 et 2 + h avec h ¹ 0, alors
f (2 + h) − f (2)
est la pente de la sécante (AM).
h
c) Le taux 2(4 + h) a pour limite 8 lorsque h tend vers
0 donc f est dérivable en 2 et f ′(2) = 8.

Chapitre 4  ★  Dérivation 57

172909_Chap04_000-000.indd 57 25/07/2019 18:51:12


4 a) La droite T passe par les points A(−1 ; 1) et −2( x − 4) − (−2 x + 5)
g ′( x) =
0 −1 1 ( x − 4)2
B(1 ; 0) donc sa pente est =− .
1 − (−1) 2 3
b) Le nombre dérivé de f en -1 est la pente de la tan- g ′( x) = .
( x − 4)2
1
gente T, ainsi f ′(−1) = − .
2 13 3h - 11 a pour limite -11 lorsque h tend vers 0
donc f ′(1) = −11.
7 g est dérivable en tout nombre réel x > 0 et
1 1 14 On effectue le calcul du taux de variation de la
g ′( x) = donc g ′(4) = .
2 x 4 fonction g entre 3 et 3 + h.
D’autre part g(4) = 2. Une équation de la tangente T
à # au point A est : 15 Louise a tort. La pente de la tangente à la courbe
1 représentative de f au point A est f ′(−1) = −2.
y = g ′(4)( x − 4) + g(4), y = ( x − 4) + 2,
4
1 16 On lit graphiquement que la pente de T est
soit y = x + 1.
4 égale à 1 donc f ′(4) = 1.
8 La courbe # est tracée ci-dessous.
17 La pente de la tangente à # au point d’abscisse
g est dérivable sur  et g ′( x) = 3 x2 . 1 1
g(1) = 1 et g ′(1) = 3 donc on trace la droite T qui 0 est donc g ′(0) = .
2 2
passe par A(1 ; 1) et de pente 3.
On peut aussi déterminer une équation de T : 18 Une équation de T est :
y = 3( x − 1) + 1, soit y = 3 x − 2 et on choisit deux y = −3( x − 1) + 2, soit y = −3 x + 5.
points de cette droite A(1 ; 1) et par exemple La réponse correcte est la réponse (2).
B(0 ; − 2).
# 19 a) Pour tout nombre réel h ¹ 0,
T
f (2 + h) − f (2) (2 + h)2 − 22 4 h + h2
= =
+3 h h h
A f (2 + h) − f (2) h(4 + h)
1 = = 4 + h.
1 h h
0 1 b) La limite de 4 + h lorsque h tend vers 0 est 4 donc
f ′(2) = 4.
-2
20 a) Pour tout nombre réel h ¹ 0,

11 a) g est dérivable sur  et pour tout nombre g(1 + h) − g(1) −(1 + h)2 + 3(1 + h) − 1 − (1)
=
réel x, g ′( x) = −12 x2 + 2 x. h h
1 g(1 + h) − g(1) −h2 + h h(−h + 1)
b) Pour tout nombre réel x, f( x) = (2 x2 − x). = = = −h + 1.
5 h h h
f est dérivable sur  et pour tout nombre réel x, b) La limite de −h + 1 lorsque h tend vers 0 est 1
1 donc g est dérivable en 1 et g ′(1) = 1.
f ′( x) = (4 x − 1).
5
c) h est le produit de deux fonctions dérivables sur  21 a) Pour tout nombre réel h avec h > −1 et
et pour tout nombre réel x, h ¹ 0,
h′( x) = 2( x2 − 2 x + 2) + (2 x − 5)(2 x − 2) 4
−4
h′( x) = 2 x2 − 4 x + 4 + 4 x2 − 14 x + 10 f (1 + h) − f (1) 1 + h 4 − 4(1 + h)
= =
h h h(1 + h)
h′( x) = 6 x2 − 18 x + 14.
f (1 + h) − f (1) −4 h −4
= = .
12 g est le quotient de deux fonctions dérivables h h(1 + h) 1 + h
sur ]− ∞ ; 4[ et ]4 ; + ∞[ donc g est dérivable sur −4
b) La limite de lorsque h tend vers 0 est -4
chacun de ces intervalles. 1+ h
Pour tout nombre réel x ¹ 4, donc f est dérivable en 1 et f ′(1) = −4.
58

172909_Chap04_000-000.indd 58 25/07/2019 18:52:22


22 a) Pour tout nombre réel h ¹ 0, 29 a) 
g(4 + h) − g(4) (4 + h)2 + 7 − 23 8h + h2 #
= = 2
h h h
g(4 + h) − g(4) h(8 + h) A
= = 8 + h. 1
h h
b) La limite de 8 + h lorsque h tend vers 0 est 8 donc 0,5
g ′(4) = 8. 0 1 2 3 4
23 a) Pour tout nombre réel h avec h > −1 et
h ¹ 0, b) On lit f ′(1) = 0 , 5.
−2 c) La pente de la tangente T à # au point A est 0,5.
− (−2)
f (h) − f (0) h + 1 −2 + 2(h + 1)
= =
h h h(h + 1) 30 a)  f ′(−1) = 3. La pente de la tangente T à # au
f (h) − f (0) 2h 2 point A est 3.
= = . b) #
h h(h + 1) h + 1
2 2
b) La limite de lorsque h tend vers 0 est 2 donc
h +1
f est dérivable en 0 et f ′(0) = 2. 1
T
24 a) La pente de la sécante (AM) est : -1
f (−1 + h) − f (−1) (−1 + h)2 − (−1)2 h2 − 2h 0 1
= = ,
h h h
-1
h(h − 2) A
soit = h − 2.
h
b) La limite de h - 2 lorsque h tend vers 0 est -2
donc la pente de la tangente T à # au point A est -2. 31 a) b) et c)
c) Et ainsi f ′(−1) = −2.
A
1
25 a) La vitesse moyenne de la bille entre les ins- 1 2 3 4 5 6
tants 1,9 et 2 est 4 , 9 × (−0 ,1) + 19 , 6 = 19 ,11 m ⋅ s−1. 0 #
-1
Entre les instants 2 et 2,1, elle est égale à : B
-2
4 , 9 × 0 ,1 + 19 , 6 = 20 , 09 m ⋅ s−1. C D
-3
b) La limite de 4 , 9h + 19 , 6 lorsque h tend vers 0 est
19,6, donc la vitesse instantanée de la bille à l’instant
t = 2 est 19 , 6 m ⋅ s−1. 32 Une équation de la tangente à # au point A est :
a)  y = x
1 b)  y = 3 x
26 a)  C(q + 1) = (q + 1)3 + 100
500 c)  y = 2
1 3 d)  y = −5 x + 1
C(q + 1) = (q + 3q2 + 3q + 1) + 100
500
1 3 3 2 3 50 001 33 a) Une équation de la
C(q + 1) = q + q + q+
500 500 500 500 tangente T à # au point A 4
3 2 3 1 d’abscisse -2 est :
b)  C(q + 1) − C(q) = q + q+ T
500 500 500 y = f ′(−2)( x − (−2)) + f (−2),
soit y = 4( x + 2) − 4 , 1
27 On lit sur le graphique : 0
y = 4 x + 4.
a)  h′(0) = 0 b)  h′(1) = −2 -2 1
b) Voir ci-contre.
#
28 On lit sur le graphique :
a)  f(0) = 2 f(2) = 0 f(4 , 5) = 2
-4
1 A
b)  f ′(0) = −4 f ′(2) = f ′(4 , 5) = 0
2
Chapitre 4  ★  Dérivation 59

172909_Chap04_000-000.indd 59 25/07/2019 18:53:15


34 a) Une équation de la tangente T à # au point 1
40 Pour tout nombre réel x ¹ 0, f ′( x) = − 2
d’abscisse 1 est : x
1
y = g ′(1)( x − 1) + g(1), donc f ′(2) = − .
4
soit y = −1( x − 1) + 2, y = −x + 3. Une équation de la tangente au point d’abscisse 2 est
1 1
b)  y = f ′(2)( x − 2) + f (2), soit y = − ( x − 2) + ,
T3 4 2
1
A y = − x + 1.
2 4
#
1 Il s’agit bien de l’équation donnée par le logiciel.
B
0 1 2 3 41 1. a) Pour tout nombre réel x, f ′( x) = 2 x
T passe par A(1 ; 2) et B(3 ; 0).  1 1
donc f ′  = .
 4  2
35 Pour tout nombre réel x, b)  #f

a)  f ′( x) = 0 b)  g ′( x) = 0 c)  h′( x) = 1
1 #g
36 Pour tout nombre réel x, f ′( x) = 3 x2 .
T
L’expression est (3).

1
37 Pour tout x de  * , g ′( x) = − . 0 1 1
x2
4
1 1
a)  g ′(1) = −1 b)  g ′(2) = − 2. a) Pour tout nombre réel x > 0, g ′( x) =
4 1 2 x
 1 donc g ′(1) = .
1 2
c)  g ′(−2) = − d)  g ′  = −4
4  2  b) Voir graphique ci-dessus.
3. a) Les droites T et T ¢ semblent parallèles.
1 b) Les droites T et T ¢ ont la même pente car
38 Pour tout nombre réel x > 0, f ′( x) = .
2 x  1 1
a) L’égalité est vraie. f ′  = g ′(1) = donc elles sont parallèles.
 4  2
b) L’égalité est fausse.
c) L’égalité est vraie. 42 Une équation de la tangente au point d’abs-
cisse 1 à la courbe :
39 a) Pour tout nombre réel x, f ′( x) = 3 x2 # f : y = f ′(1)( x − 1) + f (1), y = 2( x − 1) + 1,
donc f ′(2) = 12. y = 2 x − 1. Il s’agit de T3 .
b) Une équation de la tangente T à # au point d’abs- # g : y = g ′(1)( x − 1) + g(1), y = −1( x − 1) + 1,
cisse 2 est : y = −x + 2. Il s’agit de T1.
y = f ′(2)( x − 2) + f (2), soit y = 12( x − 2) + 8 , 1
# h : y = h′(1)( x − 1) + h(1), y = ( x − 1) + 1,
2
y = 12 x − 16. 1 1
y = x + . Il s’agit de T2 .
c)  2 2
A
8 43 a) Pour tout t de  * , f (t ) = t −3 donc n = −3.
b) Pour tout t de  * , f ′(t ) = −3t −4 .
−3
c) Donc, pour tout t de  * , f ′(t ) = 4 .
t
#
44 Pour tout nombre réel x, f ′( x) = 3 x2 + 2 x.
L’expression est donc (2).

1 45 Pour tout nombre réel x,


T
0 1 2 a)  f ′( x) = 12 x2
b)  g ′( x) = x2
c)  h′( x) = −21x2
60

172909_Chap04_000-000.indd 60 25/07/2019 18:54:28


1 55 Pour tout nombre réel t,
46 Pour tout nombre réel x ¹ 0, f ′( x) = 2 donc
x 2t + 1
f ′( x) > 0. f ′(t ) = − 2
(t + t + 1)2
Tom a raison.
1
56 Pour tout nombre réel x ¹ ,
47 Pour tout nombre réel x, 2
a)  f ′( x) = 2 x + 3 1 1
f( x) = (2 x − 1) + 3 × , donc
b)  g ′( x) = 6 x2 − 6 x + 1 3 2x − 1
c)  h′( x) = 2 x3 − 3 x2 + 7 1  2 
f ′( x) = × 2 + 3 × − 
3  (2 x − 1)2 
48 a) Pour tout nombre réel x,
u ′( x) = 2 x et v ′( x) = 3. 2 6
f ′( x) = −
b) Pour tout nombre réel x, 3 (2 x − 1)2
f ′( x) = u ′( x)v( x) + u( x)v ′( x)
1
f ′( x) = 2 x(3 x − 1) + 3( x2 + 5) 57 Pour tout nombre réel x ¹ ,
4
f ′( x) = 9 x2 − 2 x + 15.
u ′( x) = 7 et v ′( x) = 4.
49 a) Pour tout nombre réel x, 1
b) Pour tout nombre réel x ¹ ,
f ′( x) = 1( x2 + x + 2) + ( x + 1)(2 x + 1) 4
f ′( x) = x2 + x + 2 + 2 x2 + 3 x + 1 7(4 x − 1) − 4(7 x + 5)
g ′( x) =
f ′( x) = 3 x2 + 4 x + 3 (4 x − 1)2
b) Pour tout nombre réel t, −27
g ′( x) =
g ′(t ) = (3t 2 − 1)(2t − 1) + 2(t 3 − t ) (4 x − 1)2
g ′(t ) = 8t 2 − 3t 2 − 4t + 1
58 Pour tout nombre réel x ¹ 1,
50 Pour tout nombre réel x de ]0 ; + ∞[ ,
(2 x + 3)( x − 1) − ( x2 + 3 x)
1 f ′( x) =
f ′( x) = 1× x + x × ( x − 1)2
2 x
1 3 x2 − 2 x − 3
f ′( x) = x + x= x. f ′( x) =
2 2 ( x − 1)2

51 Pour tout nombre réel x de ]0 ; + ∞[ , 59 a) g est la fonction définie sur [0 ; + ∞[ par
x −1 g(X) = X .
f ′( x) = x + donc f ′(1) = 1.
2 x g est dérivable sur ]0 ; + ∞[ et pour tout nombre
La tangente à # au point d’abscisse 1 a pour équation 1
y = f ′(1)( x − 1) + f (1), soit y = 1( x − 1), y = x −1. réel X > 0, g ′(X) = .
2 X
Cette tangente est bien la droite d.
b) f est dérivable en tout nombre réel x tel que
 4 
52 a) Pour tout nombre réel x, u ′( x) = 5. 3 x + 4 > 0. Ainsi f est dérivable sur  − ; + ∞  et
b) Pour tout nombre réel x,  3 
g ′( x) = 2 × 5 × (5 x + 7) = 10(5 x + 7) 4
pour tout nombre réel x tel que x > − ,
3
1 3
53 Pour tout nombre réel x, ′
f ( x) = 3 × ′
, f ( x) = .
a)  f ′( x) = 2 × (−3)(−3 x + 5) = −6(−3 x + 5) 2 3x + 4 2 3x + 4
 1
b)  g ′( x) = 2(2 x + 4) x2 + 4 x −  60 1. B    2. C    3. A
 2

1 61 1. A, C, D     2. B, D    3. A, B, D     4. B


54 a) Pour tout nombre réel x ¹ , v ′( x) = 5.
5
1 62 1. L’affirmation est vraie.
b) Pour tout nombre réel x ¹ ,
5 En effet, pour tout nombre réel x ¹ 0,
v ′( x) 5
g ′( x) = − =− 4
v( x)2 (5 x − 1)2 f ′( x) = −4 x−5 = − 5 .
x

Chapitre 4  ★  Dérivation 61

172909_Chap04_000-000.indd 61 25/07/2019 18:55:40


2. L’affirmation est vraie.

S'entraîner
En effet, pour tout nombre réel x,
f ′( x) = 6 x2 + 6 x − 12.
Donc f ′(−2) = 0 et f ′(1) = 0.
3. L’affirmation est fausse.
1
En effet, pour tout nombre réel x > ,
4
4 2 68 1. a) Pour 0 < h < 1, la pente de la sécante
g ′( x) = = .
2 4x −1 4x −1 (OM) est :
g(h) − g(0) h2 + 2h h(h + 2)
63 h est un nombre réel, h ¹ 0. c= = = = h + 2.
h h h
a) f (1 + h) = 3(1 + h)2 = 3(1 + 2h + h2 ) L’algorithme s’écrit :
f (1 + h) = 3 + 6h + 3h2 .
h ¬1
f (1 + h) − f (1) 3 + 6h + 3h2 − 3
b)  = Tant que h > 0
h h
f (1 + h) − f (1) 3h(h + 2) c ← h+2
= = 3(h + 2). Afficher c
h h
c)  3(h + 2) a pour limite 6 lorsque h tend vers 0 donc h ← h − pas
f est dérivable en 1 et f ′(1) = 6. Fin Tant que
c) Avec pas = 0,1, on obtient :
1
64 Pour tout nombre réel x ¹ 0, f ′( x) = − 2 donc h 1 0,9 0,8 0,7 0,6 0,5 0,4 0,3 0,2 0,1
x
 1 c 3 2,9 2,8 2,7 2,6 2,5 2,4 2,3 2,2 2,1
f ′  = −4.
 2 
2. a) Lorsque h tend vers 0, la droite T position limite
La pente de la tangente T à # au point A est -4 et
des sécantes (OM) est la tangente à la courbe repré-
T est donc parallèle à d.
sentative # de g au point O.
65 a) Pour tout nombre réel x, g ′( x) = 3 x2 donc La limite de la pente c lorsque h tend vers 0 est égale
g ′(1) = 3. à 2 donc une équation de T est y = 2 x
b) Une équation de T est : b) Ce nombre est la pente de T, soit g ′(0) = 2.
y = g ′(1)( x − 1) + g(1), soit y = 3( x − 1) + 2,
y = 3 x − 1. 69 a) Pour h tel que 1 < 1 + h < 2,
1
66 a) Pour tout nombre réel x ¹ 5, −1
f (1 + h) − f (1) 1 + h −h 1
u( x) = −2 x + 3 et v( x) = x − 5. = = =−
h h h(1 + h) 1+ h
b) Pour tout nombre réel x ¹ 5,
u ′( x) = −2 et v ′( x) = 1. b) Le programme calcule et affiche le taux de varia-
c) Pour tout nombre réel x ¹ 5, tion de f entre 1 et 1+ h pour des valeurs de h de
u ′( x)v( x) − u( x)v ′( x) plus en plus proches de 0.
f ′( x) =
(v( x))2 h est initialisée à 1 et diminue de la valeur pas à
−2( x − 5) − (−2 x + 3) chaque passage dans la boucle tant que h > 0.
f ′( x) =
( x − 5)2 c) On obtient les taux de variation successifs avec un
7 pas de 0,01.
f ′( x) =
( x − 5)2
71 a) On conjecture que l’aire du triangle est
constante égale à 2.
b) Si a > 0, l’aire du triangle OAB est égale à
1 2
× 2a × = 2.
2 a
Si a < 0, ’aire du triangle OAB est égale à
1  2
(−2a) × −  = 2.
2  a 

62

172909_Chap04_000-000.indd 62 25/07/2019 18:56:40


72 a)  76 a) Pour tout nombre réel x ¹ 0,
1  1
f ′( x) = − 2 ( x2 + 5) + 1 +  × 2 x
x 
 x
 1  x2 + 5
f ′( x) = 2 x 1 +  −
 x x2
b) Pour tout nombre réel x ¹ 0,
2 x3 + 2 x2 − x2 − 5
f ′( x) =
x2
2 x3 + x2 − 5
f ′( x) =
x2
b) On conjecture que l’aire du triangle OPMQ est
constante égale à 1. 77 a) Pour tout nombre réel x ¹ 2,
c) Si a > 0, l’aire du rectangle OPMQ est égale à 1
f ′( x) = − et
1 ( x − 2)2
a × = 1.
a 4( x − 2) − (4 x − 7) −1
g ′( x) = = .
Si a < 0, l’aire du rectangle OPMQ est égale à ( x − 2)2
( x − 2)2
 1
(−a) × −  = 1. Pour tout nombre réel x ¹ 2, f ′( x) = g ′( x).
 a 
b) Pour tout nombre réel x ¹ 2,
1 4 x − 7 −4 x + 8
73 1. a) La pente de la sécante (OM) est 1. f ( x) − g( x) = − =
x−2 x−2 x−2
b) La limite de ce taux lorsque h tend vers 0 avec
−4( x − 2)
h > 0 est égale à 1. f ( x) − g( x) = = −4.
x−2
2. a) La pente de la sécante (ON) est -1. Alors pour tout nombre réel x ¹ 2,
b) La limite de ce taux lorsque h tend vers 0 avec f ′( x) − g ′( x) = 0, donc f ′( x) = g ′( x).
h < 0 est égale à -1.
f (h) − f (0) 78 Pour tout nombre réel x > −2,
3. Le taux n’a pas de limite lorsque h tend 3( x + 2) − 3 x 6 3
h f ′( x) = 2
= 2
et g ′( x) = x + .
vers 0 donc f n’est pas dérivable en 0. ( x + 2) ( x + 2) 2
3
Alors f ′(0) = g ′(0) = .
74 a) Pour tout nombre réel x > 0, 2
1 1 D’autre part f (0) = g(0) = 0 , les deux courbes ont
f ′( x) = donc f ′(1) = .
2 x 2 donc la même tangente en O.
f (1 + h) − f (1)
b) La limite de est égale à f ′(1) lorsque 79 Pour tout nombre réel x ¹ 3,
h
h tend vers 0. 4(2 x + 1)( x − 3) − (2 x + 1)2
f ′( x) =
Donc pour h proche de 0 avec h ¹ 0, ( x − 3)2
f (1 + h) − f (1) 1 (2 x + 1)[ 4( x − 3) − (2 x + 1)]
≈ . f ′( x) =
h 2 ( x − 3)2
1 (2 x + 1)(2 x − 13)
c) Alors, pour h proche de 0, f (1 + h) − f (1) ≈ h, f ′( x) =
2 ( x − 3)2
1
soit  1 + h ≈ 1 + h.
2 80 Pour tout réel x > 0,
d)  1, 02 ≈ 1 + 0 ,1, 1, 02 » 11
, 1
( x + 1) − x
0 , 996 ≈ 1 − 0 , 002, 1, 02 » 0 , 998 g ′( x) = 2 x
( x + 1)2
( x + 1) − 2 x
75 a) Pour tout nombre réel x ¹ 0, g ′( x) =
1 2 x ( x + 1)2
f ′( x) = 6 x2 − 2 1− x
x g ′( x) =
b) Pour tout nombre réel t > 0, 2 x ( x + 1)2
1 (1 − x) x
g ′(t ) = 1 − g ′( x) =
t 2 x( x + 1)2

Chapitre 4  ★  Dérivation 63

172909_Chap04_000-000.indd 63 25/07/2019 18:57:52


81 Pour tout nombre réel x ¹ 0, 85 a) Pour tout nombre réel x, f ′( x) = 8 x
 1  1  1
f ′( x) = 21 − 2  x +  donc f ′−  = −4.
 x  
 x  2 
x2 − 1 x2 + 1 1
f ′( x) = 2 × × Pour tout nombre réel x ¹ 0, g ′( x) = − donc
x2 x x2
 1
g ′−  = −4.
x −1 4  2 
f ′( x) = 2 ×
x3  1  1
f ′−  = g ′−  , donc les tangentes à # f et # g
 2   2 
82 a) Pour tout nombre réel x, 1
en leurs points d’abscisse - sont parallèles.
f ′( x) = 3 × 2(2 x − 5)2 = 6(2 x − 5)2 2
b) Pour tout nombre réel x, b) Pour tout nombre réel x ¹ 0,
1 1
g ′( x) = 4 × (−3)(7 − 3 x)3 = −12(7 − 3 x)3 f ′( x) = g ′( x) équivaut à 8 x = − 2 , soit x3 = − .
x 8
1 1
c) Pour tout nombre réel x ≠ − , Cette équation admet - pour unique solution, il
2 2
h′( x) = −3 × 2(2 x + 1)−4 = −6(2 x + 1)−4 1
n’existe pas de réel a différent de - tel que les tan-
2
1 gentes à # f et # g en leurs points d’abscisse a soient
83 a) Pour tout nombre réel x > 0, f ′( x) = − 2
x parallèles.
donc f ′(1) = −1.
b) Pour h proche de 0 avec h ¹ 0, 86 Pour tout nombre réel x, f ′( x) = 2 x.
f (1 + h) − f (1) f (1 + h) − f (1) 3
≈ f ′(1), soit ≈ −1 2 x = 3 équivaut à x = .
h h 2
c) Pour h proche de 0 avec h ¹ 0, 3
La tangente à # au point d’abscisse est parallèle à
1 2
f (1 + h) − f (1) ≈ −h, soit − 1 ≈ −h, la droite d.
1+ h
1
≈ 1 − h. 87 Pour tout nombre réel x, f ′( x) = 2 x.
1+ h
2 x = 10 équivaut à x = 5.
1 1
d)  ≈ 1 − 0 , 003, » 0 , 997 La tangente à la courbe représentative de f au point
1, 003 1, 003
d’abscisse 5 a pour équation :
1 1
≈ 1 − (−0 , 000 8), » 1, 000 8 y = f ′(5)( x − 5) + f (5)
0 , 999 2 0 , 999 2 y = 10( x − 5) + 25, y = 10 x − 25.
Il s’agit de la droite d.
84 1. La vitesse du coureur à l’instant t est donnée
1
par v(t ) = d ′(t ) = 10 − t . 88 1. a) Pour tout nombre réel x,
10
f ′( x) = 4 x − 3 donc f ′(2) = 5.
Donc v(t ) diminue proportionnellement au temps
Une équation de la tangente T à # au point d’abs-
de course.
cisse 2 est :
2. a) La vitesse du coureur s’annule à l’instant  :
y = f ′(2)( x − 2) + f (2)
t1 = 100 s. y = 5( x − 2) + 4 , y = 5 x − 6
b) La distance parcourue par le coureur est  :
b) 
1002
d(100) = 10 × 100 − = 500 m.
20
3. a) La vitesse moyenne du coureur pendant cette
500
course est = 5 m ⋅ s−1.
100
t
b) La vitesse du coureur à l’instant 1 est :
2
50 On conjecture que la courbe # est au-dessus de la
v(50) = 10 − = 5 m ⋅ s−1.
10 tangente T.
64

172909_Chap04_000-000.indd 64 25/07/2019 18:59:05


2. a) Pour tout nombre réel x, 93 a) La proposition est vraie.
f( x) − (5 x − 6) = (2 x2 − 3 x + 2) − (5 x − 6) En effet, si pour tout nombre réel x, f ( x) = g( x)
f( x) − (5 x − 6) = 2 x2 − 8 x + 8 alors f ′( x) = g ′( x).
f( x) − (5 x − 6) = 2( x − 2)2 b) La proposition est fausse.
b) Pour tout nombre réel x, 2( x − 2) > 0 2 En effet f et g définies sur  par f( x) = x et
g( x) = x + 1 ont la même fonction dérivée et elles ne
donc f( x) > 5 x − 6.
sont pas égales.
Ainsi la courbe # est au-dessus de la tangente T.
c) La proposition est vraie.
1 En effet, pour tout nombre réel f ( x) = 1 + g( x)
89 Pour tout nombre réel, f ′( x) = − .
x2 donc f ′( x) = g ′( x).
1
f ′( x) = − équivaut à x2 = 2.
2
Cette équation a pour solutions - 2 et 2.
Les coordonnées des points A et B sont :

A  2 ;
2  
 , B− 2 ; −
2 
.
Organiser son raisonnement
 2   2 

90 a) Une équation de la tangente Ta à 3 au point


d’abscisse a est : 94 Pour tout nombre réel a, A(a ; a2 ), H(0 ; a2 ) et
2 2
y = 2a( x − a) + a , soit y = 2a x − a . H′(0 ; − a2 ).
b) A appartient à la tangente Ta si, et seulement si, 2a 2
Une pente de la droite (AH′) est : = 2a.
a
−2 = 2a − a2 , soit a2 − 2a − 2 = 0.
(AH′) passe par A et a pour pente 2a, c’est donc la
Cette équation a pour solutions a′ = 1 + 3 et tangente en A à la courbe #.
a′′ = 1 − 3.
c) Les équations des tangentes à 3 qui passent par A 95 a) Pour tout nombre réel x,
sont : C′( x) = 3 x2 − 200 x + 3 000.
( )
Ta¢ : y = 2 1 + 3 x − 4 − 2 3 b) Pour tout nombre réel x,
C′( x) < 600 équivaut à 3 x2 − 200 x + 2 400 < 0.
( )
Ta¢¢ : y = 2 1 − 3 x − 4 + 2 3 Les racines du polynôme 3 x2 - 200 x + 2 400 sont

x1 =
(
20 5 + 7 )≈ 50 , 97 et
91 1. Une équation T est :
3
y = 2( x − 1) + 1, y = 2 x − 1.
2. a) Pour h ¹ 0, x2 =
(
20 5 − 7 ) ≈ 15, 69
3
yM = (1 + h)2 et yP = 2(1 + h) − 1 = 1 + 2h.
La production de l’entreprise doit être comprise entre
b) Pour h proche de 0, yM » yP , c’est-à-dire : ces deux valeurs.
(1 + h)2 ≈ 1 + 2h.
96 a) Pour tout nombre réel x,
3. (1, 001)2 ≈ 1 + 0 , 002, (1, 001)2 ≈ 1, 002,
x2 + 2 x = −x2 + 6 x − 2 équivaut à
(0 , 995)2 ≈ 1 − 0 , 01, (0 , 995)2 ≈ 0 , 99
2 x2 − 4 x + 2 = 0 , soit 2( x − 1)2 = 0.
L’unique solution de cette équation est x = 1.
92 a) La proposition est vraie. Les deux courbes ont un unique point commun, le
1
En effet, pour tout nombre réel x ¹ 0, f ′( x) = − point A de coordonnées (1 ; 3).
x2 b) f  : x  x2 + 2 x a pour dérivée f ¢ : x  2 x + 2
donc f ′( x) < 0.
donc f ′(1) = 4.
b) La proposition est vraie.
g  : x  −x2 + 6 x − 2 a pour dérivée g ¢ :
En effet, f ′(1) = −1. x  −2 x + 6 donc g ′(1) = 4.
c) La proposition est fausse. f ′(1) = g ′(1) donc les deux courbes ont une tangente
En effet, f ′(−0 , 5) = −4 et f ′(−0 , 5) < −1. commune au point A.

Chapitre 4  ★  Dérivation 65

172909_Chap04_000-000.indd 65 25/07/2019 19:00:24


97 a) La fonction Taux calcule et renvoie le taux de y = 2 a x − a 2 + 1.
variation de la fonction f  :  x  x2 entre a et a + h Une équation de la tangente Tb¢ à # 2 au point d’abs-
avec h ¹ 0. cisse b est :
b) On saisit le programme, on vérifie son fonctionne- y = g ′(b)( x − b) + g(b)
ment.
y = −2b( x − b) − b2 − 1
c) Pour changer de fonction, on modifie la ligne 2 du
programme. y = −2b x + b2 − 1.
Les tangentes Ta et Tb¢ sont confondues si, et seule-
98 Une équation de la tangente Ta à # au point ment si :
d’abscisse a est : 2a = −2b a = −b
 , soit  2
y = f ′(a)( x − a) + f (a). −a + 1 = b − 1
2 2 a = 1
 
Ta passe par l’origine du repère si, et seulement si,
On obtient donc deux tangentes communes à #1
−af ′(a) + f (a) = 0, soit
et # 2 .
−a(2a − 4) + a2 − 4 a + 5 = 0 ,
T1 = T−′ 1 tangente à #1 en A(1 ; 2) et à # 2
−a2 + 5 = 0, a2 = 5. en B(−1 ; − 2)
On obtient alors les deux points de # dont les abs- T−1 = T1′ tangente à #1 en A(−1 ; 2) et à # 2
cisses sont - 5 et 5. en B(1 ; − 2)

99 1.  102 Pour x et y réels,


a x2 + (1 − 2a)x + a = y équivaut à
a( x2 − 2 x + 1) + x = y
a( x − 1)2 + x = y
Pour x = 1 et y = 1, l’égalité précédente est vérifiée
pour tout nombre réel a.
2. Pour 0 < t < 5, v(t ) = −8t + 40.
Ainsi toutes les paraboles 3 a passent par le point
v ′(0) = 40, v ′(2) = 24, v ′(4) = 8 et v ′(5) = 0.
E(1 ; 1).
3. a) Voir le graphique.
La fonction f : x  a x2 + (1 − 2a)x + a a pour déri-
La pente de T est la vitesse à l’instant t = 0.
vée f ¢ : x  2a x + 1 − 2a et f ′(1) = 1.
b) La distance parcourue aurait été égale à :
Les paraboles 3 a ont ainsi une tangente commune
40 × 5 = 200 m.
en E, cette droite passe par E et a pour pente 1.
Il s’agit de l’ordonnée du point de T qui a pour abs-
cisse 5.
103 La courbe # passe par le point A donc f(0) = 1
100 Pour tout nombre réel x, et par le point B(2 ; 5) donc f(2) = 5.
D’après le graphique, la droite d a pour pente 1
f ′( x) = −4 x3 + 4 x + 1.
donc f ′(0) = 1.
f(−1) = 0 et f ′(−1) = 1 donc une équation de la tan-
gente T à # au point A est : f (0) = 1

y = x + 1. On résume les conditions de l’énoncé par : f ′(0) = 1

On étudie l’intersection de # et T : f (2) = 5
Pour tout nombre réel x,
Pour tout nombre réel x, f ( x) = a x2 + b x + c
f( x) = x + 1 équivaut à x 4 − 2 x2 + 1 = 0 , soit
et f ′( x) = 2a x + b.
( x2 − 1)2 = 0 , x2 − 1 = 0.
Le système précédent s’écrit :
# et T se coupent en deux points  : A(−1 ; 0) et 

B(1 ; 2). c = 1 c = 1

D’autre part f ′(1) = 1 donc T est aussi tangente à # b = 1 , soit b = 1
 
au point B(1 ; 2).
4 a + 2b + c = 5  1
a =
 2
101 Une équation de la tangente Ta à #1 au point Finalement, pour tout nombre réel x,
d’abscisse a est : y = f ′(a)( x − a) + f (a) 1
f( x) = x2 + x + 1.
y = 2a( x − a) + a2 + 1 2
66

172909_Chap04_000-000.indd 66 25/07/2019 19:01:53


104 Pour tout nombre réel x, f (h) − f (0)
Lorsque h tend vers 0, a pour limite 0
g ′( x) = 3 p x2 + 2q x. h
La courbe de g a une tangente au point de coordon- donc f est dérivable en 0 et f ′(0) = 0.
nées (1 ; − 1) parallèle à l’axe des abscisses si, et seu-
lement si, 108 a) Si f est paire alors pour tout nombre réel x,
g(1) = −1  p + q = −1 f (−x) = f ( x) donc −f ′(−x) = f ′( x),
 , soit 
g ′(1) = 0 3 p + 2q = 0 soit f ′(−x) = −f ′( x) et la fonction f ¢ est impaire.

 b) Si f est impaire alors pour tout nombre réel x,


 p − 3 p = −1 f (−x) = −f ( x) donc −f ′(−x) = −f ′( x),
 2  p = 2
, 
 3 q = −3 soit f ′(−x) = f ′( x) et la fonction f ¢ est paire.
q = − p
 2
109 1. a) Cette tangente a pour pente 0 donc
105 a) La pente de la tangente T est : f ′(0) = 0.
yB − y A 2 300 − 1200 Pour x ∈ [−2 ; 2], f ′( x) = 2a x + b donc f ′(0) = b et
= = 110.
xB − xA 30 − 20 ainsi b = 0.
b) Le coût marginal de production de la 21e tonne de f (0) = 2 c = 2
médicament est assimilé à C′(20), soit à la pente de b) Le système  équivaut à 
f (2) = 1, 5 4 a + c = 1, 5
la tangente T.
c = 2
Il s’élève donc à 110 milliers d’euros. soit 
a = −0 ,125
106 a) Pour tout nombre réel x, f ′( x) = 3 x2 − 3 donc Donc, pour x ∈ [−2 ; 2],
 1 9
f ′  = − . f( x) = −0 ,125 x2 + 2.
 2  4
 1 3 2. a) Pour x ∈ [−2 ; 2],
D’autre part f   = − .
 2  8
f(−x) = −0 ,125(−x)2 + 2
Une équation de T est :
9 1 3 9 3 f(−x) = −0 ,125 x2 + 2
y = −  x −  − , soit y = − x +
4  2 8 4 4 f (−x) = f ( x).
b) Pour tout nombre réel x,
Donc f est une fonction paire.
 9 3  (2 x − 1)2
f( x) − − x +  = ( x + 1) d’après le calcul b) La courbe # est donc symétrique par rapport à
 4 4 4
l’axe des ordonnées.
réalisé.
Pour x ∈]− ∞ ; − 1,] x + 1 < 0 3. 1re méthode
9 3 La surface de la planche est OC × OS = 4 m2
donc f( x) < − x + .
4 4 2e méthode
Pour x ∈ [−1 ; + ∞[ , x + 1 > 0
f(1) = 1, 875
9 3
donc f( x) > − x + . Pour x ∈ [−2 ; 2], f ′( x) = −0 , 25 x
4 4
donc f ′(1) = −0 , 25.
Donc la courbe # est au-dessous de T sur l’intervalle
]− ∞ ; − 1] et au-dessus de T sur l’intervalle Une équation de la droite (GH) est :
[−1 ; + ∞[. y = −0 , 25( x − 1) + 1, 875, soit y = −0 , 25 x + 2,125.
On peut remarquer que # et T se coupent aux points G a pour coordonnées (0 ; 2,125) et H a pour coor-
1 3
de coordonnées (−1 ; 3) et  ; − . données (2 ; 1, 625).
 2 8
L’aire du trapèze OCHG est donc :
107 Pour h > 0,
2,125 + 1, 625
× 2 = 3, 75 m2 .
f (h) − f (0) h h 2
= = h.
h h La forme la plus économique est celle du trapèze.

Chapitre 4  ★  Dérivation 67

172909_Chap04_000-000.indd 67 25/07/2019 19:03:06


112 Pour une production de q composants

Exploiter ses compétences


(0 < q < 200) le coût moyen est :
C(q) 1 200
= q2 +
q 3 000 q
Avec un traceur de courbes, on trace les courbes
C(q)
représentatives des fonctions q  et q  8.
110 Les informations de l’énoncé se traduisent par : q
f (0) = 0 C(q)
 On obtient < 8 pour 26 < q < 140.
f ′(0) = 0 q
 1 2
f (2) = 3 Le coût marginal est donné par C′(q) = q pour
 5 1000
f ′(2) = 0 < q < 200.
 2
C′(q) < 8 équivaut à q2 < 8 000 , c’est-à-dire q < 89.
car la pente de la tangente (OC) au raccordement en
La quantité q de composants à produire doit donc
O est égale à 0 et celle de la tangente (AB) en A au
5 être telle que 26 < q < 89.
raccordement est .
2
113 On note a l’abscisse du point M (a > 0).
Pour tout nombre réel x, f ′( x) = 3a x2 + 2b x + c et le
M a pour coordonnées (a ; 0) et H a pour coordon-
système s’écrit :
d = 0 d = 0 nées (0 ; 26).
  26
c = 0 c = 0 La pente de la droite (HM) est - et une équation
  a
8a + 4b + 2c + d = 3 , soit 8a + 4b = 3 26
  de cette droite est y = − x + 26.
 5  5 a
12a + 4b + c = 12a + 4b =
 2  2 On étudie l’intersection de la droite (HM) et de la
Ce système est équivalent : parabole # d’équation y = −x2 + 25.
d = 0  1 Pour cela, on résout l’équation :
 a = − 8 26
c = 0  −x2 + 25 = − x + 26 , c’est-à-dire :
 , soit b = 1 a
 8 a + 4 b = 3  26
 1 c = 0 x2 − x +1= 0
4 a = −  a
 2 d = 0
 26 2 676 − 4 a2
La fonction f est définie sur [0 ; 2] par : On obtient ∆ = −  − 4 =
 a  a2
1
f( x) = − x3 + x2 . 4(169 − a2 )
8 ∆= .
a2
111 Lorsque le caillou atteint la surface de l’eau • Si 0 < a < 13 alors ∆ > 0, (HM) et # se coupent
40 en deux points.
h(t ) = 0, soit 40 − 4 , 9t 2 = 0 , t2 = et
4, 9 • Si a = 13 alors ∆ = 0, (HM) et # se coupent en un
40
t= (car t > 0 ). seul point.
4, 9
• Si a > 13 alors ∆ < 0, (HM) et # ne se coupent pas.
La durée de la chute est donc t » 2, 9 s.
Donc l’observateur doit se placer à une distance mini-
La distance parcourue par le caillou à l’instant t est
male de 13 − 5 = 8 m du pied du terril pour aperce-
d(t ) = 4 , 9t 2 , la vitesse instantanée du caillou à l’ins-
voir l’extrémité du bâton.
tant t est donnée par v(t ) = 4 , 9 × 2t = 9 , 8t .
Remarque  : lorsque a = 13, la droite (HM) est la
Donc la vitesse du caillou lorsqu’il atteint la surface de
tangente à # au point d’abscisse 1.
l’eau est :
 40 
v   = 9 , 8 × 40 = 28 m ⋅ s−1.
 4 , 9  4, 9

68

172909_Chap04_000-000.indd 68 25/07/2019 19:04:13


5 Applications
de la dérivation

c) On constate que le prix de l’action est décrois-

Découvrir
sant sur les intervalles où f ′(t ) < 0 et que le prix
de l’action est croissant sur l’intervalle où
f ′(t ) > 0.

2  Signe de la dérivée et maximum


1 Signe de la dérivée
et sens de variation 1  a)  h(0) = 2 donc le poids est lancé à 2 m de hau-
teur.
1  a)  f(0) = 500 et f(30) = 680. b) On peut conjecturer que la hauteur maximale
L’action coûte 500 € en début d’étude et 680 € en atteinte par le poids est 5,2 m et que le poids se
fin d’étude. trouve à la distance horizontale de 8  m de ses
b)  pieds.

2  a) h est dérivable sur [0 ; + ∞[


et h′( x) = −0 ,1x + 0 , 8.
h′( x) = 0 si et seulement si, x = 8.
Ainsi h′( x) > 0. pour 0 < t < 6 et h′( x) < 0
c) On conjecture que le prix de l’action décroît pour tout x > 6.
sur l’intervalle [0 ; 6 ], puis croît sur l’intervalle b)  h′( x) > 0 sur [0 ; 8 ] donc h est croissante sur
[6 ; 24 ] et enfin décroît sur l’intervalle [24 ; 30 ]. [0 ; 8 ].
h′( x) < 0 pour x > 8 donc h est décroissante
2  a) f est une fonction polynôme donc f est déri- sur [8 ; + ∞[.
vable sur l’intervalle [0 ; 30 ] et pour tout nombre
h admet donc un maximum en x = 8.
réel t de l’intervalle [0 ; 30 ], c)  h(8) = 5, 2 donc la hauteur maximum atteinte
f ′(t ) = −t 2 + 30t − 144. par le poids est 5,2 m.
∆ = 302 − 4 × (−1) × (−144) = 324
3  h( x) = 0 si et seulement si
et ∆ = 18
−0 , 05 x2 + 0 , 8 x + 2 = 0.
−30 − 18 −30 + 18
t1 = = 24 et t2 = = 6. 26
2 × (−1) 2 × (−1) ∆= = 1, 04 x0 » 18 , 2 et x1 ≈ −2, 2.
25
b) Le signe de f ′(t ) est celui d’une fonction poly- La longueur du lancer de l’athlète est environ
nôme du second degré, on déduit donc le 18 m.
tableau de variations de f.
t 0 6 24 30
f ¢(t) - 0 + 0 -
500 1 076
f(t)
104 680

Chapitre 5  ★  Applications de la dérivation 69

172909_Chap05_000-000.indd 69 30/07/2019 14:47:11


On utilise le signe de a x2 + b x + c pour dresser le
Acquérir des automatismes tableau de variations de la fonction m.
x −∞ -2 2 +∞
m ¢(x) - 0 + 0 -
16
3 Par lecture graphique : m(x)
• pour tout nombre réel x de [−2 ; − 1] et de [1 ; 2], -16
g ′( x) > 0. Donc g est croissante sur chacun des inter-
m(−2) = 8 − 24 = −16.
valles [−2 ; − 1] et [1 ; 2].
Ici m est impaire donc m(2) = −m(−2) = 16.
• pour tout nombre réel x de [−1 ; 1] et [2 ; 3],
g ′( x) < 0. Donc g est décroissante sur chacun des
10 g est une fonction polynôme donc elle est déri-
intervalles [−1 ; 1] et [2 ; 3].
vable sur .
4 a) h est une fonction polynôme donc h est déri- Pour tout nombre réel x,
vable sur  et pour tout x, h′( x) = 3 x2 + 3. g ′( x) = 3 x2 − 6 x − 9 = 3( x2 − 2 x − 3)
Pour tout nombre réel x, 3 x2 + 3 > 0 donc ∆ = (−2)2 − 4 × 1× (−3) = 16 et ∆ > 0.
h′( x) > 0. −(−2) + 4 −(−2) − 4
b) Ainsi, la fonction h est strictement croissante sur x1 = = 3 et x2 = = −1
2 2
.
Le signe de a x2 + b x + c permet de dresser le
tableau de variations de la fonction g.
5 a)  k( x) = −3 x3 − 20 x
k est une fonction polynôme donc k est dérivable sur x −∞ -1 3 +∞
 et pour tout x, k ′( x) = −9 x2 − 20. g ¢(x) + 0 - 0 +
Pour tout nombre réel x, −9 x2 − 20 < 0 donc 10
k ′( x) < 0. g(x)
b) Ainsi, la fonction k est décroissante sur . -22
c)  498 < 501, 5 donc k(498) > k(501, 5).
D’après ce tableau, g(−1) = 10 est un maximum local
8 k est une fonction polynôme donc k est déri- de la fonction g et g(3) = −22 est un minimum local
de g.
vable sur .
Pour tout nombre réel x,
11 h est une fonction polynôme donc elle est déri-
k ′( x) = 3 x2 − 9 = 3( x2 − 3) = 3( x + 3 )( x − 3 )
vable sur .
k ′( x) = 0 si, et seulement si, x = 3 ou x = − 3.
Pour tout nombre réel x, h′( x) = −x2 + 8 x − 7
On utilise le signe de a x2 + b x + c pour dresser le
tableau de variations de la fonction k. ∆ = 82 − 4 × (−1) × (−7) = 36 et ∆ > 0.
x −∞ - 3 3 +∞ −8 + 6 −8 − 6
x1 = = 1 et x2 = =7
k ¢(x) + 0 - 0 + 2 × (−1) 2 × (−1)
Le signe de a x2 + b x + c permet de dresser le
6 3
k(x) tableau de variations de la fonction h.
-6 3 x −∞ 1 7 +∞
h ¢(x)
k ( 3) = 3 3 − 9 3 = −6 3 .
- 0 + 0 -
98
(
Ici k est impaire donc k − 3 = −k ) ( 3) = 6 3. h(x) 10 3
-
3
9 m est une fonction polynôme donc m est déri-
vable sur . 10
D’après ce tableau, h(1) = − est un minimum local
Pour tout nombre réel x, 3
98
m′( x) = −3 x2 + 12 = −3( x2 − 4) = −3( x + 2)( x − 2) de la fonction h et h(7) = est un maximum local
3
m′( x) = 0 si, et seulement, x = 2 ou x = −2. de h.
70

172909_Chap05_000-000.indd 70 30/07/2019 14:48:18


14 g est la somme de deux fonctions dérivables sur 16 f ′( x) est positif sur les intervalles [−3 ; 1] et
[−5 ; − 2] donc g est dérivable sur [−5 ; − 2]. Pour [3 ; 4 ].
1
tout nombre réel x, g ′( x) = 4 + 2 . Pour tout f ′( x) est négatif sur l’intervalle [1 ; 3].
x
nombre réel x, g ′( x) > 0. 17 f ′(−2) est positif et f ′(3) est négatif.
x -5 -2
18 Faustine a tort car f est croissante sur l’intervalle
g ¢(x) +
[−1 ; 0 ] (f ′( x) > 0 ) et f est décroissante sur [0 ; 1]
-7, 5 (f ′( x) < 0 ).
g (x)
-19 , 8 19 f ′( x) > 0 sur  donc f est strictement crois-
sante sur .
On obtient d’après le tableau de variations que
g(−5) = −19 , 8 est le minimum de g sur [−5 ; − 2]
20 Pour tout nombre réel x, x2 > 0 donc f est
et g(−2) = −7, 5 est le maximum de g donc pour
croissante sur .
tout réel x de [−5 ; − 2], −19 , 8 < g( x) < −7, 5.
L’affirmation (2) est donc vraie.
15 a) h est une fonction polynôme de second
21
degré donc pour tout t, h′(t ) = −4t + 5.
5 x −∞ -2 1 +∞
h′(t ) = 0 si et seulement si, −4t + 5 = 0 soit t = .
4 f ¢(x) + 0 - 0 +
h¢ est une fonction affine  ; on complète le tableau 3
ci-dessous. f(x)
5 1
t −∞ +∞
4
h ¢(t) + 0 - 22
1 x -5 -3 2 3 6
h (t) 8
f¢(x) - 0 + 0 - 0 +

b) L’équation −2t 2 + 5t − 3 = 0 a une solution évi-


23 a) On peut conjecturer que les entiers -1 et 2
c 3
dente, 1, et sa seconde racine est = sont solutions de l’équation g ′( x) = 0
a 2
b) On peut conjecturer le signe de g ′( x) dans le
 5 1
On place d’abord le sommet S ;  , puis les points tableau ci-dessous.
 4 8 
x -3 -1 2 4
d’intersection avec l’axe des abscisses A(1 ; 0) et
3  g¢(x) + 0 - 0 +
B ; 0. On peut placer un autre point C(0 ; − 3).
 2 
5
Par symétrie par rapport à la droite d’équation x =
4 24 a)  h(0) < 0 d)  h′(−3) < 0
5 
on place aussi D ; − 3. b) h′(0) > 0 e)  h(2, 5) > 0
 2 
c)  h(−3) > 0 f)  h′(−2, 5) < 0

25 f ′( x) < 0 sur l’intervalle [−2 ; − 1] donc f est


décroissante sur l’intervalle [−2 ; − 1].
f ′( x) > 0 sur l’intervalle [−1 ; 4 ] donc f est croissante
sur l’intervalle [−1 ; 4 ].

26 a) • f est décroissante sur les intervalles


[−5 ; − 3] et [1 ; 5] donc f ′( x) < 0 sur ces intervalles.
• f est croissante sur l’intervalle [−3 ; 1] donc f ′( x) > 0
sur l’ intervalle [−3 ; 1].

Chapitre 5  ★  Applications de la dérivation 71

172909_Chap05_000-000.indd 71 30/07/2019 14:50:16


b)  c)  x( x + 4) = 0 si et seulement si x = 0 ou x = −4.
3 On obtient le tableau de signes ci-dessous
2
x −∞ -4 -2 0 +∞
1
g ¢(x) + 0 - - 0 +
-5 -3 0 1 5
-8
g (x)
0
-4 d) 

27 • f est croissante sur les intervalles [−3 ; − 2] et


[0 ; 2] donc f ′( x) > 0 sur les intervalles [−3 ; − 2] et
[0 ; 2].
Seule la courbe C vérifie cela donc on fait corres-
pondre  à C .
• g est croissante sur les intervalles [−3 ; − 1] et [1 ; 2]
donc g ′( x) > 0 sur les intervalles [−3 ; − 1] et [1 ; 2]. -6 < X < 6 , pas 1 ; -20 < Y < 10 pas 2
Seule la courbe A vérifie cela donc on fait corres-
pondre  à A . 2 x( x2 + 10) − x2 × 2 x
31 a)  h′( x) =
• Finalement  correspond à B . ( x2 + 10)2
20 x x
= = 20 ⋅ 4 .
28 a) h est dérivable sur  et pour tout x, ( x2 + 10)2 x + 20 x2 + 100
1 128
h′( x) = x2 − 6 x + . b) Pour tout nombre réel x, ( x2 + 10)2 > 0 donc le
3 3
188 signe de h′( x) est celui de 20x.
∆=− < 0 donc l’équation h′( x) = 0 n’a pas de 20 x = 0 si et seulement si x = 0.
9
solutions. x −∞ 0 +∞
1
Le signe de h′( x) est donc celui de a = donc pour h ¢(x) - 0 +
3
tout nombre réel x, h′( x) > 0.
b) h′( x) > 0 pour tout x de  donc h est croissante h (x)
sur . 0
Chloé a donc raison.

29 a) Pour tout x ¹ 3, 32 k est dérivable sur  et on a pour tout x,


1⋅ (2 x − 6) − ( x + 1) × 2 −8 k ′( x) = 6 x2 − 24 ,12 x + 24 , 24.
f ′( x) = = .
(2 x − 6)2 (2 x − 6)2 k ′( x) = 0 si et seulement si x = 2 ou x = 2, 02. On
complète le tableau de signes ci-dessous
b) Pour tout x ¹ 3, (2 x − 6)2 > 0 et −8 < 0 donc
f ′( x) < 0. x −∞ 2 2,02 +∞
c) h ¢(x) + 0 - 0 +
x −∞ 3 +∞
4,24
f ¢(x) - -
h(x)
4,239 992
f (x)
Pedro a donc tort.

33 6 est un maximum local et -4 est un minimum


30 a) Pour tout nombre réel x ≠ −2,
local.
2 x( x + 2) − x2 × 1 x2 + 4 x x( x + 4)
g ′( x) = = =
( x + 2)2 ( x + 2)2 ( x + 2)2 34 g ′( x) s’annule en -1 en changeant de signe
2
b) Pour tout nombre réel x ≠ −2, ( x + 2) > 0 donc donc g admet en x = −1 un extremum local, ici par
le signe de g ′( x) est celui de x( x + 4). lecture du tableau il s’agit d’un maximum.
72

172909_Chap05_000-000.indd 72 30/07/2019 14:52:11


35 a) (3)            b) (2)            c) (2) On peut conjecturer que f admet deux extremums en
x = −2 et x = 2.
36 a)  f(0) = 2 et f ′(0) = 0. b) Pour tout x de [−2 ; 2], f ′( x) = 4 x2 − 16
b) Cette affirmation est vraie car f ′( x) = 3 x2 s’annule f ′( x) = 0 si et seulement si x = − 2 ou x = 2
en 0 sans changer de signe.
x -3 -2 2 3

37 f ′( x) = 6 x2 − 6 x − 36. f ¢(x) + 0 - 0 +
b)  f ′( x) = 0 si et seulement si x = 3 ou x = −2 64 -12
x −∞ -2 3 +∞ f(x) 3 64
-
f ¢(x) + 0 - 0 + 12 3
45 4 64
f(2) = × 23 − 16 × 2 = .
f(x) 3 3
64
-80 f est impaire donc f (−2) = −f (2) =
3
c) 45 est un maximum local de f et -80 est un mini-
40 a) Fenêtre -1 < X < 2, pas 0,5
mum local de f.
-2 < Y < 5, pas 1
38 a) Fenêtre -6 < X < 6 , pas 1
-0 , 5 < Y < 0 , 5 pas 0,5

On peut conjecturer que f admet un extremum local


4
(un minimum) en x = 1 égal à - .
On peut conjecturer que 0,5 est un maximum local et 3
que -0 , 5 est un minimum local. b) Pour tout x de [−1 ; 2], f ′( x) = 4 x3 − 4 x2
b) g est dérivable sur  et pour tout x, f ′( x) = 4 x2 ( x − 1)
2( x2 + 4) − 2 x × 2 x 2(4 − x2 ) f ′( x) = 0 si et seulement si x = 0 ou x = 1
g ′( x) = = 2
( x2 + 4)2 ( x + 4)2 Le signe de f ′( x) est celui de x - 1 car 4 x2 > 0.
2(2 − x)(2 + x) x
g ′( x) = . -1 0 1 2
( x2 + 4)2
f ¢(x) - 0 - 0 +
Pour tout nombre réel x, ( x2 + 4)2 > 0 donc le signe
4 13
de g ′( x) est celui de 2(4 − x2 ). f(x) 3 4 3
2(4 − x2 ) = 0 si et seulement si x = 2 ou x = −2. -
3
x −∞ -2 2 +∞ 4
f(1) = − est un minimum local de f sur [−1 ; 2].
g ¢(x) - 0 + 0 - 3
0,5 41 a) Fenêtre -4 < X < 0 , pas 1
g(x) -10 < Y < 6 , pas 1
-0 , 5
D’après le tableau g admet bien pour extremum
locaux -0 , 5 et 0,5.

39 a) Fenêtre -3 < X < 3, pas 1


-25 < Y < 25, pas 5 Il semblerait que f admet un maximum local en
x = −2 égal à 6.
4
b) Pour tout x de [−4 ; 0[ f ′( x) = − 2 + 1
x
x2 − 4 ( x − 2)( x + 2)
f ′( x) = = .
x2 x2

Chapitre 5  ★  Applications de la dérivation 73

172909_Chap05_000-000.indd 73 30/07/2019 14:54:19


Pour tout x de [−4 ; 0[ , x2 > 0 donc le signe de c) D’après b), 0 est le minimum local de f sur [0 ; + ∞[.
f ′( x) est celui de x2 - 4. d) D’après c), pour tout x > 0, f( x) > 0 c’est-à-dire
Sur [−4 ; 0[ x2 − 4 = 0 si et seulement si x = −2. x3 − 3 x + 2 > 0 ce qui donne x3 > 3 x - 2.

x -4 -2 0 b
45 − = 2.
f ¢(x) + 0 - 2a
6 f est décroissante sur l’intervalle ]− ∞ ; 2] et f est
f (x) croissante sur l’intervalle [2 ; + ∞[.
5
46 Elle se trompe, ce n’est pas le signe de b = −5
qui permet de donner la nature de l’extremum mais
42 a) g est dérivable et g ′( x) = 3 x2 − 18 x + 24.
celui de a = 2. a > 0 donc la fonction f admet un
g ′( x) = 0 si et seulement si x = 2 ou x = 4.
minimum sur .
x −∞ 2 4 +∞
g¢(x) + 0 - 0 + b −4
47 − =− = −2 et f(−2) = 5
17 2a −2
g(x) donc réponse (3).

13
48 f ′( x) = 2 x − 4 et f ′( x) = 0 pour x = 2
b) D’après a), g admet un maximum local égal à 17 en
x = 2 et un minimum local égal à 13 en x = 4. f ′( x) > 0 sur l’intervalle [2 ; + ∞[ et f ′( x) < 0 sur
l’intervalle ]− ∞ ; 2].
c) Sur l’intervalle [2 ; + ∞[ g admet 13 pour mini-
mum local, donc pour tout x > 2, g( x) > 13.
49 f( x) = x2 + x − 2 est une fonction polynôme
2
43 a)  h′( x) = 2 x − 3 x = x(2 − 3 x) de degré 2 donc sa courbe représentative dans un
2 repère est une parabole.
h′( x) = 0 si et seulement si x = 0 ou x = .
3 Iheb a donc raison.
2
x 0 1
3 50 A et B ont la même ordonnée et
h¢(x) 0 + 0 - xA + xB 3 +1
= = 2 = xS
4 2 2
h(x) 27 Lison a donc raison.
0 0
1
b) 0 est le minimum de h sur l’intervalle [0 ; 1], donc 51 a) f admet un minimum sur  car a = et
3
h( x) > 0 sur [0 ; 1.] a > 0.
c) La position relative, dans un repère, de la courbe b −4
Il est obtenu en − = = −6 et sa valeur est
représentative de la fonction x  x2 et de la courbe 2a 2 × 1
3
représentative de la fonction x  x3 est donnée par f(−6) = −12
le signe de x2 - x3 c’est-à-dire de h( x). Or d’après b), 3
b)  a = − et a < 0 donc l’extremum local est un
pour tout x de l’intervalle [0 ; 1], h( x) > 0 donc 2
x2 > x3 . Ainsi la courbe de la fonction carrée est maximum, il est obtenu pour
b −4 4 4 8
située au-dessus de celle de la fonction cube sur [0 ; 1]. x=− = = et sa valeur est f   = .
2a  3 3  3  3
2 × − 
44 a)  f ′( x) = 3 x2 − 3  2 
b) Sur [0 ; + ∞[ f ′( x) = 0 si et seulement si x = 1 c)  f( x) = ( x − 5)2 est la forme canonique de f( x). Ici,
x 0 1 a = 1 et a > 0 donc l’extremum local est un minimum,
+∞
il est obtenu en x = −5 et sa valeur est f(5) = 0.
f ¢(x) - 0 +
d)  f( x) = −3( x + 1)2 est la forme canonique de f( x).
Ici, a = −3 et a < 0 donc l’extremum local est un
f (x)
maximum, il est obtenu en x = −1 et sa valeur est
0 f(−1) = 0.
74

172909_Chap05_000-000.indd 74 30/07/2019 14:55:39


52 a) f est une fonction polynôme donc f est déri- 54 a) f est une fonction polynôme du second degré
vable sur . avec a = −2.
Pour tout nombre réel x, f ′( x) = −4 x − 5. a < 0 donc f admet un maximum sur , il est obtenu
5 b 9  9  25
f ′( x) = 0 si, et seulement si, x = − . pour x = − = et sa valeur est f   =
4 2a 4  4  8
5  7
x −∞ - +∞ 25
4 b) • Pour tout x de l’intervalle 1 ;  , f( x) <
 2  8
f ¢(x) + 0 -
7
49 7  1+
• f(1) = 0 et f   = 0 et 2 = 9 donc f( x) > 0
f (x) 8  2  2 4
 7
sur l’intervalle 1 ;  .
 5  49  2 
f −  =
 4  8 25
Ainsi 0 < f( x) < et donc 0 < 8f( x) < 25
b) f( x) = 0 si, et seulement si, −2 x2 − 5 x + 3 = 0. 8
1
∆ = 49, x1 = −3 et x2 = . 55 a) g est une fonction polynôme du second
2
L’ensemble des solutions de l’équation f( x) = 0 est degré donc g est dérivable sur  et pour tout x,
 1 g ′( x) = 2 x − 7.
−3 ; . 7
 2  g ′( x) = 0 si, et seulement, x = . On obtient le
2
c)  49 tableau de variations de g suivant :
––
8
7
x −∞ +∞
2
g ¢(x) - 0 +

g (x)
1
-
3 4
1
b) Pour tout x de l’intervalle ] 3 ; 4 [ , g( x) > −
1 4
donc < − 4.
1
g( x)

2
-4 5
56 a)  T : y = f ′(1)( x − 1) + f (1) avec f ′( x) = 6 x
-–
4 y = 6( x − 1) + 4
y = 6x − 2
b) g( x) = 3 x2 − 6 x + 3
g est dérivable sur  et pour tout x, g ′( x) = 6 x − 6.
-3
g ′( x) = 0 si et seulement si x = 1.
x −∞ 1 +∞

53 #1 est associé à x2 + 4 x + 4 (a = 1 est positif g ¢(x) - 0 +


b
et − = −2)
2a g (x)
3 3
# 2 est associé à x2 − 12 x + 22 (a = est positif 0
b 2 2
et − = 4) c) D’après b), 0 est le minimum de g sur  donc pour
2a
1 1 tout x de , g( x) > 0 donc f( x) > 6 x − 2
# 3 est associé à − x2 + 2 x − 1 (a = − est négatif
b 2 2 La conjecture d’Anaïs est démontrée.
et − = 2)
2a
# 4 est associé à -x2 - 4 x (a = −1 est négatif et 57 1. C            2. C            3. B            4. D
b
− = −2) 58 1. C            2. B, D            3. B, D            4. A, C
2a

Chapitre 5  ★  Applications de la dérivation 75

172909_Chap05_000-000.indd 75 30/07/2019 14:56:48


59 1. Faux. En effet, il y a un minimum en x = −2 2 x( x + 1) − x2 × 1
et un maximum en x = 2 donc f ′(−2) = 0 et 63 a) Pour tout x > 0, f ′( x) =
( x + 1)2
f ′(2) = 0. L’équation f ′( x) = 0 admet donc deux x2 + 2 x x( x + 2)
solutions. f ′( x) = = .
( x + 1)2 ( x + 1)2
2. Faux. En effet f ′(5) < 0 car f est décroissante sur
b) Pour tout x > 0, ( x + 1)2 > 0 , x + 2 > 0, donc
[2 ; 5].
f ′( x) > 0.
3. Vrai. En effet :
• f est croissante sur l’intervalle [0 , 5 ; 2] donc pour
64 a) f est dérivable sur  et pour tout x,
tout x de [0 , 5 ; 2], f ′( x) > 0 donc f ′(1, 5) > 0.
f ′( x) = 4 x3 + 4 x2
• f est décroissante sur [2 ; 5] donc pour tout x de
b) f ′( x) = 4 x2 ( x + 1).
[2 ; 5] f ′( x) < 0 donc f ′(4) < 0.
f ′( x) = 0 si, et seulement si, x = 0 ou x = −1.
• f ′( x) = 0 pour x = 2.
Ainsi f ′(1, 5) > 0 et f ′(4) < 0. x −∞ -1 0 +∞
2
4. Vrai. En effet la dérivée f ¢ de f s’annule en -2 et 2 4x + + 0 +
en changeant de signe. x+1 - 0 + +
f¢(x) - 0 + 0 +
60 a)  f ′( x) = 3 x2 + 1
Pour tout nombre réel x, 3 x2 + 1 > 0 donc f ′( x) > 0. c)
b) g ′( x) = −6 x2 − 4 = −(6 x2 + 4) x −∞ -1 0 +∞
Pour tout nombre réel x, 6 x2 + 4 > 0 donc
g ′( x) < 0. f(x)
4 4
c)  h′( x) = 1 + 2 -
3
x
Pour tout nombre réel x ¹ 0, x2 > 0 donc h′( x) > 0.
65 a)  N′(t ) = −4t 3 + 64t = 4t(−t 2 + 16)
61 a)  f ′( x) = 6 x2 + 6 x − 36 = 4t(4 − t )(4 + t )
b) f ′( x) = 0 si et seulement si x = −3 ou x = 2. Sur [0 ; + ∞[ , N′(t ) = 0 si et seulement si, t = 0 ou
c)  f ¢ est une fonction polynôme du second degré qui t = 4.
s’annule en -3 et 2. t 0 4 +∞
x −∞ -3 2 +∞ 4t 0 + +
f¢(x) + 0 - 0 + 16 - t 2
+ 0 -
N¢(t) 0 + 0 -
62 a)  f ′( x) = 4 x3 − 12 x
b) D’après le signe de N′(t ) on peut établir les varia-
b) f ′( x) = 4 x( x2 − 3) = 4 x( x − 3 )( x + 3 )
tions de N sur l’intervalle [0 ; + ∞[.
f ′( x) = 0 si et seulement si x = 0 ou x = 3 ou
t 0 4 +∞
x = − 3.
356
x −∞ - 3 0 3 +∞
N(t)
4x - - 0 + +
100
x2 - 3 + 0 - - 0 +
Le nombre de bactéries a donc augmenté sur les
f ¢(x) - 0 + 0 - 0 + 4 premières heures et diminué au-delà.

76

172909_Chap05_000-000.indd 76 30/07/2019 14:57:58


On obtient 8 points de G à coordonnées entières

S'entraîner indiqués ci-dessous :

67 a)  g(1016) = 4 et g(0) = −250.


Pour tout x > 0, g( x) < 5.
Ainsi, l’ordonnée des points d’abscisses supérieures à
1016 est comprise entre 4 et 5 donc les points à
coordonnées entières de # ont une abscisse infé-
rieure ou égale à 1016 dans [0 ; + ∞[ donc com- 70 a) On conjecture que l’aire du rectangle est
prise entre 0 et 1016. maximale pour une valeur de a environ égale à 1,8.
b) 
b) On note x = OA et S( x) l’aire du rectangle ABCD
en fonction de x (0 < x < 3).
L’ ordonnée de B est g( x) car B appartient à #.
De plus, 3 est symétrique par rapport à l’axe des
8
ordonnées donc S( x) = 2xg( x) = − x3 + 8 x.
9
On obtient 21 points de # à coordonnées entières La fonction S est dérivable sur [0 ; 3] et pour tout
indiqués ci-dessous. 8
nombre réel x de cet intervalle, S′( x) = − x2 + 8
3
8 2
Sur [0 ; 3], − x + 8 = 0 si et seulement si, x2 = 3
3
c’est-à-dire x = 3.
On dresse le tableau de variations de S.

x 0 3 3
S¢(x) + 0 -

16 3
S(x) 3
     0 0

( 3 ) = − 89 ( 3 ) 16 3
3
68 a)  h(−105) = 1 et h(4) = −108 S +8 3 = .
Pour tout x < 5, h( x) < 2. 3
Ainsi, l’ordonnée des points d’abscisses strictement L’aire du rectangle est donc maximale lorsque l’abs-
inférieures à 5 et supérieures ou égales à -105 est cisse du point A est 3.
strictement inférieure à 2 donc les points à coordon-
nées entières de G ont une abscisse comprise entre 71 a) On conjecture que l’aire du triangle ABC rec-
-105 et 4. tangle en B est maximale lorsque C a pour coordon-
b)   64 
nées 2 ; .
 9 
b) On note x l’abscisse de B (-6 < x < 6 ) et S( x)
l’aire du triangle ABC en fonction de x.
Le point C a pour coordonnées ( x ; h( x)) car C appar-
tient à #.
1 1
S( x) = AB × BC = ( x + 6) × h( x)
2 2
1  2 
S( x) = ( x + 6)− x2 + 8.
2 
 9 

Chapitre 5  ★  Applications de la dérivation 77

172909_Chap05_000-000.indd 77 30/07/2019 14:58:34


1 1  2  Sur les intervalles ]− ∞ ; − 1] et [0 ; 1],
S( x) = AB × BC = ( x + 6)− x2 + 8
2 2 
 9  f ( x) − g( x) < 0 donc # f est au-dessous de # g sur
ces intervalles.
1 2  1  4 
S′( x) = − x2 + 8 + ( x + 6)− x Sur les intervalles [−1 ; 0 ] et [1 ; + ∞[ ,

2 9  2  9 
f ( x) − g( x) > 0 donc # f est au-dessus de # g sur
1 4
S′( x) = − x2 − x + 4. ces intervalles.
3 3
# f et # g se coupent aux points de coordonnées
S′( x) = 0 si et seulement si x = 2 ou x = −6.
(−1 ; − 1), (0 ; 0) et (1 ; 1).
On dresse le tableau de variation de S.
c) On résout l’inéquation f ( x) < g( x).
x -6 2 6 2 x2 + 2 x − 1 < x équivaut à 2 x2 + x − 1 < 0.
S¢(x) 0 + 0 - 2 x2 + x − 1 = 0 si et seulement si x = −1 ou
256 x = 0 , 5.
S(x) 9 L’inéquation a pour ensemble de solution
0 0 6 = [−1 ; 0 , 5].
1  8  256 Ainsi sur l’intervalle [−1 ; 0 , 5] # f est au-dessous de
S(2) = × 8− + 8 = . L’aire du triangle ABC # g et sur les intervalles ]− ∞ ; − 1] et [0 , 5 ; + ∞[ ,
2  9  9
est maximale lorsque l’abscisse de B est égale 2. # f est au-dessus de # g .
# f et # g se coupent aux points de coordonnées
72 a) On pose h la fonction définie sur  pour (−1 ; − 1) et (0 , 5 ; 0 , 5).
h( x) = f ( x) − g( x) = x3 − 3 x − 2.
73 a) Cas h > 0.
h est dérivable sur  et pour tout x,
Pour tous nombres réels x et x + h de I, x + h > x
h′( x) = 3 x2 − 3 = 3( x2 − 1) = 3( x − 1)( x + 1).
donc f ( x + h) > f ( x) car f est croissante sur I.
h′( x) = 0 si et seulement si, x = 1 ou x = −1. f ( x + h) − f ( x)
Ainsi f ( x + h) − f ( x) > 0 donc > 0.
On dresse le tableau de variations de h. h
Cas h < 0.
x −∞ -1 1 2 +∞
Pour tous nombres réels x et x + h de I, x + h < x
h¢(x) + 0 - 0 +
donc f ( x + h) < f ( x) car f est croissante sur I
0 ( x + h) − f ( x)
Ainsi f ( x + h) − f ( x) < 0 donc > 0.
h(x) 0 h
-4 b) Pour tout x de I, le taux d’accroissement de f en
x est pour h ¹ 0 avec x + h appartient à I,
D’après le tableau de variations sur l’intervalle
f ( x + h) − f ( x)
]− ∞ ; 2], h( x) < 0 donc f ( x) < g( x), donc # f est . Ce taux a pour limite f ′( x) lorsque h
h
au-dessous de # g sur ]− ∞ ; 2]. tend vers 0.
Sur l’intervalle [2 ; + ∞[ , h( x) > 0 donc f ( x) > g( x), Ainsi f ′( x) > 0.
donc # f est au-dessus de # g sur [2 ; + ∞[.
# f et # g se coupent aux points de coordonnées 74 Notons x l’une des dimensions du rectangle.
(2 ; 8) et (−1 ; − 1) On définit la fonction p sur ]0 ; + ∞[ telle p( x)
b) On étudie le signe de f ( x) − g( x). Pour tout représente le périmètre du rectangle.
nombre réel x, 100
La deuxième dimension du rectangle est
f ( x) − g( x) = x3 − x = x( x2 − 1) = x( x − 1)( x + 1) x
 100 
Ainsi p( x) = 2 x + 
f ( x) − g( x) = 0 si et seulement si x = 0 ou x = −1  x 
ou x = 1. La fonction p est dérivable sur ]0 ; + ∞[ et pour tout
On utilise le tableau de signes de f ( x) − g( x). x > 0,
x −∞ -1 0 1 +∞ 200 2( x2 − 100) 2( x + 10)( x − 10)
p ′( x) = 2 − 2
= = .
x - - 0 + + x x2 x2
2
x -1 + - - +
Pour tout x > 0, x2 > 0 , 2( x + 10) > 0 donc le
0 0
signe de p ′( x) est celui de x - 10.
f(x) - g(x) - 0 + 0 - 0 +
x − 10 = 0 équivaut à x = 10.
78

172909_Chap05_000-000.indd 78 30/07/2019 17:03:53


x 0 10 +∞ x
77 a) Pour tout x > 0, > 0 donc le signe de
p¢(x) - 0 + 2x
f ′( x) est celui de 12 x2 − 15 x + 3.
p(x) 12 x2 − 15 x + 3 = 0 si et seulement si x = 1 ou
1
40 x= .
4
D’après le tableau 40 est le minimum de p sur On dresse le tableau de variations de f.
]0 ; + ∞[ donc pour tout x > 0, p( x) > 40. Ceci 1
x 0 1 +∞
prouve que tous les rectangles d’aire 100 ont un péri- 4
mètre supérieur ou égal à 40. f ¢(x) + 0 - 0 +
0,95
75 f est une fonction polynôme donc elle est déri- f(x)
vable et pour tout x, f ′( x) = 3 x2 − a 0,4
Pour que f admette un extremum en 2 il est néces- b) f(0 ,1) = 2, 524 0 ,1 ≈ 0 , 8
saire que f ′(2) = 0, ce qui donne a = 12. D’après le tableau de variations de f, 0,4 est le mini-
Il faut vérifier que cela est suffisant. mum de f sur [0 ,1 ; + ∞[ donc pour tout x > 0 ,1,
f( x) > 0 , 4.
f ′( x) = 3 x2 − 12 = 3( x2 − 4) = 3( x − 2)( x + 2).
f ¢ est une fonction polynôme du second degré qui MB × CB
78 a)  f( x) = AM2 +
s’annule en 2 en changeant de signe. La valeur 2
a = 12 est donc suffisante. ( 4 − x ) × 4
f( x) = x2 + = x2 − 2 x + 8.
2
76 a)  B′( x) = −3 x2 + 20 x + 3 000. b) f est dérivable sur [0 ; 4 ] et pour tout x,
f ′( x) = 2 x − 2.
Sur [0 ; 50 ], B′( x) = 0 si et seulement si f ′( x) = 0 si et seulement si x = 1.
10 + 10 91 On dresse le tableau de variations de f.
x0 = ≈ 35,131
3
x 0 1 4
x 0 x0 50
f ¢(x) - 0 +
B¢(x) + 0 -
8 16
f(x)
b)
7
x 0 x0 50
c) L’aire du carré ABCD vaut, en unité d’aire, 16.
B( x0 ) L’aire minimale de la partie colorée est 7, ce qui repré-
B(x) 7
sente = 43, 75 %
0 50 000 16
Myriam a donc raison.
B( x0 ) ≈ 74 377.
c) D’après le tableau de variations de B, le bénéfice 79 1. L’effectif de l’entreprise est 40.
est maximal pour 35,131 tonnes pour un bénéfice 10 × 15 + 5 × 18 + 13 × 20 + 4 × 25 + 6 × 30 + 2 × 40
= 21, 5
d’environ 74 377 €. 40
Le salaire annuel moyen est 21,5 k€.
d) 
2. a) 
x 16 18 19,5 21,5 33
Var(x) 72,25 54,25 46,0 42,0 174,25
b) La variance de cette série est 42.
3. a)
1
Var( x) = [10(15 − x)2 + 5(18 − x)2 + 13(20 − x)2
40
+ 4(25 − x)2 + 6(30 − x)2 + 2(40 − x)2 ]

Chapitre 5  ★  Applications de la dérivation 79

172909_Chap05_000-000.indd 79 30/07/2019 15:00:58


1 2. a) Le maximum de h sur l’intervalle [0 ; + ∞[ est
Var( x) = [10( x2 − 30 x + 225) + 5( x2 − 36 x + 324)2 1 1
40 donc pour tout nombre réel x > 0, h( x) < .
+ 13( x2 − 40 x + 400) + 4( x2 − 50 x + 625) 3 3
b) La fonction h est décroissante sur l’intervalle
+ 6( x2 − 60 x + 900) + 2( x2 − 80 x + 1600)] [5 ; + ∞[ , donc pour tout x > 5, h( x) < h(5) c’est-
1 5 5
Var( x) = [ 40 x2 − 1720 x + 20 170 ] à-dire h( x) < . Or < 0 , 3 et pour tout x > 5,
40 17 17
Var( x) = x2 − 43 x + 504 , 25 2 x > 0 et x2 + 9 > 0 donc 0 < h( x) < 0 , 3.
b) La valeur qui minimise Var( x) est :
b 43 82 a) On complète la ligne par la condition :
− = = 21, 5.
2a 2 f (k ) > 2 000
Il s’agit de la moyenne. On obtient le résultat ci-dessous.
>>>
80 1. a)  f(0) = 0 et f ′(0) = −12 Affirmation fausse
b) f(1) = −11 et f ′(1) = −9 Cela signifie qu’il existe un nombre réel x de l’inter-
2. a)  f(0) = 0 et f (0) = q donc q = 0. valle [0 ; 500 ] tel que f( x) > 2 000. L’affirmation de
f ′(0) = p et f ′(0) = −12 donc p = −12. Mélanie est donc fausse.
b)  f (1) = m + n − 12 et f(1) = −11 donc m + n = 1 b) f est le produit de deux fonctions dérivables sur
f ′(1) = 3m + 2n − 12 et f ′(1) = −9 [1 ; 500 ] donc f est dérivable sur [1 ; 500 ] et pour
donc 3m + 2n = 3  1 
3m + 2n = 3 équivaut à m + 2(m + n) = 3 c’est-à- ( )
tout x > 1, f ′( x) = 1× 24 − x + x × − 
 2 x 
dire m + 2 = 3. Donc m = 1. On en déduit n = 0.
Conclusion : f( x) = x3 − 12 x. 1 x x x
f ′( x) = 24 − x − x car = × = x.
3. f est dérivable sur  et pour tout x, 2 x x x
3
f ′( x) = 3 x2 − 12 f ′( x) = 24 − x.
2
f ′( x) = 3( x2 − 4) = 3( x − 2)( x + 2)
c)  f ′( x) > 0 si et seulement si, x < 16, c’est-à-dire
f ′( x) = 0 si et seulement si x = −2 ou x = 2.
x < 256.
On dresse le tableau de variations de f.
On dresse le tableau de variations de la fonction f.
x −∞ -2 2 +∞
x 1 156 500
f ¢(x) + 0 - 0 +
f ¢(x) + 0 -
16
2 048
f(x)
f(x)
- 16
23 500(24 − 500 )
D’après le tableau de variations 2 048 est un maxi-
81 1. a) Pour tout nombre réel x > 0,
mum local de la fonction f sur l’intervalle [1 ; 500 ]
2( x2 + 9) − 2 x × 2 x 2(−x2 + 9)
h′( x) = = donc pour tout nombre réel x de [1 ; 500 ],
( x2 + 9)2 ( x2 + 9)2 f( x) < 2 048. Ainsi M = 2 048.
2(3 + x)(3 − x)
h′( x) =
( x2 + 9)2 83 a) Le solide de révolution obtenu par rotation
b) Pour tout nombré réel x > 0, 2(3 + x) > 0 et du rectangle autour de son axe de symétrie D est un
( x2 + 9)2 > 0 donc le signe de h′( x) est celui de cylindre de diamètre x et de hauteur h.
3 - x.
c)  3 − x = 0 si et seulement si x = 3.
h
On dresse le tableau de variations de h.
x 0 3 +∞ x
h¢(x) + 0 - On sait que 2( x + h) = 60 donc h = 30 − x
1 Ainsi x appartient à l’intervalle [0 ; 30 ].
h(x) 3  x 2
Le volume du cylindre en V( x) = π  (30 − x)
0  2 

80

172909_Chap05_000-000.indd 80 30/07/2019 15:02:33


π 2 π x 0 30
V( x) = x (30 − x) = (30 x2 − x3 ).
4 4 S¢(x) +
π 3π
V ′( x) = (60 x − 3 x2 ) = x(20 − x). 450p
4 4 S(x)
V ′( x) = 0 si et seulement si x = 0 ou x = 20. 0
On dresse le tableau de variations de la fonction V.
84 a)  9( x) = AM × AQ × AI
x 0 20 30
9( x) = x × x × (6 − x) = 6 x2 − x3 avec 0 < x < 6.
V¢(x) 0 + 0 -
b) 9 ′( x) = 12 x − 3 x2 = 3 x(4 − x)
1 000p
9 ′( x) = 0 si et seulement si x = 0 ou x = 4.
V(x)
On dresse le tableau de variations de la fonction 9.
0 0
x 0 4 6
Les dimensions du rectangle pour obtenir le plus
9¢(x) 0 + 0 -
grand volume sont 20 cm et 10 cm.
32
b) L’aire latérale du cylindre de révolution est :
9(x)
 x
!( x) = 2π (30 − x) = −πx2 + 30πx 0 0
 2 
c) La fonction 9 admet un maximum local en x = 4
où x est le diamètre du cylindre avec 0 < x < 30.
donc le volume 9( x) est maximum lorsque le point
où !′( x) = −2πx + 30π M est à 4 cm du point A.
!′( x) = 0 si et seulement si x = 15.
On dresse le tableau de variations de la fonction !. 85 a) p est dérivable sur ]0 ; + ∞[ et pour tout
1 x2 − 1 ( x − 1)( x + 1)
x 0 15 30 x > 0, p ′( x) = 1 − 2 = =
x x x2
!¢(x) + 0 -
Sur ]0 ; + ∞[ , p ′( x) s’annule si et seulement si
225p x = 1 et le signe de p ′( x) est celui de x2 - 1.
!(x) On dresse le tableau de variations de la fonction p.
0 0
x 0 1 +∞
L’aire est maximale lorsque x = 15. p¢(x) - 0 +
Les dimensions du rectangle sont 15 cm et 15 cm, il
s’agit d’un carré de côté 15 cm. p(x)
c) L’aire totale du cylindre de révolution est 2
D b)  p ′( x) s’annule en x = 1 en changeant de signe
donc p admet, d’après a), un minimum sur l’intervalle
]0 ; + ∞[ égal à p(1) = 2.
2. Le périmètre du rectangle OMNA est 2(OM + OA).
x On pose OM = x avec x > 0. Le point A a donc pour
2  1
 x x
S( x) = 2π  + 2p (30 − x) avec 0 < x < 30 coordonnées 0 ; .
 2   x 
2  1
Ainsi le périmètre du rectangle OMNA vaut 2 x +  ,
π 2 π   x
S( x) = x − πx2 + 30πx =  − π x2 + 30πx. c’est-à-dire 2p( x).
2 
2 
D’après 1., p admet un minimum en x = 1 donc les
π dimensions du rectangle sont OM = 1 et OA = 1
S( x) = − x2 + 30πx
2 donc OMNA est un carré de côté 1.
S′( x) = −πx + 30π.
86 a) La deuxième dimension y d’un rectangle
S′( x) = 0 si et seulement si x = 30. 2 019
d’aire 2 019 est y = avec x > 0.
Il s’agit d’un cylindre aplati. x

Chapitre 5  ★  Applications de la dérivation 81

172909_Chap05_000-000.indd 81 30/07/2019 15:03:19


Ainsi le périmètre du L est 3 y + 3 x + ( y − x) c’est-à- f ′(2) = 0 donc la proposition est vraie.
dire 4 y + 2 x. b)  f ′(1) = −3 et −3 < 0 donc la proposition est
4 × 2 019 8 076 fausse.
Donc P( x) = 2 x + = 2x + .
x x c) Pour tout x de l’intervalle [2 ; + ∞[ , x + 2 > 0,
b) La fonction P est dérivable sur ]0 ; + ∞[ et x - 2 > 0 et x2 > 0 donc f ′( x) > 0. Ainsi f est crois-
8 076
P ′( x) = 2 − 2 sante sur [2 ; + ∞[.
x La proposition est vraie.
2( x2 − 4 038) d) Sur ]0 ; + ∞[ f ′( x) s’annule en x = 2 en chan-
P ′( x) =
x2 geant de signe, donc x0 = 2 est un extremum local.
Sur ]0 ; + ∞[ , P ′( x) = 0 si et seulement si Sur ]0 ; 2], f ′( x) < 0 sur [2 ; + ∞[ , f ′( x) > 0
x = 4 038. donc x0 = 2 est un minimum local.
On dresse le tableau de variation de P. e)  f(1) = 3 et 3 > −6 donc la proposition est fausse.

x 0 4 038 +∞ 89 La négation de P est : « Il existe un nombre réel


P¢(x) - 0 + x, f( x) > 2  ».
La négation de Q est  : «  Pour tout nombre réel x,
P(x) f ′( x) < 0  ».
4 4 038
90 a) • La réciproque de l’implication est :
8 076
P ( )
4 038 = 2 4 038 +
4 038
« Si f est croissante sur un intervalle I, alors pour tout
nombre réel x de I, f ′( x) > 0  ».
P ( )
4 038 = 4 4 038 • La contraposée de l’implication est :
c) D’après b), le périmètre minimum du polygone en «  Si f n’est pas croissante sur un intervalle I, alors il
L est obtenu pour x = 4 038 et vaut 4 1038. existe au moins un nombre réel x de I, f ′( x) < 0  ».
b) L’implication est vraie. La réciproque est vraie.
87 f est dérivable sur [0 ; 12] et pour tout x, La contraposée est fausse. En effet la fonction f défi-
 1 nie sur  par f( x) = 1 est constante et pour tout x
(2 x − 2) x +  − ( x2 − 2 x + 5) × 1
1 
 2 de , f ′( x) = 0.
f ′( x) = ×
2  2
 x + 1 
 2 
1 x2 + x − 6
= ×
2  2
 x + 1 
 2  2
 1
Pour tout x de l’intervalle [0 ; 12],  x +  > 0
 2
2
donc le signe de f ′( x) est celui de x + x − 6. Organiser son raisonnement
Sur [0 ; 12] f ′( x) = 0 si et seulement si x = 2.
On dresse le tableau de variations de f.
x 0 2 12
f ¢(x) - 0 + 91 a) f est dérivable sur  et pour tout x de ,
f ′( x) = 3 x2 − 2.
5 5
2 2
f(x) f ′( x) = 0. si et seulement si, x = ou x = − .
3 3
1
On dresse le tableau de variations de f.
D’après le tableau de variations de f, Sandrine est pas- 2 2
x −∞ - +∞
sée lors de son plongeon à 1 m minimum du fond de 3 3
la piscine. f ¢(x) + 0 - 0 +

88 a) Pour tout x ¹ 0, −45 + 4 6


f(x)
4 ( x − 2)( x + 2) 9 -45 - 4 6
f ′( x) = 1 − 2 = .
x x2 9

82

172909_Chap05_000-000.indd 82 30/07/2019 15:04:23


Le volume d’un baril de rayon r et de hauteur h est
 2   2 3 2
• f −  = −  + 2 −5 pr 2h c’est-à-dire π(9 − h2 )h.
 3   3  3
Ainsi le volume du baril en fonction de la hauteur h
 2 2 2 2 est V(h) = 9πh − πh3 avec h > 0.
f −  = − × +2 −5
 3  3 3 3 La fonction V est dérivable sur [0 ; + ∞[ et pour
tout h, V ′(h) = 9π − 3πh2 .
 2 2 2
f −  = − 6 + 6 −5 V ′(h) = 3π(3 − h2 ) = 3π 3 − h 3 + h . ( )( )
 3  9 3
Sur [0 ; + ∞[ , V ′(h) = 0 si et seulement si h = 3.
 2  −45 + 4 6 On dresse le tableau de variations de V.
f −  =
 3  9 0
h 3 +∞
3
 2  2 2 V¢(h) + 0 -
• f   =   − 2 −5

 3   3   3 6p 3
V(h)
2 2 −45 − 4 6 0
= 6− 6 −5 = .
9 3 9
( 3 ) = π(9 − )
2
b) V 3 3
n 0 1 2 3
xn 2 2,1 2,094 57 2,094 55 V ( 3 ) = 6π 3
f ( xn ) 0,000
g( xn )
0,1 0,005 4 0,000 02
001
r 2 = 9 − h2 = 6 donc r = 6.
Les dimensions du baril de volume maximum sont
f (2) r = 6 pieds et h = 3 pieds.
= −0 ,1 = 0 ,1 et 0 ,1 > 10-5 donc x prend la
g(2)
valeur 2 − (−0 ,1) = 2,1 93 1. a)  f ′( x) = 0 , 75 x 4 − 6 x2 + 12.
f (2,1) b) f ′′( x) = 3 x3 − 12 x = 3 x( x2 − 4)
≈ 0 , 005 4 et 0 , 005 4 > 10-5 donc x prend la = 3 x( x − 2)( x + 2).
g(2,1)
f (2,1) 2. a)  f ′′( x) = 0 si et seulement si, x = 0 ou x = 2
valeur 2,1 − ≈ 2, 094 57 ou x = −2.
g(2,1)
x −∞ -2 0 2 +∞
f (2, 094 57)
≈ 0 , 000 02 et 0 , 000 02 > 10-5 donc 3x - - 0 + +
g(2, 094 57)
2
x0 prend la valeur 2, 094 57 − 0 , 000 02 ≈ 2, 094 55 x -4 + 0 - - 0 +
f ¢¢(x) - 0 + 0 - 0 +
f (2, 094 55)
≈ −0 , 000 001 = 10−6 et 10−6 < 10−5
g(2, 094 55) On dresse le tableau de variations de f ¢.
l’algorithme s’arrête.
x −∞ -2 0 2 +∞
c) 
12
f ¢(x)
d)  a = 2, 094 55 arrondi au dix-millième.
0 0

92 r D’après 2. a), pour tout nombre réel x, f ′( x) > 0.


c) Pour tout x, f ′( x) > 0 donc f est croissante sur .

94 Dans le repère orthonormé, on pose x l’abscisse


de M avec x > 0. Ainsi M( x ; x ).
h 3 2
AM2 = ( x − 1)2 + ( x) = x 2 − x + 1.
Déterminer la position du point M de # le plus
proche de A c’est déterminer la position du point M
de # telle que la longueur AM est minimum, ce qui
D’après le théorème de Pythagore r 2 + h2 = 9. est équivalent à AM2 minimum.

Chapitre 5  ★  Applications de la dérivation 83

172909_Chap05_000-000.indd 83 30/07/2019 15:05:25


f est la fonction définie sur [0 ; + ∞[ par Or f (0 , 2) = 0 donc a = 6 , 25.
f( x) = x2 − x + 1. D’où f( x) = 6 , 25 x2 − 0 , 25.
f est dérivable sur [0 ; + ∞[ et pour tout x > 0,
L’ordonnée de M est donc f( x).
f ′( x) = 2 x − 1.
1 L’aire du rectangle bleu est :
f ′( x) = 0 si et seulement si x = .
2 !( x) = x f ( x) = x(0 , 25 − 6 , 25 x2 ) = −6 , 25 x3 + 0 , 25 x
On dresse le tableau de variations de f.
avec 0 < x < 0 , 2.
1
x 0
2
+∞ !′( x) = −18 , 75 x2 + 0 , 25.
f ¢(x) - 0 + Sur [0 ; 0 , 2], !′( x) = 0 si et seulement si
1 0 , 25 3
x= =
f (x) 18 , 75 15
3
4 On dresse le tableau de variations de la fonction !.

 1   1 2 1 3 x 0 3 0,2
f   =   − + 1 =
 2   2  2 4 15
! ¢(x) + 0 -
Le point de # le plus proche de A est donc
1 3  3
M ; . ! (x)
 2 2  90
0 0
95 On note x et y les dimensions d’un box.
Les douze box nécessitent 16 x + 15 y de clôture  3   3 3 3
!  = −6 , 25  + 0 , 25
 
donc 16 x + 15 y = 2. 
 15  
 15  15
L’aire des douze box est 12xy, c’est-à-dire
 3 3
 2 − 16 x  3
12 x 
1
= x(2 − 16 x) avec 0 < x < . !  =
 15  5 8  15  90
 1 L’abscisse du point M pour laquelle l’aire du rectangle
! est la fonction définie sur  0 ;  par
4 
 8  3
!( x) = (2 x − 16 x2 ) bleu est maximale est .
5 15
4
!′( x) = (2 − 32 x).
5 97 La durée du trajet de 1000 km à la vitesse
1 1000
!′( x) = 0 si et seulement si, x = . moyenne v est t = .
16 v
On dresse le tableau de variations de la fonction !.
Le salaire du chauffeur sera donc de :
1 1
x 0 1000 10 200
16 8 10 , 20 × = .
v v
!¢(x) + 0 -
La consommation en litres pour 1000 km sera de
1
6 000
!(x) 20 100(v ) = + 2v .
v
0 0
Le litre de carburant coûte 1,5 € donc le coût du car-
2 16 2 16 1 1  6 000  9 000
ymax = − xmax = − × = burant sera de 1, 5 + 2v  = + 3v .
15 15 15 15 16 15  v  v
Les dimensions d’un box qui donnent le maximum
1 Le prix de revient est alors :
d’espace aux animaux sont km c’est-à-dire 62,5 m
1 16 10 200 9 000 19 200
sur km c’est-à-dire 66,7 m. P(v ) = + + 3v = + 3v.
15 v v v
19 200
96 On note x l’abscisse du point M de P ′(v ) = − +3
v2
3(0 < x < 0 , 5).
3 a pour sommet le point S(0 ; − 0 , 25) donc la 3(−6 400 + v 2 ) 3(80 + v )(80 − v )
= = .
fonction f dont la courbe représentative est 3 est v2 v2
définie sur [0 ; 0 , 5] par f ( x) = a x2 − 0 , 25. Sur ]0 ; + ∞[ , P ′(v ) = 0 si et seulement si v = 80.
84

172909_Chap05_000-000.indd 84 30/07/2019 17:03:44


On dresse le tableau de variations de la fonction P. 99 On étudie les variations de la fonction f définie
x2
v 0 80 +∞ sur [0 ; + ∞[ par f( x) = .
x +1
P¢(v) - 0 + f est dérivable sur [0 ; + ∞[ et pour tout x,
2 x( x + 1) − x2 × 1
f ′( x) =
P(v) ( x + 1)2
480 x2 + 2 x x( x + 2)
f ′( x) = 2
= .
Pour minimiser les coûts, la vitesse moyenne doit être ( x + 1) ( x + 1)2
de 80 km ⋅ h−1 et le prix de revient est alors 480 €. Pour tout nombre réel x > 0, x + 2 > 0,
2
1000 ′
( x + 1) > 0 donc f ( x) > 0 donc f est croissante sur
La durée du trajet, en h, est alors = 12, 5 soit
80 [0 ; + ∞[.
12 h 30 mn.
f conserve l’ordre donc 1, 089 > 1, 088 implique
1, 0892 1, 0882
f (1, 089) > f (1, 088) c’est-à-dire > .
98 a) Dans la fenêtre -6 < X < 6 (pas = 1) et 2, 089 2, 088
0 < Y < 1 (pas = 1). On peut conjecturer que m = 0
et M = 1. 100 a)  j est dérivable sur [−1 ; 1] et pour tout x,
ϕ ′( x) = f ′( x) − f ′(−x)
1 1 1 1
ϕ ′( x) = 2
− 2
= 2
− =0
1+ x 1 + (−x) 1+ x 1 + x2
b) Pour tout nombre réel x de [−1 ; 1], ϕ ′( x) = 0
donc j est une fonction constante sur [−1 ; 1]
Pour tout nombre réel x de [−1 ; 1,]
b) f est dérivable sur  et pour tout x, ϕ( x) = ϕ(0) = f (0) + f (−0) = 0 + 0 = 0.
8 x(4 + x 4 ) − 4 x2 × 4 x3 Ainsi pour tout nombre réel x de [−1 ; 1], ϕ( x) = 0.
f ′( x) =
(4 + x 4 )2 c) Pour tout nombre réel x de [−1 ; 1] :
f ( x) + f (−x) = 0 donc f (−x) = −f ( x).
8 x(4 − x 4 ) 8 x(2 − x2 )(2 + x2 ) f est donc impaire.
f ′( x) = =
(4 + x 4 )2 (4 + x 4 )2 d) 

=
8 x(2 + x2 ) ( 2−x )( 2+x ) p
4–
p–
(4 + x 4 )2 6
- 3
3
f ′( x) = 0 si et seulement si x = 0 ou x = 2 ou -1 O 1
3
x = − 2. p 3
- 6–
Le signe de f ′( x) est celui de 8 x(2 − x2 ) car - p4–
(4 + x 4 )2 > 0 et 2 + x2 > 0.
x −∞ - 2 0 2 +∞
101 To find latitude and longitude of the southern-
8x - - 0 + +
most point at which the full eclipse could viewed, we
2 - x2 - 0 + + 0 - will find the extremum values of the function f.
f ¢(x) + 0 - 0 + 0 - f ′( x) = 0 , 025 x − 1157
, .
1 1 ′
f ( x) = 0 if and only if x = 46 , 28.
P(x) x 15 46,28 90
0 f ¢(x) - 0 +

D’après le tableau de variations de f, pour tout 8,321 5 19,984


nombre réel x, f( x) < 1. De plus 4 x2 > 0 et f (x)
4 + x 4 > 0 donc f( x) > 0. - 3,908 98

Donc pour tout x, 0 < f( x) < 1. Ainsi M = 1 et So the number of degrees of longitude east of the prime
m = 0. meridian is 46°16’48” and the latitude is 3°54’32” South.

Chapitre 5  ★  Applications de la dérivation 85

172909_Chap05_000-000.indd 85 30/07/2019 15:07:51


102 Pour tout nombre réel a, f ′(a) = −2a + 1. La moyenne arithmétique des deux vitesses est
1 20 + x
f ′(a) s’annule en a = en changeant de signe donc .
2 2
1
f admet en un extremum local. On note f( x) la différence des deux vitesses :
2
 20 + x 40 x
1 1  f( x) = − .
Sur  − ∞ ;  , f ′(a) > 0 et sur  ; + ∞  , f ′(a) < 0 2 x + 20
 2   2 
 1 2 f est dérivable sur ]0 ; + ∞[ et pour tout x > 0,
donc il s’agit d’un maximum local égal à   + 6 ,
2 1 800
f ′( x) = −
soit 6,25. 2 ( x + 20)2
103 a) D’après le théorème de Pythagore h2 + r 2 = 3. ( x + 20)2 − 1600 ( x + 60)( x − 20)
1 1 f ′( x) = = .
Le volume du cône est πr 2 × h = π(3 − h2 )h 2( x + 20)2 2( x + 20)2
3 3
La fonction V définie sur [0 ; + ∞[ par Pour tout nombre réel x > 0, 2( x + 20)2 > 0 et
1 x + 60 > 0, donc le signe de f ′( x) est celui de
V(h) = π(3h − h3 ) est dérivable et pour tout h,
3 x - 20.
1 Sur [0 ; + ∞[ , f ′( x) = 0 si et seulement si x = 20.
V ′(h) = π(3 − 3h2 ) = π(1 − h2 )
3 On dresse le tableau de variations de f.
V ′(h) = π(1 − h)(1 + h).
x 0 20 +∞
Sur [0 ; + ∞[ , V ′(h) = 0 si et seulement si h = 1.
f ¢(x) - 0 +
On dresse le tableau de variation de la fonction V.
10
h 0 1 +∞
f (x)
V¢(h) + 0 -
0
2p
V(h) Le minimum de f est 0 donc on peut affirmer que la
3
vitesse arithmétique est supérieure ou égale à la
0
vitesse moyenne.
Le volume du cône est maximal lorsque h = 1 m et
r = 3 − 12 = 2 m. A
106
2p 3
b) Ce volume maximal est m .
3
D B
104 f est dérivable sur [0 , 5 ; 3, 4 ] et pour tout x,
f ′( x) = 3 x2 − 12 x + 9.
f ′( x) = 0 si et seulement si x = 1 et x = 3. C
On dresse le tableau de variations de la fonction f. ABCD est un losange de périmètre p tel que BD = L
x 0,5 1 3 3,4 et AC = .
f ¢(x) + - +
L
0 0 L’aire du losange est .
2
6 2,544  L 2   2  p 2
f(x) D’après le théorème de Pythagore,   +   =  
 2   2   4 
2 2 p2
5,125 2 donc L +  = .
4
Par lecture du tableau m = 1, c = 2, d = 3 et L
L’aire est maximum lorsque est maximum, ce qui
u = 6. 2 2 2
L
équivaut à maximum (la fonction carré est
d 4
105 Le temps t1 mis pour l’aller est t1 =
20 croissante sur [0 ; + ∞[ ).
d
Le temps t2 mis pour le retour est t2 =  p2  p2 2
x L2  2 = L2  − L2  = L − L4 .
La vitesse moyenne aller-retour est :  4  4
2d 2d 40 x
= = . On note f la fonction définie sur [0 ; + ∞[ par
t1 + t2 d d x + 20
+ p2 2
20 x f (L) = L − L4 .
4
86

172909_Chap05_000-000.indd 86 30/07/2019 15:10:55


f est dérivable sur [0 ; + ∞[ et pour tout nombre réel Cette consommation vaut, arrondie au centième,
L on a : 2,67 litres pour 100 km.
p2  p2  e) La valeur de la variable x à la fin de l’exécution de
f ′(L) = L − 4L3 = L  − 4L2 . l’algorithme est 51.
2  2 
p En effet, f(50) = 4 et sur l’intervalle [30 ; 75] la fonc-
f ′(L) = 0 si et seulement si, L = 0 ou L = . tion f est décroissante par conséquent l’algorithme
2 2
s’arrête juste après 50 donc à 51.
p
L 0 +∞ Cela signifie que la consommation est strictement
2 2
inférieure à 4  L aux 100  km lorsque la vitesse sera
f ¢(L) + 0 -
supérieure à 51 km ⋅ h−1.
p4
f (L) 64
0

Exploiter ses compétences


La fonction f admet un maximum sur [0 ; + ∞[ en
p
L= .
2 2
p2 p2 p2 p2 p
2 = − L2 = − = donc  = .
4 4 8 8 2 2
108 D’après le Doc. 2, f(0) = 2, 2 et f ′(0) = 2.
Ainsi, parmi les losanges de périmètre p, il en existe
un dont l’aire est maximum : c’est le carré (les diago- D’après le Doc. 3, f (0) = c et f ′(0) = b
nales du losange sont de même longueur). On déduit que c = 2, 2 et b = 2.
Ainsi f( x) = −0 , 5 x2 + 2 x + 2, 2.
44 D’après le Doc. 3, le panier est à 3 m du joueur et se
107 a)  f(30) = ≈ 14 , 7
3 situe à 3,7 m du sol.
f(50) = 4 f(3) = −0 , 5 × 32 + 2 × 3 + 2, 2 = 3, 7.
À 30 km ⋅ h−1 la consommation est 14,7 litres pour Le joueur a des chances de marquer son panier.
100 km et à 50 km ⋅ h−1 la consommation est de 4 L
pour 100 km. 109 On étudie la fonction g définie sur [0 ; + ∞[
b) f est dérivable sur [30 ; 130 ] et pour tout x, par g(P) = f (P) − P
(16 x − 800)x2 − (8 x2 − 800 x + 30 000) × 2 x g(P) = −0 , 025P2 + 3P
f ′( x) =
x4 g est dérivable et g ′(P) = −0 , 05P + 3
x(16 x2 − 800 x − 16 x2 + 1600 x − 60 000) g ′(P) = 0 si et seulement si P = 60
f ′( x) =
x4 On dresse le tableau de variations de g.
800 x − 60 000 P 0 60 +∞
f ′( x) =
x3 g¢(P) + 0 -
c) Pour tout nombre réel x de l’intervalle [30 ; 130 ],
90
x3 > 0 donc le signe de f ′( x) est celui de
g(P)
800 x - 60 000.
60 000 0
f ′( x) = 0 si et seulement si x = = 75.
800 Le RMD est de 90 000 lièvres.
On dresse le tableau de variations de la fonction f.
110 x+2
x 30 75 130
f ¢(x) - 0 +
612
y
44
f (x) 3 169 x
8
3 On note x et y les dimensions de la piscine.
d) La consommation est minimum pour la vitesse de 16
On a donc xy = 16 donc y = .
75 km ⋅ h−1. x

Chapitre 5  ★  Applications de la dérivation 87

172909_Chap05_000-000.indd 87 30/07/2019 15:13:24


D’après le Doc. 1, la surface de la margelle est : 111 D’après les Doc. 1 et 2, le bénéfice quotidien
16 est :
2 × 1× ( x + 2) + y × 1× 2 = 2 x + 4 + 2 ×
x B( x) = R( x) − 2 = −x 4 + 6 x3 − 12 x2 + 10 x − 2
32
c’est-à-dire 2 x + 4 + . D’après le Doc. 3., B′( x) = −2( x − 1)2 (2 x − 5).
x
Pour tout x compris entre 0 et 10, ( x − 1)2 > 0 donc
On note f la fonction définie sur ]0 ; + ∞[ par
32 le signe de B′( x) est celui de −2(2 x − 5).
f( x) = 2 x + 4 + . 5
x B′( x) = 0 si et seulement si x = 1 ou x = .
2
32 2( x2 − 16) 2( x − 4)( x + 4) On dresse le tableau de variations de R.
f ′( x) = 2 − 2
= 2
= .
x x x2 5
Le signe de f ′( x) est celui de x - 4. x 0 1 10
2
Sur ]0 ; + ∞[ , f ′( x) = 0 si et seulement si x = 4. R¢(x) + 0 + 0 -
On dresse le tableau de variations de la fonction f. 2,687 5
x 0 4 +∞ R(x)
f ¢(x) - 0 + - 2 - 5 102
La fonction R admet un maximum en 2,5 égal à
f (x) 2, 687 5.
20 Le bénéfice quotidien est donc maximum lorsque
l’entreprise produit et vend 250 litres de parfum et il
20
Louis utilisera donc = 80 caillebotis. s’élève à 2 687,5 €.
0 , 25
2 bidons de 1 L coûtent 29,80 € pour un rendement
de 24 m2 .
1 bidon de 2,5 L coûte 29,90 € pour un rendement de
30 m2 .
1 bidon de 5 L coûte 47,90 € pour un rendement de
40 m2 .
Louis doit donc prendre 2 bidons de 1 L.

88

172909_Chap05_000-000.indd 88 30/07/2019 15:14:07


6 Fonction exponentielle

N(7) e7
Découvrir
3  a)  = 1 ≈ 403, 429
N(1) e
N(5) e5
= ≈ 2, 718
N(4) e 4
N(6) e6
= ≈ 54 , 598
1 La fonction exponentielle N(2) e2
b) On remarque :
1  f(0) = 1. N(7) N(5) N(6)
= N(6) ; = N(1) et = N(4).
N(1) N(4) N(2)
2  Cette contrainte se traduit par :
pour tout nombre réel x, f ′( x) = f ( x). ea
Il semble que = ea−b où a et b désignent
eb
3  a) b)  deux nombres réels.

Acquérir des automatismes

c)  a  2, 7183.
3 f( x) = (5 + x)e x
2 Découvrir des propriétés f est dérivable sur . Pour tout nombre réel x,
de la fonction exponentielle u( x) = 5 + x u ′( x) = 1
x
v( x) = e v ′( x) = e x
1   t 0 1 2 3 f ′( x) = 1× e x + (5 + x) × e x
N(t) 1 2,718 7,389 20,086 f ′( x) = e x (1 + (5 + x))
t 4 5 6 7 f ′( x) = e x (6 + x)
N(t) 54,598 148,413 403,429 1 096,633
4 a)  f( x) = (2 x − 5)e x
2  a)  N(2) × N(3) = e2 × e3 ≈ 148 , 413
f est le produit de deux fonctions dérivables sur 
N(3) × N(4) = e3 × e 4 ≈ 1096 , 633 donc dérivable.
N(1) × N(5) = e1 × e5 ≈ 403, 429 Pour tout nombre réel x,
b) On remarque que : u( x) = 2 x − 5 u ′( x) = 2
x
N(2) × N(3) = N(5), N(3) × N(4) = N(7) v( x) = e v ′( x) = e x
N(1) × N(5) = N(6) f ′( x) = 2 × e + (2 x − 5) × e x
x

Il semble que ea × eb = ea+b où a et b désignent f ′( x) = e x (2 + (2 x − 5))


deux nombres réels. f ′( x) = e x (2 x − 3)

Chapitre 6  ★  Fonction exponentielle 89

172909_Chap06_000-000.indd 89 25/07/2019 19:16:00


b)  g(t ) = t et 8 La fonction f est dérivable sur .
La fonction g est le produit de deux fonctions déri- Pour tout nombre réel x,
1
vables sur  donc dérivable sur .  1 − x
f ′( x) = −0 , 00182 × − e 7
Pour tout nombre réel t,  7 
u(t ) = t u ′(t ) = 1 13 −7 x
1
t
v(t ) = e v ′(t ) = et f ′( x) = e
50000
g ′(t ) = 1× et + t × et
11 a)  A = e10 x−8 × e1−2 x = e10 x−8+1−2 x = e8 x−7
g ′(t ) = et (1 + t )
e x+1
b)  B = = e x+1−(4 x+3) = e x+1−4 x−3 = e−3 x−2
5 a)  f (t ) = 4 et − 1 4 x +3
f est dérivable sur . c)  C = (e5 x+2 )2 = e(5 x+2)×2 = e10 x+4
Pour tout nombre réel t, f ′(t ) = 4et .
b)  g( x) = (−4 x − 5)e x + 10 12 a)  A = e−2 ,5 x × e × e0 ,5 x−1
A = e−2 ,5 x+1 × e0 ,5 x−1
g est dérivable sur .
A = e−2 ,5 x+1+0 ,5 x−1
Pour tout nombre réel x,
A = e−2x
u( x) = −4 x − 5 u ′( x) = −4
x
v( x) = e v ′( x) = e x (e x )2 e2 x
b)  B = −2 9x
= −2+9 x = e2 x−(−2+9 x)
e ×e e
g ′( x) = −4 × e x + (−4 x − 5) × e x + 0
  B = e2 x+2−9 x = e−7 x+2
g ′( x) = e x (−4 + (−4 x − 5))
g ′( x) = e x (−4 x − 9) 13 A = e−2a (e 4a − 1) − (e−a + ea )2
A = e−2a+4a − e−2a − (e−a×2 + 2 × e−a+a + ea×2 )
6 a) La fonction f est le produit de deux fonctions
A = e2a − e−2a − e−2a − 2e0 − e2a
dérivables donc dérivable sur .
A = −2
Pour tout nombre réel x,
u( x) = x2 u ′( x) = 2 x 14 a) Pour tout nombre n de ,
x
v( x) = e v ′( x) = e x un+1 = e3(n+1) = e3n+3 = e3n × e3 = e3 × un
f ′( x) = 2 x × e x + x2 × e x La suite (un ) est donc géométrique de raison q = e3 .
x 2
f ′( x) = e ( x + 2 x) Son premier terme est u0 = e3×0 = e0 = 1.
b) La fonction g est le produit de deux fonctions déri- b)  S = u0 + u1 +  + u5
vables sur  donc dérivable sur . S = 1 + e3 + (e3 )2 +  + (e3 )5
Pour tout nombre réel x,
1 − (e3 )5+1
u( x) = x2 + 5 x − 3 u ′( x) = 2 x + 5 S=
x 1 − e3
v( x) = e v ′( x) = e x
1 − e18
g ′( x) = (2 x + 5) × e x + ( x2 + 5 x − 3) × e x S=
1 − e3
g ′( x) = e x (2 x + 5 + x2 + 5 x − 3)
g ′( x) = e x ( x2 + 7 x + 2) 15 a) Pour tout nombre n de ,
un+1 = 2e−0 ,5(n+1) = 2e−0 ,5n−0 ,5
7 a) La fonction f est dérivable sur . un+1 = 2e−0 ,5n × e−0 ,5 = e−0 ,5 × un
Pour tout nombre réel x, La suite (un ) est donc géométrique de raison
f ′( x) = 7 × e7 x = 7e7 x . q = e−0 ,5 .
b) La fonction g est dérivable sur . Son premier terme est u0 = 2e−0 ,5×0 = 2.
Pour tout nombre réel x, b) Pour tout nombre n de ,
g ′( x) = 1, 5 × 0 , 4 × e0 ,4 x = 0 , 6e0 ,4 x . v n = un2 = (2e−0 ,5n )2 = 4 e−n
c) La fonction h est dérivable sur . v n+1 = 4 e−(n+1) = 4 e−n−1 = 4 e−n × e−1 = e−1 × v n
Pour tout nombre réel t, La suite (v n ) est donc géométrique de raison e-1.
h′(t ) = −70 × (−0 ,1)e−0 ,1t = 7e−0 ,1t . Son premier terme est v 0 = 4 e−0 = 4.
90

172909_Chap06_000-000.indd 90 25/07/2019 19:17:32


18 a) La fonction f est le produit de deux fonctions e 4 x +6
dérivables sur , donc dérivable sur . 22 A = 3− x
= e 4 x+6−(3− x)
e
Pour tout nombre réel x,
= e 4 x+6−3+ x = e5 x+3
u( x) = 5 x + 10 u ′( x) = 5
x L’affirmation (2) est donc exacte.
v( x) = e v ′( x) = e x
f ′( x) = 5 × e x + (5 x + 10) × e x 23 B = (e5 x+1)2 = e(5 x+1)×2 = e10 x+2
f ′( x) = e x (5 + 5 x + 10) Florent a donc tort.
x
f ′( x) = e (5 x + 15).
b) Pour tout nombre réel x, e x > 0, donc f ′( x) est du 24 a)  e 4 × e6 × e−5 = e 4 +6+(−5) = e5
signe de 5 x + 15. b)  e × (e3 )4 = e1 × e3×4 = e1+12 = e13
x −∞ - 3 +∞ (e1,5 )2 e1,5×2 e3
5 x + 15 = 0 c)  = = = e3−1 = e2
f ¢(x) - 0 + e e1 e1
5 x = −15 e−0 ,4 × e3 e−0 ,4 +3 e 2 ,6
f (x) d)  2 ,2
= 2 ,2
= 2 ,2 = e2 ,6−2 ,2 = e0 ,4
x = −3 e e e
-5e-3
25 a)  e12 × e6 = e18 b)  e−1 × e9 = (e 4 )2
f(−3) = (5 × (−3) + 10)e−3 = −5e−3 .
e14 ,5 1
c)  = e10 d)  e × = e−0 ,5
19 a) La fonction g est le produit de deux fonctions e1,5 × e3 e1,5
dérivables sur , donc dérivable sur .
Pour tout nombre réel x, e2 × e5 1 e 2 +5 × 1 e7
26 A = × = =
u( x) = −x u ′( x) = −1 e−5 e−7 e−5+(−7) e−12
v( x) = e x v ′( x) = e x = e7−(−12) = e19
g ′( x) = −1× e x + (−x) × e x Manon a donc tort.
x
g ′( x) = e (−1 − x).
b) Pour tout nombre réel x, e7 > 0 , donc g ′( x) est e5,6 × e−4 1 e5,6−4 × 1
27 × =
du signe de -1 - x. e1 (e0 ,3 )2 e1 × e0 ,3×2

x −∞ - 1 +∞ e1,6 e1,6


= = = e1,6−1,6 = e0 = 1
g¢(x) + 0 - e1+0 ,6 e1,6
−1 − x = 0
−x = 1
(1 − e−3 )2 − e−5 (e5 − 2e2 + e−1)
e-1
g(x) x = −1 = 12 − 2 × 1× e−3 + (e−3 )2 − e−5+5 + 2e−5+2 − e−5−1
= 1 − 2e−3 + e−6 − e0 + 2e−3 − e−6
g(−1) = −(−1)e−1 = e−1. = 1− 1
=0
20 Les fonctions f et g sont dérivables sur .
Pour tout nombre réel t, 28 a)  A = e1− x × e3 x−2 = e1− x+3 x−2 = e2 x−1
f ′(t ) = 2 e2t et g ′(t ) = −0 , 5e−0 ,5t . b)  B = e2 x × e3−5 x + e2 x+3−5 x+1 = e−3 x+4
Pour tout nombre réel t, e2t > 0 et 2 > 0 donc
c)  C = (e x )2 × e7+4 x = e2 x × e7+4 x = e2 x+7+4 x
f ′(t ) > 0 et f est croissante sur .
Pour tout nombre réel t, e−0 ,5t > 0 et −0 , 5 < 0 C = e6 x+7
donc g ′(t ) < 0 et g est décroissante sur . d)  D = (e2 x )3 × e−3 = e6 x × e−3 = e6 x−3
La courbe #1 est donc celle de la fonction g et # 2
est celle de la fonction f. 29 a)  A = (et +1)3 = e(t +1)×3 = e3t +3
b)  B = (e−2−3t )10 = e(−2−3t )×10 = e−20−30t
21 a)  e3 × e−1 = e3+(−1) = e2
c)  C = (e0 ,6+1,4 t )2 = e(0 ,6+1,4 t )×2 = e1,2+2 ,8t
(e 4 )3 e 4×3 e12
b)  2 = 2 = 2 = e12−2 = e10 2 2 2
e e e d)  D = (et +t −1 2
) = e(t +t −1)×2
= e2t +2t −2

Chapitre 6  ★  Fonction exponentielle 91

172909_Chap06_000-000.indd 91 25/07/2019 19:18:48


e5a+7 (e x − 1)2 (e x )2 − 2 × 1× e x + 12
30 a)  A = = e5a+7−3a = e2a+7 34 a)  1 − 2x
= 1−
e 3a
e e2 x
e 4 a+1 e 2 x − 2 e x + 1 e 2 x − e 2 x + 2e x − 1
b)  B = = e 4 a+1−(−9) = e 4 a+10 = 1− =
e−9 e2 x e2 x
e−3a 2e x − 1
c)  C = = e−3a−(1−4 a) = e−3a−1+4 a =
e1−4 a e2 x
= ea−1 1 3 − ex 1× e x 3 − ex ex + 3 − ex
b)  x
+ 2x = x x
+ 2x =
e2a+1 e e e ×e e e2 x
d)  D = 1−a
= e2a+1−(1−a) = e2a+1−1+a 3
e = 2x
= e3a e

35 Pour tout nombre n de ,


e x × e3 x e x +3 x e4 x
31 a)  A = = = un+1 = −3e11, (n+1) = −3e11, n+11,
e− x × e 4 x e− x+4 x e3 x
4 x−3 x x un+1 = −3e11, n × e11, = e11, × un .
=e =e
La suite (un ) est donc géométrique de raison e11,.
(e− x )3 e− x×3 e−3 x Son premier terme est u0 = −3e11, ×0 = −3e0 = −3.
b)  B = = =
e x+4 e x+4 e x+4
−3 x−( x + 4 ) −3 x − x − 4
=e =e = e−4 x−4 36 Pour tout nombre n de ,
1 1 1
c)  C = (e0 ,5 x )2 × = e0 ,5 x×2 × e− x v n+1 = e5−0 ,6(n+1) = e5−0 ,6 n−0 ,6
ex 3 3
1
= e x × e− x = e x+(− x) = e0 = 1 v n+1 = e5−0 ,6 n × e−0 ,6 = e−0 ,6 × v n .
3
e3 x × e 6 −5 e3 x+6+(−5) La suite (v n ) est donc géométrique de raison e-0 ,6
d)  D = × e =
e × e4 x e1+4 x 1 1
et de premier terme v 0 = e5−0 ,6×0 = e5 .
e3 x+1 3 3
= 1+ 4 x
= e3 x+1−(1+4 x)
e 37 La fonction f est dérivable sur .
= e3 x+1−1−4 x = e− x Pour tout nombre réel x, f ′( x) = e x .
2
32 a)  e x × e x = e x+ x = e2 x ≠ e x 38 La fonction f est dérivable sur .
La proposition est donc fausse. Pour tout nombre réel x, f ′( x) = −0 ,1e x .
b)  e0 × e = e0 × e1 = e0+1 = e1 ≠ e0 L’affirmation (2) est exacte.
La proposition est donc fausse.
39 Les fonctions f, g, h, i, j et k sont dérivables sur .
1 x ex Pour tout nombre réel x,
c)  x × e = = e x−( x−2)
e × e−2 e x−2 a)  f ′( x) = e x
= e x− x+2 = e2 ≠ e−2 b)  g ′( x) = 2, 7 e x
La proposition est donc fausse.
c)  h′( x) = 5 e x + 1
0
1 e
d)  = 3 x = e0−3 x = e−3 x d)  i ′( x) = 3 − 3e x
e3x e
La proposition est donc vraie. e)  j ′( x) = 15 x2 − 9 e x
f)  k ′( x) = −e x
33 a)  A = (et − 1)2 = (et )2 − 2 × et × 1 + 12
40 a) La fonction f est le produit de deux fonctions
= e 2 t − 2e t + 1
dérivables sur , donc dérivable sur .
b)  B = e2t (et − e−2t ) = e2t × et − e2t × e−2t Pour tout nombre réel t,
= e 3t − e 0 = e 3t − 1 u(t ) = 1 − t u ′(t ) = −1
t
c)  C = 3et (et − e−t ) − 5e2t v(t ) = e v ′(t ) = et

= 3et × et − 3et × e−t − 5e2t f ′(t ) = −1× et + (1 − t ) × et


= 3e2t − 3e0 − 5e2t f ′(t ) = et (−1 + 1 − t )
= −2e 2 t − 3 f ′(t ) = −t et
92

172909_Chap06_000-000.indd 92 25/07/2019 19:20:56


41 Les fonctions f, g et h sont les produits de deux 44 La fonction f est dérivable sur .
fonctions dérivables sur  donc dérivables sur . Pour tout nombre réel x,
Pour tout nombre réel x, f ′( x) = 5 × e5 x = 5e5 x
a)  u( x) = 2 x − 7 u ′( x) = 2
x 45 La fonction f est dérivable sur .
v( x) = e v ′( x) = e x
Pour tout nombre réel x,
f ′( x) = 2 × e x + (2 x − 7) × e x f ′( x) = −5, 8 × e−5,8 x+1 = −5, 8e−5,8 x+1
f ′( x) = e x (2 + 2 x − 7) L’affirmation (3) est donc exacte.
f ′( x) = e x (2 x − 5)
46 a) La fonction f est dérivable sur .
b)  u( x) = x u ′( x) = 1 Pour tout nombre réel x,
v( x) = e x v ′( x) = e x f ′( x) = 0 , 7 × e0 ,7 x−8 = 0 , 7e0 ,7 x−8
g ′( x) = 1× e x + x × e x b) La fonction g est dérivable sur .
Pour tout nombre réel x,
g ′( x) = e x (1 + x)
g ′( x) = 6 × 2 × e2 x + 1 = 12e2 x + 1
c)  u( x) = 3 x2 − 2 u ′( x) = 6 x c) La fonction h est le produit de deux fonctions déri-
v( x) = e x v ′( x) = e x vables sur  donc h est dérivable sur .
h′( x) = 6 x × e x + (3 x2 − 2) × e x Pour tout nombre réel x,
h′( x) = e x (6 x + 3 x2 − 2) u( x) = 2 x + 1 u ′( x) = 2
− 0 ,2 x
v( x) = e v ′( x) = −0 , 2 × e−0 ,2 x
h′( x) = e x (3 x2 + 6 x − 2)
h′( x) = 2 × e−0 ,2 x + (2 x + 1) × (−0 , 2e−0 ,2 x )
42 La fonction f est le quotient de deux fonctions déri- h′( x) = e−0 ,2 x (2 + (2 x + 1) × (−0 , 2))
vables sur [0 , 5 ; + ∞[ donc dérivable sur [0 , 5 ; + ∞[. h′( x) = e−0 ,2 x (2 − 0 , 4 x − 0 , 2)
Pour tout nombre réel x de l’intervalle [0 , 5 ; + ∞[ , h′( x) = e−0 ,2 x (1, 8 − 0 , 4 x)
u( x) = e x u ′( x) = e x
v( x) = x v ′( x) = 1 47 f( x) f ′( x)
x x x
e × x − e × 1 e ( x − 1)
f ′( x) = = e2x • • 4e2x
x2 x2
2e x • • 2e-2x
43 a) La fonction f est le quotient de deux fonc-
2e2x • • 2e2x
tions dérivables sur  donc dérivable sur .
Pour tout nombre réel x, -e-2x • • 2e x

u( x) = 3 x + 1 u ′( x) = 3
x 48 La fonction f est dérivable sur .
v( x) = e v ′( x) = e x
Pour tout nombre réel t,
3 × e x − (3 x + 1) × e x  7 − t
7
f ′( x) = f ′(t ) = 4 ,1× −  × e 3
(e x )2  3 
7 7
e x (3 − 3 x − 1) 2 − 3 x −28 , 7 −3 t −287 −3 t
f ′( x) = = f ′(t ) = e = e
(e x )2 ex 3 30
b) La fonction f est le quotient de deux fonctions déri- 49 a) La fonction f est dérivable sur . Pour tout
vables sur  donc dérivable sur . nombre réel x,
Pour tout nombre réel t, u( x) = e−2 x+1 u ′( x) = −2e−2 x+1
u(t ) = 1 + et u ′(t ) = et v( x) = e 5 x −4
v ′( x) = 5e5 x−4
t
v(t ) = e v ′(t ) = et
−2e−2 x+1 × e5 x−4 − e−2 x+1 × 5e5 x−4
t t t t f ′( x) =
e × e − (1 + e ) × e (e5 x−4 )2
f ′(t ) =
(et )2 −7e5 x−4 × e−2 x+1 −7e−2 x+1
f ′( x) = =
et (/e/t − 1 − /e/t ) −et 1 (e5 x−4 )2 e 5 x −4
f ′(t ) = t 2
= t 2 =− t
(e ) (e ) e f ′( x) = −7e−2 x+1−(5 x−4) = −7e−7 x+5

Chapitre 6  ★  Fonction exponentielle 93

172909_Chap06_000-000.indd 93 25/07/2019 19:23:50


e−2 x+1 54 a) La fonction f est le produit de deux fonctions
b)  f( x) = 5 x −4
= e−2 x+1−(5 x−4) = e−7 x+5 dérivables sur  donc f est dérivable sur .
e
Pour tout nombre réel x,
Ainsi f ′( x) = −7e−7 x+5
u( x) = 4 − 3 x u ′( x) = −3
x
50 a) La fonction f est dérivable sur . v( x) = e v ′( x) = e x
Pour tout nombre réel x, f ′( x) = −3 × e x + (4 + 3 x) × e x
f ′( x) = 2e2 x .
f ′( x) = e x (−3 + 4 − 3 x)
Or pour tout nombre réel x, e2 x > 0 et 2 > 0 donc
f ′( x) > 0. f ′( x) = e x (1 − 3 x)
La fonction f est donc croissante sur  et l’affirma- f ′( x) = −e x (3 x − 1)
tion est exacte. b) Pour tout nombre réel x, e x > 0 donc f ′( x) est du
b) La fonction f est dérivable sur . signe de 1 - 3 x.
Pour tout nombre réel x, 1 1− 3 x = 0
x −∞ +∞
f ′( x) = −3e−3 x . 3 −3 x = −1
Or pour tout nombre réel x, e−3 x > 0 et −3 < 0. f ¢(x) + 0 - x=
−1
Ainsi f ′( x) < 0 et la fonction f est décroissante sur . −3

L’affirmation est donc fausse. c) 


1
51 Pour tout nombre réel x, 6 , 3 > 0 et e−2 x > 0 x −∞ +∞
3
donc f ′( x) > 0. f ¢(x) + 0 -  1
La fonction f est donc croissante sur  et Pauline a f   ≈ 4 , 2
 1  3
raison. f  
f (x)  3 

52 La fonction g est dérivable sur .


Pour tout nombre réel x,
55 La fonction f est le produit de deux fonctions
g ′( x) = 5 × (−4 , 5)e−4 ,5 x = −22, 5e−4 ,5 x .
dérivables sur  donc f est dérivable sur .
Or pour tout nombre réel x, −22, 5 < 0 et e−4 ,5 x > 0
Pour tout nombre réel x,
donc g ′( x) < 0.
La fonction g est donc décroissante sur . u( x) = x2 − 4 u ′( x) = 2 x
x
v( x) = e v ′( x) = e x
53 a) La fonction f est le produit de deux fonctions f ′( x) = 2 x × e x + ( x2 − 4) × e x
dérivables sur  donc f est dérivable sur .
f ′( x) = e x ( x2 + 2 x − 4)
Pour tout nombre réel x,
Pour tout nombre réel x, e x > 0 donc f ′( x) est du
u( x) = 6 x − 2 u ′( x) = 6
signe de x2 + 2 x − 4.
v( x) = e x v ′( x) = e x
f ′( x) = 6 × e x + (6 x − 2) × e x x −∞ -1 - 5 −1 + 5 +∞

f ′( x) = e x (6 + 6 x − 2) f ¢(x) + 0 - 0 +
x
f ′( x) = e (4 + 6 x) f(−1− 5 )
b) Pour tout nombre réel x, e x > 0, f ′( x) est donc du f (x)
signe de 4 + 6 x. f(−1 + 5 )

2 4 + 6x = 0 ∆ = 22 − 4 × 1× (−4) = 20
x −∞ - +∞
3 6 x = −4
−2 − 20
f ¢(x) - 0 + x=
−4 x1 = = −1 − 5
6 2
 2 x2 = −1 + 5
c) La fonction f est donc décroissante sur  −∞ ; − 
 3  f(−1 − 5 ) ≈ 0 , 25
 2 
et croissante sur l’intervalle  − ; + ∞  . f(−1 + 5 ) ≈ −8 , 51
 3 
94

172909_Chap06_000-000.indd 94 25/07/2019 19:26:20


56 La fonction g est le quotient de deux fonctions 59 Pour tout nombre réel t de l’intervalle [0 ; 1, 5],
dérivables sur [1 ; 3], donc g est dérivable sur [1 ; 3]. 2t < 3t
Pour tout nombre réel t, Or la fonction exponentielle est croissante sur l’inter-
u(t ) = et u ′(t ) = et valle [0 ; 1, 5] donc e2t < e3t et f (t ) < g(t ).
v(t ) = 2t v ′(t ) = 2 La fonction f est donc représentée par la courbe #1
et × 2t − et × 2 et (2t − 2) 2t − 2 et la fonction g par la courbe # 2 .
g ′(t ) = = =
(et )2 (et )2 et
60 a) La fonction f est dérivable sur  et pour tout
Pour tout nombre réel t de [1 ; 3], et > 0, g ′(t ) est
nombre réel x,
donc du signe de 2t - 2.
f ′( x) = 0 , 82e0 ,82 x .
t 1 3 Pour tout nombre réel x, e0 ,82 x > 0 et 0 , 82 > 0 donc
2t − 2 = 0 f ′( x) > 0 et f est croissante sur .
g¢(t) 0 +
2t = 2 x -3 -2 -1 0 1 2 3
e3 2
g(t) t = =1 f (x) 0,09 0,19 0,44 1 2,27 5,16 11,71
e1 6 2
2
c) 
11
10
57 La fonction h est le quotient de deux fonctions
9
dérivables sur , donc h est dérivable sur .
8
Pour tout nombre réel x,
7
u( x) = x2 + 2 x u ′( x) = 2 x + 2
x
6
v( x) = e v ′( x) = e x
5
(2 x + 2) × e x − ( x2 + 2 x) × e x 4
h′( x) =
(e x )2 3
e x (2 x + 2 − x2 − 2 x) −x2 + 2 2
h′( x) = =
(e x )2 ex 1
Pour tout nombre réel x, e x > 0, g ′(t ) est donc du -3 -2-1 0 1 2 3
signe de −x2 + 2.
1
Échelle :
x −∞ - 2 2 +∞ 2
h¢(x) - 0 + 0 - 61 #f #g #i

h( 2 )
h(x)
h(− 2 ) #h
1
2
−x + 2 = 0
0 1
x2 = 2
x = − 2 ou x = 2 62 La fonction h est le quotient de deux fonctions
h(− 2 ) ≈ −3, 41 dérivables sur l’intervalle [−3 ; 1], h est donc dérivable
h( 2 ) ≈ 117
, sur l’intervalle [−3 ; 1].
Pour tout nombre réel x de l’intervalle [−3 ; 1],
58 f(1) = e1 g(1) = e−0 ,5 h(1) = e0 ,1   u( x) = 5 − 4 x u ′( x) = −4
−0 , 5 < 0 ,1 < 1 x
v( x) = e v ′( x) = e x
Or la fonction exponentielle est croissante sur  donc
h′( x) = −4 × e x + (5 − 4 x) × e x
e−0 ,5 < e0 ,1 < e1
g(1) < h(1) < f (1). h′( x) = e x (−4 + 5 − 4 x)
La courbe # 3 est donc la courbe représentative de la h′( x) = e x (1 − 4 x)
fonction f, la courbe #1 est celle de la fonction h et la b) Pour tout nombre réel x de [−3 ; 1], e x > 0, h′( x)
courbe # 2 est celle de la fonction g. est du signe de 1 - 4 x.

Chapitre 6  ★  Fonction exponentielle 95

172909_Chap06_000-000.indd 95 25/07/2019 19:27:39


x -3 0,25 1 b) La fonction f est dérivable sur l’intervalle [0 ; 5] et
pour tout nombre réel t de l’intervalle [0 ; 5],
1− 4 x = 0
h¢(x) + 0 - f ′(t ) = 35 × (−1, 6)e−1,6t
4x = 1
4 e0 ,25 1 f ′(t ) = −56e−1,6t
h(x) x=
4 Pour tout nombre réel t de [0 ; 5], −56 < 0 et
17e-3 e1 e−1,6t > 0 , donc f ′(t ) < 0.
c)  La température des frites est donc décroissante.
c)  f(1, 5) = 35e−1,6×1,5 − 30  −26 , 8
f(1, 5) < −24
La température sera donc conforme.
#h
1 68 a) Il semble que la fonction f admette un maxi-
mum en x = 1, 5.
0 1
Il faut donc vendre environ 150 parois de douche
pour réaliser un bénéfice maximum.
63 f(5) ≈ 13, 5 ce qui signifie qu’à 5 h il y a 13,5 mg
de médicament dans le sang. b)  f(1, 5) = (5 − 2 × 1, 5)e1,5 = 2e1,5  8 , 963.
f(13, 5) = 100e−0 ,4×13,5 ≈ 0 , 45 Le bénéfice maximum est de 8 963 €.
Au bout de 13 h 30, la quantité de médicament est de
69 1. C      2. B     3. A     4. C     5. D
0,45 mg.
Stéphane a donc tort.
70 1. B, C, D     2. A, D     3. A, B
64 La fonction f semble admettre un maximum
71 1. L’affirmation est vraie.
égal à 23, atteint en x = 3.
En effet,
Le bénéfice maximum est donc d’environ 2 300 €.
e x × (e− x+1)2 e x × e−2 x+2 e x −2 x +2
L’affirmation (1) est exacte. g( x) = = =
e− x+4 e− x+4 e− x+4
65 a) La fonction f est dérivable sur l’intervalle e− x+2−(− x+4) = e− x+2+ x−4 = e−2
[0 ; 10 ] et pour tout nombre réel x de l’intervalle La fonction g est donc une fonction constante.
[0 ; 10 ], 2. L’affirmation est vraie.
f ′( x) = 1013, 25 × (−0 ,12)e−0 ,12 x En effet, pour tout nombre n de ,
un+1 = 5e0 ,1(n+1)−1 = 5e0 ,1n+0 ,1−1
f ′( x) = −121, 59e−0 ,12 x
Or pour tout nombre réel x, e−0 ,12 x > 0 et un+1 = 5e0 ,1n−1 × e0 ,1
−121, 59 < 0 donc f ′( x) < 0. = e0 ,1un
La fonction f est donc décroissante sur l’intervalle
La suite (un ) est donc géométrique de raison
[0 ; 10 ].
q = e0 ,1 et de premier terme :
Ainsi plus l’altitude x augmente, plus la pression
5
atmosphérique diminue. u0 = 5e0 ,1×0−1 = 5e−1 = 1 .
e
b) À partir de 9 m, l’altitude reste inférieure à 350 hec-
3. L’affirmation est fausse.
topascals.
En effet, f(−2)  −0 , 812, f(−1, 9)  −0 , 8376 et
66 a) La fonction f modélise la valeur de la machine, donc f (−1, 9) < f (−2).
en milliers d’euros, t années après l’acquisition de 4. L’affirmation est fausse.
celle-ci. En effet, pour tout nombre réel x, e x > 0.
b) En tabulant f à l’aide de la calculatrice on obtient :
f(6 , 9)  5, 0158 et f(7)  4 , 9659. 72 a)  e6 × e 4 = e6+4 = e10
C’est donc au bout de 7 ans que la machine aura b)  (e−1,5 )4 = e−1,5×4 = e−6
perdu la moitié de sa valeur.
e11
c)  = e11−6 = e5
67 a)  f(0) = 35e−1,6×0 − 30 = 35 × e0 − 30 = 5. e6
La température des frites à l’entrée du tunnel est de 5 °C. d)  e5 x+6 × e−8 x = e5 x+6−8 x = e−3 x+6
96

172909_Chap06_000-000.indd 96 25/07/2019 19:28:40


e)  (e 4 x−3 )2 = e2(4 x−3) = e8 x−6

f) 
e7 x
= e7 x−( x+5) = e7 x− x−5 = e6 x−5
S’entraîner
e x +5

73 Les fonctions f, g, h et i sont dérivables sur .


a) Pour tout nombre réel x,
78 a)
f ′( x) = 5e x
t > 50 Vrai Vrai Vrai Vrai Vrai Vrai Vrai Faux
b) Pour tout nombre réel x,
n 0 1 2 3 4 5 6 7 –
g ′( x) = 3, 6 × (−0 , 5)e−0 ,5 x = −1, 8e−0 ,5 x
t 100 89,6 80,3 71,9 64,4 57,7 51,7 46,3 –
c) Pour tout nombre réel x,
u( x) = 3 x + 5 u ′( x) = 3 b) La valeur de n obtenue à la fin de l’exécution de
v( x) = e x
v ′( x) = e x l’algorithme est 7.
Cela signifie que le ventilateur s’arrête au bout de
h′( x) = 3 × e x + (3 x + 5) × e x
7 minutes.
h′( x) = e x (3 + 3 x + 5)
79
h′( x) = e x (3 x + 8)
d) Pour tout nombre réel x, s < 3, 9 Vrai Vrai Vrai Vrai Vrai Vrai
u( x) = 2 x u ′( x) = 2 n 0 1 2 3 4 5 6
x s 0,12 0,51 0,89 1,3 1,6 2,0 2,4
v( x) = e v ′( x) = e x
s < 3, 9 Vrai Vrai Vrai Vrai Vrai Faux
2 × ex − 2x × ex
i ′( x) = n 7 8 9 10 11 –
(e x )2
s 2,7 3,1 3,5 3,8 4,2 –
e x (2 − 2 x)
= b) La valeur de n obtenue à la fin de l’exécution de
(e x )2
l’algorithme est n = 11.
2 − 2x
= Cela signifie que l’eau n’est plus utilisable 11 minutes
ex après le début de l’incident.
74 Pour tout réel x, e x > 0 donc f ′( x) est du signe 81 1. 
de 25 - 10 x.
x −∞ 2,5 +∞
f ¢(x) + 0 -

f (x)
Il semble que, quel que soit le nombre réel a, la
courbe # est située au-dessus de la tangente T.
75 # 2. a) Une équation de la tangente T est :
f
#g y = f ′(a)( x − a) + f (a)
1 c’est-à-dire y = −e−a ( x − a) + e−a .
b) La fonction d est dérivable sur  et
0 1 d ′( x) = −e− x + e−a .
Or, d ′( x) < 0 si, et seulement si, -e- x > -e-a ,
soit e- x < e-a ,
76 On résout graphiquement f( x) > 2. c’est-à-dire -x < -a et x > a.
On obtient comme ensemble solutions l’intervalle x −∞ a +∞
[5 ; 8 ].
d¢(x) - 0 +
En tabulant la fonction f avec la calculatrice avec un
pas de 0,001 on obtient f(5,198) < 2 et f(5,199) > 2.
d(x)
C’est donc à partir de 5  199  personnes connectées
simultanément que la durée de chargement dépasse 0
2 s. Ainsi, pour tout nombre réel x, d( x) > 0.

Chapitre 6  ★  Fonction exponentielle 97

172909_Chap06_000-000.indd 97 25/07/2019 19:29:25


c) Quel que soit le nombre réel a et pour tout nombre • Si a < b alors a − b < 0 et on obtient :
réel x,
x −∞ 0 +∞
e− x > −e−a ( x − a) + e−a .
(a - b)t + 0 -
Ainsi, quel que soit le nombre réel a, la courbe # est
située au-dessus de la tangente T. • Si a > b alors a − b > 0 et on obtient :
x −∞ 0 +∞
82 1. 
(a - b)t - 0 +

Ainsi :
• Si a > b , # f et # g sont confondues.
•  Si a < b , # f est au-dessus de # g sur l’intervalle
]− ∞ ; 0[ ,
# f est en dessous de # g , sur l’intervalle ]0 ; + ∞[
Il semble que, quel que soit le nombre réel a, la et # f et # g se coupent en t = 0.
courbe # est située au-dessous de la tangente T. •  Si a > b , # f est au-dessus de # g sur l’intervalle
2. a) Une équation de la tangente T est : ]0 ; + ∞[ , # f est en dessous de # g sur l’intervalle
y = g ′(a)( x − a) + g(a) ]− ∞ ; 0[ et # f et # g se coupent en t = 0.
y = −e−a ( x − a) + 2 − ea .
85 Christophe pense que les deux courbes qu’il a
b) La fonction d est dérivable sur  et
tracées à l’écran de sa calculatrice sont confondues
d ′( x) = −e x + ea .
sur l’intervalle ]− ∞ ; − 3].
d ′( x) > 0 si, et seulement si -e x > -ea , soit
Fatima pense qu’une exponentielle peut être nulle.
e x < ea ,
5(1 + e 4 x+7 ) = 5
c’est-à-dire x < a.
c’est-à-dire 1 + e 4 x+7 = 1 donc e 4 x+7 = 0 ce qui est
x −∞ a +∞ impossible.
d¢(x) + 0 - L’équation n’a donc aucune solution.
0
d(x) 86 a) La fonction g est dérivable sur .
Pour tout nombre réel x,
u( x) = 3 − x u ′( x) = −1
Ainsi, pour tout nombre réel x, d( x) < 0. x
v( x) = e v ′( x) = e x
c) Quel que soit le nombre réel a et pour tout nombre
réel x, g ′( x) = −1× e x + (3 − x) × e x
−e x < −ea ( x − a) − ea g ′( x) = e x (−1 + 3 − x)
donc 2 − e x < −ea ( x − a) + 2 − ea . g ′( x) = e x (2 − x)
Quel que soit le nombre réel a, la courbe # est située b) Pour tout nombre réel x, e x > 0, donc g ′( x) est
au-dessous de la tangente T. du signe de 2 - x.
x −∞ 2 +∞
83 Pour tout nombre réel x,
g¢(x) + 0 -
(e x − 1)(e x + 1) (e x )2 − 12 e2 x − 1
= =
e2 x e2 x e2 x e2
g(x)
e2 x 1
= 2 x − 2 x = 1 − e−2 x
e e
c) Une équation de la tangente T0 à # g au point
84 Pour tout nombre réel t, d’abscisse 0 est :
f (t ) − g(t ) = eat − ebt T0 : y = g ′(0)( x − 0) + g(0)
f (t ) − g(t ) > 0 si, et seulement si eat - ebt > 0 y = 2x + 3
soit  eat > ebt c’est-à-dire at > bt ou encore En effet g ′(0) = e0 (2 − 0) = 2 et
(a − b)t > 0. g(0) = (3 − 0)e0 = 3 .
98

172909_Chap06_000-000.indd 98 25/07/2019 19:30:34


d)  89 Pour tout nombre réel x, e x > 0 donc f( x) est
7 du signe de −20 x2 + 43 x − 2,1.
6
x −∞ 0,05 2,1 +∞
5
4 f(x) - 0 + 0 -
3 −20 x2 + 43 x − 2,1 = 0
2 ∆ = 1681
1
−43 − 1681
x1 = = 2 ,1
- 4 - 3- 2 - 10 1 2 3 2 × (−20)
−43 + 1681
87 a) Les fonctions f et g sont dérivables sur  et x2 = = 0 , 05
2 × (−20)
pour tout nombre réel x,
Ainsi f( x) > 0 pour x ∈ [0 , 05 ; 2,1] et f( x) < 0 sinon.
u( x) = 2 − x u ′( x) = −1
v( x) = e x v ′( x) = e x
f ′( x) = −1× e x + (2 − x) × e x 90 a) f est dérivable sur l’intervalle [0 ; 3].
f ′( x) = e x (−1 + 2 − x) Pour tout nombre réel x de l’intervalle [0 ; 3],
f ′( x) = e x (1 − x) 1
u( x) = ( x − 4) u ′( x) =
1
4 4
u( x) = x2 u ′( x) = 2 x
v( x) = e x v ′( x) = e x v( x) = e x v ′( x) = e x
g ′( x) = 2 x × e x + x2 × e x 1 1
f ′( x) = 0 + × e x + ( x − 4) × e x
g ′( x) = e x ( x2 + 2 x) 4 4
b) Pour tout nombre réel x, e x > 0, donc f ′( x) est du  1 1 
f ′( x) = e x  + x − 1
 4 4 
signe de 1- x et g ′( x) est du signe de x2 + 2 x.
 −3 1  
x −∞ 1 +∞ f ′( x) = e x  + x
 4 4 
f ¢(x) + 0 -
Pour tout nombre réel x de l’intervalle [0 ; 3], e x > 0
et −3 1
donc f ′( x) est du signe de + x.
4 4
x −∞ -2 0 +∞ x2 + 2 x = 0
x 0 3
g¢(x) + 0 - 0 + x( x + 2) = 0 −3 1
+ x=0
f ¢(x) - 0 4 4
c) La fonction f est croissante sur l’intervalle ]− ∞ ; 1,] 1 3
x=
la courbe # 2 est donc celle de la fonction f et la f(x) 4 4
courbe #1 est celle de la fonction g. x=3

88 a) La fonction f est dérivable sur . La fonction f est donc décroissante sur l’intervalle [0 ; 3].
Pour tout nombre réel x, L’affirmation est donc vraie.
u( x) = x2 + 1 u ′( x) = 2 x b) La tangente T a pour équation :
x
v( x) = e v ′( x) = e x y = f ′(0)( x − 0) + f (0)
3
2 x × e x − ( x2 + 1) × e x y =− x+4
f ′( x) = 4
(e x )2
L’affirmation est donc fausse.
e x (2 x − x2 − 1) c) La tangente à la courbe # au point d’abscisse 2 a
=
(e x )2 pour équation :
−x2 + 2 x − 1 −( x − 1)2 y = f ′(2)( x − 2) + f (2)
= =
ex ex −e2 e2
y= ( x − 2) + 5 −
Pour tout nombre réel x, e x > 0 et ( x − 1)2 > 0 donc 4 2
−( x − 1)2 −e 2
e 2
e 2
−e2
< 0. = x+ − +5 = x+5
ex 4 2 2 4
La fonction f est donc décroissante sur . L’affirmation est donc fausse.

Chapitre 6  ★  Fonction exponentielle 99

172909_Chap06_000-000.indd 99 25/07/2019 19:31:38


91 La fonction f est dérivable sur l’intervalle d)  #g
[−4 ; 1, 5].
Pour tout nombre réel x de l’intervalle [−4 ; 1, 5], 1
u( x) = −2 x2 + 3 x u ′( x) = −4 x + 3 0 1
x
v( x) = e v ′( x) = e x
T
f ′( x) = (−4 x + 3) × e x + (−2 x2 + 3 x) × e x
f ′( x) = e x (−2 x2 − x + 3).
Pour tout nombre réel x de l’intervalle [−4 ; 1, 5],
94 a)  f(0) = 3 − 2e−5×0 = 3 − 2e0 = 1
e x > 0, f ′( x) est donc du signe de −2 x2 − x + 3.
b) La fonction f est dérivable sur l’intervalle [0 ; 1] et
x -4 -1,5 1 1,5 pour tout nombre réel x de l’intervalle [0 ; 1],
f ¢(x) - 0 + 0 - f ′( x) = −2 × (−5) × e−5 x = 10e−5 x
f(−4) f(1) Pour tout nombre réel x de l’intervalle [0 ; 1],
f (x) e−5 x > 0 et 10 > 0 donc f ′( x) > 0.
f(−1, 5) 0 La fonction f est donc croissante sur l’intervalle [0 ; 1].
c) La fonction f est croissante sur l’intervalle [0 ; 1],
−2 x 2 − x + 3 = 0
donc pour tout nombre réel x de l’intervalle [0 ; 1] :
∆ = 25
f ( x) > f (0) soit f( x) > 1.
1− 5
x1 = =1 La fonction f est donc strictement positive sur l’inter-
−4
valle [0 ; 1].
1+ 5
x2 = = −1, 5 d)  f( x) = 3 lorsque 3 − 2e−5 x = 3
−4
c’est-à-dire −2e−5 x = 0 ou encore e−5 x = 0 ce qui
f(−4) ≈ −0 , 81
est impossible.
f(−1, 5) ≈ −2, 01 L’équation f( x) = 3 n’admet donc aucune solution.
f(1) ≈ 2, 72
95 1. a) Il a voulu résoudre l’équation 2 + e5 x = 10.
Une fenêtre graphique adéquate est alors :
b) L’équation admet une solution a.
xmin = −4
0 , 41 < α < 0 , 42.
xmax = 1, 5 2. L’équation e-0,55x-4 = 2 a pour solution x  -8 , 5.
ymin = −2,1
ymax = 2, 8 −1
96 a)  g ′(0) =
2
92 a) La fonction f est dérivable sur  et pour tout b) La fonction g est dérivable sur  et pour tout
nombre réel x, nombre réel t,
f ′( x) = 1, 5 × 1, 6e1,6 x = 2, 4 e1,6 x g ′(t ) = kekt
1
b) Pour tout nombre réel x, e1,6 x > 0 et 2, 4 > 0 0 1 − t

c)  g (0) = ke = k = − et g(t ) = e .
2
donc f ′( x) > 0. 2
La fonction f est donc croissante sur .
97 a) Pour tout nombre n de ,
c) Pour tout nombre réel x, e1,6 x > 0 et 1, 5 > 0 donc
un+1 = 5e−0 ,2(n+1) = 5e−0 ,2n−0 ,2
f( x) > 0.
un+1 = 5e−0 ,2n × e−0 ,2 = e−0 ,2un .
93 a) La fonction g est dérivable sur . La suite (un ) est donc géométrique de raison e-0 ,2
Pour tout nombre réel t, et de premier terme u0 = 5e0 = 5.
b)  S = u0 + u1 +  + u6
g ′(t ) = 0 , 2 × (−1)e−t = −0 , 2e−t
b) Pour tout nombre réel t, de e−t > 0 et −0 , 2 < 0 S = 5 + 5e−0 ,2 + 5(e−0 ,2 )2 +  + 5(e−0 ,2 )6
donc g ′(t ) < 0. S = 5(1 + e−0 ,2 + (e−0 ,2 )2 +  + (e−0 ,2 )6 )
La fonction g est donc décroissante sur . 1 − (e−0 ,2 )6+1 1 − e−1,4
c) La tangente T a pour équation : S = 5× = 5 ×
1 − e−0 ,2 1 − e−0 ,2
y = g ′(0)(t − 0) + g(0) Ainsi S  20 , 781
y = −0 , 2t + 0 , 2 − 2 = −0 , 2t − 1, 8 L’écrivain a vendu 20 781 livres de 2012 à 2018.
100

172909_Chap06_000-000.indd 100 25/07/2019 19:32:59


98 Partie A 120et (et + 3) − 120et × et
a)  f(1) = 10(1 − e−0 ,3 ) ≈ 2, 59 f ′(t ) =
(et + 3)2
b) La fonction f est dérivable sur l’intervalle [0 ; 5].
Pour tout nombre réel t de l’intervalle [0 ; 5], 120e2t + 360et − 120e2t
f ′(t ) =
f ′(t ) = 10 × (0 − (−0 , 3)e−0 ,3t ) = 3e−0 ,3t . (et + 3)2
Pour tout nombre réel t de [0 ; 5], e−0 ,3t > 0 et 3 > 0 360et
f ′(t ) =
donc f ′(t ) > 0. (et + 3)2
La fonction f est donc croissante sur l’intervalle [0 ; 5]. c) Pour tout nombre réel t de l’intervalle de [0 ; + ∞[ ,
c) La fonction f admet f(5)  7, 8 comme maximum et > 0, 360 > 0 et (et + 3)2 > 0 , donc f ′(t ) > 0.
sur l’intervalle [0 ; 5]. d)  t 0 +∞
L’équation f (t ) = 10 n’admet donc aucune solution.
f ¢(t) +
d)  2, 31 < α < 2, 32.
Partie B
a) La superficie de forêt brûlée après une journée est f (t)
d’environ 2,6 hectares. 30
20 e) La fonction f est strictement croissante sur l’inter-
b)  50 × = 10.
100 valle [0 ; + ∞[ , f(2) < 100 et f(3) > 100.
20 % de 50 hectares représente 10 hectares. Au bout de 3 mois, le chiot dépassera 1 m.
Or f (t ) = 10 n’a pas de solution.
L’affirmation du journaliste est incorrecte. 101 a) Pour tous nombres réels x et y,
c) 5  hectares de forêt ont été brûlés, au bout de ex + e y ex ey
x+ y
= x + y + x + y = e x− x− y + e y − x− y
2,32 jours. e e e
= e− y + e− x .
99 1. a) La suite (un ) est géométrique de raison
L’affirmation est donc vraie.
e0 ,02 et de premier terme u0 = 1.
b) La fonction f : x  e-4 x-5 est dérivable sur [0 ; + ∞[.
b) Pour tout nombre n de , Pour tout nombre réel x positif f ′( x) = −4 e−4 x−5
un = u0 × q n = 1× (e0 ,02 )n = e0 ,02n Pour tout nombre réel x positif, e−4 x−5 > 0 et
c) −4 < 0 donc f ′( x) < 0 et f est décroissante sur l’in-
tervalle [0 ; + ∞[.
Or f(0) ≈ 0 , 007 soit f(0) < 1.
L’affirmation est donc vraie.
fenêtre : −1 < X < 10 , pas 1 et −0 ,1 < Y < 2, pas 0,1. c) La fonction exponentielle est strictement crois-
La courbe # f semble réaliser un prolongement sante sur  donc sur ] − ∞ ; 0 ].
continu du nuage de points. Pour tout nombre réel x négatif, e x < e0 soit e x < 1.
b)  f(0 , 5) = e0 ,01 ≈ 1, 010 L’affirmation est donc fausse.
Au 1er juillet 2018, le village comptait 1 010 habitants. d)  e−1 + e−2 ≈ 0 , 5 et e−1+(−2) = e−3 ≈ 0 , 05 donc
f(1, 5) = e0 ,03 ≈ 1, 030 e−1 + e−2 > e−1+(−2)
L’affirmation est donc vraie.
Au 1er juillet 2019, le village comptait 1 030 habitants.
73 e) Pour tout nombre réel x, e x+3 = e x × e3 donc
 1  73 
f 6 +  = f   = e 600 ≈ 1129
, a = e3 .
 12  12 
L’affirmation est donc vraie.
Au 1er juillet 2024, le village comptera 1 129 habitants.
102 a) La fonction exponentielle est strictement
120e0 120 croissante sur  donc :
100 a)  f(0) = 0 = = 30
e + 3 4 si x = y , alors e x = e y et l’affirmation est vraie.
Le chiot mesurait 30 cm le jour de l’adoption. b) Pour les mêmes raisons que précédemment,
b) La fonction f est dérivable sur [0 ; + ∞[. si e x < e y alors x < y.
Pour tout nombre réel t de l’intervalle [0 ; + ∞[ , L’affirmation est exacte.
u(t ) = 120et u ′(t ) = 120et c)  −2 < 0 et e−(−2) = e2 ≈ 7, 4 et e2 > 1
v(t ) = et + 3 v ′(t ) = et L’affirmation est donc fausse.

Chapitre 6  ★  Fonction exponentielle 101

172909_Chap06_000-000.indd 101 25/07/2019 19:34:15


−7 −7
103 a)  e ≈ 0 , 00091 donc e < 0 , 001. L’entreprise doit donc vendre 1  870  cafetières pour
Il existe un nombre réel x, tel que e x < 0 , 001. obtenir un bénéfice maximal de 50 435 €.
b) La fonction f définie sur  par f (t ) = e−2t est
décroissante sur . 106 a)  a  0 , 33.
Il existe un nombre réel a, tel que la fonction f définie b) La fonction exponentielle est strictement crois-
sur  par f (t ) = eat n’est pas croissante sur . sante sur  donc e x = e y équivaut à x = y.
c)  f( x) = 0 lorsque e x = e 4 x−1 ce qui équivaut à
1
x = 4 x − 1 soit −3 x = −1 et x = .
3
L’équation f( x) = 0 admet donc comme solution

Organiser son raisonnement


1
x= .
3
107 La fonction f est dérivable sur .
Pour tout nombre réel x,
u( x) = 2 − 0 , 5 x u ′( x) = −0 , 5
104 Pour tout entier n de , x
v( x) = e v ′( x) = e x
e2 ,8(n+1) e 2 ,8 n × e 2 ,8
un+1 = 2 0 ,8(n+1)−1
= 2 0 ,8 n−1 f ′( x) = −0 , 5 × e x + (2 − 0 , 5 x) × e x
e e × e 0 ,8
e 2 ,8 e 2 ,8 n = e x (−0 , 5 x + 1, 5)
un+1 = 2 0 ,8 × 0 ,8 n−1 = 2e2 × un Pour tout nombre réel x, e x > 0 donc f ′( x) est du
e e
La suite (un ) est donc géométrique de raison 2e2 et signe de −0 , 5 x + 1, 5.
e0 x −∞ 3 +∞
de premier terme u0 = 2 = 2e.
e0−1 f¢(x) + 0 -
105 La fonction f est dérivable sur [0 ; 3].
Pour tout nombre réel x de l’intervalle [0 ; 3], f(x)
u( x) = 0 , 22 x2 − 1, 6 x + 3 u ′( x) = 0 , 44 x − 1, 6
v( x) = e x v ′( x) = e x
La fonction f admet donc un maximum en x = 3 égal
f ′( x) = (0 , 44 x − 1, 6) × e x + (0 , 22 x2 − 1, 6 x + 3) × e x à f(3)  10 , 04.
f ′( x) = e x (0 , 22 x2 − 116
, x + 1, 4). Ainsi pour tout nombre réel x, f ( x) < f (3) < 11.
Pour tout nombre réel x de l’intervalle [0 ; 3], e x > 0,
donc f ′( x) est du signe de 0 , 22 x2 − 116
, x + 1, 4. 108 f ( x) = (a x + b)e x .
116
, - 0 ,1136 D’après le tableau de variations f(6) = e6 et
x 0 3 f ′(6) = 0.
0 , 44
La fonction f est dérivable sur  et pour tout nombre
f¢(x) + 0 -
réel x,
u( x) = a x + b u ′( x) = a
f(x) x
v( x) = e v ′( x) = e x
f ′( x) = a × e x + (a x + b) × e x
0 , 22 x2 − 116
, x + 1, 4 = 0
∆ = 0 ,1136 f ′( x) = e x (a x + a + b).
116
, − 0 ,1136 Or f ′(6) = 0, on déduit que :
x1 =  1, 8704
2 × 0 , 22 e6 (6a + a + b) = 0
116
, + 0 ,1136 0
x2 =  3, 40 soit 7a + b = 6 = 0 et b = −7a.
2 × 0 , 22 e
La fonction f admet donc un maximum égal à
De f(6) = e6 , on déduit que :
116, − 0 ,1136 
f   (6a + b)e6 = e6
 0 , 44 
e6
, − 0 ,1136 
116 soit 6a + b = =1
f    5, 0 435 e6
 0 , 44  ainsi 6a + b = 1
102

172909_Chap06_000-000.indd 102 25/07/2019 19:35:33


f ( x)
Or b = −7a donc 6a − 7a = 1 • Pour x = −3 et x = 2 = 1 et f ( x) = g( x).
g( x)
on obtient −a = 1 donc a = −1 et
La conjecture émise au a) est donc démontrée.
b = −7a = −7 × (−1) = 7.
Ainsi f( x) = (−x + 7)e x . 111 1. a) La tangente T0 à # en M0 a pour équation :
T0 : y = f ′(0)( x − 0) + f (0)
109 L’aire !( x) du panneau publicitaire est donné,
y = x +1
pour tout nombre réel x de l’intervalle [0 ; 10 ] par :
!( x) = x f( x) = 4 xe−0 ,4 x b)  N1 est le point de T0 d’abscisse h donc son ordon-
!( x) est dérivable sur l’intervalle [0 ; 10 ] et née est y = 1 + h et N1(h ; 1 + h)
u( x) = 4 x u ′( x) = 4 2. a)  f (h) = 1 + h et f ′(h) = f (h) = 1 + h.
−0 , 4 x
v( x) = e v ′( x) = −0 , 4 e−0 ,4 x La tangente T1 à # en M1 a pour équation :
!′( x) = 4 e−0 ,4 x + 4 x × (−0 , 4) × e−0 ,4 x T1 : y = f ′(h)( x − h) + f (h)
y = (1 + h)( x − h) + 1 + h
!′( x) = e−0 ,4 x (−1, 6 x + 4).
y = x − h + hx − h2 + 1 + h
Pour tout réel x de l’intervalle [0 ; 10 ], e x > 0 donc
y = (1 + h)x − h2 + 1
!′( x) est du signe de 4 - 1, 6 x.
b) Le point N2 de T1 a pour abscisse 2h et a pour
x 0 2,5 10
ordonnée :
!¢(x) + 0 - 4 − 1, 6 x = 0 y = (1 + h)2h − h2 + 1
4 = 2h + 2h2 − h2 + 1
x= = 2, 5
!(x) 1, 6
= h2 + 2h + 1
= (h + 1)2
L’aire sera donc maximale lorsque OB = 2, 5. Donc N2 (2h ; (h + 1)2 )
Les dimensions du panneau publicitaire sont alors de
3. a) 
2,5 m sur 1,47 m.

110 a) Il semble que # f soit au-dessous de # g sur


l’intervalle ]− ∞ ; − 3[ ∪ ]2 ; + ∞[ , que # f et # g se
coupent en x = −3 et x = 2 et que # f soit en des-
sous de # g sur l’intervalle ]− 3 ; 2[.
2
f ( x) 0 ,1e x −6 2
b)  = −x
= e x −6+ x
g( x) 0 ,1e b) 
2
Or e x + x−6 > 1, si et seulement si, x2 + x − 6 > 0
∆ = 25
−1 − 25
Il y a deux solutions x1 = = −3 et
2 ×1
−1 + 25
x2 = = 2.
2 ×1

x −∞ -3 2 +∞

x2 + x - 6 + 0 - 0 + c) Il faut donner la valeur -0 , 01 à h pour que la


courbe soit tracée sur l’intervalle [−4 ; 0 ].
• Ainsi pour tout nombre réel x de l’intervalle
]− ∞ ; − 3[ ∪ ]2 ; + ∞[ , x2 + x − 6 > 0 , c’est-à-dire 1
f ( x) 112 a) Lorsque n est très grand, h = est très petit.
> 1 et f ( x) > g( x). (Pour tout réel x, g( x) > 0 ). n
g( x) 1 n
n×  1
• Pour tout nombre réel x de l’intervalle ]− 3 ; 2[ , On a alors enh ≈ (1 + h)n soit e n ≈ 1 + 
 n
f ( x)
x2 + x − 6 < 0 , soit < 1 et f ( x) < g( x). (Pour  n
g( x) 1
c’est-à-dire e ≈ 1 +  .
tout réel, g( x) > 0 ).  n

Chapitre 6  ★  Fonction exponentielle 103

172909_Chap06_000-000.indd 103 25/07/2019 19:36:58


 1
n
v( x) = e− x+2 v ′( x) = −1e− x+2
lim 1 +   e.
n→+∞  n f ′( x) = 6 × e− x+2 + (6 x − 4) × (−1e− x+2 ) + 2
La suite (un ) définie pour tout n Î  par : f ′( x) = e− x+2 (6 + (6 x − 4) × (−1)) + 2
 1
n
f ′( x) = (10 − 6 x)e− x+2 + 2
un = 1 +  a pour limite le nombre e.
 n f ′( x) = g( x)
b) Lorsque l’on saisit N = 10, l’algorithme affiche : b) 
k 1 2 3 4 5 6 7 8 9 10 x 0,5 2 3
Valeur f¢(x) + 0 -
2 2,25 2,37 2,44 2,49 2,52 2,55 2,57 2,58 2,59
affichée

113 • On sait que f(0) = −1 or e0 − e = 1 − e ≠ −1. f(x)


On élimine donc la 3e fonction.
• La fonction x  e1,5 x - 2 est dérivable sur  et La fonction f admet un maximum en x = 2.
pour tout nombre réel x, sa fonction dérivée est la Il faut donc extraire 2  000  tonnes de minerai pour
fonction x  1, 5e1,5 x . obtenir un résultat d’exploitation maximum.
Or 1, 5 > 0 et e1,5 x > 0 donc la fonction x  e1,5 x - 2
est croissante sur . 115 1. La fonction N est dérivable sur l’intervalle
La fonction f correspondant au tableau de variation [0 ; + ∞[.
est donc la fonction x  e x ( x −1). Pour tout nombre réel t de l’intervalle [0 ; + ∞[ ,
N′(t ) = −0 ,121N0 e−0 ,121t
114 1. a) La fonction g est dérivable sur [0 , 5 ; 3]. −0 ,121N0 < 0 et e−0 ,121t > 0 donc N′(t ) < 0 et la
Pour tout nombre réel x de cet intervalle, fonction N est strictement décroissante sur l’intervalle
u( x) = 10 − 6 x u ′( x) = −6 [0 ; + ∞[.
− x +2
v( x) = e v ′( x) = −1e− x+2 donc N0
2. a)  N(T) =
g ′( x) = −6 × e− x+2 + (10 − 6 x) × (−1e− x+2 ) + 0 2
g ′( x) = e− x+2 (−6 + (10 − 6 x) × (−1)) N0 1
donc N0 e−0 ,121t = et e−0 ,121t =
g ′( x) = e− x+2 (−16 + 6 x) 2 2
b) 
b) Pour tout nombre réel x de l’intervalle [0 , 5 ; 3],
e− x+2 > 0, donc g ′( x) est donc du signe de
−16 + 6x.
8
x 0,5 2 3
3
g¢(x) - 0 + −16 + 6 x = 0
6 x = 16
g(0 , 5) g(3) 8
g(x) 8 x=
0 g  3
 3 

g(0 , 5) ≈ 33, 4 fenêtre : 0 < X < 7, pas 1 et -0 ,1 < Y < 1, 2, pas 0,1.
8 T » 5, 728.
g   ≈ −11
, 3. a)  N(2 T) = N0 e−0 ,121×2 T
 3 
g(3) ≈ −0 , 9 = N0 (e−0 ,121T )2

c)  g(2) = (10 − 6 × 2)e−2+2 + 2 = −2e0 + 2 = 0  1 2


= N0 × 
 2 
d) On obtient le tableau de signes suivant :
N0
x 0,5 2 3 =
4
g(x) + 0 -
b)  2T » 11, 456.
2. a) La fonction f est dérivable sur [0 , 5 ; 3]. Au bout de 11 456 ans le nombre de noyaux radioac-
Pour tout nombre réel x de cet intervalle, tifs de carbone 14 n’est plus égal qu’au quart de sa
u( x) = 6 x − 4 u ′( x) = 6 valeur initiale.
104

172909_Chap06_000-000.indd 104 25/07/2019 19:38:04


e La pièce pourra donc être démoulée au bout de
116 e x−1 × e2 x−5 =
ex 12 h 12 min au minimum.
5. a)  f (t ) = 25
soit e3 x−6 = e1− x ce qui équivaut à 3 x − 6 = 1 − x −5
7 Si 1370 e−0 ,065t + 30 = 25 soit e−0 ,065t = ce
Ainsi 4 x = 7 et x = 1370
4 qui est impossible.
6 = {1, 75}. La pièce ne pourra donc pas atteindre une tempéra-
ture de 25° C.
117 e1 ´ e2 ´ e3 ´ ´ e100 b) Ce résultat était prévisible car la température
= e1+2+3++100 ambiante de la pièce est de 30° C.
(1+100)×100
=e 2

= e5050 .

Exploiter ses compétences


118 La fonction f définie sur  par f( x) = 3 x vérifie
f(1) = 31 = 3.
Pour tous nombres réels x et y,
f ( x + y) = 3 x+ y = 3 x × 3 y = f ( x) × f ( y)

119 On pose X = e x 121 Dater les fragments de charbon de bois revient


L’équation e2 x + 2e x − 3 = 0 s’écrit à résoudre l’équation N0 e−0 ,121t = 0 ,125N0 soit
alors (e x )2 + 2e x − 3 = 0 e−0 ,121t = 0 ,125.
X 2 + 2X − 3 = 0 On obtient une valeur approchée de la solution à
∆ = 22 − 4 × 1× (−3) = 16 l’aide de la calculatrice.

−2 − 16 −2 + 16
X1 = = −3 et X2 = =1
2 2
Or X = e x
L’équation e x = −3 n’admet aucune solution.
L’équation e x = 1 équivaut à e x = e0 donc x = 0.
L’équation e2 x + 2e x − 3 = 0 admet comme solution
le nombre 0.

120 1.  f(0) = 1370e−0 ,065×0 + 30 = 1400.


La température à la sortie du four est de 1 400 °C. fenêtre  : 0 < X < 20 , pas 1 et -0 ,1 < Y < 1, 2, pas
2. a) La fonction f est dérivable sur l’intervalle 0,1.
[0 ; + ∞[. On obtient t  17, 2.
Pour tout nombre réel t de l’intervalle [0 ; + ∞[ ,
Les fragments de charbon de bois ont environ
f ′(t ) = 1370 × (−0 , 065)e−0 ,065t + 0 17 000 ans.
f ′(t ) = −89 , 05e−0 ,065t .
Pour tout nombre réel t > 0, 122 Pour tout nombre réel x,
e−0 ,065t > 0 et −89 , 05 < 0 donc f ′(t ) < 0. f (−x) = f ( x)
La fonction f est donc strictement décroissante sur La fonction f est donc paire sur .
l’intervalle [0 ; + ∞[. La courbe # est donc symétrique par rapport à l’axe
b) Une fois sortie du four, la température de la pièce des ordonnées.
diminue. La hauteur de l’arche est donc f(0).
3.  f(5)  1020 f(0) = 209 , 6 − 8 , 81(e0 + e0 )
Or 1020 > 650 = 191, 98
La pièce ne peut pas être démoulée après 5 h. Pour déterminer la largeur de l’arche, il faut résoudre
4. À l’aide de la calculatrice, on cherche une valeur l’équation f( x) = 0.
approchée de la solution a de l’équation f (t ) = 650. À l’aide de la calculatrice, on obtient une valeur
On obtient a  12,198 approchée des solutions a et b.

Chapitre 6  ★  Fonction exponentielle 105

172909_Chap06_000-000.indd 105 25/07/2019 19:39:00


a  95, 99 β  −95, 99. 124 La tension f vérifie, pour tout t > 0
−1
f ′(t ) = 3 f (t )
10 × 3, 2 × 10−3
D’après le doc. 2 :
1
− t
3 ,2
f (t ) = A e +B
Or à l’instant t = 0, la tension est égale à 4,6 V, ainsi
f(0) = 4 , 6 et A + B = 4 , 6.
1
-
D’après le doc. 3, si t est très grand alors e 3,2  0.
La largeur de l’arche est donc de 191,98 m et sa hau- Or lorsque t est très grand, la tension est nulle, ainsi
teur est également de 191,98 m. A × 0 + B = 0 et B = 0.
Ainsi A = 4 , 6.
123 La représentation de la courbe d’équation
1
polaire r = e0 ,17θ est obtenue à l’aide de GeoGebra. −
3 ,2
t
On obtient f (t ) = 4 ,6 e .
La tangente T au point d’abscisse 0 a pour équation :
T : y = f ′(0)(t − 0) + f (0)
−1
y= × 4 , 6(t − 0) + 4 , 6
3, 2
−4 , 6
y= t + 4, 6
3, 2
La tangente T coupe l’axe des abscisses lorsque
−4 , 6 −4 , 6
t + 4 , 6 = 0 soit en t = = 3, 2
3, 2 −4 , 6
L’épigraphe sur la tombe «  eadem mutata resurgo  »
3, 2
signifie « déplacée, je réapparais à l’identique ».
Lorsque l’on agrandit, ou réduit le repère sur GeoGe-
bra, on a l’impression que la même courbe réapparait.

Le graveur n’a pas tracé une spirale logarithmique


mais une spirale d’Archimède.

106

172909_Chap06_000-000.indd 106 25/07/2019 19:39:23


7 Trigonométrie

π π
b)  cos  ≈ 0 , 81 et sin  ≈ 0 , 59.
Découvrir 5 5
On retrouve les résultats obtenus au 1  c) 
 = 2π et IP
c)  IN  = 4π .
5 5
 2π 
L’abscisse du point N est cos  ≈ 0 , 31
1 Une nouvelle mesure d’angle :  5 
le radian  4π 
L’abscisse du point P est cos  ≈ −0 , 81
5
1  a) Si la mesure de l’angle est 180°, l’arc de cercle
est un demi-cercle de rayon 1. Son périmètre 3  La largeur de la fenêtre vaut donc 0 , 81 + 0 , 31,
vaut donc p m. c’est-à-dire 1,12 m.
b) Pour un angle au centre de 180°, la longueur La hauteur de la fenêtre vaut donc 2 ´ 0 , 59 ,
 vaut p m. Ainsi, par proportionnalité,
de l’arc AB c’est-à-dire 1,18 m.
la longueur de l’arc AB vaut 80 ×π , c’est-à-dire
180
environ 1,4 m.

 vaut donc, en simplifiant, 4p rad.


2  L’angle AOB
Acquérir des automatismes
9

2  Cosinus et sinus d’un angle

1  a) La longueur de l’arc IM  vaut 2p , c’est-à-dire


10
p
. 3 La mesure de l’angle MON est égale à 5π − π ,
5 6 4
b) La mesure de l’angle IOM  est 360 = 36°. 7π
10 =
donc MON 
rad. Ainsi la longueur de l’arc MN
12
c) Soit H le projeté orthogonal du point M sur la
7π 7p
droite (OI). Ainsi les coordonnées du point M est égale à × 3 cm, c’est-à-dire cm.
12 4
sont (OH ; MH).
Or le triangle OH est rectangle en H = 180° π
OH 4 a)  IOM × = 30°.
donc cos(36°) = = OH π 6
OM 16π 2π 18π 2π
MH b)  − = − = − 3 × 2π
et sin(36°) = = MH. 3 3 3 3
OM
Arrondies au centième, les coordonnées de M 16π 2p
donc − et ont le même point image sur le
sont (0 , 81 ; 0 , 59). 3 3
cercle trigonométrique.
 mesure p d’après
2  a) L’arc IM   a) et a pour  = 120°.
1 Donc ION
5
 = π rad. = 180° π
rayon 1 donc IOM c)  IOP × = 45°.
5 π 4

Chapitre 7  ★  Trigonométrie 107

172909_Chap07_000-000.indd 107 29/07/2019 17:03:28


19π 3π 16π 3π  7π  π 2
d)  = + = + 2 × 2π et sin  = −sin  = − .
4 4 4 4  4   4  2
19p 3p 5π
donc et ont le même point image sur le b) C est le point image du nombre réel − . Comme
4 4 6
cercle trigonométrique. 5π π
3π π − = − π, le point C est le symétrique du point
Or = 3 × donc, pour placer R, on reporte trois 6 6
4 4 p
D image du nombre réel par rapport à l’origine du
p 6
fois la longueur dans le sens direct à partir de I.
4  5π  π 3
repère. Ainsi, cos−  = −cos  = −
J +  6   6  2
N
R P
 5π  π 1
M et sin−  = −sin  = − .
 6 6 2
J +
O I B
D

O I
7 Pour tout nombre réel x,
cos2( x) + sin2( x) = 1. C
A
 5 2
Ainsi, sin ( x) = 1 − cos ( x) = 1 −  
2 2
 3 
16π  π 1
5 4 10 a)  cos = −cos  = −
soit sin2( x) = 1 − . D’où sin2( x) = .  3   3  2
9 9
2 2 16π  π 3
et sin sin 
On en déduit que sin( x) = ou sin( x) = − .  3  = −  3  = − 2
3 3
 π   23π  π
Or x appartient à l’intervalle − ; 0 donc sin( x) < 0. b)  cos  = cos  = 0
 2   2  2
2  23π  π
Ainsi sin( x) = − . et sin = −sin  = −1
3  2   2 
8 Pour tout nombre réel x,
π π
cos2 ( x) + sin2 ( x) = 1. 11 a)  rad = 30° b)  rad = 90°
6 2
 1 2 15 c)  π rad = 180°
Ainsi, cos2 ( x) = 1 − sin2 ( x) = 1 − −  = .
 4  16
15 15 π π
On en déduit que cos( x) = ou cos( x) = − . 12 a) 30° = rad b) 45° = rad
4 4 6 4
 π  π π
Or x ∈ − ; 0 donc cos( x) > 0. c) 60° = rad d) 90° = rad
 2  3 2
15
Ainsi cos( x) = . e) 180° = π rad
4
7p 13 Agnès a tort. En effet, la longueur d’un arc de
9 a) A est le point image du nombre réel .
4 cercle de rayon 2 cm et d’angle au centre de mesure
7π π π × 45 π
Comme = − + 2π, le point A est le symétrique 45° est  = 2 × , soit  = cm.
4 4 180 2
p
du point B image du nombre réel par rapport à
4 14 Réponse (2). En effet, la longueur d’un arc de
l’axe des abscisses. cercle de rayon 3 cm et d’angle au centre de mesure
 7π  π 2 p π
Ainsi, cos  = cos  = rad est  = 3 × , soit  = π cm.
 4   4  2 3 3
108

172909_Chap07_000-000.indd 108 30/07/2019 14:49:59


5π 23 a)  3π = π × α soit α = 3rad
15 a)  l = 8 × = 20π
2 b)  300 = 150 ×α soit α = 2rad
3π 3π
b)  3π = R × soit R =
4 3π 24 a) def Longueur (r, a)  :
4 l = r*a
Ainsi R = 4.
return l
16 Nicolas a tort. En effet, on a  : 3 = 2 ×α donc b) La commande Longueur (10, 0.1) retourne 1.
3 c) La valeur renvoyée par Longueur (10,3*pi) n’a pas
α = rad.
2 de sens puisque l’arc ferait un cercle complet et la
p
a est donc différent de rad. moitié d’un. La mesure de l’angle au centre doit varier
2
L’angle n’est pas droit. entre 0 et 2p.

p p
17 a) 140°     b) 75°     c) 67,5°     d) 144°. 25 a) p     b)       c)  .
2 18

8p 7p  = 18°.
26 IOM
18 a)  rad b)  rad
9 18
29p 7p
c)  rad d)  rad p 3p 5p 7p
36 20 27 a) A : ; B : ; C : ; D :
4 4 4 4
p 2p 4p 5p
5π b) I : 0 ; M : ; N : ; P : p ; Q : ; R : .
19 a)  l = 6 × = 5π cm soit l » 15, 71 cm 3 3 3 3
6
8π 24π
b)  l = 3 × = m soit l » 10 , 77 m π 5p
7 7 28 Lily a tort. − et ont bien le même point
3 3
11π
c)  l = 12 × = 26 , 4π km soit l » 82, 94 km 5p
5 image mais n’appartient pas à l’intervalle
3
[2π ; 4π[.
π × 150 5π
20 a)  l = 2 × = m soit l » 5, 24 m.
180 3 p 2p 3π
29 A  : ; B  : ; C  : − ; I : 1002π; J  : 7p  ;
π × 80 16π 6 3 4
b)  l = 4 × = cm soit l » 5, 59 cm.
180 9 π π
K : − ; L : −
π × 5 5π 2 6
c)  l = 30 × = mm soit l » 2, 62 mm.
180 6
30 p
– +
 = 60° ; ION
21 a)  IOM  = 150° 2
M 1
N
5p p
––– –
6 3
p 0 ; 2p
0 0 1

= 5π π π
b)  MON − = rad. Ainsi la longueur de l’arc
6 3 2
3p
π
 est l = 2, 5 × cm soit l = 3, 93 cm –––
MN 2
2

π × 30 240
22 a)  40 = R × soit R = cm donc
180 π
R » 76 , 4 cm
1
b)  1 = R × π soit R= m donc R » 0 , 318 m
p

Chapitre 7  ★  Trigonométrie 109

172909_Chap07_000-000.indd 109 29/07/2019 17:04:58


31 2p p + 35 2p p +
––– – ––– –
3 3 3 3
1 1
M

3p 6p
––– –––
0 1 3 0 1 3

4p 2p p 4p 5p
––– ; - ––– -– ––– –––
3 3 3 3 3

32 + 36 a)  6p
7p ––– 5p
8p ––– 12 ––– 4p
12 ––– +
––– 12
5p 1 9p 12 12 3p
p 13p ––– 1 –––
––– 10p12 M 12
6 – ; –––– –––– p
6 6 12 –
11p 6 p
–––– –––
12 12
12p 24p
0 1 ––––
12
––––
0 1 12
13p 23p
–––– ––––
12 12
5p p 14p 22p
- ––– -– –––– ––––
6 6 12 15p 21p12
–––– ––––
12 16p 20p 12
–––– 17p 18p 19p ––––
12 –––– –––– –––– 12
33 + 12 12 12
3p 19p p  puisque
––– ; –––– 1 b) On trace la bissectrice de l’angle IOM
4 4 –
4 π 1 π
= ×
12 2 6
c) Voir la figure à la question a).
0 1
p 7π 13π
37 C :  ;   B : −  ;   K : −  ;   I : 2v
3 4 6
3p p 7p 2p 5π
- ––– -– G : 15p ;   H :  ;   D : ;   E : −
4 4 6 3 4
34 13p p 17p 5p 8π
–––– + 38 G : ;    N : ;    K : ;    L : − ;
2 13p 6 6 4 3
–––– 3p 11π 5p 9π
1 3 M : ;    J : − ;    H : ;    P : −
2 2 3 4
39 Faux. Le cercle étant coupé en 8 arcs de même
p 2p 3p
longueur, il s’agit des nombres réels  ; ; ;
0 1 4 4 4
4p 5p 6p 7p 8p
; ; ; ; dans le repère ci-dessous.
13p 4 4 4 4 4
- ––––
13p 6
–––– 13p
4 - ––––
3

110

172909_Chap07_000-000.indd 110 29/07/2019 17:05:18


40 a) et b)  p 5π 7p 19p
44 • ; − ; ;
J 3 3 3 3
M 29π 19p
• 0 ; – 8p • p ; – 9p
; •−
4 4
L1 23π p 13p 3π 13p
•− ; ; • − ;
6 6 6 4 4
O I
# 7p 9π
45 a)  ; −
4 4
L2
π 7p
b)  − ;
2 2
N
7p 19p
2 c)  ;
 1 2  3  6 6
 
ON=  +   = 1 donc N Î #.
 2   2  5π 7p
d)  − ; .
3 3
1 = 2π π π
c)  IOL − = rad donc L1 est le point image
3 2 6 46 a) La commande Mpi(pi/3, 7*pi/3) renvoie « oui ».
p b) Les points sont les mêmes.
du nombre réel .
6 c) On remplace « Oui » par : « Les nombres réels a et b
• L2 est le symétrique de L1 par rapport à O donc L2 ont le même point image sur un cercle trigonomé-
π 5π trique ».
est le point image du nombre réel − π, soit − .
6 6 On remplace «  Non  » par  : «  Les nombres réels a et b
• N est le symétrique de M par rapport à O donc N est n’ont pas le même point image sur un cercle trigonomé-
2π π trique ».
le point image du nombre réel − π, soit − .
3 3
47 a) L’affirmation est fausse.
= π π π
41 •  COA + = rad donc la longueur de π  13π  28π π 27π
6 3 2 En effet, − −  = 2π mais − = qui

7  7  7 7 7
π 3π
l’arc rouge est l 1 = 1, 5 × = cm soit l 1 » 2, 4 cm n’est pas un multiple de 2p.
2 4
b) L’affirmation est fausse.
 = 3π − π = 5π rad donc la longueur de l’arc
•  AOB 28π  27π 
4 3 12 En effet, − − = 11π qui n’est pas un mul-
5  5 
5π 5π
vert est l 2 = 1, 5 × = cm soit l 2 » 2 cm tiple de 2p.
12 8
c) L’affirmation est vraie.
 = 2π − π − 5π = 13π rad donc la longueur
•  BOC 3π  107π 
2 12 12 En effet, − −  = 10π = 5 × 2π.
13π 13π 11  11 
de  l’arc bleu est l 3 = 1, 5 × = cm soit
12 8
l 3 » 5,1 cm 48 a) cos( x ) = 0 , 6 ; sin( x ) = 0 , 8.
b) sin( y ) = 0 , 4 ; cos( y ) ≈ −0 , 9.
27π 3π 24π 3π
42 a)  = + = + 3 × 2π.
4 4 4 4  π 1  π 3
3p 49 a)  cos−  = ; sin−  = −
b)  a donc le même point image que le nombre  3  2  3  2
4
 5π  3  5π  1
27p b)  cos  = − ; sin  =
réel .  6  2  6  2
4
 3π  2  3π  2
c)  cos−  = − ; sin−  = −
 4  2  4  2
101π π 102π π
43 a)  − = − = − 17 × 2π
3 3 3 3
p π π
b)  a donc le même point image que le nombre 50 a)  cos  ≈ 0 , 81 ; sin  ≈ 0 , 59
3  5   5 
101π π π
réel − . b)  cos  ≈ 0 , 97 ; sin  ≈ 0 , 26
3 12  12 

Chapitre 7  ★  Trigonométrie 111

172909_Chap07_000-000.indd 111 29/07/2019 17:06:07


 3π   3π   4π  3
c)  cos−  ≈ 0 , 22 ; sin−  ≈ −0 , 97 54 a)  sin  = −
 7   7   3  2
 19π   19π 
d)  cos− ≈ 0 , 38 ; sin− ≈ −0 , 92  3π 
 8   8  b)  sin  = −1
 2 
51 a) + 11π  1
π c)  sin =−
3π 1 4  6  2
4 –
M
 29π  1
55 a)  cos
49 = b)  cos(17π) = −1
1  3  2
O
 7π  2
11π 5π 7π c)  cos−  =
– ;
4 4
 4  2
4

 7π   7π  5π π 5p p
2 2 56 •  = π− donc et ont des points
b)  cos  = ; sin  = − 6 6 6 6
 4  2 C06_fig118 
 4  2
images symétriques par rapport à l’axe des ordonnées.
 3π  2  3π  2
cos  = − ; sin  =  5π  π 1
Ainsi, sin  = sin  = .
 4  2  4  2  6   6  2
 5π  2  5π  2
cos  = − ; sin  = − 23π π 23π p
 4  2  4  2 •  − = − 3 × 2π donc − et ont le même
4 4 4 4
 11π  2  11π  2 point image sur le cercle trigonométrique. Ainsi,
cos− =− ; sin− =−
 4  2  4  2  23π  π 2
cos− = cos  = .
 4   4  2
52 a) 2π π 11π
;–
3 3 3
1 +  2π  3
57 a) Faux. En effet, sin  =
 3  2
π 3
et 2 sin  = 2 × = 3.
O 1  3  2

b) Vrai. En effet, 0 < 0 , 9π < π.

2π 5π π π 2
– c) Vrai. En effet, cos  = sin  = .
3 3  4   4  2
 2π  1  2π  3
b)  cos−  = − ; sin−  = − π 1 π 3
 3 C06_fig119
2  3  2 d) Faux. En effet, cos  = et sin  = .
3 2 3 2
 5π  1  5π  3
cos  = ; sin  = −
 3  2  3  2
π 2
 2π  1  2π  3 sin 
cos  = − ; sin  =  π 
58 a)  tan  =
 4  2
 3  2  3  2 = = 1.
 4   π 
 2
 11π  1  11π  cos 
3 4 2
cos− = ; sin− =
 3  2  3  2 π 3
sin 
π 3
 5π  b)  tan  = = 2 = 3.
2  3   π  1
53 a)  cos  = − cos 
 4  2 3 2
 7π  1
b) cos−  =  7π 
sin  1
 3  2 −
 7π  6 2 1
c)  tan  = = = .
 5π  3  6   7π  3 3
c)  cos  = − cos  −
 6  2 6 2
112

172909_Chap07_000-000.indd 112 29/07/2019 17:06:43


59 Matthieu a raison comme le montre la figure ci- b) Il s’agit de x1 » 2, 21 et x2 = −x1 ≈ −2, 21
dessous.
+ p 2p
64 1. M : et N :
1 6 3
5p 1  a pour longueur 2π − π = π .
MN
––– ––
6 2 3 6 2
 a pour longueur :  7π − π = 11π .
2. a)  MN
0 1 6 4 12
 a pour longueur : 11π π 4 π
b)  MN − = .
6 2 3
5
65 cos2 ( x) =
5p 49
Il s’agit de .
6 66 Romain a raison. En effet, pour tout nombre
p 3p π 5π 5π réel x, cos2 ( x) + sin2 ( x) = 1.
60 a)     b)     c)  − et −    d) −
3 4 6 6 6 1 2
Ainsi sin2 ( x) = 1 − cos2 ( x) = 1 − = .
3 3
π
61 a)  − + 2kπ avec k Î .
4 67 Réponse (3). En effet,
2π  1 2 3
b)  − + 2kπ avec k Î . cos2 ( x) = 1 − sin2 ( x) = 1 −   =
3  2  4
5p
c)  . 3p
2 68 •  : 
4
d) Aucune solution.
 3π   3π 
cos  < 0 et sin  > 0
62 1. a)  x » 0 , 2 b)  x ≈ −0 , 7  4   4 
2. a) L’équation admet deux solutions sur l’intervalle 7π
•  − : 
[0 ; π]. 12
+  7π   7π 
cos−  < 0 et sin−  < 0
 12   12 
1
p
0,4 •  : 
x2 x1 9
π π
cos  > 0 et sin  > 0
 9   9 
0 1
π
•  − : 
3
 π  π
cos−  > 0 et sin−  < 0
 3   3 
b) Il s’agit de x1 » 0 , 41 et x2 = π − x1 ≈ 2, 73
 5π 
69 sin  > 0
63 1. a)  x » 1, 3 b)  x » 2, 6  7 
2. a) L’équation admet deux solutions sur l’intervalle  3π 
cos−  < 0
]− π ; π].  4 
+ 13π 
sin >0
x1 1  5 

70 Pour tout nombre réel x, cos2 ( x) + sin2 ( x) = 1.


 3 2 16 4
- 0,6 Ainsi, sin2 ( x) = 1 − −  = soit sin( x) = ou
 5  25 5
0 1
4
sin( x) = − .
5
4
x2 Or x ∈ [0 ; π] donc sin( x) > 0. Ainsi sin( x) = .
5
Chapitre 7  ★  Trigonométrie 113

172909_Chap07_000-000.indd 113 29/07/2019 17:07:29


71 Pour tout nombre réel x, cos2 ( x) + sin2 ( x) = 1. 31p p
Donc et ont le même point image sur le
 12 8 8 3 3
Ainsi, cos2 ( x) = 1 −   = soit cos( x) = ou cercle trigonométrique.
 3  9 3
 31π  π 1
8 Soit cos  = cos  = .
cos( x) = − .  3  3 2
3
3. Faux. En effet, par exemple,
π  8
Or x ∈  ; π donc cos( x) < 0. Ainsi cos( x) = − .  π 2  π 2
 2  3 cos−  = et sin−  = − .
 4  2  4  2
72 1. a) et 2. a)  4. Vrai. En effet, les points images des nombres réels
+ p π
et π + sont symétriques par rapport à l’origine
p 3 3
1 – du repère.
M 5 5. Vrai. En effet, les points images des nombres réels
p π
et π − sont symétriques par rapport à l’axe des
7 7
0 1 ordonnées.

π 180°
76 a)  rad = = 20°.
p 6p p 9 9
-–
p + – = ––– 5 b) 
5 5
b) Pour tout nombre réel x, cos2 ( x) + sin2 ( x) = 1.
p

2 π   5 + 12 16 − (5 + 2 5 + 1) 9
Ainsi sin   = 1 − 
  =
 5   4  16 3 cm
π π
 π  10 − 2 5 c)  l = R × α = 3 × = cm
soit sin2   = 9 3
 5  16 Ainsi l » 1, 05 cm
π 10 − 2 5
Ainsi sin  = 4p +
5 4 77 - –––
3
π 1
10 − 2 5
ou sin  = − .
5 4
π π 10 − 2 5
Or sin  > 0 donc sin  = .
5  5  4 0 1
 π π 5 +1 7p
2. b)  cos−  = cos  = –––
 5   5  4 6 5p
–––
 π π 10 − 2 5 4
sin−  = −sin  = −
 5   5  4
78 p
 6π  π 5 +1 –
cos  = −cos  = − 2 p +
 5   5  4 3 –
––– 3 p
2 –
 6π  π 10 − 2 5 2 4
sin  = −sin  = − . ––– p
 5   5  4 2 –
1 6

73 1. C   2. C   3. B   4. D   5. D   6. C 2

74 1. B, D    2. B   3. B, C, D 0


O 1 2 3
– ––– –––
4 × 180° 2 2 2
75 1. Vrai. En effet, = 80°.
9
31π π 30π π 15π π
2. Vrai. En effet, = + = + 5 × 2π. 79 a)  − 2 × 2π = −
3 3 3 3 4 4
114

172909_Chap07_000-000.indd 114 29/07/2019 17:07:54


b)  +

3p N
–––
1 M p
– S'entraîner
4 4

0 1
82 a) Voici le tableau de suivi des valeurs des
variables k et S lors de l’exécution de l’algorithme.
P 15p
–––– k
4 - 3 - 2 - 1
3 3 1
 3π  π 2 S 0 - 1 -1 - -1 - -
c)  sin  = sin  = 2 2 2
4 4 2
k 0 1 2 3
15π  π 2
cos cos  = 3 1 3
 4  =  4  2 S -1 - - -1 - - 1 0
2 2 2

80 1. a)  + La valeur stockée dans la variable S à la fin de l’exécu-


p tion de l’algorithme est 0.
1 –
4
M Cette valeur est celle de la somme :
p  π  π  π π π π
– sin−  + sin−  + sin−  + sin(0) + sin  + sin  + sin .
8  2   3   6   6   3   2 
0 1I  π  π  π π π π
sin−  + sin−  + sin−  + sin(0) + sin  + sin  + sin .
 2   3   6   6   3   2 
b) Sur le cercle trigonométrique ci-dessous, on
remarque que les valeurs des nombres réels apparte-
 π 
π 1 π nant à l’intervalle − ; 0  annulent celles des
b)  = × .  2 
8 2 4  π
nombres réels appartenant à l’intervalle  0 ;  . De

On trace alors la bissectrice de l’angle IOM.  2 
plus, sin(0) = 0.
π
c) On en déduit que sin  > 0. Ainsi la somme est nulle.
 8 
p

2. Pour tout nombre réel x, cos2 ( x) + sin2 ( x) = 1. 2 p
— +
3
2
π π   p
 2 + 2  —
Ainsi sin2   = 1 − cos2   = 1 −   soit 6
 8   8   2 

π 2+ 2 2− 2 0
sin2   = 1 − = . O
 8  4 4
p
-—
π 2− 2 π 2− 2 6
Donc sin  = ou sin  = − .
 8  2 
8 2 p
-—
p 3
π -—
Or, d’après la question 1. c) , sin  > 0 2
8
π 2− 2 83 a) Voici le tableau de suivi des valeurs des
donc sin  = .
 8  2 variables k et S lors de l’exécution de l’algorithme.
k 0 1 2 3
2 2
S 0 0 +1 2 +1
2 2
k 4 5 6 7 8
2 2
S 2 +1 +1 0 0
2 2

La valeur affichée par l’algorithme est 0.

Chapitre 7  ★  Trigonométrie 115

172909_Chap07_000-000.indd 115 30/07/2019 14:50:18


Cette valeur est celle de la somme : 87 a) Dans le triangle OMH rectangle en H, le théo-
π π  3π  rème de Pythagore donne :
sin(0) + sin  + sin  + sin  + sin(π)
 4   2   4  OM2 = OH2 + MH2 ,
 5π   3π   7π  c’est-à-dire 1 = OH2 + MH2 .
+ sin  + sin  + sin  + sin(2π)) 1
 4   2   4  Or OH = OM donc 1 = 2OH2 soit OH2 = .
2
p
b)  – + 1 2
2 Ainsi, OH = = .
3p 1 p 2 2
––– –
4 2 4 b) Le triangle OMH est isocèle rectangle en H donc
–––  et HMO
 ont pour mesure 45°.
2 les angles HOM
p 0  est p rad.
c) La mesure de l’angle HOM
0 1 2p 4
Donc :
2 π
5p -–––
2 7p  ) = OH = OH = OH = 2
cos  = cos(HOM
––– –––  4  OM 1 2
4 4
et
3p
––– π  ) = HM = HM = HM = 2
2 sin  = sin(HOM
 4  OM 1 2
Sur le cercle trigonométrique ci-dessus, on remarque
que les valeurs des nombres réels appartenant à l’in-
101π 5π 96π
tervalle ]0 ; π[ annulent celles des nombres réels 88 − = = 8 × 2π.
6 6 6
appartenant à l’intervalle ]π ; 2π[.
101p 5p
De plus, sin(0) = sin(2π) = 0. Les nombres réels et ont donc le même
6 6
point image sur un cercle trigonométrique.
85 a) Lorsqu’on déplace le curseur X, on lit dans
l’affichage Algèbre que l’aire du losange semble égale 89 On suppose que la longueur de l’arc est supé-
3 rieure à 0,5.
à soit environ 0,87 lorsque X » 1,05 rad.
2 On a alors (1 − x) × x > 0 , 5 soit −x2 + x − 0 , 5 > 0.
Comme ∆ = −1 < 0, −x2 + x − 0 , 5 est du signe de
b) L’aire du losange est égale à 1× sin(X) soit sin(X).
a donc −x2 + x − 0 , 5 < 0 ce qui est contradictoire
3 donc la longueur de l’arc est inférieure à 0,5.
La solution de l’équation sin(X) = dans l’inter-
2
 π p 90 a) 
valle  0 ;  est le nombre soit environ 1,05.
 2  3

86 a) Lorsqu’on déplace le curseur X, on lit dans


l’affichage Algèbre que l’aire du triangle OHM est
égale à quatre fois celle du triangle IMH lorsque
X » 0 , 64rad. b) On peut émettre la conjecture
pour tout nombre réel x,
cos(X) × sin(X)
b) L’aire du triangle OHM est égale à (cos( x) + 2 sin( x))2 + (2 cos( x) − sin( x))2 = 5
2
(1 − cos(X)) × sin(X) En effet, (cos( x) + 2 sin( x))2 + (2 cos( x) − sin( x))2
L’aire du triangle IMH est égale à
2 = cos2 ( x) + 4 cos( x)sin( x)) + 4 sin2 ( x)
cos(X)sin(X) (1 − cos(X))sin(X) +4 cos2 ( x) − 4 cos( x)sin( x) + sin2 ( x)
L’équation =4 est
2 2 = 5 cos2 ( x) + 5 sin2 ( x)
équivalente à 5 cos(X)sin(X) = 4 sin(X)
= 5(cos2 ( x) + sin2 ( x))
Comme M ¹ I, X ¹ 0 donc sin(X) ≠ 0.
= 5 ×1 = 5
Ainsi, l’équation est équivalente à 5 cos(X) = 4 soit
4
cos(X) = .
5
À l’aide de la calculatrice X » 0 , 644.
116

172909_Chap07_000-000.indd 116 30/07/2019 14:51:54


91 a)  + 94 • Arc rouge :
1 π
M l1 = 2 × =π
x 2
• Arc bleu :
π
=π l2 = 4×
4
0 1 Les deux arcs sont de même longueur.

95 a) Le périmètre de la base est 2 × 2, 4 × π soit


N 4 , 8p cm.
b) Ainsi 4 , 8π = 4 × α d’où α = 1, 2π rad.
b) Le point N est l’image du nombre réel π + x.
c) N est le symétrique de M par rapport à O. π
96 On a 2 = R = .
Ainsi les coordonnées de N sont les opposées de 5
10
celles de M, soit : Ainsi R = m.
π
cos(π + x) = −cos( x) et sin(π + x) = −sin( x).
La distance séparant Zoé du miroir est R » 3,18 m.
92 Le périmètre du terrain rectangulaire est
97 a)  LO = LS car O et S appartiennent à un même
(45 + 80) × 2 soit 250 m.
cercle de centre L.
Le périmètre du second terrain est 2 × 45 + l 1 + l 2 LO = OS car L et S appartiennent à un même cercle
où l 1 est la longueur du petit arc de cercle et l 2 est de centre O.
la longueur du grand arc de cercle. Ainsi, LO = LS = OS et le triangle SOL est équilatéral.
2π 2π
Or l 1 = 40 × = 16π m et l 2 = 85 × = 34π m  = 60° soit SOL
b) On a donc SOL  = π rad.
5 5 3
Ainsi le périmètre du second terrain est 2π
c)  l = 696 000 × km
90 + 16π + 34π soit environ 247 m. 3
On en déduit que le terrain ayant le plus grand péri- soit l » 1457 699 km.
mètre est le rectangulaire.
98 a) π
3 +
π 5π 1
93 a)  l = 5 × = cm C A
3 3
5π π 20
b)  = R × donc R = cm.
3 4 3 O 1
π π
c)  rad = 60° et rad = 45°
3 4
D
B
π π 2π
b) B : − c) C : π − =
3 3 3
π C06_fig116

p d) D : −π = −
– 3 3
3
5 cm
99 1. D J
B +
K 1
C

E
O 1 I
p

4 G
20
––– cm F
3

Chapitre 7  ★  Trigonométrie 117


C06_fig117

172909_Chap07_000-000.indd 117 29/07/2019 17:09:25


p p π 7p 3p  π 2
2. B : ; C: ; E:p;F: − ; G: ; K: π2 − 4 − 
3 6 3 6 4  π  6
104 a)  cos−  ≈
 6   π 2
101π 5π π2 + − 
100 a)  = + 16 × 2π  6
3 3
On peut donc effectuer 16 tours entiers. 8 2
 π  π  π  32

b) Il reste alors1× cm de fil soit environ 5,2 cm. soit cos−  ≈ 9 d’où cos−  ≈
3  6  37 2  6  37
π
36
 = 2 × ACB
101 a)  AOB  = 120° 32
b)  » 0 , 864 864 86 et
 = 120° − 90° = 30°
b)  IOB 37
I
OC = BOC  + I OB = 120° + 30° = 150°  π
cos−  =
3
≈ 0 , 866 025 40
= 2π  6 2
c)  AOB rad
3
105 a) 
 = π rad
IOB G
6
0,5 T
 = 5π rad
IOC
6 0,5
p π 5π - 0,5 O
d) I : 0 ; A : ; B : − ; C : −
2 6 6
- 0,5
I
102 1. a)  R
13p 7p p
Valeur de x 2π π π
3 3 3 =
b)  GOT − = rad
x > 0 Vrai 3 6 2
Vrai Vrai Faux
c) TRIG est un quadrilatère dont les diagonales se
x > p
coupent en leur milieu O, sont de même longueur 2
13p
Lorsque x prend la valeur , la valeur de x à la fin et sont perpendiculaires. TRIG est donc un carré.
p 3
de l’algorithme est .
3 106 1. a)  J
b) 19π 7π 5p M
Valeur de x − − P x
6 6 6
x > 0 Faux

x < - p Vrai Vrai Faux O I


19π
Lorsque x prend la valeur − , la valeur de x à la fin
6 N
5p
de l’algorithme est .
6
2. Le rôle de cet algorithme est de stocker dans la b) N est le point image du nombre réel -x.
variable x le nombre réel de l’intervalle ] − π ; π] qui a c) Ainsi cos(−x) = cos( x) et sin(−x) = −sin( x).
le même point image sur un cercle trigonométrique 2. P est le symétrique de N par rapport à O.
que le nombre réel affecté à x. P est le point image du nombre réel π − x.
Ainsi cos(π − x) = −cos( x) et sin(π − x) = sin( x).
π π 1 1
103 a)  cos  + sin  = + = 1 107 a) On peut émettre la conjecture  : pour tout
 3   6  2 2
nombre réel x,
 π π 2 2 (1 + cos( x) + sin( x))2 − 2(1 + cos( x))(1 + sin( x)) = 1
b)  cos−  − sin  = − =0
 4   4  2 2 b)  (1 + cos( x) + sin( x))2 − 2(1 + cos( x))(1 + sin( x))
 2π   7π   1  1 5 = 1 + cos2 ( x) + sin2 ( x) + 2 cos( x) + 2 sin( x)
c)  3 cos  − 8 sin  = 3 × −  − 8−  =
 3   6   2   2 2 +2 cos( x)sin( x) − 2 − 2 sin( x)
π  3π  −2 cos( x) − 2 cos( x)sin( x)
d)  cos  − sin  = 0 − (−1) = 1.
 2   2  = 0 car cos2 ( x) + sin2 ( x) = 1.
118

172909_Chap07_000-000.indd 118 29/07/2019 17:10:05


108 1. c) On peut émettre la conjecture. N est le Ainsi la longueur totale du tracé vert est :
point image du nombre réel 2X. 5π 3 2
 = 2× + 2× + 2× soit
 π π 12 2 2
2. •  cos2 ×  = 1 − 2 sin2   soit
 12  12  5π
= + 3 + 2.
π π 3 π 6
cos  = 1 − 2 sin2   d’où = 1 − 2 sin2  . On a donc  » 5, 76
 6  12  2 12 
3
−1 110 a) Dans le triangle MOA rectangle en M,
π 2
2

On en déduit que sin   = soit ) = OM 1
12  −2 cos(MOA = .
OA 2
 π  2− 3  = 60° et M est l’image du nombre réel p .
sin2   = Ainsi MOA
12  4 3

π 2− 3 π 2− 3 111 Dans le triangle ANB rectangle en A,


Donc sin  = ou sin  = − AN
12  2 12  2 tan(35°) =
100
π π 2− 3 Dans le triangle ARB rectangle en A,
or sin  > 0 donc sin  = .
12  12  2 AR
tan(55°) =
 π π π 100
• sin2 ×  = 2 cos sin  soit Donc AN » 70,02 m et AR » 142,81m.
 12  12  12 
π π π Ainsi, NR » 72,79 m.
sin  = 2 cos sin 
 6  12  12 
112 a) L’affirmation est fausse. En effet, il en existe
1 π 2− 3
d’où = 2 cos  × une infinité.
2 12  2
En voici deux : cos−1(0 , 7) et cos−1(0 , 7) + 2π.
π 1 2+ 3 b) L’affirmation est fausse. En effet, pour tout
Ainsi cos  = =
12  2 nombre réel x, -1 < sin x < 1.
2 2− 3

2π 2π 113 a) La réciproque est fausse.


109 1. a)  x = et x = − .
3 3 En effet, si cos( x) > sin( x), alors x peut appartenir
4p  3π π 
b) L’arc de cercle rouge a pour longueur . à l’intervalle − ; 
3  4 4  p
 2π  3  2π  3 –
c) Comme sin  = et sin = −  = − , 4
 3  2  3  2
la longueur totale du tracé rouge est :
4π 3 3 4π
= + + soit  = + 3.
3 2 2 3
On a donc  » 5, 92.
π 5π
2. a)  x = et x =
6 6 3p
3π π - –––
4
b)  x = − et x = −
4 4 b) La réciproque est fausse. En effet, par exemple,
π π π  π 1
c) Les deux arcs de cercle ont pour longueur + π π
6 4 cos  = cos−  = et ≠ − .
5p  3   3  2 3 3
soit .
12 c) La réciproque est fausse. En effet,
π 3  3π   3π 
De plus, cos  = cos−  = sin−  = −
2 3π π
et − ≠ .
 6  2  4   4  2 4 4
 5π  3
cos  = −
 6  2  π
π 3
114 a) Pour x = − , sin−  = − ce qui ne
 π  2 3 
 3 2
cos−  =
 4  2 peut être égal à un nombre positif.
 3π  2 3π  3π 
cos−  = − . b) Pour x = , sin  = −1
 4  2  2 
2

Chapitre 7  ★  Trigonométrie 119

172909_Chap07_000-000.indd 119 30/07/2019 14:53:18


 
 = sin−1 29 , 7 
Organiser son raisonnement
Ainsi DAC 
 1323, 09 

 )= AI
• sin(DAE où I est le milieu du segment [AB].
DI
1
1 1, 8 m Or AI = AB = 14 , 85
115 a) L’échelle étant  , AB = = 0 , 9 m. 2
2 2
b) 1 et DI2 = DA2 + AI2 = 212 + 14 , 852
≈ π – 1,12 ≈ 1,12
A 0,9 donc DI = 611, 522 5
 
 = sin−1  14 , 85 
Ainsi DAE  661, 522 5 
C  
B O 1
 = 180° − DAE
• Or AED  − DAC
 soit AED » 89 , 997
π 
x est le nombre réel de l’intervalle  ; π tel que
 2  Marc a tort, l’angle est quasi droit.
sin( x) =C06_fig127
0 , 9.
On a sin−1(0 , 9) ≈ 112
, 119 1. a) Le polygone est régulier à 6 côtés ; ainsi le
d’où x ≈ π − 112,
 = π rad. et
triangle OAJ est équilatéral. Donc AOJ
Donc x » 2, 02 m. 3
La longueur de l’arc AC  est donc 2,02 m environ sur π
 = rad. A est donc le point image du nombre
IOA
la maquette, et 4,04 m en réalité. 6
p
réel .
116 En appliquant successivement le théorème de 6
Pythagore dans les triangles rectangles colorés, on π 1
b)  AH = sin  =
obtient les longueurs inscrites sur le schéma.  6  2
1 1 π 1
1 sin 
1 π 6 2 1
BI = tan  = =
 6   π  3 3
1 6 
 7

8 1 cos 

5 6 2

4 
9
1 1 c) Le demi-périmètre du polygone rouge est :
α4
α3

α5

α2 α6 7

3 α1 α
α8
C 1
A p1 = × 6 = 3.

2 B 11 2
1 1 Le demi-périmètre du polygone bleu est :
1 1 2 6
1 1 1 p2 = × ×6 = .
Ainsi, sin(α1) C06_fig130
= ; sin(α 2 ) = ;  ; sin(α 8 ) = . 2 3 3
4 5 11
6
On a alors : On a donc 3 < π <
3
 = sin−1 1  + sin−1 1  + … + sin−1 1 
ABC    5   2. 
 4  11
 ≈ 179 , 02° et ABC
ABC  ≠ 180°.
Les points A, B, C ne sont pas alignés.

117
b) Archimede (4) renvoie (3.141  031  950  890  510  2,
3.142 714 599 645 368 7)
223
c)  » 3,140 845 07
71
 ) = DC or DC = 29 , 7
118 •  sin(DAC 22
AC » 3,142 857143
7
et AC2 = AD2 + DC2 = 212 + 29 , 72 Remarque  : Archimède n’avait pas les techniques
donc AC = 1323, 09 actuelles pour effectuer ces calculs.
120

172909_Chap07_000-000.indd 120 29/07/2019 17:11:19


120 1. a) 10° degrés séparent les extrémités de cette ) = AM 2 1  = 30°.
sin(ASM = = donc ASM
frontière. SA 4 2
b)   ) = TH .
Dans le triangle STH, rectangle en H, tan(HST
SH
TH
Ainsi, tan30° =
63 8
71 10°
km 8 3
d’où TH = 8 tan(30°) = et TH » 4,62 m.
3
La frontière a une longueur égale à :
Le diamètre du disque formé par l’ombre sur le mur
π × 10
l 1 = 6 371× 16 3
180 est égal à m soit environ 9,24 m.
6 371 3
= π km soit l 1 » 2 002 km
18 BC BD
2. • Déterminons le rayon HA du 49° parallèle. 122 On a tan(45°) = et tan(60°) =
AB AB
 ) = AH donc AH = 6 371× sin(41°) km
sin(AOH Ainsi ABtan(45°) = BC = BD − 7 = ABtan(60°) − 7
AO
donc AB(tan(45°) − tan(60°)) = −7
A H
H A 7
soit AB =
49° 49° O tan(60°) − tan(45°)
0
sin(60°)
or tan(60°) = = 3
cos(60°)
    sin(45°)
• Ensuite, le document permet d’estimer les degrés et tan(45°) = =1
cos(45°)
séparant les extrémités de la frontière, on lit
123° − 95° = 28°. 7
donc AB = ≈ 9 , 56 m
3 −1

123 1. On peut émettre la conjecture :


28° AH
AB2 + CD2 = 84
2. a) Dans le triangle OHB rectangle en H, on applique
le théorème de Pythagore :
La longueur de cette frontière est donc égale à OB2 = OH2 + HB2
π × 28
l 2 = AH × soit Ainsi HB2 = OB2 − OH2 .
180
OH
π × 28 Or sin(α) = donc OH = OM sin(α).
l 2 = 6 371× sin(41°) × km OM
180
On a donc l 2 » 2 043 km. On a alors HB2 = OB2 − OM2 sin2 (α).
b) Le triangle AOB étant isocèle en O, H est le milieu
121 1. H est le point d’intersection de la droite (SA)
de [AB].
avec le mur.
T est le point d’intersection du mur avec la tangente Ainsi AB2 = (2HB)2
au cercle issue de S (en le point M). = 4HB2
T = 4 OB2 − 4 OM2 sin2 (α)
c) De même, OC2 = OK 2 + KC2
M  ) = OK donc
Ainsi KC2 = OC2 − OK 2 or cos(MOK
OM
H OK = OM cos(α)
S
A On a alors KC2 = OC2 − OM2 cos2 (α)
On en déduit que :
CD2 = (2KC)2
= 4KC2
Dans le triangle ASM, rectangle en M : = 4 OC2 − 4 OM2 cos2 (α)

Chapitre 7  ★  Trigonométrie 121

172909_Chap07_000-000.indd 121 29/07/2019 17:11:52


On a donc : 125 1. a) Dans le triangle BDC rectangle en D
AB2 + CD2  π  BD
cos  = donc
= 4 OB2 − 4 OM2 sin2 (α) + 4 OC2 − 4 OM2 cos2 (α)  6  BC
= 4 OB2 + 4 OC2 − 4 OM2 (cos2 (α) + sin2 (α)) π π 3
BD = BC cos  = 1× cos  = .
= 4 OB2 + 4 OC2 − 4 OM2 6 6 2
= 4 × 25 + 4 × 25 − 4 × 4 Or AD = AB − BD.
= 84 3
Ainsi AD = 1 − .
2
p  π  CD
124 On remplace x par , ce qui donne : b) Dans le triangle BCD rectangle en D, sin  =
5  6  BC
π π π  π   π  1
sin(π) = 16 sin5   − 20 sin3   + 5 sin  donc CD = BC sin  = 1× sin  = .
5 5 5 6 6 2
π 1
On pose X = sin  , on a ainsi : Ainsi CD = .
 5  2
2. a) Le théorème de Pythagore dans le triangle ACD
16X5 − 20X3 + 5X = 0.
rectangle en D permet d’écrire AC2 = CD2 + AD2 .
π
Or sin  ≠ 0 donc l’équation est équivalente à :  1 2 
2
5 
2 3 
D’où AC =   + 1 −  .
 2   2 3
16X 4 − 20X2 + 5 = 0. 2 1
Donc AC = + 1 − 3 + = 2 − 3.
On pose Y = X , on a ainsi 16 Y 2 − 20 Y + 5 = 0.
2
4 4
Le discriminant valant 80, cette équation admet deux Ainsi AC = 2 − 3 .
solutions distinctes.
 = π − ADC
b)  ACD  − CAD
20 − 80 5− 5 5+ 5
Y1 = = et Y2 = π 1 π
32 8 8 = π − − π − 
2 2  6
5− 5 5+ 5
Ainsi X12 = et X22 = π
8 8 =
12
On a alors :
1
5− 5 5− 5  π  CD 2 2+ 3

c)  cos  =
12  CA =
X1 = ou X1 = − =
8 8 2− 3 2
5+ 5 5+ 5 3
et X2 = ou X1 = − 1−
8 8  π  AD 2 = 2− 3 = 2− 3
sin  = =
π π π 12  CA 2
De plus, < < 2− 3 2 2− 3
6 5 4
p 126 H
2 –
––– 4 L
2
0,5 p
– q
6 T K
La tangente (TH) au cercle issue de T est perpendicu-
laire au rayon [HL ]. Ainsi le triangle THL est rectangle
en H.
Les triangles THL et TKL ont deux côtés respective-
π ment égaux et sont rectangles. D’après le théorème
2
Donc 0 , 5 < sin  < parmi les 4 valeurs, la de Pythagore, ils ont donc trois côtés respectivement
 5  2  2  égaux et sont alors isométriques.
seule  appartenant à l’intervalle  0 , 5 ; est
 2   = LTK
Ainsi HTL =θ
5− 5 2
X1 =
8  θ  HL HL
Ainsi, sin  = donc TL =
π 5− 5  2  TL θ
Donc sin  = sin 
5 8 2
122

172909_Chap07_000-000.indd 122 29/07/2019 17:12:30


3  = cos−1(0 , 525)
on en déduit que EOL
× 6 371
soit TL ≈ 11  = 1 cos−1(0 , 525)
d’où OLH
sin(0 , 25) 2
c’est-à-dire TL » 398 217 km et LA » 3,67 cm donc LA > LE.

127 Chaque point rouge est donc équidistant des 129 Le théorème de Pythagore dans le triangle ABC
deux autres. Le triangle formé par ces points est donc rectangle en B permet d’écrire :
équilatéral, et les angles aux sommets sont donc AC2 = AB2 + BC2 .
p Ainsi AC2 = 12 + 12
égaux rad.
3 et AC = 2.
On se place alors dans le repère (O ; I ; J). Le cercle Le théorème de Pythagore dans le triangle AEC rec-
rouge est donc de rayon 1. Les arcs rouges AB , BC
 et tangle en A permet d’écrire :

AC sont de même longueur et donc de longueur EC2 = AE2 + AC2 .
2p 2
. Ainsi EC2 = 12 + 2 .
3
d’où EC2 = 3 et EC = 3 .
Sur le cercle rouge trigonométrique, les points A et B
) = EA 1
p π Donc dans le triangle AEC, sin(ECA =
sont les images des réels et − . EC 3
3 3
π 3  π 3  = sin−1 1 .
d’où ECA
Or sin  = et sin−  = −  
 3   3
2  3 2
Le triangle AOC est isocèle en O
Ainsi AB = 3.
 = 180° − 2 sin−1 1 .
donc AOC  
 a pour longueur AB × π = 3π .
L’arc bleu AB  3
3 3  ≈ 109 , 5°.
AOC
Le tracé bleu a donc une longueur égale à l = 3π
Le périmètre du cercle rouge est égal à p = 2π 130 a) D’après le théorème de Pythagore, on a :
2 l 2 = R2 − h2
On a donc p = l.
3 = 202 − h2
128 Le théorème de Pythagore dans le triangle LEO = 400 − h2
rectangle en L permet d’écrire : L’aire de la base vaut ! = πl 2
OE2 = LE2 + OL2 = π(400 − h2 )
Ainsi LE2 = OE2 − OL2 Le volume du cône vaut donc
d’où LE2 = 4 2 − 2,12 = 11, 59. 1
V(h) = π(400 − h2 )h
Donc LE = 11, 59 cm soit LE » 3,4 cm. 3
• OLA est isocèle en O donc le pied H de la hauteur 1
b)  V(h) = π(400h − h3 )
issue de O est le milieu de [LA ]. 3
1
E 4 cm O A V ′(h) = π(400 − 3h2 )
3
c m

On a V ′(h) = 0 ⇔ 400 − 3h2 = 0


2,1

H
I L G 400
⇔ h2 =
1 3
LA
C06_fig131
 LH 2 20 20
Ainsi, cos(OLH) = = ⇔h= ou h = − .
LO 2 ,1 3 3
 ).
et LA = 4 , 2 cos(OLH On en déduit le tableau de variation de la fonction V.
20
 = 1 OLG
Or OLH  = 1 EOL
. h 0 20
2 2 3
En effet, OLG est aussi isocèle en L et (HL) est bissec- V¢(h) + 0 -
.  20 
trice de l’angle OLG V  
V(h)  3 
Comme cos(EOL  ) = OL = 2,1 = 0 , 525
EO 4 0 0

Chapitre 7  ★  Trigonométrie 123

172909_Chap07_000-000.indd 123 29/07/2019 17:13:20


20 En réalité, AH = 1, 5 × 45 = 67, 5 cm
Le volume du cône est maximal lorsque h = cm.
3 3
HB = HC = × 45 ≈ 39 cm
c) Le périmètre de la base est alors égal à : 2
p = 2π Installation :
 On perce le point A.
= 2π 400 − h2
 À l’aide du niveau, on trace un trait vertical partant
400 de A et on place H à 67,5 cm de A.
= 2π 400 −
3  À l’aide du niveau, on trace un trait horizontal pas-
2 sant par H et on perce les point B et C tels que
= 40π cm
3 HB = HC = 39 cm.
La longueur du cercle de base est aussi la longueur
de l’arc de cercle du carton restant après avoir ôté le 132 On réalise un schéma.
secteur circulaire d’angle au centre de mesure a.  . On utilise la formule d’Al-
• Déterminons l’angle ACB
2 Kashi dans le triangle ABC.
On a donc, 40π = 20(2π − α)
3 A 7,32 m
2 2 B
Ainsi 20α = 40π − 40π d’où α = 2π − 2π rad
3 3
soit α ≈ 66°.
E
16 m
20 m
D
9,15 m

Exploiter ses compétences


C
2 2 )
AB = AC + BC2 − 2AC × BC × cos(ACB
2 2 2
 ) = 7, 32 − 16 − 20
Ainsi cos(ACB
131 Tout d’abord, on réalise un croquis : ABC est un −2 × 16 × 20
triangle équilatéral, d’après le  Doc.  1, inscrit dans le 
soit cos(ACB) = 0 , 941277 5
cercle de rayon 1 (unité = 45 cm d’après le Doc. 2).
 est égal à environ 0,344 rad
L’angle ACB
 AC
Les arcs de cercle AB,  et BC sont donc de même
 représentant le mur a une lon-
• L’arc de cercle DE
2p
longueur . gueur égale à  ≈ 9 ,15 × 0 , 344 m soit  » 3,15 m
3
+
133 Coût
On complète le plan de face du Doc. 2 par le triangle
3 3 ABC équilatéral de côté 2 m.
-––– –––
2 2  est donc égal à p rad.
L’angle ABC
0 1 3
H  a pour longueur  = π × 2 m.
Ainsi l’arc de cercle AB
C B 3
A

Ainsi A, B, C sont les points image respectifs des réels


p π 5π
, − et − .
2 6 6
 π  1  π 3 p/3
Or sin−  = − , cos−  = et B C
 6  2 
 6 2 2m
 5π  3
cos−  = −
 6  2 Soit  » 2,1 m donc  ≈ 1, 5 m + 6 × 0 ,1 m
3 Bruno peut choisir entre des rouleaux de 1 m ou 2 m
Ainsi, sur le croquis, AH = 1, 5, HB = HC = .
2 de large.
124

172909_Chap07_000-000.indd 124 30/07/2019 14:53:59


Coût pour des rouleaux de 1 m de large Roue Bielle Piston
Le coût serait de 4 ´ (120 € + 6 ´ 10 €) soit 720 € M
Coût pour des rouleaux de 2 m de large
Le coût serait de 2 ´ (280 € + 6 ´ 13 €) soit 716 €. O p
– H1 P
4
Bruno va donc choisir les rouleaux de 2 m de large.
Déclaration en mairie
H est le pied de la hauteur issue de A dans le triangle 2
Ainsi, MH = OH = m De plus, le théorème de
 π  AH π 2
ABC. Ainsi sin  = donc AH = AB sin  soit Pythagore dans le triangle MHP rectangle en H
 3  AB 3
3  2 2
AH = 2 × m c’est-à-dire AH » 1, 73 m ce qui est donne  : MP = MH + HP donc HP = 3 −  
2 2 2 2 2
2  2 
inférieur à 1,80 m. 17
soit HP = m.
Ainsi la serre de Bruno ne nécessite aucune formalité 2
particulière. 2 17
On en déduit que la distance d vaut + m
2 2
134 Après 15,125 secondes, la roue aura fait, d’après soit environ 3,62 m.
1
le doc 2, 15 tours et de tour. Le point M est donc
8
2p p
l’image du nombre réel , c’est-à-dire sur le
8 4
cercle trigonométrique représenté par la roue.

Chapitre 7  ★  Trigonométrie 125

172909_Chap07_000-000.indd 125 30/07/2019 14:53:41


172909_Chap07_000-000.indd 126 29/07/2019 17:13:59
8 Fonctions sinus
et cosinus

3  a) 

Découvrir sin(x)
x
sin(x + 2p)
x + 2p
O cos(x) O cos(x + 2p)

    
1 Les fonctions trigonométriques Pour tout x, sin( x + 2π) = sin( x)
et cos( x + 2π) = cos( x).
1  b) On remarque que point U appartient au cercle b)  Graphiquement on peut tracer les courbes
trigonométrique. représentatives des fonctions cos et sin sur les
Pour tout nombre réel t de l’intervalle [0 ; 2π], intervalles [−3π ; − π], [π ; 3π],  par transla-
OU2 = cos2 t + sin2 t = 1 donc OU = 1.  
tion de vecteur −2πi , 2pi ,…
2  b)  M(t ; sin(t )) avec 0 < t < 2p.
Le point M décrit donc la courbe représentative
de la fonction sinus sur l’intervalle [0 ; 2π].

3  b)  P(t ; cos(t )) avec 0 < t < 2p. Acquérir des automatismes
Le point P décrit donc la courbe représentative
de la fonction cosinus sur l’intervalle [0 ; 2π].

2  Parité et périodicité de cos et sin 3 a) Pour tout nombre réel x,


g(−x) = 2 cos(−x) − cos(−2 x)
1  On sait que cos(0) = 1 et sin(0) = 0 donc la g(−x) = 2 cos( x) − cos(2 x)
courbe rouge représente la fonction cos et la g est donc une fonction paire.
courbe bleue représente la fonction sin.
La fonction cosinus est paire, il en est de même des
2  a)  On observe que la courbe rouge est symé- fonctions x  cos(a x + b) (avec a ∈  ∗ et b Î  ).
trique par rapport à l’axe des ordonnées et que b) g est une fonction paire donc sa courbe est symé-
la courbe bleue est symétrique par rapport à trique par rapport à l’axe des ordonnées du repère.
l’origine du repère. On peut conjecturer que la Donc la courbe de g est affichée sur l’écran 2.
fonction cos est paire et que la fonction sin est
impaire. 4 a) Pour tout nombre réel x,
 4 π    4π 
b)  h x + 
 = cos1, 5 x + 
sin(x)  3    3 
x cos(x) x  4 π   4π 
h x +
  = cos1, 5 x + 1, 5 × 
O -x O -x 3   3
cos(- x) 
sin(- x) 4 π 
h x +  = cos(1, 5 x + 2π) = cos(1, 5 x) = h( x).
      3 
4p
cos(−x) = cos( x) sin(−x) = −sin x. Donc la fonction h est périodique de période .
3
Chapitre 8  ★  Fonctions sinus et cosinus 127

172909_Chap08_000-000.indd 127 30/07/2019 15:06:13


p 13 a) Pour tout nombre réel x,
b) Une graduation en abscisse représente donc la f (−x) = sin(−2 x) = −sin(2 x) = −f ( x).
3
partie de la courbe qui se « répète » doit être tracée La fonction f est donc impaire.
sur 4 graduations, ce qui est le cas de l’écran . b) On peut déduire que la courbe représentative de la
fonction f est symétrique par rapport à l’origine du
π 1 π 2
5 a)  sin  = b)  sin  = repère.
 6  2  4  2
π 14 a) Pour tout nombre réel x,
c)  sin  = 1 d)  sin(π) = 0 g(−x) = −x + sin(−x)
 2 
g(−x) = −x − sin( x)
6 Pour tout nombre réel x, g(−x) = −( x + sin( x))
sin( x + 9π) = sin( x + π + 4 × 2π) g(−x) = −g( x).
= sin( x + π) = −sin( x). La fonction g est donc impaire.
Julie n’a donc pas raison. b) On peut déduire que la courbe représentative de la
fonction g est symétrique par rapport à l’origine du
7 On conjecture que la fonction f est impaire et
repère.
périodique de période 1.
15 1. 
8 La fonction sinus est croissante sur l’intervalle
 π   π 
− ; 0 donc f est décroissante sur − ; 0 .
 2   2 
L’affirmation (2) est donc exacte.

9 La fonction f est paire et la fonction g est impaire.


b) La fonction f est périodique de période p.
2. a)  (0 ; 0)             b)  (3π ; 0)             c)  (−2π ; 0)
p
La fonction g est périodique de période .
 2 16 a) Pour tout nombre réel x,
c)  La translation du vecteur pi laisse invariante la π 
courbe # f . f( x + 8) = 3 sin ( x + 8)
 4 
p
La translation du vecteur i laisse invariante la π 
2 f( x + 8) = 3 sin x + 2π
courbe # g .  4 
 π 
10 a) h a pour période 0,5. f( x + 8) = 3 sin x
 4 
b) Pour tout nombre réel x,
f ( x + 8) = f ( x).
h( x + 1) = h( x + 0 , 5 + 0 , 5) = h( x + 0 , 5) = h( x)
La fonction f est donc périodique de période 8.
11 b) Pour tout nombre réel x,
π 
f(−x) = 3 sin (−x)
 4 
 π 
f(−x) = −3 sin ( x)
 4 
f (−x) = −f ( x).
La fonction f est donc impaire.
c)  À partir du tracé de la courbe représentative de f
sur l’intervalle [0 ; 4 ], on obtient par symétrie de
12 a)  f est impaire (sa courbe est symétrique par
centre l’origine du repère la courbe sur l’intervalle
rapport à O) et périodique de période 2. 
[−4 ; 4 ] puis par translation de vecteur 8i la courbe
b) Pour tout nombre réel x,
sur l’intervalle [−4 ; 12].
f (−x) = −2 sin(−2 x) = −(−2 sin(2 x)) = −f ( x)
f est donc impaire. 17 a) Pour tout nombre réel t > 0,
Pour tout nombre réel x, π  π 
C(t + 4) = 250 sin (t + 4) = 250 sin t + 2π
f( x + 2) = −2 sin(π( x + 2)) = −2 sin(πx + 2π)  2   2 
= −2sin(πx) = f ( x) π 
(La fonction sinus est périodique de période 2p). = 250 sin t  = C(t ).
 2 
128

172909_Chap08_000-000.indd 128 30/07/2019 15:07:00


b) On peut en déduire que la concentration du médica-
3 π
ment sera nulle au bout de 4 h dans le sang du patient. b)  Dans [−π ; π], sin(t ) = équivaut à x = ou
2π 2 3
x= .
π 3
18 a)  Dans [0 ; π], sin( x) = 0 , 5 équivaut à x =  π 2π 
5π 6 L’ensemble des solutions est 6 =  ; .
ou x = .  3 3 
6
 π 5π  π 7π 2π 8π
L’ensemble des solutions est 6 =  ; . c)  + 2π = et + 2π = .
 6 6  3 3 3 3
π On utilise le fait que la fonction sin est périodique de
b) Dans [−π ; π], sin( x) = 0 , 5 équivaut à x = ou
5π 6 période 2π.
x= .
6  π 5π   7π 8π 
L’ensemble des solutions est 6 =  ; L’ensemble des solutions est 6 =  ; .
.  3 3 
 6 6 
π 13π 5π 17π 3 π
c)  + 2π = et + 2π = d) Dans , sin( x) = équivaut à x = + k 2π ou
6 6 6 6 2 3
On utilise le fait que sin est de période 2p. L’en- 2π
x= + k 2π, avec k Î .
13π 17π  3
semble des solutions est 6 =  ; .
 6 6  L’ensemble des solutions est :
π  π 2π 
d) Dans , sin( x) = 0 , 5 équivaut à x = + k 2π ou 6 =  + k 2π ; + k 2π  avec k Î .
5π 6  3 3 
x= + k 2π avec k Î .
6  π
L’ensemble des solutions est : 21 1. La fonction sinus est croissante sur  0 ;  et
π   2 
π 5π 
6 =  + k 2π ; + k 2π avec k Î . décroissante sur  ; π .
 6 6   2 
2. a) • L’intervalle [−π ; π] est symétrique par rapport
19 a)  Dans [0 ; π], 1 + 2 sin( x) = 0 équivaut à à O donc c’est la parité qui permet d’obtenir le sens
sin( x) = −0 , 5. de variations de f sur [−π ; π]
Or sur [0 ; π], sin( x) > 0 donc l’équation n’a pas de
•  0 + 2π = 2π et π + 2π = 3π
solution.
L’ensemble des solutions est 6 = ∅. C’est la périodicité qui permet d’obtenir le sens de
5π variations de f sur [2π ; 3π]
b) Dans [−π ; π], sin( x) = −0 , 5 équivaut à x = −
6 π 3π
π •  0 − 2π = −2π et − 2π = −
ou x = − . 2 2
6  5π π
L’ensemble des solutions est 6 = − ; − . C’est la périodicité qui permet d’obtenir le sens de
 6 6  variations de f sur [−4π ; − 3π]
5π 7π π 11π b)
c)  − + 2π = et − + 2π = . π p
6 6 6 6 x −π − 0 p
2 2
On utilise le fait que la fonction sin est périodique de
0 1
période 2p.
 7π 11π  f (x)
L’ensemble des solutions est 6 =  ; .
 6 6  -1 0

d) Dans , sin( x) = −0 , 5 équivaut à x = − + k 2π 5p
6 x 2p 3p
π 2
ou x = − + k 2π avec k Î .
6 1
L’ensemble des solutions est :
f (x)
 5π π 
6 = − + k 2π ; − + k 2π  k Î . 0 0
 6 6 

20 a)  Dans [0 ; π], −4 sin(t ) + 2 3 = 0 équivaut à x −2π −
2
3 π 2π 1
sin(t ) = équivaut à x = ou x = .
2 3 3
 π 2π  f (x)
L’ensemble des solutions est 6 =  ; . 0
 3 3 

Chapitre 8  ★  Fonctions sinus et cosinus 129

172909_Chap08_000-000.indd 129 30/07/2019 15:08:09


22 1. a) Périodicité : 3π − 2 × 2π = −π et 30 a) f semble paire et sa période semble être 2.
4π − 2 × 2π = 0. b) • Pour tout nombre réel x,
b) Parité : les intervalles [- 4p ; - 3p] et [3p ; 4p] sont f (−x) = −2 cos(π(−x)) = −2 cos(πx) = f ( x)
symétriques par rapport à 0. (La fonction cos est paire).
c) Parité : l'intervalle [- p ; p] est symétrique par rap- f est donc paire.
port à 0. • Pour tout nombre réel x,
f( x + 2) = −2 cos(π( x + 2)) = −2 cos(πx + 2π)
2. π
x −π − 0 = −2cos(πx)
2
(La fonction cos a pour période 2p).
0 0
f est donc périodique de période 2.
sin (x)
- 1 31 a) Pour tout nombre réel x,
π p f(−x) = cos(2(−x)) = cos(−2 x) = cos(2 x)
x −π − 0 p
2 2 (La fonction cos est paire).
0 1 Donc f est paire.
sin (x) 0 b) On peut déduire que la courbe représentative dans
- 1 0 un repère orthogonal est symétrique par rapport à
l’axe des ordonnées.

x −4π − −3π
2
32 1.
1
sin (x)
0 0

π 1 π 2
23 a)  cos  = b)  cos  =
 3  2  4  2
π
c)  cos  = 0 d)  cos(π) = −1.
 2 
2. a)  x = 0      b)  x = 3π      c)  x = −2π      d)  x = 2π.
24 Pour tout nombre réel x,
cos( x + 10π) = cos( x + 5 × 2π) = cos( x) car la fonc- 33 a) Pour tout nombre réel t > 0,
tion cos a pour période 2p.  8π 
P(t + 0 , 75) = 100 − 20 cos (t + 0 , 75)
Fiona a donc raison.  3 
 8π 
P(t + 0 , 75) = 100 − 20 cos t + 2π
25 On peut conjecturer que f est paire et de période 1.  3 
 8π 
 π P(t + 0 , 75) = 100 − 20 cos t 
26 (3) f est croissante sur −π ; −  et croissante  3 
 3π   2  (La fonction cos a pour période 2p).
sur π ; .
 2  P(t + 0 , 75) = P(t ).
b) Un cycle dure 0,75 s donc dans 60 secondes il y a
27 a) h a pour période 1. 60
= 80 cycles c’est-à-dire 80 battements de cœur.
b) Pour tout nombre réel x, h( x + 2 × 1) = h( x). 0 , 75

28 a) f et g sont des fonctions paires. 2 π


34 a) Dans [0 ; π], cos( x) = équivaut à x = .
p 2 4
b) f a pour période p et g a pour période .  π 
2 L’ensemble des solutions est 6 =  .
 4 
29 b)  La fonction cos est paire donc dans [−π ; π],
2 π π
cos( x) = équivaut à x = ou x = − .
2 4 4
 π π 
L’ensemble des solutions est 6 = − ; .
 4 4 
π 7π π 9π
c)  − + 2π = et + 2π =
4 4 4 4
130

172909_Chap08_000-000.indd 130 30/07/2019 15:08:49


On utilise le fait que cos a pour période 2p.
x 6p 7p 8p
 7π 9π 
L’ensemble des solutions est 6 =  ; .
 4 4  1 1
2 π f (x)
d)  Dans , cos( x) = équivaut à x = + k 2π
2 4 -1
π
ou x = − + k 2π avec k Î .
4 x −4π −3π
L’ensemble des solutions est : 1
 π π 
6 =  + k 2π ; − + k 2π avec k Î . f (x)
 4 4  -1

35 a)  Dans [0 ; π], 3 + 2 cos( x) = 0 équivaut à 37 1. a)  π − 2π = −π et 3π − 2π = π.


3 5π C’est donc la périodicité qui permet d’obtenir le
cos( x) = − équivaut à x =
2 6 tableau de variations de cos sur [−π ; π].
 5π 
L’ensemble des solutions est 6 =  . b)  C’est la parité qui permet d’obtenir le tableau de
 6 
variations de cos sur [−3π ; − π].
b)  La fonction cos est paire donc dans [−π ; π],
c)  π + 2π = 3π et 3π + π = 4π
3 5π 5π
cos( x) = − équivaut à x = ou x = − . C’est donc la symétrie d’axe x = 3π qui permet d’ob-
2 6 6 tenir le tableau de variations de cos sur [2π ; 4π].
 5π 5π 

L’ensemble des solutions est 6 = − ; . 2.
 6 6  x −π 0 p
5π 17π 5π 7π
c)  + 2π = et − + 2π = . 1
6 6 6 6
On utilise le fait que cos a pour période 2p. cos (x)
 7π 17π  -1 -1
L’ensemble des solutions est 6 =  ; .
 6 6 
x −3π −2π −π
3 5π
d) Dans , cos( x) = − équivaut à x = + k 2π 1
2 6
5π cos (x)
ou x = − + k 2π, k Î .
6 -1 -1
L’ensemble des solutions est :
 5π 5π  x 2p 3p 4p
6= + k 2π ; − + k 2π k Î .
 6 6  1 1
cos (x)
36 1. La fonction cos est décroissante sur [0 ; π] et
- 1
croissante sur [π ; 2π].
2. a) • C’est la parité qui permet d’obtenir à partir des 38 1. B            2. D            3. C            4. A            5. B
variations de f sur [0 ; 2π] celles sur [−2π ; 0 ].
•  0 + 3 × 2π = 6π et 2π + 3 × 2π = 8π. 39 1. B. C.
C’est donc la périodicité qui permet d’obtenir le sens 2. C.
de variations de f sur [6π ; 8π] à partir de la question 1. 3. A. B. D.
•  0 − 2 × 2π = −4π et π − 2 × 2π = −3π. 4. D.
C’est donc la périodicité qui permet d’obtenir le sens
40 1. • Pour tout nombre réel x,
de variations de f sur [−4π ; 2π].
f (−x) = 2 cos(3(−x)) = 2 cos(−3 x) = 2 cos(3 x) = f ( x)
b) f est donc paire.
x −2π −π 0
• Pour tout nombre réel x,
1 1 g(−x) = 2 sin(3(−x)) = 2 sin(−3 x) = −2 sin(3 x) = −g( x)
f (x) g est donc impaire.
- 1 L’affirmation est donc fausse.

Chapitre 8  ★  Fonctions sinus et cosinus 131

172909_Chap08_000-000.indd 131 30/07/2019 15:09:41


π
2.  f(0) = 2 cos(0) = 2 et f   = 2 cos(π) = −2
 π 
 3 
p S'entraîner
f (0) ≠ f   donc f n’est pas périodique de période .
 3  3
L’affirmation est donc fausse.
3. Pour tout nombre réel x, −1 < cos(3 x) < 1 donc
−2 < 2 cos(3 x) < 2 c’est-à-dire −2 < f( x) < 2. 46
3
L’affirmation est donc vraie.
4. Pour tout nombre réel x,
g( x + 2π) = 2 sin(3( x + 2π)) = 2 sin(3 x + 6π)
= 2 sin(3 x) = g( x)
(La fonction sinus est périodique de période 2p).
L’affirmation est donc vraie.

41 • La courbe représentative de la fonction sinus


est symétrique par rapport à l’origine du repère et elle
π  47
passe par le point de coordonnées  ; 1 , il s’agit
 2 
donc de la courbe .
• La courbe représentative de la fonction cosinus est
symétrique par rapport à l’axe des ordonnées et
passe par le point de coordonnées (π ; −1), il s’agit
donc de la courbe .

42 a) La période est 3.


b) La période est p.
49 a)  Lorsqu’on déplace le curseur de 0 à 8, on
43 • Pour tout nombre réel x, obtient la courbe ci-dessous.
f(−x) = sin2 (−x) − 3 cos(−x)
f(−x) = (−sin( x))2 − 3 cos( x) (La fonction cos est
paire et la fonction sin est impaire).
f(−x) = sin2 ( x) − 3 cos( x)
f (−x) = f ( x).
La fonction f est donc paire. b)  On peut conjecturer qu’on obtient une sinusoïde
• Pour tout nombre réel x, de période 4.
g(−x) = sin(−x)cos(−x)
g(−x) = −sin( x)cos( x) (La fonction cos est paire et la 50 a)  Lorsqu’on déplace le curseur de 0 à p, on
fonction sin est impaire). obtient la courbe ci-dessous.
g(−x) = −g( x).
La fonction g est donc impaire.

44

b)  On peut conjecturer qu’on obtient une sinusoïde


p
de période .
2
51 a) Pour tout nombre réel x,
f ( x) = cos(a x + b) = cos(a x + b + 2π)
  2π    2π 
= cosa x +  + b = f  x + 
  a 
 
 a

(car la fonction cos a pour période 2π). Ainsi T = .
a
132

172909_Chap08_000-000.indd 132 30/07/2019 15:10:12


b)  À partir du graphique, on constate que g a pour  π p
1 3. • Pour tout x de l’intervalle  0 ;  on a, 0 < x <
période 4 et que g(0) = .  2  2
2 donc 0 < 2 x < p.
2π 2π π La fonction cos étant décroissante sur [0 ; π] on
Donc a = = =
T 4 2 déduit que la fonction x  cos(2 x) est décroissante
π   π
g( x) = cos x + b sur  0 ;  et donc f également.
 2   2 
1 π  p
g(0) = cos(b) = . • Pour tout x de l’intervalle  ; π on a < x < p
2  2  2
donc p < 2 x < 2p.
π 1 π La fonction cos étant croissante sur [π ; 2π] on déduit
Or 0 < b < donc cos(b) = équivaut à b = .
2 2 3 π 
π que la fonction x  cos(2 x) est croissante sur  ; π
π  2 
Ainsi g( x) = cos x + . et donc f également.
 2 3

52 On pose h la fonction définie sur [0 ; π] par  π π


54 a)  Dans − ;  , 2 cos( x) − 3 = 0 équivaut
h( x) = f ( x) − g( x)  2 2 
h( x) = cos2 ( x) − cos( x) = cos( x)(cos( x) − 1) 3 π π
à cos( x) = équivaut à x = ou x = − .
Pour tout nombre réel x de [0 ; π], cos( x) < 1 donc 2 6 6
cos( x) −1 < 0.  π π 
L’ensemble des solutions est 6 = − ; .
 π π   6 6 
Sur  0 ;  , cos( x) > 0 et sur  ; π , cos( x) < 0.
 2   2  b) Il s’agit du cercle trigonométrique .
On dresse le tableau de signes de f( x).  π π
c)  Dans − ;  , 2 cos( x) − 3 > 0 équivaut à
p
 2 2 
x 0 p
2 3
cos( x) > équivaut à x appartient à l’intervalle
cos( x) -1 0 - - 2
cos( x) + 0 -  π π
− ;  .
0 - 0 +  6 6 
h( x)
L’ensemble des solutions dans  de l’inéquation
La position relative de # f et # g est donnée par le
2 cos( x) − 3 > 0 est donc :
signe de h( x).
D’après le tableau # f est au-dessous de # g sur l’in-  π π 
6 = − + k 2π ; + k 2π avec k Î .
 π  6 6 

tervalle  0 ;  et # f est au-dessus de # g sur l’inter-
 2 
π  55 1. a)  On trace en gras
valle  ; π . # f et # g se coupent aux points de sur le cercle trigonométrique
 2 
π  l’ensemble des points M asso- p 0
coordonnées (0 ; 0),  ; 0. ciés aux nombres réels x tels O
 2 
que sin( x) > 0.
53 1. a) Pour tout nombre réel x, On déduit que l’ensemble des
f (−x) = 3 cos(2(−x)) = 3 cos(−2 x) = 3 cos(2 x) = f ( x) solutions dans l’intervalle
La fonction f est donc paire. [0 ; 2π] de l’inéquation sin( x) > 0 est 6 1 = [0 ; π].
b) D’après a), la courbe représentative de la fonction f b)  On trace en gras sur le p

4
est symétrique par rapport à l’axe des ordonnées, on cercle trigonométrique l’en-
peut donc restreindre l’étude de f à l’intervalle semble des points M associés p 0
[0 ; + ∞[. aux nombres réels x tels que O 2p
2. a) Pour tout nombre réel x, 2
cos( x) > . 7p

f( x + π) = 3 cos(2( x + π)) = 3 cos(2 x + 2π) 2 4
= 3 cos(2 x) = f ( x). On déduit que l’ensemble des solutions dans l’inter-
La fonction f est donc périodique de période p. valle [0 ; 2π] de l’inéquation − 2 + 2 cos( x) > 0 est
b)  On peut restreindre l’étude de f à un intervalle  π   7π 
6 2 = 0 ;  ∪  ; 2π .
d’amplitude p, donc à [0 ; π].  4   4 

Chapitre 8  ★  Fonctions sinus et cosinus 133

172909_Chap08_000-000.indd 133 30/07/2019 15:11:22


2. a)  58 a)  On peut conjecturer que la fonction g a un
p 7p
x 0 p maximum en x ≈ −1 égal à 1 et un minimum en
4 4
2p
x » 2 égal à -1.
sin( x) 0 + + 0 - - 0
 π
+ 0 - - 0 + b) Sur [−π ; π], l’équation cos x +  = 1 équivaut à
− 2 + 2 cos( x)  3
π π
(− 2 + 2cos( x)) sin( x) 0 + 0 - 0 + 0 - 0 x + = 0 c’est-à-dire x = − .
3 3
b)  D’après le tableau de signes obtenu au a), on  π 
déduit que l’ensemble des solutions de l’inéquation E g −  = cos(0) = 1 et on sait que pour tout x de
 3 
π   7π   π
est 6 =  ; π  ∪  ; 2π  . [−π ; π], cos x +  < 1 donc 1 est atteint par g
 4   4   3
donc le maximum de g sur [−π ; π] est 1 obtenu en
56 1. Pour tout nombre réel x, −1 < sin( x) < 1 π
x x x − .
donc − 1 < + sin( x) < 1 + 3 
2 2 2 π
Sur [−π ; π], l’équation cos x +  = −1 équivaut à
x x  3
c’est-à-dire − 1 < f( x) < + 1.
2 2 π π 2π
x + = π ou x + = −π c’est-à-dire x = ou
2. a) La courbe # est située entre les droites d1 et d2 3 3 3
x x 4π
d’équations respectives y = − 1 et y = + 1. x=− .
2 2 3
x
b)  L’équation f( x) = − 1 équivaut à sin( x) = −1 4π
Or − ∉ [−π ; π].
π2 3
c’est-à-dire à x = − + k 2π avec k Î .
2  2π 
# coupe donc d1 en une infinité de points. g   = cos(π) = −1 et pour tout x de [−π ; π],
 3 
x  π
L’équation f( x) = + 1 équivaut à sin( x) = 1 c’est-à- cos x +  > − 1 donc -1 est atteint par g donc le
2  3
π 2p
dire à x = + k 2π avec k Î . minimum de g sur [−π ; π] est -1 obtenu en .
2 3
# coupe donc d2 en une infinité de points.
 1 59 1. a)  On trace à la calculatrice dans la fenêtre
d1 et d2 ont le même coefficient directeur   , donc
 2  p p p
elles sont parallèles. < X < , pas ; 0 < Y < 10 , pas 2 la courbe
4 2 8
représentative de R.
57 a)  f(0) = 1 et T = 2
b) • Pour tout nombre réel x > 0,
f ( x) = 2 sin(a x + b) = 2 sin(a x + b + 2π)
(La fonction sin a pour période 2p )
  2π  
f ( x) = 2 sina x +  + b
  a 
 2π 
f ( x) = f  x + .
 a
2π On conjecture que le maximum de R(θ) est 6,25 m.
Or T est la période donc = T c’est-à-dire aT = 2π.
a 100  π  π
b)  R(θ) = sin2θ −  = 6 , 25 sin2θ − .
•  f(0) = 1 équivaut à 2 sin(b) = 1. 16 
 4 
 4
aT = 2π π π
a et b vérifient donc le système  . Pour tout nombre réel q de l’intervalle  ;  ,
 2 sin(b) = 1   4 2 
π
 R(θ) < 6 , 25 car sin2θ −  < 1.
2π 2π  4
c)  aT = 2π donc a = = = π. π π  π
T 2 Sur  ;  , R(θ) = 6 , 25 équivaut à sin2θ −  = 1
 π  4 2  
 4
Sur  0 ;  l’équation équivaut à 2 sin(b) = 1 équi-
 2  π π 3π
équivaut à 2θ − = équivaut à θ = .
1 2 π 4 2 8
vaut à sin(b) = = équivaut à b = . 3π
2 2 4 La valeur 6,25 est atteinte pour θ = donc R(θ) est
 π  3p 8
Ainsi f( x) = 2 sinπx + . maximum pour .
 4 8
134

172909_Chap08_000-000.indd 134 30/07/2019 15:12:48


v 2  π b) Pour tout réel t ∈ [0 ; 5],
2. R(θ) = sin2θ − 
16  4  2π   2π  1
π π •  0 , 6 sin t  = 0 , 3 équivaut à sin t  =
Pour tout nombre réel q de l’intervalle  ;  ,  5   5  2
 4 2 
v2  π 2π π 2π 5π
R(θ) < car sin2θ −  < 1. équivaut à t = ou t=
16  4 5 6 5 6
5 25
π π v2  π équivaut à t = ou t =
Sur  ;  , R(θ) = équivaut à sin2θ −  = 1 12 12
 4 2  16 
 4 5
π π 3π La personne inspire 0 , 3 L ⋅ s−1 à t = s et à
c’est-à-dire 2θ − = ce qui donne θ = . 25 12
4 2 8 t= s.
v2 v2 12  2π 
est atteint par R donc le maximum de R est • Pour t ∈ [0 ; 5], 0 , 6 sin t  = −0 , 3 équivaut à
16 16  5 
qui dépend de v.
 2π 
La vitesse influence donc la distance horizontale −sin t  = −0 , 5
 5 
maximale.
2π 7π 2π 11π
équivaut à t= ou t=
60 1. H(0) = −6 , 4 sin(0) − 11, 6 cos(0) + 13, 9 = 2, 3 5 6 5 6
B(0) = −7, 4 sin(0) − 9 , 5 cos(0) + 5, 4 = −4 ,1. 35 55
équivaut à t = ou t =
L’écart de températures au 1er janvier est de 12 12
35
2, 3 − (−4 ,1) = 6 , 4 °C. La personne expire 0 , 3 L ⋅ s−1 à t = s et à
2. a)  La valeur x = 0 correspond à la valeur t = 0 55 12
t= s.
c’est-à-dire au 1er janvier. 12
La valeur e = 6 , 4 correspond à l’écart de tempéra-
tures au 1er janvier. 62 1. a)  On peut conjecturer que f est paire et
b)  x représente le temps en mois, e l’écart de tempé- périodique de période 2p.
ratures à l’instant t et m le mois où l’écart de tempéra- b) Pour tout réel x,
1 1
tures est le plus grand. f (−x) = = = f ( x).
2 + cos(−x) 2 + cos( x)
c) On obtient l’affichage ci-dessous.
1 1
f ( x + 2π) = = = f ( x).
2 + cos( x + 2π) 2 + cos( x)
c)  D’après b), f est paire donc on peut restreindre
Le plus grand écart de températures est de 10 , 8 °C l’inter­valle d’étude à [0 ; + ∞[.
et il est obtenu au cours du 5e mois. De plus, f est périodique de période 2p, donc on
3.  peut étudier le sens de variations de f sur [0 ; π].
2. a) Pour tout nombre réel x de [0 ; π],
sin( x)
f ′( x) = .
(cos( x) + 2)2
sin( x) > 0 et (cos( x) + 2)2 > 0 donc f ′( x) > 0, f est
donc croissante sur [0 ; π].
b)  On obtient la courbe représentative de f sur
[−π ; π] par la symétrie d’axe des ordonnées puis on
 
applique les translations de vecteurs 2pi et −2πi .

63 1. a) Pour tout nombre réel x,


g( x + 2π) = (1 + cos( x + 2π))sin( x + 2π)
g( x + 2π) = (1 + cos( x))sin( x) car sin et cos ont pour
61 a) Pour tout réel t de [0 ; 5], période 2p.
 2π   2π  g( x + 2π) = g( x).
f (t + 5) = 0,6 sin (t + 5) = 0,6 sin t + 2π La fonction g est donc périodique de période 2π.
 5   5 
 2π  b) Pour tout nombre réel x,
= 0,6 sin t  = f (t ). g(−x) = (1 + cos(−x))sin(−x)
 5 
g(−x) = (1 + cos( x)) × (−sin x)
La période d’un cycle est donc T = 5 s.

Chapitre 8  ★  Fonctions sinus et cosinus 135

172909_Chap08_000-000.indd 135 30/07/2019 15:13:57


La fonction cos est paire et la fonction sin est impaire. 0
g(−x) = −g( x). a) 
66f(0) = + cos(0) = 1
2
La fonction g est donc impaire. 2π
f (0 + 2π) = f (2π) = + cos(2π) = π + 1
c)  La parité de g permet de restreindre l’étude de la 2
fonction g à l’intervalle [0 ; + ∞[ et sa périodicité à f (0) ≠ f (0 + 2π) donc l’affirmation est fausse.
l’intervalle [0 ; π]. π π π π π 2
b)  ∈ [0 ; 1] et f   = + cos  = + et
 π 4 
 4  8 
 4  8 2
2. a) Par la lecture graphique, g ′( x) > 0 sur  0 ;  et
 3  π 2
π  + >0
g ′( x) < 0 sur  ; π et g ′( x) = 0 si et seulement si 8 2
π  3 
x= . L’affirmation est donc fausse.
3
b) 
p
x 0 p
3

Organiser son raisonnement


g ¢( x) + 0 -
3 3
g( x) 4

0 0
5π π p 5p 3 a)Pour tout réel x,
67
x −2π − −π − 0 p 2p
3 3 3 3 π  π 
f( x + 10) = sin ( x + 10) = sin x + 2π
0  5  
5 
3 3 3 3
g( x) 4 0 0 4 0 π 
3 3 3 3 = sin x = f ( x).
0
-
4
-
4
 5 
π  π 
g( x + 6) = sin ( x + 6) = sin x + 2π
π  3  
3 
64 a)  sin  = 1
 2  π 
π = sin x = g( x).
Or > 1  3 
2
La proposition est donc fausse. Ainsi f est de période T1 = 10 et g de période T2 = 6.
2 2
π π  2   2  T1 a a
b)  sin2   − cos2   =   −   = 0. b) On pose
T2
= avec
b b
irréductible.
 4   4   2   2 
La proposition est donc vraie. Ainsi bT1 = aT2 .
c) Pour tout nombre réel x, Pour tout nombre réel x,
π  π  π  h( x + bT1) = f ( x + bT1) + g( x + aT2 )
cos ( x + 12) = cos x + 2π = cos x = f ( x) + g( x) = h( x)
 6  
6   6 
donc T = bT1 = aT2
π  π  π 
cos ( x − 24) = cos x − 4π = cos x − 2 × 2π (car f a pour période T1 et g a pour période T2 ).
 6   6   6  T 10 5
π  c)  1 = = donc a = 5, b = 3
= cos x. T2 6 2
 6  et T = 5 ´ 6 = 3 ´ 10 = 30.
La proposition est donc vraie.
68 1. a) On pose X = sin( x).
65 a) Pour tout nombre réel x, −1 < sin( x) < 1. L’équation devient 2X2 + X − 1 = 0.
x
Si x est solution de (E) alors sin( x) = donc b)  2X2 + X − 1 = 0 si et seulement si X = −1
x 2 1
-1 < < 1 donc -2 < x < 2. ou X =
2 2  1
x
b) Si x est solution de (E) alors sin( x) = L’ensemble des solutions est 6 = −1 ; .
x 2
c)  X = sin x  2 
donc −sin( x) = −
2 3π
Dans [0 ; 2π[ , sin x = −1 si et seulement si x =
−x 2
donc sin(−x) = 1 π 5π
2 sin x = si et seulement si x = ou x = .
donc -x est solution de (E). 2 6 6
136

172909_Chap08_000-000.indd 136 30/07/2019 15:15:01


L’ensemble des solutions de (E) est : The maximum number of owls is 180, this occurs after
 π 5π 3π  5 years.
6= ; ; .
 6 6 2  b) For any real number t > 0, M(t ) > 300
2. Pour tout nombre réel x, cos2 ( x) + sin2 ( x) = 1 π π
M(t ) = 300 equals sin t +  = −1
10 20 
donc cos2 ( x) = 1 − sin2 ( x).
L’équation (F) est équivalente à : π π 3π
equals t + = + k 2π with k Î 
1 − sin2 x + 2 sin2 ( x) = 2 10 20 2
29
c’est-à-dire à sin2 ( x) = 1. equals t = + 20k with k Î 
2
On pose X = sin( x). The minimum number of mice is 300, this occurs after
L’équation (F) s’écrit X2 = 1 qui admet pour solution 14,5 years.
X = −1 ou X = 1. c) When the number of mice increases, the owl popu-
π lation increases too, and reaches the maximum, howe-
Dans [−π ; π], sin( x) = 1 si et seulement si x =
π 2 ver the mice population often plummets, leaving too
sin( x) = −1 si et seulement si x = −
2  little food for owls whose population also then crashes.
π π
L’ensemble des solutions de (F) est 6 = − ; .
 2 2   π
71 a) Pour tout nombre réel x de  0 ;  :
 2 
 2π 
69 a) Pour tout nombre réel t de  0 ; ,  1 
 13  cos( x) − (2 cos( x) + 2) = 2 cos ( x) + 2 cos( x)
2
 2 
−0 , 65 < v(t ) < 0 , 65. −cos( x) − 1
2
v(t ) = 0 , 65 si et seulement si sin(13t ) = 1 = 2 cos ( x) + cos( x) − 1.
π  π
si et seulement si 13t = b) Pour tout x de l’intervalle  0 ;  , 2 cos( x) + 2 > 0.
2  2 
π  π 1 π
si et seulement si t = Sur  0 ;  , cos( x) − = 0 équivaut à x = .
26  2  2 3
v(t ) = −0 , 65 si et seulement si sin(13t ) = −1 p p

2 —
3π 3
si et seulement si 13t =
2
3π 0
si et seulement si t = O 1
26 —
2
Ainsi, la vitesse maximale est 0,65 m × s- 1 à l’instant
π
t= s et la vitesse minimale est - 0,65 m × s- 1 à L’ensemble des points M associés aux nombres réels
26 3π
l’instant t = s. 1
x tels que cos( x) − > 0 est tracé en gras.
26 2
b)  v(t ) = 0 si et seulement si sin(13t ) = 0 L’ensemble des solutions de l’inéquation
si et seulement si 13t = 0 ou 13t = π ou 13t = 2p, 1  π
cos( x) − > 0 est 6 =  0 ;  .
π 2π 2  3 
si et seulement si t = 0 ou t = ou t = .
13 13 p p
 2π   2π  x
h  = 0 , 05 cos13 ×  = 0 , 05 cos(2π) = 0 , 05. 0 3 2
 13   13  2 cos( x) + 2 + +
 π    π 
h  = 0 , 05 cos13  1
+ -
13   13  cos( x) - 0
2
= 0 , 05 cos(π) = −0 , 05 et h(0) = 0,05.
2 cos2 ( x) + cos( x) − 1 + 0 -

Les vitesses sont nulles lorsque le piston est dans sa L’inéquation a pour ensemble de solution l’intervalle
position haute et basse.  π
0 ;  .
 3 
70 a) For any real number t > 0, O(t ) < 180
π  72 Le point M appartient au cercle # de centre O
O(t ) = 180 equals 30 sin t  = 30
10  et de rayon 1
π π
equals t = + k 2π Ses coordonnées dans le repère orthonormé (O, A, C)
10 2 p
equals t = 5 + 20k with k Î  sont (cos(t ) ; sin(t )) avec 0 < t < .
2
Chapitre 8  ★  Fonctions sinus et cosinus 137

172909_Chap08_000-000.indd 137 30/07/2019 15:16:17


Le point P a donc pour coordonnées (sin(t ) ; sin(t )) • Le potentiel émotionnel a pour période 28 jours
car il appartient à la droite (OB) d’équation y = x. 2π 2π
donc = 28 ce qui donne ω= .
Le point Q a pour coordonnées (sin(t ) ; 0). ω 28
L’aire du rectangle PQAR est QA ´ QP. • Le potentiel physique a pour période 33 jours donc
2π 2π
QA = 1 − sin(t ) et QP = sin(t ). = 33 ce qui donne ω = .
ω 33
Donc l’aire de PQAR est (1− sin(t )) × sin(t ) c’est-à-
b) Le potentiel physique d’un individu est à 100 %
dire −sin2 (t ) + sin(t ).  2π 
 π si et seulement si sin t  = 1
Pour tout nombre réel t de l’intervalle  0 ;  .  23 
 2 
2π π
−sin2 (t ) + sin(t ) − 0 , 25 = −(sin2 (t ) − sin(t ) + 0 , 25) si et seulement si t = + k 2π avec k Î .
23 2
= −(sin(t ) − 0 , 5)2 23
 π si et seulement si t = + 23k avec k Î .
Pour tout nombre réel t de l’intervalle  0 ;  , 4
 2  Le potentiel émotionnel d’un individu est à 100 % si
−(sin(t ) − 0 , 5)2 < 0
 2π 
donc −sin2 (t ) + sin(t ) − 0 , 25 < 0 et seulement si sin t  = 1 si et seulement si
 28 
c’est-à-dire −sin2 (t ) + sin(t ) < 0 , 25.
2π π
On a donc démontré que quelle que soit la position t = + k ′2π avec k ′ ∈ .
28 2
du point M, l’aire du rectangle PQAR est inférieure ou 28
si et seulement si t = + 28k ′ avec k ′ ∈ .
égale à 0,25. 4
Les potentiels physique et émotionnel sont à 100 % si
73 Pour tout nombre réel x de l’intervalle
et seulement si, il existe k Î  et k ′ ∈ . tels que
 π π
− ;  , f( x) = sin( x) × (1 − sin( x)) 23 28 5
 2 2  + 23k = + 28k ′ c’est-à-dire 23k − 28k ′ =
4 4 4
Pour tout x, sin( x) < 1 donc 1 − sin( x) > 0. soit 4(23k − 28k ′) = 5.
 π 
Pour tout x de l’intervalle − ; 0 , sin( x) < 0 et Or 23k − 28k ′ est un nombre entier.
 2  Donc on aurait 4 qui divise 5 qui est un nombre pre-
 π
pour tout x de l’intervalle  0 ;  , sin( x) > 0. mier. Ce qui absurde.
 2  Les potentiels physique et émotionnel ne peuvent
On dresse le tableau de signes de f( x). être simultanément à 100 %. Il en est donc de même
π p pour les trois potentiels.
x − 0
2 2  2π 
1- sin( x) + + 0 c)  50 sin × 7 305 + 50 ≈ 18 , 4
 23 
sin( x) - 0 +  2π 
50 sin × 7 305 + 50 ≈ 18 , 8
f( x) - 0 + 0  28 
La position relative de la courbe représentative de f  2π 
50 sin × 7 305 + 50 ≈ 87, 8
par rapport à l’axe des abscisses du repère est don-  33 
 π π Une personne de 20 ans a un très bon potentiel intel-
née par le signe de f( x) sur − ;  . lectuel et des potentiels physique et émotionnel
 2 2 
 π  beaucoup plus faibles.
Sur − ; 0 , f( x) < 0 donc # est au-dessous de
 2 
 π 75 1. a) 
l’axe des abscisses, et sur  0 ;  , f( x) > 0 donc #
 2 
est au-dessus de l’axe des abscisses.
π
f(0) = 0 et f   = 0 donc # coupe l’axe des abs-
 2 
π 
cisses aux points de coordonnées (0 ; 0) et  ; 0.
 2 
b) On peut conjecturer que la translation de vecteur
74 • La fonction t  50 sin(ωt ) + 50 a pour
3π 
2π − i permet de passer de # f à # g .
période . 4
ω c) Pour tout nombre réel x,
• Le potentiel physique a pour période 23 jours donc
2π 2π  π  3π 3π 
= 23 ce qui donne ω = . g  x −  = sin x − +  = sin( x) = f ( x).
 4 
 4 4
ω 23
138

172909_Chap08_000-000.indd 138 30/07/2019 15:17:30


Donc les point M d’abscisse x de # f et N d’abscisse 77 a)  La longueur de la ligne brisée OABCD est
3π OA + AB + BC + CD.
x− de # g ont la même ordonnée.
4  π 2  π  π2 1 π2 + 8
d)  On peut donc construire # g à partir de # f par OA2 =   + sin2   = + =
 4   4  16 2 16
3π 
une translation de vecteur − i . π2 + 8
4 donc OA =
2. Pour tout nombre réel x, 4
2 2
h( x) = 2 sin( x) = 2f ( x). π π  π  π 
AB2 =  −  + sin  − sin 
Donc l’ordonnée du point P d’abscisse x de # h est  2 4    2   4 
le double de l’ordonnée du point M de # f de même 2
abscisse x. π2  2  π2 3 − 2 2
AB2 = + 1 −  = +
Pour construire # h à partir de # f , on prend un 16  2  16 2
point M de # f et on construit le point P de # h dont
π2 + 24 − 16 2
l’ordonnée est le double de celle de M. Donc AB=
4
3. 
Par symétrie ; BC = AB et CD = OA.
D’où
π2 + 8 π2 + 24 − 16 2
OA + AB + BC + CD = +
2 2
OA + AB + BC + CD ≈ 3, 79
b)  Pour tout n = 1000 et n = 2 000 on obtient les
résultats ci-dessous.

76 a)  A(0) = 500 + cos(0) + 3 sin(0) = 501 On peut donc supposer qu’une valeur approchée à
Le point de référence est à 501 m. 2 chiffres significatifs après la virgule est 3,82.
b) Pout tout nombre réel x, Pour déterminer le plus petit entier n, on modifie le
 x π
500 + 2 sin +  programme.
 4 6 
  x  π  π   x 
= 500 + 2  sin cos  + sin cos 
  4   6   6   4 

 x π   3  x 1  x  
500 + 2 sin +  = 500 + 2  sin  + cos 
 4 6   4  2  4 
 2
 x π  x  x
500 + 2 sin +  = 500 + 3 sin  + cos 
 4 6   4   4 
= A( x).
c) L’altitude maximale est 502 m.
 4π 
En effet pour tout x, A( x) < 502 et A   = 502.
 3 
Une personne part à 25 km à l’est du point de réfé- On obtient
rence ( x = 25) pour aller à 25 km à l’ouest ( x = −25)
donc x appartient à l’intervalle [−25 ; 25].

La fonction A est périodique de période T= = 8π
1 78 a)  Dans le triangle OHC rectangle en H,
4π 28π
+ 8π = > 25 4 OH = cos(α) et et HC = sin(α) donc l’aire du rec-
3 3
4π 20π tangle ABCD est 2OH × 2HC = 4 cos(α)sin(α).
− 8π = − > −25 Le volume du parallélépipède rectangle est donc
3 3
4π 4π 3 × 4 cos(α)sin(α) c’est-à-dire 12sin(α)cos(α).
− 2 × 8π = − < −25.
3 3 b) Le volume du parallélépipède peut s’écrire
La personne atteint donc 2 fois l’altitude maximale. 6 sin(2α).

Chapitre 8  ★  Fonctions sinus et cosinus 139

172909_Chap08_000-000.indd 139 30/07/2019 15:18:08


π 81 1. a)  # f semble au-dessous de # g sur [0 ; + ∞[
Il est maximum lorsque sin(2α) = 1 avec 0 < α <
2 b) La valeur de l’abscisse x pour laquelle l’écart entre
π π
c’est-à-dire lorsque 2α = soit α = . les deux courbes # f et # g semble maximal est 1,6.
2 4
2. Pour tout nombre réel x > 0,
h( x) = g( x) − f ( x)
79 Dans le repère C M h( x) = e− x − e− x cos( x)
P
(O, B, C), M a pour
h( x) = e− x (1 − cos( x)).
coordonnées
Pour tout x > 0, 1 − cos( x) > 0 et e− x > 0 donc
(cos(t ) ; sin(t )) avec
h( x) > 0.
0<t <π A O B
C’est-à-dire f ( x) < g( x).
P a pour coordonnées (1 ; sin(t )) et A a pour coor-
# f est donc au-dessous de # g sur l’intervalle
données (−1 ; 0)
[0 ; + ∞[.
AM = 2MP équivaut à AM2 = 4MP2
3. a) • Pour tout nombre réel
équivaut à (1 + cos(t ))2 = 4(1 − cos(t ))2 —p
 π 4
équivaut à x de l’intervalle  0 ;  ,
 2  —
2
cos2 (t ) + 2 cos(t ) + 1 + sin2 (t ) = 4(1 − 2 cos(t ) + cos2 (t )) π π π p 2
− < x− < O
équivaut à 2(cos(t ) + 1) = 4(1 − 2 cos(t ) + cos2 (t )) 4 4 4
équivaut à 2 cos2 (t ) − 5 cos(t ) + 1 = 0.  π  2 -— p
donc cos x −  > 4
 4 2
On pose X = cos(t ).
 π
L’équation s’écrit 2X2 − 5X + 1 = 0. et 2 cos x −  − 1 > 0.
 4
5 + 17 5 − 17
D = 17 et X1 = et X2 = • Pour tout nombre réel x de p
4 4 —
4
π 
5 − 17 l’intervalle  ; 2π ,
X1 > 1 donc cos(t ) =  2  p
4 O
π π 7π 2
ce qui donne t » 1,35 rad. < x− < —
2
4 4 4 7p
Il existe donc un unique point M tel que AM = 2MP. —4
 π 2
donc cos x −  < ,
 4  2
80 Un motif bleu est représenté sur une période.  π
2p et 2 cos x −  − 1 < 0.
La fonction g semble périodique de période .  4
3
Pour tout nombre réel x, b)
p
 2π    2π    2π  x 0 2p
g  x +  = sin3 x + cos6 x +  2
 3  
 
 
3     3 
e-x + +
 2π 
g  x +  = sin(3 x + 2π)cos(6 x + 4π)  π
 3 2 cos  x −  − 1 + 0 -
 4 
 2π  h¢( x)
g  x +  = sin(3 x)cos(6 x) = g( x). + -
 3 π

2p e 2
La fonction g a donc pour période .
3 2π h( x)
On cherche le plus grand entier k tel que k × < 2 019
3 0 0
6 057
c’est-à-dire k < . On obtient k = 964. p
2p La fonction h admet un maximum en donc l’abs-
Il y a donc 964 périodes sur l’intervalle [0 ; 2 019 ]. 2
Par symétrie (la fonction g est impaire) on peut cisse x pour laquelle l’écart entre les deux courbes
p
conclure qu’il y a 2 ´ 964 soit 1928 motifs bleus. # f et # g est maximal est .
2

140

172909_Chap08_000-000.indd 140 30/07/2019 15:19:11


84 A I H B

Exploiter ses compétences h


x 30

D 7 7 C
3 À partir du Doc. 1, la période du mouvement est
82 Dans le triangle HBC rectangle en H :
2π HC
T= . cos( x) = donc h = 30cos( x).
n 30
D’après le Doc. 3, HB
2π π sin( x) = donc HB = 30sin( x).
•  T = 16 donc = 16 , et n = . 30
n 8 1
•  x(2) = 0 donc Csin(2n + α) = 0 comme C ¹ 0 AB = 7 + HB = 7 + 30 sin( x)
2
π
2n + α = 0 donc α = −2n = − donc AB = 2(7 + 30 sin( x)).
4
π  π π L’aire du trapèze ABCD est :
•  v(4) = 2π donc × Ccos4 × −  = 2π (CD + AB) × h [14 + 2(7 + 30 sin( x))]× 30 cos( x)
8  8 4 =
π 2 2
donc Ccos  = 16 c’est-à-dire 30(14 + 30 sin( x))cos( x).
 4 
 π
donc C = 16 2. On note f la fonction définie sur  0 ;  par
 2 
L’amplitude du mouvement est 16 2 m. f( x) = 30(14 + 30sin( x))cos( x).
D’après le Doc. 3, f ′( x) = 900 cos(2 x) − 420 sin( x)
83 • D’après le Doc. 1 : Donc f ′( x) = 900[−2 sin2 ( x) + 1] − 420 sin( x)
L(0) = 40 000 et L(6) = 25 000. f ′( x) = −1800 sin2 ( x) − 420 sin( x) + 900
D’après le Doc. 2 : L(0) = a + b et L(6) = a On pose X = sin( x).
On déduit que a = 25 000 et b = 15 000. f ′( x) = 0 s’écrit donc −1800X2 − 420X + 900 = 0
• D’après le Doc. 1 : C(0) = 5 000 et C(6) = 7 000 3 5
∆ = 6 656 400 et X = ou X = − .
D’après le Doc. 2 : C(6) = c + d et C(0) = c 5 6
On déduit que c = 5 000 et c = 2 000.  5  3 
f ′( x) = −1800sin( x) + sin( x) − 
On trace les courbes représentatives des fonctions C  6  5
et L.  π
Pour tout nombre x de l’intervalle  0 ;  , sin( x) > 0
Les populations de lapins et de coyotes ont le même 5  2 
donc sin( x) + > 0.
cycle qui est de 6 ans. 6
La population de lapins est au maximum (40 000) en 3 p 3
0 < < donc il existe a tel que sinα = .
2012. Et décroît jusqu’en 2024. Celle des coyotes en 5 2 5
2012 est de 5 000 individus et croît jusqu’en 2018. Pour tout x > a, sin( x) > sin(α) car sin est crois-
Puis décroît jusqu’en 2030.  π 3
sante sur  0 ;  donc sin( x) − > 0.
Ceci explique le cycle lapins-coyotes du document 3.  2  5
On dresse le tableau de variations de f.
p
x 0 a
2
 5
−1800 sin( x) +  - -
 6 

3
sin( x) − - 0 +
5
f ¢( x) + -

768

f( x)

420 0

Chapitre 8  ★  Fonctions sinus et cosinus 141

172909_Chap08_000-000.indd 141 30/07/2019 15:20:13


 3 4 85 En utilisant le tableur de GeoGebra on obtient
f( x) = 30(14 + 30 sin(α))cos( x) = 3014 + 30 ×  × l’écran ci-dessous. On peut estimer qu’au mois de
 5 5
2 2 9 16 4 décembre la température était de −21, 7 °C.
cos α = 1 − sin α = 1 − = donc cosα =
25 25 5
π
car 0 < α <
2
À la calculatrice α ≈ 0 , 64 rad soit environ 36 , 9 °.
L’équilibre est maximum pour a rad.

142

172909_Chap08_000-000.indd 142 30/07/2019 15:20:19


9
Produit scalaire
et calcul vectoriel

L’affirmation de Fred est donc vraie : il suffit de

Découvrir
mesurer les longueurs AB et AH .

 ) = 180° − HAC
2  a)  (BAC  , d’où
 ) = cos(180° − HAC
cos(BAC  ) = −cos(HAC  ).
b) Dans le triangle ACH , rectangle en H ,
1  Le travail d’une force en physique  ) = AH .
cos(HAC
AC
Donc, AH = AC × cos(HAC ) = −AC × cos(BAC) .
1  a)   
D’où, AB ⋅ AC = AB × (−AH), c’est-à-dire
 0° 30° 45° 60° 120°  
BAC AB ⋅ AC = −AB × AH.
W 50 000 25 000 3 25 000 2 25 000 - 25 000 c) Il suffit donc de mesurer les longueurs AB et
 
 = 90° d’où cos(BAC
b) Dans ce cas, BAC  ) = 0, AH pour calculer le produit scalaire AB × AC  :
donc W = 0. En conséquence, le travail (l’effica- Fred a donc raison.
cité) de la force est nulle. Louise a donc raison.

2  a)  W = AB × F × cos(BAC  ).

Comme AB > 0 et F > 0, il vient :
W > 0 équivaut à cos(BAC  ) > 0,

soit 0° < BAC < 90°.
) = 0 ,
Acquérir des automatismes
W = 0 équivaut à cos(BAC
 = 90°.
soit BAC
W < 0 équivaut à cos(BAC ) < 0 ,

soit 90° < BAC < 180°.  
3 a)  DE ⋅ DC = DE × DC × cos(CDE  ).
b)  W < 0 équivaut à BAC  est obtus, ce qui  = 60°,
Le triangle AED est équilatéral donc ADE
signifie que le vent exerce une force contraire au  = 90° − 60° = 30° .
d’où CDE
sens de déplacement de Vincent.   3
DE ⋅ DC = 4 × 5 × cos(30°) = 20 × = 10 3
2
 
b)  ABCD est un rectangle donc CB = DA.
2  Une expression du produit    
)
scalaire D’où CB ⋅ DE = DA ⋅ DE = DA × DE × cos(ADE
 
  CB ⋅ DE = 4 × 4 × cos(60°) = 8
)
1  AB ⋅ AC = AB × AC × cos(BAC
  et le triangle
Dans les deux figures : BAC = HAC 4 a) Le triangle ABC est isocèle en C , donc le
ACH est rectangle en H . projeté orthogonal du point C sur la droite (AB)
est I.
D’où cos(BAC  ) = AH
 ) = cos(HAC    
AC D’où BC ⋅ BA = BI ⋅ BA
 
) .
soit AH = AC × cos(BAC Les vecteurs BA et BI sont colinéaires et de même
   
Donc AB ⋅ AC = AB × AH. sens, d’où : BC ⋅ BA = BI × BA = 2, 5 × 5 = 12, 5.

Chapitre 9  ★  Produit scalaire et calcul vectoriel 143

172909_Chap09_000-000.indd 143 29/07/2019 17:49:55


b) Le projeté orthogonal du point A sur (CI) est I 15 ABCD est un parallélogramme donc
 
 
2   = 30° (angles alternes-internes)
donc CA ⋅ CI = CI = CI2 . BAC = DCA
D’après le théorème de Pythagore, CI2 = CA2 − AI2 ,   15 3 15
AB ⋅ AC = 3 × 5 × cos(30°) = = 3.
donc CI2 = 36 − 6 , 25 = 29 , 75. 2 2
 

D’où CA ⋅ CI = 29 , 75. L’affirmation d’Elida n’est donc pas correcte.

 
7 a)  H est le projeté orthogonal du point C sur la 16 a)  u × v est un nombre réel et non un vecteur.
     
droite (AB). AB et AH sont colinéaires et de sens b)  u ⋅ v = 0 équivaut à u et v orthogonaux.
 
opposés, donc AB ⋅ AC = −AB × AH, 1
  c) L’écriture  n’a pas de sens car 1 est un nombre
AB ⋅ AC = −4 × 2 = −8.  v
  réel et v un vecteur.
 ).
b) D’autre part, AB ⋅ AC = AB × AC × cos(BAC      
d)  AB ⋅ AC = AB ⋅ (− CA) = − AB ⋅ CA
 ),
D’où −8 = 4 × 5 × cos(BAC
) = −8 2 17 a) L’affirmation est fausse. En effet,
c’est-à-dire cos(BAC =− .  
20 5 )
AB ⋅ AC = AB × AC × cos(BAC
Donc avec la calculatrice, il vient α ≈ 114°.   1
AB ⋅ AC = 6 × 6 × cos(60°) = 36 × = 18
    2
8 AB(2 ; 2) et AC(4 ; 1) donc AB ⋅ AC = 10. b) L’affirmation est vraie. En effet,
 
Par projection orthogonale sur (AB), AB ⋅ AC = AB × AC × cos(BAC  ).
  10 10    
AB ⋅ AC = AB × AK . D’où, AK = = AB ⋅ AC = 5 × 6 × cos(0°) d’où AB ⋅ AC = 30.
AB 8  = 90°
c) L’affirmation est vraie. En effet, BAC
soit AK » 3,5.  
donc cos(BAC ) = 0 , d’où AB ⋅ AC = 0.
     
11 DE ⋅ AF = (DA + AE) ⋅ (AD + DF),
           = 0° donc cos(BAC
18 a)  BAC  ) = 1. D’où :
DE ⋅ AF = DA ⋅ AD + DA ⋅ DF + AE ⋅ AD + AE ⋅ DF,  
       ) = 6 × 2 ×1
AB ⋅ AC = AB × AC × cos(BAC
DE ⋅ AF = −AD2 + DA ⋅ DF + AE ⋅ AD + AE × DF.  
  AB ⋅ AC = 12
Les vecteurs DA et DF sont orthogonaux ainsi que
       = 180° donc cos(BCA
b)  BCA  ) = −1.
les vecteurs AE et CD , donc DA ⋅ DF = AE ⋅ AD = 0.  
   ) = 2 × 4 × (−1)
CA ⋅ CB = CA × CB × cos(BCA
DE ⋅ AF = −AD2 + AE × DF = −4 + 1× 4 = 0.  
Donc les droites (DE) et (AF) sont perpendiculaires. CA ⋅ CB = −8.

 
 19  
12 Dans le repère orthonormé (A ; AI, AJ) :
 
4  AB AC  en rad dans ]− p ; p]
BAC AB × AC
A(0 ; 0), D(0 ; 2), K  ; 0 , C(3 ; 2).
 3 
   4 3 2 p
 8 12
Donc, AC(3 ; 2) et DK  ; − 2. 2 4
 3 
  5 8 2p
4 - 20
Ainsi, AC ⋅ DK = 3 × + 2 × (− 2) = 4 − 4 = 0 3
3
p
et les droites (AC) et (DK) sont perpendiculaires. 2 4 4 2
4
  π
13 AB ⋅ AC = AB × AC × cos(BAC ), 2 7,5 − 7,5
  3
1
d’où AB ⋅ AC = 4 × 3 × cos(60°) = 12 × = 6.
2  
)
20 a)  EB ⋅ EC = EB × EC × cos(BEC
  
 
 AB ⋅ AC EB ⋅ EC = 4 × 4 × cos(60°) = 8
14 cos(BAC) =  
AB × AC b)  CE ⋅ CF = 4 × 2 × cos(60°) = 4
−15  = 90° − 60° = 30°
c)  DCE
) =
d’où cos(BAC = −1  
15 CD ⋅ CE = 4 × 4 × cos(30°) = 8 3
 
 = π rad.
soit BAC d)  BA ⋅ BF = BA × BF × cos(90°) = 4 × 4 × 0 = 0
144

172909_Chap09_000-000.indd 144 30/07/2019 15:33:55


21 L’hexagone est constitué exactement de six  
24 AB ⋅ AC = AB × AC × cos(BAC)
triangles équilatéraux identiques de côté 1.
  Le théorème de Pythagore conduit à :
1
a)  OF ⋅ OA = 1× 1× cos(60°) = AC2 = 22 + 22 = 8 , soit AC = 2 2.
2  
 = COD
b)  DOA  + BOC + AOB
 = 3 × 60° = 180° AB ⋅ AC = 4 × 2 2 × cos(135°),
  135° = 180° − 45° donc cos(135°) = −cos(45°).
OA ⋅ OD = OA × OD × cos(180°) = 1× 1× (−1)
     2
OA ⋅ OD = −1 AB ⋅ AC = 4 × 2 2 × −  = −8
  1  2 
c)  OF ⋅ OB = 1× 1× cos(120°) = −
2
25 Le projeté orthogonal de C sur (AB) est B, donc
d)  OA = AB = BC = CO donc OABC est un losange  
  AB ⋅ AC = AB2 = 9.
d’où AB = OC.
    1  
OA ⋅ AB = OA ⋅ OC = 1× 1× cos(120°) = − . 26 a)  u ⋅ v = 4 × 2 = 8
2  
e)  DC = DO et BC = BO , donc (DB) est la média- b)  u ⋅ v = 4 × 2 = 8
   
trice de [CO] d’où DB et CO orthogonaux. c)  u ⋅ v = −4 × 4 = −16
 
COED est un losange, d’où CO = DE.    
    27 Dans chaque cas, on pose u = OA et v = OB.
Finalement, DB ⋅ DE = DB ⋅ CO = 0.    est aigu.
  a)  u ⋅ v > 0 car AOB
f)  DOFE est un losange, donc OF = DE .    est obtus.
    b)  u ⋅ v < 0 car AOB
D’où OF ⋅ DA = DE ⋅ DA = DE × DA × cos(60°).    est obtus
  1 c)  u ⋅ v < 0 car AOB
OF ⋅ DA = 1× 2 × = 1
2
28 • Le projeté orthogonal de C sur (AB) est B,
 
22 a) Le triangle ABC est équilatéral donc la donc AB ⋅ AC = AB2 = 16.
médiane (AA ′) est aussi la hauteur relative à [BC]. Le résultat d’Ambre est correct.
Donc le triangle AA ¢B est rectangle en A ¢. • On note I¢ le milieu de [CD].
D’après le théorème de Pythagore, il vient : Le triangle CDI est isocèle en I, donc le projeté
AA ′2 = AB2 − BA ′2 , soit AA ′2 = 16 − 4 = 12. orthogonal de I sur (CD) est I¢.
 
Donc, AA ′ = 12 = 2 3. Les vecteurs CI¢ et CD sont colinéaires et de même

 
b) Le triangle ABC est équilatéral donc la médiane sens, donc CI ⋅ CD = CI′ × CD = 2 × 4 = 8.
.
(AA ′) est aussi la bissectrice intérieure de BAC Le résultat d’Ambre n’est pas correct.

Donc BAA ′ = 30°. • Le projeté orthogonal de A sur (CB) est B, donc
   
AA ′ ⋅ AB = AA ′ × AB × cos(BAA ′) CA ⋅ CB = CB2 = 9.
  3 Le résultat d’Ambre n’est pas correct.
AA ′ ⋅ AB = 2 3 × 4 × cos(30°) = 8 3 × = 12
2
  29 a) D et E ont le même projeté orthogonal sur
(AA ′) est perpendiculaire à [BC] donc AA ′ ⋅ BC = 0.
(AB), C et F ont le même projeté orthogonal sur
   
  
23 On note C¢ le point tel que AC ’ = BC. (AB) d’où AB ⋅ FE = AB ⋅ CD.
b) Par projection orthogonale sur (AB), on a :
   
AB ⋅ GJ = −AB ⋅ CD.
c) Par projection orthogonale sur (AB), on a :
   
EH ⋅ AB = −AB ⋅ CD.
d) Par projection orthogonale sur (JK ), on a :
  
′ = 180° − 60° = 120°
BAC CD ⋅ JK = JK 2 .
    Par projection orthogonale sur (AB), on obtient :
AB ⋅ BC = AB ⋅ AC ′ = AB × AC′ × cos(120°)     
   1 AB ⋅ CD = AB ⋅ JK = AB × JK
AB ⋅ BC = 3 × 2 × −  = −3    
 2  JK 2 ≠ AB × JK . Donc CD ⋅ JK ≠ AB ⋅ CD.
Remarque : la linéarité du produit scalaire permet de
répondre plus rapidement, mais elle n’a peut-être pas 30 a) Par projection orthogonale des points C et D
 
été abordée à ce stade. sur (AB) : AB ⋅ DC = AB × DC = 24.

Chapitre 9  ★  Produit scalaire et calcul vectoriel 145

172909_Chap09_000-000.indd 145 29/07/2019 17:52:18


     
Remarque : on peut aussi utiliser le fait que AB et b)  (AB + AH) ⋅ AB = AB2 + AH ⋅ AB.

DC sont colinéaires et de même sens. Le projeté orthogonal de B sur (AH) est H, donc
  
b) Par projection orthogonale du point C sur (AB) : (AB + AH) ⋅ AB = AB2 + AH2 = 9 + 5 = 14.
 
AB ⋅ AC = AB × DC = 24.
     
c) Par projection orthogonale du point C sur (AD) : 36 AB ⋅ AD = (AE + EB) ⋅ (AE + ED)
 
AC ⋅ AD = AD2 = 4. E est le milieu des diagonales du parallélogramme
 

d) Par projection orthogonale des points B et C sur ABCD, donc ED = −EB.
        
(AD) : BC ⋅ AD = AD2 = 4. D’où AB ⋅ AD = (AE + EB) ⋅ (AE − EB) = AE2 − EB2 .
   
e)  AD et DC orthogonaux donc AD ⋅ DC = 0 .
     
f)  BA et DA orthogonaux donc BA ⋅ DA = 0. 37 u ⋅ v = − 10 + 10 = 0
 
u et v sont donc orthogonaux.
   
31 a)  AB ⋅ CA = −AB ⋅ AC = −4
       
b)  AB ⋅ CB = BA ⋅ BC = −3 38 a)  u(2 ; 0) et v(2 ; 1) donc u ⋅ v = 4 + 0 = 4.
 
b)  u(−4 ; 1) et v(2 ; − 3)
     
32 a)  −3u ⋅ (4v ) = −12 u ⋅ v = 24 donc u ⋅ v = −8 − 3 = −11.
 2
b)  u 2 = u = 1  
      39 a)  u ⋅ v = −2 − 12 = −14 . Réponse (2).
c)  (u + v )2 = u 2 + 2u ⋅ v + v 2 = 1 − 4 + 5 = 2  
( )
b)  t −5 ; 3 et i (1 ; 0)
 
    donc t ⋅ i = −5 + 0 = −5 . Réponse (2).
33 a)  u et v orthogonaux donc u ⋅ v = 0.
    2      
(u − 2v ) ⋅ (u + v ) = u + u ⋅ v − 2u ⋅ v − 2v 2 c)  w = (−1)2 + 52 = 26 . Réponse (1).
    1 1
(u − 2v ) ⋅ (u + v ) = 1 + 0 − 0 − 2 × =  
4 2 40 AB(1 ; 2) et AC(5 ; 1) .
Réponse (3).  
a)  AB ⋅ AC = 5 + 2 = 7
      1 3    
b)  (u − v ) ⋅ (u + v ) = u 2 − v 2 = 1 − = b)  AB ⋅ CA = −AB ⋅ AC = −7
4 4  2
Réponse (1). c)  AB = 12 + 22 = 5
      1 5 d)  AB = 5
c)  (u − v )2 = u 2 − 2u ⋅ v + v 2 = 1 − 0 + =
4 4
Réponse (3). 41 a) 
     
34 AC ⋅ DB = (AB + BC) ⋅ (DA + AB)
     2    
AC ⋅ DB = AB ⋅ DA + AB + BC ⋅ DA + BC ⋅ AB
 
Or, ABCD est un rectangle donc AB et DA sont
 
orthogonaux, ainsi que BC et AB,
   
donc AB ⋅ DA = BC ⋅ AB = 0.
   
D’où AC ⋅ DB = AB2 + BC ⋅ DA
   
Or, BC = AD, donc AC ⋅ DB = AB2 − AD2 .
 
Finalement, AC ⋅ DB = 25 − 4 = 21.
A(0 ; 0), D(2, 5 ; 0), E(0 ; 2, 5) .
      
35 1.  (HC + BC) ⋅ AC = HC ⋅ AC + BC ⋅ AC. B(5 ; 0) et C(0 ; 5) donc les coordonnées du milieu F
Le projeté orthogonal de A sur (HC) ou (BC) est H et de [BC] sont : F(2, 5 ; 2, 5).
   
les vecteurs BC et HC sont colinéaires et de même b) On déduit de a), AF(2, 5 ; 2, 5) et ED(2, 5 ; − 2, 5).
 
sens, donc : Par conséquent, AF ⋅ ED = 2, 52 − 2, 52 = 0.
    
(HC + BC) ⋅ AC = HC2 + BC × HC = 9 + 5 × 3 = 24 D’où AF et ED sont orthogonaux.
2. a) En appliquant le théorème de Pythagore dans le
 
triangle ABH rectangle en H, on obtient : 42 Dans le repère donné, AC(a ; b) et DB(a ; − b).
 
AH = 32 − 22 = 5 . Donc AC ⋅ DB = a2 − b2 .
146

172909_Chap09_000-000.indd 146 30/07/2019 15:12:26


43 L’affirmation est exacte. En effet, pour tous vec- 1  
  50 p = (BC2 + AC2 − AB2 ), donc p = CB ⋅ CA .
teurs u et v, 2
 2    
u + v = u 2 + 2u ⋅ v + v 2 et 
 
 2     )
51 EF ⋅ EG = EF × EG × cos(FEG
u − v = u 2 − 2u ⋅ v + v 2
 2  2   
Or, −1 < cos(FEG) < 1 , d’où
d’où u + v + u − v = 2u 2 + 2v 2 , 
 
 2  2 2 2 −EF × EG < EF ⋅ EG < EF × EG, soit
u +v + u −v = 2 u +2 v , 
 
 2  2 2 2 −10 < EF ⋅ EG < 10. Or, 12 ∉ [−10 ; 10 ],
u + v + u − v = 2( u + v ).
donc la construction du triangle EFG est impossible.
44 L’affirmation est fausse. En effet, pour tous vec-
  52 a) Par projection orthogonale sur (AB),
teurs u et v,  
 2     AB ⋅ AC = AB2 = 9.
u + v = u 2 + 2u ⋅ v + v 2 et  
 ),
 2     b)  CB ⋅ CE = CB × CE × cos(BCE
u − v = u 2 − 2u ⋅ v + v 2  
 2  2   CB ⋅ CE = 3 × 2 × cos(60°) = 3.
d’où u + v − u − v = 4u ⋅ v .
  c) Les diagonales d’un carré sont perpendiculaires,
Il suffit de choisir u et v non orthogonaux pour inva-  
donc AC ⋅ BD = 0.
lider l’affirmation.
53 ABCD est un parallélogramme donc
45 L’affirmation est exacte. En effet, pour tous vec-  
  CB = DA .
teurs u et v,    
  1    2   D’où AD ⋅ CB = AD ⋅ DA = −AD2 .
u ⋅ v = (u 2 + v 2 − u − v ), et en posant u = AB
2 L’affirmation est exacte.
 
et v = AC , il vient :  
     54 a) AB(1 ; 1) et AC(3 ; − 1).
u − v = AB − AC = CB d’où  
  1 Donc AB ⋅ AC = 3 − 1 = 2.
AB ⋅ AC = (AB2 + AC2 − CB2 ).  
2  ),
b)  AB ⋅ AC = AB × AC × cos(BAC
    AB = 12 + 12 = 2 et AC = 32 + (−1)2 = 10 .
46 En posant u = AB et v = AC,
 2 2 2 D’où 2 = 2 × 10 × cos(BAC  ), soit
u − v = u + v équivaut à CB2 = AB2 + AC2 .
) = 2
On retrouve donc le théorème de Pythagore et sa cos(BAC .
2 × 10
réciproque, appliqués au triangle ABC.
 ≈ 63°.
Avec la calculatrice, on obtient BAC
  1
47 AB ⋅ AC = (AB2 + AC2 − CB2 )
2 55 a) Le triangle ABC est équilatéral
  1  = 60°.
AB ⋅ AC = (9 + 36 − 25) = 10 donc BAC
2  
 ),
AB ⋅ AC = AB × AC × cos(BAC
 
  1  2   AB ⋅ AC = 6 × 6 × cos(60°) = 18.
48 u ⋅ v = ( u + v − u 2 − v 2 )
2 b)  CB = CA et DB = DA donc la droite (CD) est la
  1 médiatrice de [AB] et passe donc par O.
u ⋅ v = (16 − 9 − 25) = −9
2 D’où les projetés orthogonaux respectifs de B et D sur
  1 (CO) sont O et D.
49 a)  AB ⋅ AC = (AB2 + AC2 − CB2 )  
2 CO et OD sont colinéaires et de même sens, donc
  1  
AB ⋅ AC = (25 + 49 − 9) = 32, 5 CO ⋅ BD = CO × OD.
2 Le théorème de Pythagore appliqué aux triangles rec-
  1    2  2  2 
b)  AB ⋅ AD =  AB + AD − AB − AD . tangles COB et ODB, conduit à :
2   CO = 3 3 et OD = 3 2.
ABCD est un parallélogramme  
   Donc CO ⋅ BD = 9 6.
d’où AB + AD = AC. c) Par projection orthogonale de B et D sur (OA), il
  1  
AB ⋅ AD = (AC2 − AB2 − AD2 ) vient : OA ⋅ BD = OA × BO = 9.
2      
  1 d)  CA ⋅ BD = (CO + OA) ⋅ (BO + OD)
AB ⋅ AD = (49 − 25 − 9) = 7, 5          
2 CA ⋅ BD = CO ⋅ BO + CO ⋅ OD + OA ⋅ BO + OA ⋅ OD

Chapitre 9  ★  Produit scalaire et calcul vectoriel 147

172909_Chap09_000-000.indd 147 29/07/2019 17:54:51


       
CO et BO sont orthogonaux ainsi que OA et OD 60 u − v = u + v équivaut à
     2  2    
donc CO ⋅ BO = OA ⋅ OD = 0. u − v = u + v , soit −2u ⋅ v = 2u ⋅ v , c’est-à-dire
       
CA ⋅ BD = 9 6 + 9. 4u ⋅ v = 0 , soit u ⋅ v = 0, ce qui équivaut à u et v
Remarque : introduire un repère orthonormé pour orthogonaux.
utiliser l’expression analytique du produit scalaire Exemple de représentation :
peut être pertinent.

 
)
56 a)  AB ⋅ AD = AB × AD × cos(BAD
 
AB ⋅ AD = 4 × 2 × cos(60°) = 4.
  2  2    2
b)  AB + AD = AB + 2AB ⋅ AD + AD
  2
AB + AD = 16 + 8 + 4 = 28  
61 a)  AB(−6 ; 1) et AC(1 ; 6).
c) • ABCD est un parallélogramme donc  
   Donc AB ⋅ AC = −6 + 6 = 0.
AB + AD = AC.  
b)  AB et AC sont orthogonaux donc le triangle ABC
D’après b), AC2 = 28 , d’où AC = 2 7.
 2   est rectangle en A.
•  BD2 = BD = (BA + AD)2 On peut même préciser que ABC est isocèle en A car
 
BD2 = BA2 + 2BA ⋅ AD + AD2 AB = AC = 37.
 
BD2 = BA2 − 2AB ⋅ AD + AD2  
62 AB(3 ; − 2) et CD(−4 ; − 6).
BD2 = 16 − 8 + 4 = 12  
soit BD = 12 = 2 3. Donc AB ⋅ CD = −12 + 12 = 0.
D’où les droites (AB) et (CD) sont perpendiculaires.
57 a) 
63 a) Ligne 5 : x*u+y*v==0
Ligne 6 : vecteurs orthogonaux
Ligne 8 : vecteurs non orthogonaux
 
b) Avec t (3 ; − 2) et w(−4 ; − 6) le programme
affiche : vecteurs orthogonaux.
 
Avec t (3 ; − 2) et w(−4 ; 6) le programme affiche :
b) Le projeté orthogonal du point C sur (AB) est B, vecteurs non orthogonaux.
 
donc AB ⋅ AC = AB2 = 25.
 
c) Le projeté orthogonal du point B sur (AC) est H, 64 u et v orthogonaux équivaut à
 
AH et AC sont colinéaires et de même sens, donc ( x + 3)(−7 − x) + ( x + 6)( x + 9) = 0, soit
  33
AB ⋅ AC = AH × AC. 5 x + 33 = 0 , c’est-à-dire x = − = −6 , 6.
5
Le théorème de Pythagore appliqué au triangle ABC
rectangle en B, conduit à AC = 29. 65 On introduit le repère orthonormé
   
Ainsi, 25 = AH 29 , soit AH =
25
. A , 1 AB, AD.
 2 
29
 2 
AH » 4,6 cm. A(0 ; 0), C( 2 ; 1), D(0 ; 1) et E  ; 0.
 2 
     2 
58 u ⋅ v = ( 2 − 1)( 2 + 1) + (4 + 5 )(4 − 5 )
  D’où AC( 2 ; 1) et DE  ; − 1.
u ⋅ v = 2 − 1 + 16 − 5 = 12    2 
  AC ⋅ DE = 1 − 1 = 0.
12 ¹ 0 donc u et v ne sont pas orthogonaux.
Donc les droites (AC) et (ED) sont perpendiculaires.

59 a) L’affirmation est vraie. En effet, ABC est un 66 1. A   2. C   3. B   4. B   5. D
triangle rectangle en A ou A = B ou A = C.
  67 1. B, C, D  2. B, C, D  3. C, D  4. A, B, D
Dans tous les cas, AB ⋅ AC = 0.
b) L’affirmation est vraie. En effet, la médiatrice 68 1. Faux. En effet,
(CD) de [AB] est perpendiculaire à la droite (AB),   16 2
  AB ⋅ AC = 4 × 4 × cos(45°) = = 8 2.
donc AB ⋅ CD = 0. 2
148

172909_Chap09_000-000.indd 148 30/07/2019 15:13:06


2. Vrai. En effet,
  1   1
AB ⋅ AB′ = AB ⋅ AC = × 8 2 = 4 2.
2 2 S'entraîner
 180 − 45
3. Vrai. En effet, ACB = = 67, 5°.
  2
CB ⋅ CA = CB × CA × cos(ACB ),
 
CB ⋅ CA = CB × 4 × cos(67, 5°). 73  
 2  2  
4. Vrai. En effet, BC = BC = (BA + AC)2 ,
 
donc BC2 = BA2 − 2 × AB ⋅ AC + AC2
BC2 = 32 − 2 × 8 2 = 32 − 16 2 .
5. Vrai. En effet, ABC est isocèle en A, donc par proje-
 
tion orthogonale sur (BC) et comme CB et CA ¢ sont
colinéaires et de même sens :
 
CB ⋅ CA = CB × CA ′.
 
69 a)  OA ⋅ OA ′ = OA × OA ′ × cos(AOA′),
  2 9
OA ⋅ OA ′ = 3 × 3 × cos(45°) = 9 × = 2.
  2 2
′),
b)  OA ⋅ OB′ = OA × OB′ × cos(AOB
  1
OA ⋅ OB′ = 3 × 4 × cos(60°) = 12 × = 6.
  2
′),
c)  OA ⋅ OC′ = OA × OC′ × cos(AOC
  −1
OA ⋅ OC′ = 3 × 4 × cos(120°) = 12 ×
  2
OA ⋅ OC′ = −6.
70 a) Le projeté orthogonal du point C sur (AB)
  Deux points de coordonnées (2 ; 4) et (3 ; 4) appa-
est B, donc AB ⋅ AC = AB2 = 9. raissent quand on exécute le programme.
 
b)  AB ⋅ AC = AB × AC × cos(BAC  ).
 
AB ⋅ AC = 4 × 2 × cos(45°) = 4 2 . 74 On note ( x ; y) les coordonnées du point M.
 
c)  AB(4 ; − 1) et AC(1 ; − 3). On a, MA2 = x2 + y 2 et MB2 = (5 − x)2 + y 2 .
 
AB ⋅ AC = 4 + 3 = 7. D’où MA2 − MB2 = −15 équivaut à
  1 x2 − (5 − x)2 = −15.
d)  AB ⋅ AC = (AB2 + AC2 − BC2 ),
2 Programme en langage python :
  1
AB ⋅ AC = (25 + 36 − 16) = 22, 5.
  2  
e)  AB ⋅ AC = −BA ⋅ AC
 
Or, ABCD est un parallélogramme donc AC = BD.
     
AB ⋅ AC = AB ⋅ BD = −BA ⋅ BD.
  1
BA ⋅ BD = (BA2 + BD2 − AD2 ),
2 Affichage :
  1 3
BA ⋅ BD = (9 + 4 − 16) = − .
2 2
  3
D’où AB ⋅ AC = .
2
71 a) Le projeté orthogonal du point C sur (AB)
 
est H. AB et AH sont colinéaires et de même sens
 
donc AB ⋅ AC = AB × AH = 3 × 4 = 12.
b) Le projeté orthogonal du point B sur (AC) est K.
 
AC et AK sont colinéaires et de même sens donc
 
AB ⋅ AC = AK × AC = 6AK .
Ainsi, 6AK = 12, soit AK = 2.

Chapitre 9  ★  Produit scalaire et calcul vectoriel 149

172909_Chap09_000-000.indd 149 30/07/2019 15:14:08


76 a) À l’aide du logiciel, on conjecture que lorsque a x = −b y conduit à akb = −b y,
a varie (BE) et (CF) restent perpendiculaires. c’est-à-dire −ka = y.
     
b)  BE ⋅ CF = (BC + CE) ⋅ (CD + DF),

          2e cas : b = 0.
BE ⋅ CF = BC ⋅ CD + BC ⋅ DF + CE ⋅ CD + CE ⋅ DF. a x = −b y conduit à ax = 0, c’est-à-dire a = 0
 
Les vecteurs BC et CD sont orthogonaux, ainsi que ou x = 0.
   
les vecteurs CE et DF. Or a = 0 est impossible car u ¹ 0. Donc x = 0.
    Sous cette condition on note k le nombre réel,
donc BC ⋅ CD = CE ⋅ DF = 0.

  1 1 y
D’où BE ⋅ CF = −a × a + a × a = 0. k= . Ainsi, x = kb et y = −ka.
3 3 −a
Donc les droites (BE) et (CF) sont perpendiculaires. • Réciproquement, s’il existe un nombre réel k tel
 
que x = kb et y = −ka, alors u ⋅ v = kab − kab = 0.
 
77 a)  Donc u et v sont orthogonaux.

 2  2
79 AB2 − AC2 = AB − AC
    
AB2 − AC2 = (AH + HB)2 − (AH + HC)2 .
   
Or, (AH + HB)2 = AH2 + 2AH ⋅ HB + HB2 et
   
(AH + HC)2 = AH2 + 2AH ⋅ HC + HC2 , donc
  
AB2 − AC2 = 2AH ⋅ (HB − HC) + HB2 − HC2
 
AB2 − AC2 = 2AH ⋅ CB + HB2 − HC2 .
 
On conjecture que lorsqu’on fait varier a, les droites (AH) est la hauteur issue de A donc AH et CB sont
 
(AH) et (FB) restent perpendiculaires. orthogonaux, soit AH ⋅ CB = 0.
 1  
b)  AH = (AE + AD). Finalement, AB2 − AC2 = HB2 − HC2 .
2
Par conséquent,
   80 1. a) Les projetés orthogonaux des points I et J
1    
AH ⋅ FB = (AE + AD) ⋅ (FA + AB) sur la droite (AB) sont respectivement I et A.
2     
   1         AB ⋅ IJ = AB ⋅ IA.
AH ⋅ FB = (AE ⋅ FA + AE ⋅ AB + AD ⋅ FA + AD ⋅ AB).
2
Or les droites (AB) et (AD) sont perpendiculaires
   
donc AD ⋅ AB = 0 et AE ⋅ FA = 0.
 
AE et AB sont colinéaires et de même sens
  1 1
donc AE ⋅ AB = a × a = a2 .
  2 2
AD et FA sont colinéaires et de sens contraires
  1 1
donc AD ⋅ FA = − a × a = − a2 .
   2 2
D’où AH ⋅ FB = 0.
Ce résultat ne dépend donc pas de a.
Pour tout nombre réel a strictement positif, les droites
(AH) et (FB) sont perpendiculaires. Le projeté orthogonal du point H sur la droite (AB)
        
est I, donc AB ⋅ HA = AB ⋅ IA. D’où AB ⋅ IJ = AB ⋅ HA.
  2 2
78 1.  u = v équivaut à u = v , b) On procède de la même façon : les projetés ortho-
   
soit u 2 = v 2 , c’est-à-dire u 2 − v 2 = 0 , gonaux des points A et H sur la droite (AC) corres-
   
ce qui s’écrit (u − v ) ⋅ (u + v ) = 0. pondent respectivement aux projetés orthogonaux
   
Ainsi, u - v et u + v sont orthogonaux. de I et J sur (AC).
   
2. On raisonne en deux temps. On obtient AC ⋅ IJ = AC ⋅ AH.
   1  
• On suppose que v( x ; y) et u(a ; b) sont orthogo-
2.  A ¢ est le milieu de [BC ] donc AA ′ = (AB + AC).
naux. 2
    1   
u ⋅ v = 0 donc a x + b y = 0, soit a x = −b y. AA ′ ⋅ IJ = (AB + AC) ⋅ IJ,
1er cas : b = 0. 2
  1   1  
x AA ′ ⋅ IJ = AB ⋅ IJ + AC ⋅ IJ,
On note k le nombre réel . Ainsi, x = kb. 2 2
b
150

172909_Chap09_000-000.indd 150 30/07/2019 15:25:40


  1   1   84 Le projeté orthogonal de C sur (AH) est P, donc
AA ′ ⋅ IJ = AB ⋅ HA + AC ⋅ AH,    
2 2 HA ⋅ HC = HA ⋅ HP.
  1    1  
AA ′ ⋅ IJ = HA ⋅ (AB + CA) = HA ⋅ CB. Le projeté orthogonal de A sur (HC) est Q, donc
2 2    
HA ⋅ HC = HQ ⋅ HC.
H est le projeté orthogonal de A sur (BC) donc    
  Finalement, HA ⋅ HP = HQ ⋅ HC, soit
HA ⋅ CB = 0.    
  HP ⋅ HA = HQ ⋅ HC.
Finalement, AA ′ ⋅ IJ = 0. Donc les droites (AA ′) et
(IJ) sont perpendiculaires.  
85 a) ABCD est un parallélogramme donc AD = BC
 
     et AB = DC. 
81 1. Si u = 0, alors u ⋅ (v + w ) = 0 et 2  2  2  2
    AB2 − AD2 = AB − AD = AB − BC ,
u ⋅ v + u ⋅ w = 0 + 0 = 0,    
       AB2 − AD2 = (AB + BC) ⋅ (AB + CB),
d’où u ⋅ (v + w ) = u ⋅ v + u ⋅ w .     
   
De même pour v = 0 ou w = 0. AB2 − AD2 = AC ⋅ (DC + CB) = AC ⋅ DB.
          
2. a)  u = OA et v + w = OB + BC = OC, donc b)  AC et DB orthogonaux équivaut à AC ⋅ DB = 0,
    
u ⋅ (v + w ) = OA ⋅ OC. soit AB2 = AD2 , c’est-à-dire AB = AD.

b) Le projeté orthogonal de C sur (OA) est C¢ et OA Un parallélogramme dont les diagonales sont per-
 pendiculaires est donc un losange.
et OC¢ sont colinéaires et de même sens, d’où
  
u ⋅ (v + w ) = OA × OC′.  

c) Le projeté orthogonal de B sur (OA) est B¢ et OA 86 a)  CA ⋅ CB = CA2 = 9 par projection orthogo-
 nale de B sur (AC).
et OB¢ sont colinéaires et de même sens, d’où :  
    b)  AB ⋅ AE = AE2 par projection orthogonale de B sur
u ⋅ v = OA ⋅ OB = OA × OB′
  (AE).
OA et B¢C¢ sont colinéaires et de même sens, d’où
Dans le triangle ABE rectangle en E,
par projection orthogonale sur (OA)  :
    3
u ⋅ w = OA ⋅ BC = OA × B′C′. AE = cos(30°) × AB, soit AE = × 4 = 2 3.
      2
Finalement, u ⋅ v + u ⋅ w = OA × OB′ + OA × B′C′.
Finalement, AB ⋅ AE = 12.
d)  B¢ appartient au segment [OC′ ] d’où  
c)  AB ⋅ AF = AE × AF par projection orthogonale de
OC′ = OB′ + B′C′.
B sur (AF).
Donc, OA × OC′ = OA × OB′ + OA × B′C′ ,  = 90° − 30° = 60°.
       CAF
c’est-à-dire u ⋅ (v + w ) = u ⋅ v + u ⋅ w .
Dans le triangle ACF rectangle en F,
  1 3
82 •  BA(3 ; 3) et BC(4 ; − 3). AF = cos(60°) × AC , soit AF = × 3 = .
    2 2
D’où BA. ⋅ BC = 12 − 9 = 3 et BA ⋅ BC ≠ 0.   3
  Finalement, AB ⋅ AF = 2 3 × = 3 3.
•  AB(−3 ; − 3) et AC(7 ; 0). 2
          
D’où AB ⋅ AC = −21 et AB ⋅ AC ≠ 0. d)  AC ⋅ (BA + AE) = AC ⋅ BA + AC ⋅ AE.
     
•  CA(−7 ; 0) et CB(−4 ; 3). AC et AB orthogonaux donc AC ⋅ BA = 0,
     
D’où CA ⋅ CB = 28 et CA ⋅ CB ≠ 0. AC ⋅ AE = AF × AE par projection orthogonale de C
Finalement, le triangle ABC n‘est pas rectangle. sur (AE).
  3
D’où AC ⋅ AE = × 2 3 = 3 3.
83 On note ( x ; y) les coordonnées du point C.
    2    
AB(−4 ; − 2) et AC( x − 2 ; y − 4). e)  AB ⋅ (CA + AF) = AB ⋅ CA + AB ⋅ AF,
     
AB ⋅ AC = 0 équivaut à −4( x − 2) − 2( y − 4) = 0 , AB et CA orthogonaux donc AB ⋅ CA = 0,
    
soit −4 x − 2 y + 16 = 0. AB ⋅ (CA + AF) = AB ⋅ AF = 3 3 (d’après c)).
  
Le point C( x ; y) appartient à la droite d donc y = x. f)  CB ⋅ FE = FE × FE par projection orthogonale des
De −4 x − 2 x + 16 = 0 , points C et B sur (FE).
16 8 3 4 3 −3
on en déduit que x = = . FE = AE − AF = 2 3 − = .
6 3 2 2
8 8   −24 3 + 57
Finalement les coordonnées de C sont  ; . D’où CB ⋅ FE = .
 3 3 
4

Chapitre 9  ★  Produit scalaire et calcul vectoriel 151

172909_Chap09_000-000.indd 151 30/07/2019 15:26:41


87 a) On introduit le repère orthonormé (A ; I, J) 90 1re méthode : en utilisant le produit scalaire.

 1  
 1  On note a la longueur AB (avec a > 0 ).
tel que AI = AB et AJ = AD.
b b On introduit le repère orthonormé (A ; I, J) tel que
  

CQ(− b ; − DQ) et DP(AP ; − b). 1  
 1 
AI = AB et AJ = AD.
 = 45° et MQD  = 90° donc le triangle DQM est a a
QDM
B(a ; 0), D(0 ; a).
rectangle isocèle en Q.
La longueur d’une hauteur dans un triangle équilaté-
D’où QD = QM.
3
Comme APMQ est un rectangle, il vient : ral de côté a est égale à a, d’où
2
AP = QM = QD. 1 3   3 1 
  E  a ; a + a et F− a ; a.
De CQ(− b ; − DQ) et DP(DQ ; − b), on déduit  2 2    2 2 
 
CQ ⋅ DP = − b × DQ + DQ × b = 0. 
 3 1 1 3  
Donc, EF− a − a ; − a − a et BD(− a ; a).
Finalement, les droites (CQ) et (DP) sont perpendi-  2 2 2 2  
culaires. 
  3 2 1 2 1 2 3 2
b)  CQ = (− b)2 + (− DQ)2 = b2 + DQ2 et Donc, EF ⋅ BD = a + a − a − a = 0.
2 2 2 2
DP = DQ2 + (− b)2 = DQ2 + b2 . Par conséquent, les droites (BD) et (DE) sont per-
Par conséquent, CQ = DP. pendiculaires.
2e méthode : sans utiliser le produit scalaire.
      En utilisant le même repère que dans la première
88 a)  AC ⋅ DB = (AD + DC) ⋅ (DA + AB),
        méthode, on a :
AC ⋅ DB = −AD2 + AD ⋅ AB + DC ⋅ DA + DC ⋅ AB
     
 3 1 1 3 
AD et AB sont orthogonaux ainsi que DC et DA AC(a ; a) et EF− a− a ;− a− a .
     2 2 2 2 
donc AD ⋅ AB = DC ⋅ DA = 0.  

  On teste la colinéarité de AC et EF  :
DC et AB sont colinéaires et de même sens, donc
   1   
DC ⋅ AB = DC × AB = c × a. − a − 3 a × a − − 1 a − 3 a × a = 0 ,
 2   
   2   2 2 
Finalement, AC ⋅ DB = − b2 + ac .  
  Donc AC et EF sont colinéaires.
b)  DB et AC orthogonaux équivaut à − b2 + ac = 0,
Les diagonales d’un carré sont perpendiculaires donc
soit ac = b2 .   

AC et BD sont orthogonaux. On en déduit que EF

89 1. a)  et BD sont orthogonaux.
  
91 a)  CF ⋅ CG = CF × CG × cos(FCG )
  1
CF ⋅ CG =  × (a − ) × cos(60°) = (a − )
2
      2  
b)  FG ⋅ FC = (FC + CG) ⋅ FC = FC + CG ⋅ FC
  1 1 1
FG ⋅ FC =  2 − (a − ) =  2 − a +  2
2 2 2
  3 1
FG ⋅ FC =  2 − a
2 2
b) Lorsqu’on fait varier le curseur a et que l’on c) CGF est un triangle rectangle en F équivaut à
  3 2 1
déplace le point E, on conjecture que les droites (BE) FG ⋅ FC = 0, c’est-à-dire  − a = 0 , soit
et (DF) sont perpendiculaires. 1 2 2
(3 − a) = 0.
2. On introduit le repère orthonormé (B ; I, J) tel que 2
 1   1  Finalement, CGF est un triangle rectangle en F équi-
BI = BC et BJ = BA. a
 a
  a vaut à  = 0 ou  = .
BE(a ; CE) et DF(CF ; − a) 3

  Or,  = 0 est impossible car  > 0 donc, une condi-
Donc BE ⋅ DF = a × CF − a × CE.
  tion nécessaire et suffisante pour que le triangle CGF
Comme CF = CE , il vient BE ⋅ DF = 0. a
soit rectangle en F est  = .
D’où les droites (BE) et (DF) sont perpendiculaires. 3
152

172909_Chap09_000-000.indd 152 30/07/2019 15:26:35


92 a) ABC est un triangle rectangle en A donc 95 a) 
  A
AB ⋅ AC = 0. 4
   
D’où (AH + HB) ⋅ (AH + HC) = 0, c’est-à-dire, 3
      B
2
AH2 + AH ⋅ HC + HB ⋅ AH + HB ⋅ HC = 0.
  1
Or, les vecteurs AH et HC sont orthogonaux, ainsi C
 
que les vecteurs HB et AH, c’est-à-dire O 1 2 3 4 5
     
AH ⋅ HC = HB ⋅ AH = 0 et HB et HC sont colinéaires    
b)  CA(1 ; 3) et CB(4 ; 1). Donc CA ⋅ CB = 4 + 3 = 7.
et de sens opposés, donc AH2 − HB × HC = 0 , 
c) Le projeté orthogonal de A sur (CB) est H, CH et
soit AH2 = BH × CH. 
b)  B¢ est le milieu de [AC] et C¢ est le milieu de [AB], CB sont colinéaires et de même sens, donc
 
 1    1   CA ⋅ CB = CH × CB.
donc HB′ = (HA + HC) et HC′ = (HA + HB).  
2 2 CA ⋅ CB 7 7
  1       D’où CH = = = .
2
HB′ ⋅ HC′ = (HA + HA ⋅ HB + HC ⋅ HA + HC ⋅ HB). CB 2
4 +1 2 17
  4  
Or, HA ⋅ HB = HC ⋅ HA = 0 De plus, CA = 12 + 32 = 10 .
  1
donc HB′ ⋅ HC′ = (HA2 − HC × HB) = 0 d’après a). En appliquant le théorème de Pythagore au triangle
4 ACH rectangle en H, il vient :
Donc les droites (HB′) et (HC ′) sont perpendiculaires.
49 11
AH = 10 − = .

       17 17
93 a)  AI ⋅ DB = (AD + DI) ⋅ (DA + AB),

         En unités d’aire, l’aire ! du triangle ABC est :
AI ⋅ DB = − AD2 + AD ⋅ AB + DI ⋅ DA + DI ⋅ AB.
    BC × AH 17 × 11 11
AD et AB sont orthogonaux ainsi que DI et DA != = = .
     2 2 × 17 2
donc AD ⋅ AB = DI ⋅ AB = 0.

 
DI et AB sont colinéaires et de même sens, donc   1

  96 AB ⋅ AM = (AB2 + AM2 − BM2 ).
1 2
DI ⋅ AB = DI × AB = a2 .   1
2 AB ⋅ AM = (64 + 16 − 36) = 22.

  a2 a2 2
Finalement, AI ⋅ DB = − b2 + = − b2 . Par ailleurs, le projeté orthogonal de M sur (AB) est H
2 2  
b) À l’aide du théorème de Pythagore, il vient : et AB et AH sont colinéaires et de même sens,
 
a2 donc AB ⋅ AM = AB × AH.
AI = + b2 et DB = a2 + b2 .
4 22 11

  En conséquence, AH = = = 2, 75.
I).
c)  AI ⋅ DB = AI × DB × cos(BE 8 4

  H est donc situé sur [AB] à une distance de 2,75 m du
En translatant les vecteurs AI et DB en la même ori-
gine E, il vient : point A.

a2 a2      


− b2 = + b2 × a2 + b2 × cos(BE I), 97 a)  AB ⋅ AC = (AD + DB) ⋅ (AD + DC).
2 4  
Or, D est le milieu de [BC], donc DB = −DC.
I) = a2 − 2b2      
soit cos(BE . AB ⋅ AC = (AD − DC) ⋅ (AD + DC) = AD2 − DC2 .
a2 + 4 b 2 × a2 + b 2
b) ABC est un triangle isocèle en A, donc la médiane
d) Pour a = b = 4, ( ABCD est alors un carré de
(AD) est aussi la hauteur issue de A.
 ) = − 10 et BEI
côté 4), il vient cos(BEI  ≈ 108°. D’après le théorème de Pythagore appliqué au
10
triangle ABD rectangle en D, il vient :
94   AD2 = AB2 − DB2 , soit AD2 = AB2 − DC2 .
 
D’où AB ⋅ AC = AB2 − DC2 − DC2 = AB2 − 2DC2 .
 
c)  BC = 4 donc DC = 2. D’où AB ⋅ AC = 9 − 8 = 1.
 
 ), soit
Par ailleurs, AB ⋅ AC = AB × AC × cos(BAC
 
 ) = AB ⋅ AC = 1 .
cos(BAC
AB × AC 9
 ≈ 84°.
Ainsi, BAC

Chapitre 9  ★  Produit scalaire et calcul vectoriel 153

172909_Chap09_000-000.indd 153 29/07/2019 18:02:50


98 D’après le théorème de Pythagore, 101 a) « Si P, alors Q » est vraie. En effet,
    
AC2 = 22 + 22 = 8 , AB2 = 32 + 22 = 13, si u = 3v alors u = 3v = 3 v ( 3 > 0 ).
BC2 = 32 + 22 = 13. « Si Q, alors P » est fausse :
  1 1
AB ⋅ AC = (AB2 + AC2 − BC2 ) = × 8 = 4. on considère dans un repère le contre-exemple :
2 2  
u(0 ; 3) et v(1 ; 0).
Par ailleurs, dans le plan (ABC), Comme l’implication précédente est fausse, il ne peut
 
AB ⋅ AC = AB × AC × cos(BAC  ), soit
pas y avoir équivalence entre P et Q.
4 = 13 × 8 × cos(BAC  ), donc   
b) « Si P, alors Q » est vraie. En effet, si u + v = 0 alors
     
 ) = 4 = 26 .
cos(BAC (u + v ) ⋅ (u − v ) = 0, soit u 2 − v 2 = 0.
104 13 «  Si Q, alors P » est fausse. On peut considérer le
 ≈ 67°.
À l’aide de la calculatrice, on obtient BAC    
contre-exemple : v = u avec u ¹ 0.
Comme l’implication précédente est fausse, il ne peut
99 a) L’affirmation est fausse. En effet, il suffit de
 pas y avoir équivalence entre P et Q.
considérer le contre-exemple avec des vecteurs u et
 
v , tous les deux orthogonaux à un vecteur w
comme indiqué sur la figure suivante :

u
w Organiser son raisonnement
v

  102 Comme H est situé sur [BC],


Dans ce cas, u et v ne sont pas égaux.
b) L’affirmation est fausse. En effet, il suffit de consi- BC = BH + HC = 4 + 16 = 20
   
dérer le contre-exemple suivant dans lequel u et v ABC est un triangle rectangle en A, donc AB ⋅ AC = 0.
   
sont deux vecteurs colinéaires et de même sens. D’où (AH + HB) ⋅ (AH + HC) = 0, ce qui conduit à
     
u v AH2 + AH ⋅ HC + HB ⋅ AH + HB ⋅ HC = 0.
   
u+v Or AH et HC sont orthogonaux ainsi que HB et AH
   
      donc AH ⋅ HC = HB ⋅ AH = 0.
On a u + v = u + v sans avoir u et v orthogo-  
naux. Par conséquent, AH2 + HB ⋅ HC = 0.
 
c) L’affirmation est vraie. En effet, pour tous vec- Comme HB et HC sont colinéaires et de sens oppo-
  sés, il vient AH2 = −HB × HC.
teurs u et v , non nuls,
   
u ⋅ v = u × v × cos(θ). D’où AH2 = 4 × 16 = 64 , soit AH = 8.
   
| cos(θ)| < 1et |u ⋅ v | = u × v ×|cos(θ)| À l’aide du théorème de Pythagore appliqué au
   
donc u ⋅ v < u × v . triangle ABH rectangle en H puis au triangle ABC rec-
d) L’affirmation est vraie. En effet, pour tous vec- tangle en A, on obtient :
     
teurs u et v , u + v < u + v équivaut à BA2 = AH2 + BH2 , soit BA = 4 5
 2  
u + v < ( u + v )2 , et AC2 = BC2 − AB2 , soit AC = 400 − 80 = 8 5.
   
soit 2u ⋅ v < 2 u × v ,
    103 1. a) 
c’est-à-dire u ⋅ v < u × v .
Cette dernière inégalité est toujours vraie d’après c).

  BC2


100 BA ⋅ BC = équivaut à
2
1 BC2
(BA2 + BC2 − CA2 ) = , soit
2 2
1
(BA2 − CA2 ) = 0 , c’est-à-dire BA = CA.
2
  BC2
Finalement, BA ⋅ BC = équivaut à ABC est un b) Lorsque a varie, on conjecture que la mesure de
2
triangle isocèle en A, c’est-à-dire P équivaut à Q.  reste constante.
l’angle ICA
154

172909_Chap09_000-000.indd 154 30/07/2019 15:28:40


2. a) On introduit le repère orthonormé 106 1. 
 1  1  
A , AB, AD.
 a a 
 1 
A(0 ; 0), C(a ; a) et I0 ; a donc
 2 
 
 a
CA(− a ; − a) et CI− a ; − .
 2

  1 3
CI ⋅ CA = a2 + a2 = a2 .

  2 2
b)  CI ⋅ CA = CI × CA × cos(I CA).

a2 5
Or, CI = a2 + = a et CA = 2a,
4 2

  5   
d’où CI ⋅ CA = a × 2a × cos(I
CA), c’est-à-dire
2 2. a)  A ¢ est le milieu de [BC] donc A ′B + A ′C = 0.
      

  10 2 OB + OC = OA ′ + A ′B + OA ′ + A ′C = 2OA ′
CI ⋅ CA = a × cos(I
CA).    
2 b)  OH = OA + OB + OC, donc
    

  3 2 OA + AH = OA + OB + OC , d’où
a   
C I ⋅ CA 3 AH = OB + OC.
c)  cos(I
CA) = = 2 = .     
10 2 10 2 10 D’après 2. a), OB + OC = 2OA ′ donc, AH = 2OA ′.
a a
2 2 c) O est le centre du cercle circonscrit au triangle ABC,
3
est indépendant de a, donc la mesure de l’angle donc (OA ′) est la médiatrice de [BC], d’où les droites
10
 est constante et ICA  ≈ 18°. (OA ′) et (BC) sont perpendiculaires.
ICA
D’après 2. b), les droites (AH) et (OA ′) sont parallèles
104 On note a la longueur AB avec a > 0 et b la lon- donc les droites (AH) et (BC) sont perpendiculaires.
gueur AE avec b > 0. d) On raisonne comme dans 2. b) et 2. c) pour obte-
 1  1   nir que les droites (BH) et (AC) sont perpendicu-
On introduit le repère orthonormé  A, AB, AD. laires.
 a a 
e) A est un point de la droite (AH) et (AH) est per-
 a −b 
A(0 ; 0), B(a ; 0) et G(0 ; − b) donc I ; . pendiculaire à (BC), donc (AH) est la hauteur issue
 2 2 
de A.
E(−b ; 0), D(0 ; a). De même (BH) est la hauteur issue de B.

 a b 
D’où, AI ; −  et ED(b ; a). De même (CH) est la hauteur issue de C .
 2 2

  ab ab H appartient à ces trois hauteurs qui ne sont pas
AI ⋅ ED = − = 0, donc les droites (IA ) et (ED) confondues sinon les côtés du triangle ABC seraient
2 2
sont perpendiculaires. parallèles entre eux, ce qui est impossible.
Par conséquent, les trois hauteurs du triangle ABC
105 On pose S = AB2 + BC2 + CD2 + DA2 . sont concourantes en H.
Comme ABCD est un parallélogramme, AB = CD et Remarque : H est l’orthocentre du triangle ABC.
  
BC = AD. 3. a)  A ¢ est le milieu de [BC], donc GB + GC = 2GA ′.
    
S = 2AB2 + 2AD2 = 2(AB2 + AD2 ). D’où GA + GB + GC = GA + 2GA ′ ,
     
On note I le milieu des diagonales [BD] ou [AC ]. GA + GB + GC = GA + 2(GA + AA ′),
 2  2 
   
       
S = 2(AB + AD ) = 2((AI + IB)2 + (AI + ID)2 ) GA + GB + GC = 3GA + 2AA ′ ,

     2   
S = 2(2AI2 + 2AI ⋅ (IB + ID) + IB2 + ID2 ) De AG = AA ′ on déduit 3GA = 2A ′A.

    3    
I est le milieu de [BD] donc IB + ID = 0.
Finalement, GA + GB + GC = 0.
  AC 2  BD 2     

S = 2(2AI2 + 2IB2 ) = 22  + 2   b)  GA + GB + GC = 0 donc,
     
  2   2  
 GB + BA + GB + GB + BC = 0,
  
S = AC2 + BD2 . 3GB = AB + CB.

Chapitre 9  ★  Produit scalaire et calcul vectoriel 155

172909_Chap09_000-000.indd 155 29/07/2019 18:05:20


Or, B¢ est le milieu de [AC], donc Par ailleurs, dans le triangle DEF,
    
DE ⋅ DF = DE × DF × cos(EDF  ).
AB + CB = 2B′B.
  5 2
Finalement, 3BG = 2BB′.   a
 DE ⋅ DF 8 5
Remarque : G est le centre de gravité du triangle Donc, cos(EDF) = = = .
DE × DF 3 2 6
ABC. a
    4
4. a)  3OG = OG + OG + OG,  est indépendant de a et EDF
D’où EDF  ≈ 34°.
      
3OG = OA + AG + OB + BG + OC + CG,
       110 a)  EBC  = 90° − 60° = 30°.
3OG = OA + OB + OC + AG + BG + CG,   
     ),
3OG = OH + 0 = OH. Or, BC ⋅ BE = BC × BE × cos(EBC
   
b) De 3OG = OH, on déduit que les points O, G et H 3
donc BC ⋅ BE = 1× 1× cos(30°) = .
sont alignés (dans cet ordre).   2
ABCD est un carré donc DA = CB,
    
107 D est le projeté orthogonal de A sur (BF) donc d’où DA ⋅ BE = CB ⋅ BE.
         
BF ⋅ BD = BF ⋅ BA. 3
DA ⋅ BE = −BC ⋅ BE = − .
Le projeté orthogonal de F sur (BA) est E donc 2
        1
BF ⋅ BA = BE ⋅ BA. b)  EA ⋅ EB = 1× 1× cos(60°) = .
2
E est le projeté orthogonal de C sur (BA) donc  = 90° − 30° = 60° donc le
    c)  BC = BF et CBF
BE ⋅ BA = BC ⋅ BA. triangle BCF est équilatéral.
Le projeté orthogonal de A sur (BC) est C donc    1
    d)  BC ⋅ BF = 1× 1× cos(60°) = .
BC ⋅ BA = BC ⋅ BC = BC2 . 2
          
BF et BD sont colinéaires et de même sens, d’où 1
  DA ⋅ EG = CB ⋅ EG = CB ⋅ BF = −BC ⋅ BF = − .
BF ⋅ BD = BF × BD.     2
e)  AE ⋅ EG = −EA ⋅ EG = −EA × EG × cos(AEG  ),
Finalement, BF × BD = BC2 .  
BC2 16 8 AE ⋅ EG = −1× 1× cos(90° + 60°),
Par conséquent, BF = = = .  
BD 6 3 3
AE ⋅ EG = −1× (−sin(60°)) = .
      2
108 On note ( x ; y) les coordonnées du point C. f)  DE ⋅ BG = (DA + AE) ⋅ (BE + EG),
             
AB(−2 ; −4), CC′(2, 5 − x ; 4 − y) DE ⋅ BG = DA ⋅ BE + DA ⋅ EG + AE ⋅ BE + AE ⋅ EG.
   
AC( x − 3 ; y − 5), BB′(3 ; 3) 3 1 1 3
    DE ⋅ BG = − − + + = 0.
CC′ ⋅ AB = 0 et BB′ ⋅ AC = 0 d’où : 2 2 2 2
Les droites (DE) et (BG) sont donc perpendiculaires.
−21 + 2 x + 4 y = 0 et −24 + 3 x + 3 y = 0
g) La droite (BG) est perpendiculaire à la droite (DE)
( x ; y) est donc solution du système
et à la droite (EF) ([EF] et [BG] sont les diagonales du
2 x + 4 y = 21  x = 5, 5
 , ce qui conduit à  . carré BFGE).
3 x + 3 y = 24  y = 2, 5
Donc les droites (DE) et (EF) sont parallèles et elles
  sont confondues car elles ont au moins un point com-
109 On calcule le produit scalaire DE × DF de deux mun E.
façons. Finalement, D, E et F sont alignés.
3
Dans le triangle équilatéral ADC, on a DE = a. De
2 111 ACGE est un rectangle car il s’agit d’un parallélo-
3
même DF = a. gramme dont les diagonales [CE ] et [AG] sont de
2
même longueur.
Dans le triangle équilatéral ABC, le théorème des      
AB a GA ⋅ EC = (GE + EA) ⋅ (EG + GC),
milieux conduit à EF = = .  
2 2 EA = GC donc,
     
  1 GA ⋅ EC = (GE + EA) ⋅ (−GE + EA) = EA2 − GE2 .
DE ⋅ DF = (DE2 + DF2 − EF2 ),
2 EA = a et GE = 2a ([GE ] est la diagonale d’un
  1  3 2 3 2 a2  5 2 carré de côté a).
DE ⋅ DF =  a + a −  = a .  
2  4 4 4  8 Donc GA ⋅ EC = a2 − 2a2 = −a2 .
156

172909_Chap09_000-000.indd 156 29/07/2019 18:06:32


Par ailleurs, en translatant les vecteurs à la même ori-
 
gine I, il vient : GA ⋅ EC = GA × EC × cos(α).
Or, GA = EC = a2 + 2a2 .
−a2 1
D’où cos(α) = 2 2
=− .
a + 2a 3
Donc a est indépendant de a et α ≈ 109°.
Remarque : une autre méthode consiste à calculer le

  
produit scalaire IA × IC à l’aide du triangle IAC isocèle
en I et en utilisant le milieu J de [BD].

 aigu
112 1. a) 1er cas : BAC
Dans le triangle ACH rectangle en H,
On souhaite montrer que la droite (IA ′) est perpendi-
 ) = CH . Or, H appartient au segment [AB]
sin(HAC
AC culaire à la droite (BC).
     

donc HAC = BAC. IA ′ ⋅ BC = (IC′ + C′A ′) ⋅ (BA + AC),
         
 ) = CH.
Par conséquent, AC × sin(BAC IA ′ ⋅ BC = IC′ ⋅ BA + IC′ ⋅ AC + C′A ′ ⋅ BA + C ′A ′ ⋅ AC,
1 1  ), d3 est perpendiculaire à (AB) et passe par C¢ , d’où
S = AB × CH = AB × AC × sin(BAC    
2 2
1 IC¢ et BA sont orthogonaux soit IC′ ⋅ BA = 0.
 ).
S = bc sin(BAC
2 Les projetés orthogonaux de A, B et C sur la droite
 obtus
2e cas : BAC (A ′C′) sont respectivement A ¢ , B¢ et C¢ donc
       
Dans le triangle ACH rectangle en H, C′A ′ ⋅ BA = C′A ′ ⋅ B′A ′ et C′A ′ ⋅ AC = C′A ′ ⋅ A ′C′.
  
 ) = CH . Or HAC
sin(HAC  = 180° − BAC , De plus, IC ′ = IB′ + B′C′.
AC Par conséquent,
 ) = sin(BAC
et sin(180° − BAC  ).         
IA ′ ⋅ BC = (IB′ + B′C′) ⋅ AC + C′A ′ ⋅ B′A ′ + C′A ′ ⋅ A ′C′ ,
 ) = CH.
Par conséquent, AC × sin(BAC          
1 1 IA ′ ⋅ BC = IB′ ⋅ AC + B′C′ ⋅ AC + C′A ′ ⋅ B′A ′ + C′A ′ ⋅ A ′C′.
S = AB × CH = AB × AC × sin(BAC  ),    
2 2 IB¢ et AC sont orthogonaux soit IB′ ⋅ AC = 0.
1  ). Les projetés orthogonaux de A et C sur la droite
S = bc sin(BAC
2 (B′C′) sont respectivement A ¢ et C¢ donc
b) En permutant les rôles de A et B dans la réponse    
B′C′ ⋅ AC = B′C′ ⋅ A ′C′.
à 1. a), on obtient :
D’où,
1  ).        
S = ac sin(CBA
2 IA ′ ⋅ BC = B′C′ ⋅ A ′C′ + C′A ′ ⋅ B′A ′ + C′A ′ ⋅ A ′C′ ,
       
1  ). IA ′ ⋅ BC = A ′C′ ⋅ A ′B′ + B′C′ ⋅ A ′C′ + C′A ′ ⋅ A ′C′ ,
c)  S = absin(ACB      
2
IA ′ ⋅ BC = A ′C ⋅ (A ′B′ + B′C ’ + C′A ′),
2. Ainsi :    
1  ) = 1 ac sin(CBA  ) = 1 ab sin(ACB
 ). IA ′ ⋅ BC = A ′C ⋅ 0 = 0.
bc sin(BAC
2 2 2 Donc la droite (IA ′) qui passe par I et A ¢ , est perpen-
En divisant tous les membres de ces égalités par diculaire à la droite (BC).
abc  ) sin(CBA
sin(BAC  ) sin(ACB ) D’où la droite (IA ′) correspond à la droite d1.
, il vient : = = .
2 a b c Finalement les trois droites d1, d2 et d3 sont concou-
 = 180° − (75° + 50°) = 55°.
3.  BCA rantes en I.
sin(50°) sin(55°)
= d’où AC » 4 , 7.      
AC 5 114 1. a)  AB ⋅ AM = (AO + OB) ⋅ (AO + OM)
       
sin(55°) sin(75°) AB ⋅ AM = AO2 + AO ⋅ OM + OB ⋅ AO + OB ⋅ OM
= d’où BC » 5, 9.    
5 BC
OB et AO sont orthogonaux donc OB ⋅ AO = 0.
113 Les droites d2 et d3 ne sont pas parallèles Le projeté orthogonal de M sur la droite (AO) est A et
sinon (AB) et (AC) seraient parallèles. le projeté orthogonal de M sur la droite (BO) est B,
   
On note I le point d’intersection des droites d2 et donc AO ⋅ OM = AO2 et OB ⋅ OM = OB2 .
 
d3 . AB ⋅ AM = AO2 − AO2 + 0 + OB2 = OB2 .

Chapitre 9  ★  Produit scalaire et calcul vectoriel 157

172909_Chap09_000-000.indd 157 30/07/2019 15:29:53


b) Le projeté orthogonal du point M sur (AB) est D et 1 1    
  (AC2 − AB2 ) = (AC + AB) ⋅ (AC − AB)
les vecteurs AB et AD sont colinéaires de même 2 2
  
 
sens donc AB ⋅ AM = AB × AD. 1 2 2
(AC − AB ) = AI ⋅ BC
Les diagonales du rectangle OAMB sont de même 2
  1
longueur d’où AB = OM = 1. AM ⋅ BC = (AC2 − AB2 ) équivaut donc à
  2
Donc AB ⋅ AM = 1× AD = AD.      
 
AM ⋅ BC = AI ⋅ BC , c’est-à-dire (AM − AI) ⋅ BC = 0,
À l’aide de 1. a) on déduit AD = OB2 .  
A, D et B sont alignés dans cet ordre et AB = 1, d’où soit encore à IM ⋅ BC = 0.
     L’ensemble cherché est donc la médiatrice du seg-
AD = AD AB. Par conséquent, AD = OB2 AB.
ment [BC].
2. a)  M( x ; y) donc A( x ; 0) et B(0 ; y).

Donc OB2 = y 2 et AB(− x ; y). 116 On note I le milieu du segment [AB].
       
On note ( xD ; yD ) les coordonnées du point D. MA ⋅ MB = (MI + IA) ⋅ (MI + IB),
 x − x = y 2 × (− x)        
  MA ⋅ MB = (MI + IA) ⋅ (MI − IA),
AD = OB2 AB équivaut à  D c’est-  
 yD = y 2 × y MA ⋅ MB = MI2 − IA2 .
 x = x(1 − y 2 ) 
à-dire  D . On note K le milieu du segment [AC ]. On a de même,
 y = y 2 × y  
 D MA ⋅ MC = MK 2 − KA2 .
   
Or OM2 = 1, donc x2 + y 2 = 1, (MA ⋅ MB) × (MA ⋅ MC) < 0 équivaut à
   
 x = x3 MA ⋅ MB > 0 MA ⋅ MB < 0
soit 1 − y 2 = x2 . On obtient donc :  D .    
ou    .
 y = y 3 MA ⋅ MC < 0 MA ⋅ MC > 0
 D   
b)  y 2 = 1 − x2 équivaut à y = 1 − x2 ou MA ⋅ MB > 0 équivaut à MI2 > IA2 , soit
MI > IA , c’est-à-dire M n’appartient pas au disque
y = − 1 − x2 .
fermé D1 de diamètre [AB].
Or, y 3 = yy 2 = y(1 − x2 ) donc y 3 = 1 − x2 (1 − x2 )  
MA ⋅ MC < 0 équivaut à M appartient au disque
ou y 3 = − 1 − x2 (1 − x2 ). ouvert D2 de diamètre [AC ].
c)  D2 Ì D1 donc l’ensemble des points M qui vérifient
 
MA ⋅ MB > 0

   est l’ensemble vide.
MA ⋅ MC < 0

On raisonne de même pour le 2e cas :
 
MA ⋅ MB < 0

   conduit à M appartient à D1 auquel on
MA ⋅ MC > 0

retire D2 et le cercle de diamètre [AC ].
Finalement,
   
(MA ⋅ MB) × (MA ⋅ MC) < 0 équivaut à M appartient à
Affichage à l’issue de l’exécution du programme : D1 auquel on retire D2 et le cercle de diamètre [AC].
La zone colorée (les frontières des disques étant
exclues) correspond à l’ensemble des points cher-
chés.

A K C I B

115 On note I le milieu du segment [BC]. 117 On nomme ABCD un quadrilatère quelconque, I
 
1 1 le milieu du segment [AC ] et J le milieu du segment
(AC2 − AB2 ) = (AC2 − AB2 )
2 2 [BD].
158

172909_Chap09_000-000.indd 158 29/07/2019 18:09:00


B Dans le triangle BIC rectangle en B :
1 1
A 
cos(BIC) = 2 = 2 . Donc B IC ≠ 60°.
J
1 5
I 1+
4 4
C 
  
  
D 2. a)  IC ⋅ GC = (IB + BC) ⋅ GC,

      
Dans le triangle ABC : IC ⋅ GC = IB ⋅ GC + BC ⋅ GC.
   
  
  Or, GC = FB et FB orthogonal à IB, donc GC ortho-
AB2 + BC2 = AB2 + BC2 


     gonal à IB.
AB2 + BC2 = (AI + IB)2 + (BI + IC)2  
De plus, BC orthogonal à GC,
1 
 
AB2 + BC2 = 2IB2 + AC2 (formule de la médiane). donc IC ⋅ GC = 0 + 0 = 0.
2 
 
De même dans le triangle ACD : b) De 2. a) on déduit que IC orthogonal à GC, c’est-
1 à-dire le triangle ICG est rectangle en C. D’après le
CD2 + DA2 = 2ID2 + AC2 . théorème de Pythagore (appliqué à deux reprises) :
2
Donc,  1 2  1 2 3
IJ2 = IC2 + CJ2 = 12 +   +   = .
 2   2  2
AB2 + BC2 + CD2 + DA2 = AC2 + 2(IB2 + ID2 ).
1 3
Dans le triangle IBD, IB2 + ID2 = 2IJ2 + DB2 . D’où IJ = .
2 2
Finalement,   1
c)  EI ⋅ EJ = (IE2 + EJ2 − IJ2 ).
AB2 + BC2 + CD2 + DA2 = AC2 + DB2 + 4 IJ2. 2
Or IJ2 > 0 d’où l’inégalité d’Euler. Le théorème de Pythagore conduit à :
1 5
 IE2 = + 1 =
118 On note (a ; b) les coordonnées du vecteur u 4 4
 1 9
et (c ; d ) celles du vecteur v . EJ2 = 2 + = .
4 4
On constate que b = 2a et c = −2d .   1  5 9 3 
 
Donc u(a ; 2a) et v (−2d ; d ). Donc EI ⋅ EJ =  + −  = 1.
    2  4 4 2 
D’où u ⋅ v = −2ad + 2ad = 0, c’est-à-dire u et v  
sont orthogonaux. Or, dans le plan (EIJ), EI ⋅ EJ = EI × EJ × cos(I
EJ), donc
 
EI ⋅ EJ 1
cos(I EJ) = =  ≈ 53°.
, et IEJ
119 1. a) L’affirmation est vraie. En effet, dans le EI × EJ 5 9
×
plan (ABC) le projeté orthogonal du point C sur la 4 4
 

droite (AI) est B et les vecteurs AB et AI sont coli-
   1
néaires de même sens, donc AC ⋅ AI = AB × AI = .
2
b) L’affirmation est fausse. En effet, I est le milieu de

 
 
 
[AB] donc AI = IB et AI ⋅ IB = AI2 = .
1 Exploiter ses compétences
4
   1   1
D’après a), AC ⋅ AI = et AC ⋅ AI ≠ .
2 4
c) L’affirmation est vraie. En effet,      
        
   
AB ⋅ IJ = AB ⋅ (IC + CJ) = AB ⋅ IC + AB ⋅ CJ. 120 F = F1 + F1 donc F = F1 + F1 ,

 1     
 Par conséquent,
CJ = AE et AE orthogonal à AB, donc AB ⋅ CJ = 0. 2   2  
2
     F = F1 + F1 = (F1 + F1)2 ,
D’où AB ⋅ IJ = AB ⋅ IC.  2  2    2
F = F1 + 2F1 ⋅ F1 + 2F2 ,
d) L’affirmation est fausse. En effet, d’après 1. c) 2  
     F = F12 + 2F1 ⋅ F1 + F22 .
AB ⋅ IJ = AB ⋅ IC. Or, I est le milieu de [AB], donc
 
   
  F1 = 600 et F2 = 50 (en kN).
AB = 2IB, d’où AB ⋅ IJ = 2IB ⋅ IC.  
  F1 ⋅ F1 = F1 × F2 × cos(45°),
AB ⋅ IJ = 2IB × IC × cos(B IC), soit   2
  F1 ⋅ F1 = 600 × 50 × .

AB ⋅ IJ = AB × IC × cos(BIC). 2
Chapitre 9  ★  Produit scalaire et calcul vectoriel 159

172909_Chap09_000-000.indd 159 29/07/2019 18:10:09


2 On raisonne maintenant dans le triangle CEF.
Finalement, F = 362 500 + 30 000 2 , soit  
  I est le milieu de [EF] donc IF = −IE et
F = 362 500 + 30 000 2 et F ≈ 636 kN.   
    
CE ⋅ CF = (CI + IE) ⋅ (CI + IF),
  
    
CE ⋅ CF = (CI + IE) ⋅ (CI − IE) = CI2 − IE2 ,
121 On considère le triangle ABC tel que AS = 4 , 07,
   IJ 2 1 1 1
BS = 3, 84 et AB = 5 (en hm). CE ⋅ CF =   − IE2 = a2 − a2 = − a2 .
 2  8 4 8
S  
 ),
Par ailleurs, CE ⋅ CF = CE × CF × cos(ECF
 
 ) = CE ⋅ CF .
d’où cos(ECF
CE × CF
4,07 3,85 Le théorème de Pythagore permet d’obtenir :
1 1 3
CE2 = CI2 + EI2 = a2 + a2 = a2 .
8 4 8
Finalement, comme CE = CF, il vient :
A B 1
− a2
 8 1  ≈ 109 , 5°.
H cos(ECF) = = − . Donc ECF
5 3 2 3
a
  1 8
AS ⋅ AB = (AS2 + AB2 − BS2 ),
2 123 Dans le repère orthonormé (O ; I, J), on note
  1
AS ⋅ AB = (4 , 072 + 52 − 3, 84 2 ) = 13, 409 7. ( x ; y) les coordonnées du point P.
2
On note H le pied de la hauteur issue de S. Par projec- P appartient au demi-cercle de centre O et de rayon 9,
tion orthogonale du point S, sur (AB), il vient : donc x2 + y 2 = 81, soit y 2 = 81 − x2 .
    De plus, D(6 ; 0) et C(− 3 ; 0).
AS ⋅ AB = AH × AB (le produit scalaire AS × AB est  
  D’où PC(− 3 − x ; − y) et PD(6 − x ; − y).
positif donc AH et AB sont dans le même sens).  
  PC ⋅ PD = (−3 − x)(6 − x) + y 2 ,
AS ⋅ AB 13, 409 7  
D’où AH = = = 2, 68194. PC ⋅ PD = −18 − 3 x + x2 + 81 − x2 = − 3 x + 63.
AB 5
En appliquant le théorème de Pythagore au triangle PC2 = (− 3 − x)2 + y 2 = 6 x + 90.
ASH rectangle en H, on obtient : De même PD2 = (6 − x)2 + y 2 = 117 − 12 x.
 
 ),
Par ailleurs, PC ⋅ PD = PC × PD × cos(CPD
SH = AS2 − AH2 .
SH » 3,06 hm. donc
 
Donc la hauteur de la tour est bien supérieure à  PC ⋅ PD −3 x + 63
cos(CPD) = = .
300 m. PC × PD 6 x + 90 × −12 x + 117
D’où cos(CPD ) = f ( x) avec x ∈ [− 9 ; 9 ].
122 On note a la longueur de l’arête [EG].
La fonction f définie sur [− 9 ; 9 ] par
Les triangles EHG et HFG sont des triangles équilaté- −3 x + 63
raux identiques d’où JE = JF. f( x) = admet un mini-
6 x + 90 × −12 x + 117
I est le milieu de [EF] donc IE = IF. 27
mum, atteint une seule fois en x = .
Par conséquent dans le plan (EFJ), (IJ) est la média- 7
trice de [EF]. Comme la fonction cosinus est décroissante sur
Ainsi, le triangle EJI est rectangle en I.  est maximum uniquement pour x = 27 ,
[0 ; π], CPD
 27 18  7
3
Sachant que la hauteur EJ est égale à a, le théo- c’est-à-dire pour P ; 10  puisque
2  7 7 
rème de Pythagore conduit alors à :
3 1 1  27 2 18
IJ2 = EJ2 − IE2 = a2 − a2 = a2 . 81 −   = 10 .
4 4 2  7  7

160

172909_Chap09_000-000.indd 160 30/07/2019 15:30:57


10
Applications
du produit scalaire

 
4  M( x ; y) d’où AB(−0 , 2 ; v ) et BM( x − 2, 8 ; y − v )
Découvrir
 
et AB ⋅ BM = 0 soit −0 , 2( x − 2, 8) + v( y − v ) = 0
d’où −0 , 2 x + v y = v 2 − 0 , 56.

1  Calcul d’un angle de tir


  
1  a) D’après la relation de Chasles AB = AT + TB
Acquérir des automatismes
  
donc TB − TA = AB.
 2    
b)  AB = (TB − TA)2 = TB2 + TA2 − 2 TB ⋅ TA
 ).
donc AB2 = TB2 + TA2 − 2 TA ⋅ TB cos(ATB 3 a) Un point M appartient à % si, et seulement si,
  1
MA ⋅ MB = 8, c’est-à-dire MI2 − AB2 = 8.
2  a)  TA2 = TC2 + CA2 = 252 + 202 = 1025 4
1 2 1
TB2 = TC2 + CB2 = 252 + 27,322 = 1371, 3 824 Or AB = × 4 = 1 donc M appartient à # si,
4 4
)
b)  AB2 = TB2 + TA2 − 2 TA ⋅ TB cos(ATB et seulement si, MI2 = 9 soit MI = 3.
2 2 2
 ) = TB + TA − AB
donc cos(ATB % est le cercle de centre I et du rayon 3 cm.
2 TA × TB b) Un point M appartient à % si et seulement si
   8 , 9° .
d’où cos(ATB) = 0 , 9 880 d’où ATB   1
MA ⋅ MB = 15, c’est-à-dire MI2 − AB2 = 15.
1 1 4
2  Déterminer les équations Or AB2 = × 4 = 1 donc M appartient à %, si et
4 4
d’un cercle et de deux tangentes seulement si MI2 = 16 soit MI = 4.
1  1. N appartient à G si et seulement si AN = 1, % est le cercle de centre I et du rayon 4 cm.
c’est-à-dire AN2 = 1. 4 a) 
D’où ( x − 3)2 + ( y − 0)2 = 1  2  2    
AB2 + AC2 = AB + AC = (AK + KB)2 + (AK + KC)2
soit x2 − 6 x + 9 + y 2 = 1 d’où :
Donc N ∈ Γ si, et seulement si,  
AB2 + AC2 = AK 2 + KB2 + 2AK ⋅ KB + AK 2 + KC2
x 2 − 6 x + y 2 + 8 = 0.  
+ 2AK ⋅ KC
2  B(2,8 ; y) appartient à G, donc soit :
  
2, 82 − 6 × 2, 8 + y 2 + 8 = 0 soit y 2 = 0 , 96 AB2 + AC2 = 2AK 2 + KB2 + KC2 + 2AK(KB + KC).
d’où B(2,8 ; 0 , 98). 1
Or K est le milieu de [BC], donc KB = KC = BC
   2
3  La droite T est la tangente en B au cercle G si, et et KB + KC = 0.
seulement si, les droites T et (AB) sont perpendi- 1
Donc AB2 + AC2 = 2AB2 + BC2
culaires. 2
1
Ainsi un point M appartient à la tangente T si, et b)  2, 52 + 4 2 = 2AK 2 + × 62
2
seulement si, les droites (AB) et (BM) sont per- 1
pendiculaires, c’est-à-dire si, et seulement si, les d’où AK 2 = (6 , 25 + 16 − 18) soit AK 2 = 2,125.
  2
vecteurs AB et BM sont orthogonaux. Ainsi AK = 2,125 soit AK  1, 5 cm.

Chapitre 10  ★  Applications du produit scalaire 161

172909_Chap10_000-000.indd 161 30/07/2019 17:17:26


 
7 a)  AB ⋅ AD = AB × AD × cos(120°)  ) ≈ 25 + 24 , 5 − 49 ≈ 0 , 01
d’où cos(ABC
   1 2 × 5 × 4,95
AB ⋅ AD = 6 × 5 × −  = −15  ≈ 89 , 42°.
 2  et ABC
b) • D’après la formule d’Al-Kashi appliquée au
triangle ABD : 13 a) 
BD2 = AB2 + AD2 − 2 × AB × AD × cos(120°) 5
BD2 = 36 + 25 + 30 = 91 d A
Donc BD = 91. 2
 = 180° − BAD
•  ABC  = 180° − 120° = 60° 1
D’après la formule d’Al-Kashi appliquée au triangle O 1 4
ABC : D

AC2 = BA2 + BC2 − 2 × BA × BC × cos(60°) b) Un vecteur normal n à la droite D est un vecteur
1 directeur de la droite d.
AC2 = 36 + 25 − 2 × 6 × 5 × 
2 Donc, par exemple n(−2 ; 1).
AC2 = 36 + 25 − 30 = 31
D a une équation cartésienne de la forme :
Donc AC = 31.
−2 x + y + c = 0
Or, A appartient à D
8 a) D’après la formule d’Al-Kashi appliquée au
donc −2 × (−1) + 3 + c = 0 soit c = −5.
triangle ABC :
) Une équation cartésienne de D est :
AC2 = BA2 + BC2 − 2 × BA × BC × cos(ABC
−2 x + y − 5 = 0
soit AC2 = 4 2 + 82 − 2 × 4 × 8 × cos(150°)
D’où AC2  135, 43 et AC  11, 64. 14 Méthode de la complétion du carré :
 
)
b)  AB ⋅ AD = AB × AD × cos(BAD •  x2 + 2 x = ( x + 1)2 − 12 = ( x + 1)2 − 1
= 4 × 8 × cos(30°)  27, 71. •  y 2 − 2 y = ( y − 1)2 − 12 = ( y − 1)2 − 1
c) D’après la relation de Chasles
     Un point M( x ; y) appartient à ^ si, et seulement si
BD = BA + AD d’où BD2 = (BA + AD)2 ( x + 1)2 − 1 + ( y − 1)2 − 1 = 4
 
soit BD2 = BA2 + AD2 + 2BA ⋅ AD c’est-à-dire ( x + 1)2 + ( y − 1)2 = 6.
d’où BD2 ≈ 16 + 64 + 2 ×(27, 71) Donc ^ est le cercle de centre A(−1 ; 1) et de rayon
donc BD2 » 135, 42 et BD » 11, 64 cm. 6.

9 D’après la formule d’Al-Kashi appliquée au 15 x2 + y 2 − 2 x − 4 y = 7, 25


triangle ABC : On utilise la méthode de complétion au carré.
)
BC2 = AB2 + AC2 − 2 × AB × AC × cos(BAC Les termes en x :
)
25 = 16 + 36 − 2 × 4 × 6 × cos(BAC x2 − 2 x = ( x − 1)2 − 1
 ) = 16 + 36 − 25 = 27 = 9
cos(BAC Les termes en y :
2× 4 ×6 48 16 y 2 − 4 y = ( y − 2)2 − 4
 ≈ 56°.
Avec la calculatrice, on obtient BAC
Ainsi M( x ; y) appartient à l’ensemble & si et seule-
ment si ( x − 1)2 − 1 + ( y − 2)2 − 4 = 7, 25 c’est-à-dire
10 D’après la formule d’Al-Kashi appliquée au
( x − 1)2 + ( y − 2)2 = 12, 25
triangle ABC :
) Donc & est le cercle de centre A(1 ; 2) et de rayon
BC2 = AB2 + AC2 − 2AB × AC × cos(BAC
12, 25 .
soit BC2 = 52 + 72 − 2 × 5 × 7 cos(45°)

  
d’où BC2 » 24 , 50 et BC » 24 , 50 16 IA ⋅ IB = −6 , 25
soit BC » 4 , 95 cm.
  1 9 7
D’après la formule d’Al-Kashi appliquée au triangle 17 MA ⋅ MB = MI2 − AB2 = 4 − =
ABC : 4 4 4
AC2 = AB2 + BC2 − 2 × BA × BC × cos(ABC ) 18 C appartient au cercle de centre I et de diamètre
2 2 2
) = AB + BC − AC [AB], le triangle ABC est rectangle en C donc
soit cos(ABC  
2 × BA × BC CA ⋅ CB = 0. Jessica a raison.
162

172909_Chap10_000-000.indd 162 30/07/2019 16:21:17


19 a)    1
A 26 MA × MB < 8 équivaut à MI2 - AB2 < 8 avec
5
4
4 1
I milieu de [AB] d’où MI2 − × 4 < 8 soit MI2 < 9
3 4
I d’où MI < 3.
2
1 L’ensemble des points M cherché est donc le disque
B de centre I et de rayon 3 cm, frontière incluse.
0
1 2 3 4  
    27 M( x ; 0) est tel que MA ⋅ MB = 8 si et seule-
MA ⋅ MB = 0 si, et seulement si, MA et MB sont 1
ment si MI2 − AB2 = 6
orthogonaux, donc M appartient au cercle de dia- 4
mètre [AB] 7 2 25
MI2 =  − x + ; AB2 = 52 + 12 = 26
 2  4
20 a)  A D
3 7 2 25 26 7 2 25
I J d’où  − x + − = 6 et  − x = .
2  2  4 4  2  4
1
B C 7 5
On a donc − x = soit x = 1
0 1 2 3 4 2 2
7 −5
ou − x = soit x = 6.
b) Les points communs aux deux ensembles sont les 2 2
points A et B. Les points recherchés sont M1(1 ; 0) et M2 (6 ; 0).

21 M appartient à # si, et seulement si,  


  28 M(0 ; y). MA ⋅ MB = 8. équivaut à
1  
MA ⋅ MB = −9 soit MI2 − AB2 = −9 avec I milieu 1
4 MA ⋅ MB = MI2 − AB2 = 8
1 4
de [AB]. Soit MI2 = × 4 2 − 9 = −5 (impossible) MI2 = 9 + (−3 − y)2 et AB2 = 16 + 4 = 20
4   1
Il n’existe pas de point M tel que MA ⋅ MB = −9 d’où 9 + (−3 − y)2 − × 20 = 8
4
 
22 M appartient à # si, et seulement si, MA ⋅ MB = 49 soit (−3 − y)2 = 12 d’où :
1 −3 − y = 2 3 et y = −2 3 − 3
soit MI2 − AB2 = 49 avec I milieu de [AB].
4 ou −3 − y = −2 3 et y = 2 3 − 3
1 Les points M recherchés sont M1(0 ; − 2 3 − 3) et
Soit MI − × 14 2 = 49 c’est-à-dire MI2 = 98.
2
4 M2 (0 ; 2 3 − 3)
L’ensemble # est le cercle de centre I et de rayon
7 2. 29 a) M appartient à % si et seulement si
1 1
MI2 − AB2 = k soit MI2 − × 144 = k
23 M appartient à # si, et seulement si, MI = 2. 4 4
  1 et MI2 = k + 36.
Or, MA ⋅ MB = −5 soit MI2 − AB2 = −5
4 b) • Si k < −36 MI2 < 0 : % = ∅.
1 • Si k = −36 MI2 = 0 : % est réduit au point I.
avec I milieu de [AB]. Donc 22 − AB2 = −5 soit
4 • Si k > −36 % est le cercle de centre I et de rayon
AB2 = 36 et AB = 36 = 6 cm. k + 36.
24 M appartient au cercle de centre I et de rayon 3  2  2
  30 a)  AB2 + AC2 = AB + AC d’où
si, et seulement si, MI = 3. Or MA ⋅ MB = k soit 
   
 
1 1 AB2 + AC2 = (AI + IB)2 + (AI + IC)2
MI2 − AB2 = k c’est-à-dire 9 − × 4 2 = k d’où 
  
  
4 4 = AI2 + 2AI ⋅ IB + IB2 + AI2 + 2AI ⋅ IC + IC2
k = 5. 
  
= 2AI2 + 2AI(IB + IC) + IB2 + IC2
Comme I est le milieu de [BC].
25 M appartient au cercle de centre I et de rayon 
    BC
 
8 cm si, et seulement si, MI = 8. Or MA ⋅ MB = k donc IB + IC = 0 et IB = IC =
2
1 1 1 2 1 2
MI2 − AB2 = k soit 64 − × 4 = k d’où k = 63. 2 2 2
On obtient AB + AC = 2AI + BC + BC
4 4 4 4

Chapitre 10  ★  Applications du produit scalaire 163

172909_Chap10_000-000.indd 163 30/07/2019 16:22:33


1 )
BD2 = AB2 + AD2 − 2AB × AD × cos(DAB
soit AB2 + AC2 = 2AI2 + BC2
2 2 2 2
soit BD = 6 + 4 − 2 × 6 × 4 × cos(60°) = 28
1
b)  4 + 5 = 2AI + × 62 soit AI2 = 11, 5
2 2 2
d’où BD = 28.
2
 = 180 − 60 = 120°
c)  ABC
d’où AI = 3, 4 cm.
d)  AC2 = BA2 + BC2 − 2BA × BC × cos(120°)
31 a)   1
A AC2 = 36 + 16 − 2 × 6 × 4 × − 
 2 
K J
AC2 = 76
I
B C d’où AC = 76 .

b)  37 a) D’après la formule d’Al-Kashi dans le triangle



  
  
AB + AC = (AI + IB)2 + (AI + IC)2
2 2 AVB :

   
   )
AB2 = VA2 + VB2 − 2 VA × VB × cos(AVB
AB2 + AC2 = 2AI2 + 2AI ⋅ IB + IB2 + AI2 + 2AI ⋅ IC + IC2

     AB2 = 81 + 25 − 2 × 9 × 5 × cos(60°)
AB2 + AC2 = 2AI2 + 2AI ⋅(IB + IC) + IB2 + IC2 1
AB2 = 106 − 90 ×
1 1 2
d’où AI2 = (AB2 + AC2 ) − BC2
2 4 AB2 = 61.
2 1 2 2 1 2 Donc AB = 61 et AB » 7, 8 milles.
soit AI = (3,5 + 2,5 ) − 4
2 4
et AI2 = 5, 25 donc AI  2, 3 cm 38 D’après la formule d’Al-Kashi dans le triangle
1 1 ABC :
De même BJ2 = (BA2 + BC2 ) − AC2 = 12, 56
2 4 )
BC2 = AB2 + AC2 − 2AB × AC × cos(BAC
donc BJ  3, 5 cm
BC2 ≈ 72 900 + 202 500 − 83111 ≈ 192 289
1 1
et CK 2 = (CA2 + CB2 ) − AB2 = 8 , 0625 D’où BC » 439  m.
2 4
donc CK  2, 8 cm
39 D’après la propriété d’Al-Kashi appliquée au
1 triangle ABC on obtient
32 AC2 = 22 + 4 2 − 2 × 2 × 4 × = 12
2 BC2 = AB2 + AC2 − 2AB × AC × cosa
soit 25 = 36 + 16 − 48 cos(α)
) = 25 + 9 − 49 1 36 + 16 − 25
33 Réponse (2) cos(ABC =− d’où cos(α) = = 0 , 5625 soit a  56°.
30 2 48

34 BC2 = 13 40 On utilise le théorème d’Al-Kashi dans le triangle


AB2 + AC2 − 2AB × AC × cos(60°) = 13 dessiné sur le billard.
Vincent a raison. 11002 = 14002 + 12002 − 2 × 1400 × 1200 × cos(α)
cos(α) ≈ 0 , 652 d’où α  49°.
35 D’après la formule d’Al-Kashi appliquée au
triangle ABC : 41 La formule d’Al-Kashi appliquée dans le triangle
)
BC2 = AB2 + AC2 − 2AB × AC × cos(BAC ABC donne :
soit BC2 = 40 − 12 = 38 BC = 38 BC2 = AB2 + AC2 − 2AB × AC × cos(A )
2 2 2
D’où cosA = AB + AC − BC = 134
36   2AB × AC 442
D C Donc A ≈ 72° avec la calculatrice.
CA2 + CB2 − AB2 444
De même, cos( C) = =
2CA × CB 612
60 ° donc C ≈ 43°.
A B On a donc B ≈ 180 − (72 + 43) ≈ 65°.

b) D’après la propriété d’Al-Kashi appliquée au 42 a)  AB2 = (−2)2 + (−4)2 = 20


triangle ABD : d’où AB = 2 5 .
164

172909_Chap10_000-000.indd 164 30/07/2019 16:23:12


AC2 = 52 + (−5)2 = 50 d’où AC = 5 2 cm. b) AC est la solution positive de l’équation
BC2 = 72 + (−1)2 = 50 d’où BC = 5 2 cm. x2 − 3 x − 28 = 0
b) La formule d’Al-Kashi appliquée au triangle ABC ∆ = 121 ; x1 = 7  ; x2 = −4 et donc AC = 7.
entraîne AB2 = CA2 + CB2 − 2CA × CB cos(ACB)
2 2 2
47 1. a)  AB2 = 12 + 62 = 37 d’où AB = 37
 = CA + CB − AB soit cos(ACB
d’où cosACB ) = 4 AC2 = 22 + 32 = 13 d’où AC = 13
2CA × CB 5
 ≈ 36 , 9°. BC2 = 4 2 + 4 2 = 32 d’où BC = 4 2
et ACB
2 2 2 b) La formule d’Al-Kashi appliquée au triangle ABC
 ) = BA + BC − AC = 1
De même cos(ABC permet d’écrire
2BA × BC 10 )
BC2 = BA2 + AC2 − 2AB × AC × cos(BAC
 
d’où ACB ≈ 71, 6° et BAC = 180 − (36 , 9 + 71, 6)  ) ≈ 0 , 41 et BAC
d’où cos(BAC  = 65, 8°
= 71, 6°.
2. a) Dans le triangle ACH rectangle en H
43 a) Dans le triangle DCE on a :  ) = CH d’où CH = AC × sin(CAH
sin(CAH  ) soit CH » 3, 3
AC
CE2 = CD2 + DE2 = 20 d’où CE = 2 5. 1
Dans le triangle BCF on a b) Aire (ABC) = × CH × AB ≈ 10 u.a
2
64 208 4 13
CF2 = CB2 + BF2 = 16 + = d’où CF= 
9 9 3   2 
48 a)  d1 : u(−3 ; 2), v(2 ; 3), A 1 ; .
Dans le triangle EAF, on a :  
 3 
16 52 2 13 b)  d2 : u(0 ; 1), v(1 ; 0), B(7 ; 3).
EF2 = AF2 + AE2 = +4= d’où EF =
9 9 3
b) La propriété d’Al-Kashi appliquée au triangle ECF 49 Réponse (2)
permet d’écrire EF2 = CE2 + CF2 − 2CE × CF cos(ECF )
2 2 2
50 a)  d2 d’équation x + 5 y − 4 = 0 est parallèle à
 ) = CE + CF − EF = 7
d’où cos(ECF la droite d1.
2CE × CF 65
 b)  d3 −5 x + y + 1 = 0 est perpendiculaire à d1.
on a donc ECF ≈ 30°
51 a) Une équation cartésienne de la droite d est
44 Dans le triangle HGK rectangle en G
de la forme 2 x + y + c = 0.
HK 2 =HG2 + GK 2 = a2 + 9a2 = 10a2 , donc
A(4 ; 1) appartient à d, donc 8 + 1 + c = 0 et c = - 9.
HG = a 10.
2 x + y − 9 = 0 est une équation cartésienne de d.
Dans le triangle JKF rectangle en F 
b)  n(2 ; 1) est un vecteur normal à d.
JK 2 = JF2 + KF2 = 5a2 , donc JK = a 5. c)  A(0 ; 9) et B(2 ; 5) sont des points de d.
Le théorème d’Al-Kashi dans le triangle HJK permet
d’écrire : 52 a) Une équation cartésienne de d est de la
HJ2 = HK 2 +JK 2 − 2HK × JK cos(α) forme −3 x − 5 y + c = 0.
soit a2 = 10a2 + 5a2 − 2a 10 × a 5 cos(α) A(4 ; 2) appartient à (d), d’où −12 − 10 + c = 0 soit
14 a2 7 c = 22. Une équation cartésienne de d est
donc cos(α) = = , cos(α) ≈ 0 , 99
10 2a2 5 2 −3 x − 5 y + 22 = 0.
donc α ≈ 8°. b) L’équation réduite de d est 5 y = 3 x − 22 soit
3 22
y =− x+ .
45 PB2 = PF2 + FB2 = 100 , 36 5 5 
PD2 = FB2 + (0 , 6 − 0 , 4)2 = 100 , 04 c) Un vecteur directeur de d est v(5 ; − 3) sa pente h
Le théorème d’Al-Kashi dans le triangle PDB donne : 3
est .
DB2 = PD2 + PB2 − 2PD × PB × cos( P) 5
200 , 24  
cos(
P) ≈ ≈ 0 , 999 d’où 
P ≈ 2° . 53 AB(−6 ; 2) et n(1 ; 3).
200 , 4   
AB(−6 ; 2) = −6 + 6 = 0 donc n est orthogonal àAB
46 a) D’après la formule d’Al-Kashi appliquée au Une équation cartésienne de la droite (AB) est de la
triangle ABC on a : forme x + 3 y + c = 0.
 ) = BC2
AB2 + AC2 = 2AB × AC × cos(BAC A(5 ; 1) appartient à (AB) donc 5 + 3 + c = 0.
d’où 9 + x2 − 3 x = 37 soit x2 − 3 x − 28 = 0. Une équation cartésienne de (AB) est x + 3 y − 8 = 0.

Chapitre 10  ★  Applications du produit scalaire 165

172909_Chap10_000-000.indd 165 30/07/2019 16:24:40


 
54 a) Un vecteur normal à d1 est n1(1 ; 2). 60 a)  n1(1 ; 1) est un vecteur normal à d1
 
Un vecteur normal à d2 est n2 (−3 ; − 6). d2 est perpendiculaire à d1 donc n1 est un vecteur
    directeur de d2 .
b)  n2 = −3n1. Les vecteurs n1 et n2 sont colinéaires,
donc les droites d1 et d2 sont parallèles. Une équation cartésienne de d2 est de la forme
x − y + c = 0.
  4 3 A(2 ; 1) appartient à d2 donc 2 − 1 + c = 0
55 a)  n1− ;  est normal à la droite d1.
 5 5  d’où c = −1

n2 (3 ; 4) est normal à la droite d2 . Une équation cartésienne de d2 est donc :
  4 3 x − y − 1 = 0.
b)  n1 ⋅ n2 = − × 3 + × 4 = 0
5 5 b) Les coordonnées du point d’intersection des
c) Les vecteurs n1 et n2 sont orthogonaux, les droites  x + y = 1
d1 et d2 sont donc perpendiculaires. droites d1 et d2 sont solutions du système 
 x − y = 1
soit x = 1 et y = 0

56 AB(−6 ; 3) est un vecteur normal à la hauteur d d1 et d2 sont sécantes en M(1 ; 0).
issue de C dans le triangle ABC.
Une équation cartésienne de d est de la forme 61 a)  d1 d2
4
−6 x + 3 y + c = 0. A
3
C(−1 ; − 3) appartient à d donc :
2
−6 × (−1) + 3 × (−3) + c = 0 , soit c = 3.
1
Une équation cartésienne de d est −6 x + 3 y + 3 = 0 ,
soit −2 x + y + 1 = 0. 0 1 2 3 4
 B
57 AB(−3 ; 1) est un vecteur normal à la médiatrice D
C
d du segment [AB]. Une équation de d est : 
− 3 x + y + c = 0. b)  BC(5 ; − 1) est un vecteur normal à d1
Le milieu I de [AB] a pour coordonnées Une équation cartésienne de d1 est de la forme
 5 + 2 −2 − 1 7 3 5 x − y + c = 0.

 2
;  , soit  ; − 
2  2 2 A appartient à d1 d’où
7 3 10 − 3 + c = 0 soit c = −7.
I appartient à d donc −3 × − + c = 0 , soit
2 2 Une équation cartésienne de d1 est 5 x − y − 7 = 0.
c = 12. 
Une équation cartésienne de d est −3 x + y + 12 = 0. c) Un vecteur normal à d2 est n2 (−5 ; 1).
   
 n2 = −BC, BC et n2 sont colinéaires donc les droites
58 a)  AB(3 ; − 1) est un vecteur normal à d d1 et d2 sont parallèles.

Une équation cartésienne de d est de la forme d) Le vecteur BC(5 ; − 1) est un vecteur normal à la
3 x − y + c = 0. médiatrice [BC]. d a donc une équation de la forme
A(1 ; 3) appartient à d donc 3 − 3 + c = 0. 5x − y + c = 0
D’où c = 0. 3 3
Or le milieu I ; −  de [BC] appartient à d, ainsi
Une équation cartésienne de d est 3 x − y = 0. 2 2
b) Une équation cartésienne de la parallèle à d qui 15  3 
− −  + c = 0 et c = - 9.
passe par B est de la forme 3 x − y + c = 0 ; B(4 ; 2) 2  2 
appartient à cette droite, donc 12 − 2 + c = 0 soit d a pour équation 5 x − y − 9 = 0.
c = −10. 
62 a)  n(−3 ; 1) est un vecteur directeur de d1, c’est
Une équation de cette droite est 3 x − y − 10 = 0.
un vecteur normal à d2 est :

59 a)  n2 (2 ; − 1) est normal à la droite d2 . −3 x + y + c = 0
 
b)  n1 ⋅ n2 = 1× 2 + 2 × (−1) = 0 donc d1 et d2 sont A(2 ; 6) ∈ d2 donc −6 + 6 + c = 0
perpendiculaires. Une équation de d2 est −3 x + y = 0.
c) Les coordonnées de M point d’intersection de d1 et b) Les coordonnées de d1 sont solution du système
d2 sont solutions du système :  x + 3 y − 7 = 0

 x + 2 y − 8 = 0 −3 x + y = 0

2 x − y + 4 = 0  7 21
On trouve H ; .
On trouve M(0 ; 4). 10 10 

166

172909_Chap10_000-000.indd 166 30/07/2019 16:26:33


c) La distance de A à d1 est la distance AH 127
On a donc − 5 x − 6(− 9 , 5 + 6 x) = − 6 , 5 et x =
7  2
 21  2
169 13 10 82
AH =  − 2 +  − 6 = = 127  17
10  10  10 10 On obtient y = − 9 , 5 + 6 ×  =− .
 82  82
 127 17 
63 a)  BC(−4 ; − 1) est un vecteur normal à la Ainsi K  ; − 
 82 82 
médiatrice d issue de A. Une équation cartésienne de
2
 127   2
la droite d est de la forme −4 x − y + c = 0. c)  KA = 5 −  + 2 + 17  = 56 425
A appartient à d d’où −4 × 2 − (−1) + c = 0 soit
 82   82  3 362
c = 7. Une équation de la droite d est donc  2 2
127   17  56 425
−4 x − y + 7 = 0 soit 4 x + y − 7 = 0. KB = − 1 −  + 3 +  =
  82   82  3 362
b) Un vecteur normal à d2 est AC(−2 ; 3).
2 2
Une équation de la droite d2 est de la forme  27   17  56 425
KC = 0 −  + − 4 +  =
−2 x + 3 y + c = 0. B(4 ; 3) appartient à d2 , d’où  82   82  3 362
−8 + 9 + c = 0 et c = −1. Ainsi KA = KB = KC.
Une équation de la droite d2 est −2 x + 3 y − 1 = 0. Le point K est bien le centre du cercle circonscrit au
c) Les coordonnées du point d’intersection des triangle ABC.
droites d1 et d2 sont solution du système
4 x + y = 7 2
  1 2  3 
−2 x + 3 y = 1 65 a)  OA =   +   = 1, donc A appartient
 2   2 
10 9  à #.
On trouve H ; .
 7 7   1 3 
 b) Un vecteur normal à T est OA  ;
d)  CH vecteur directeur de la droite (CH) est .
 2 2 
 10 5   
CH ; − . AB(2 ; 4) est un vecteur directeur de n(1 ; 3 ) est un vecteur normal à d.
 7 7   10  1   
5
la droite (AB). CH ⋅ AB = × 2 − × 4 d’où OA = n ; OA et n sont colinéaires donc les droites
  7 7 2
CH ⋅ AB = 0. (CH) est perpendiculaire à (AB) ; c’est T et d sont parallèles.
la troisième hauteur du triangle ABC.
2
 66 a)  AB = 12 + 3 = 2 donc B appartient au
64 a)  AB(−6 ; 1) est un vecteur normal à la média- cercle #.
trice d1 du segment [AB]. 
 5
( )
b)  AB 1 ; 3 est un vecteur normal à T et B appar-
C′2 ;  est le milieu du segment [AB]. tient à T.
 2
Une équation cartésienne de d1 est de la forme Une équation cartésienne de T est de la forme
−6 x + y + c = 0 et −12 + 2, 5 + c = 0 soit c = 9 , 5. x + 3 y + c = 0 et 4 + 2 + 3 ( ) 3 + c = 0 soit
La droite d1 a donc pour équation −6 x + y + 9 , 5 = 0 c = −7 − 2 3.
La médiatrice d2 du segment [AC] a pour vecteur nor- Une équation cartésienne de T est :

mal le vecteur AC(−5 ; − 6). x + 3 y − 7 − 2 3 = 0.
La droite d2 a donc une équation de la forme 4 + x′
c) B¢(x¢ ; y¢). A est le milieu de [BB′ ], soit 3 =
−5 x − 6 y + c = 0 2
2 + 3 + y′
5  et x ′ = 2 et 2 = et y ′ = 2 − 3.
Or le milieu J ; − 1 du segment [AC] appartient à 2
2  T2 est parallèle à T1 donc une équation cartésienne
5 de T2 est de la forme x + 3 + c ′ = 0 avec
d2, ainsi − 5 × − 6 ×(−1) + c = 0 et c = 6 , 5.
2
La droite d2 a donc pour équation ( )
2 + 3 2 − 3 + c ′ = 0 soit c ′ = 1 − 2 3.

− 5 x − 6 y + 6 , 5 = 0. Une équation de T2 est x + 3 y + 1 − 2 3 = 0.


b) Les coordonnées du point K vérifient le système :
− 6 x + y = − 9 , 5 67 a) Cercle de centre A(3 ; 1) et de rayon 3.

− 5 x − 6 y = − 6 , 5 b) Cercle de centre A(0 ; − 5) et de rayon 2.
 y = − 9 , 5 + 6 x c) Cercle de centre A(−2 ; 5) et de rayon 4.
qui équivaut à 
− 5 x − 6 y = − 6 , 5 d) Cercle de centre A(0 ; 0) et de rayon 5.

Chapitre 10  ★  Applications du produit scalaire 167

172909_Chap10_000-000.indd 167 30/07/2019 16:27:55


68 a)  (2 − 2)2 + (4 + 1)2 = 25, Ida a raison. d’où x2 − 2 x + y 2 − 4 y − 5 = 0 équivaut à
b)  (5 − 2)2 + (3 + 1)2 = 25, B appartient à #. ( x − 1)2 + ( y − 2)2 − 1 − 4 − 5 = 0
(7 − 2)2 + (−2 + 1)2 = 26 , C n’appartient pas à #. c’est-à-dire ( x − 1)2 + ( y − 2)2 = 10.
(6 − 2)2 + (2 + 1)2 = 25, D appartient à #.
75 a)  x2 − 10 x = ( x − 5)2 − 25
69 a)  ( x − 2)2 + ( y − 5)2 = 9 y 2 + 4 y = ( y + 2)2 − 4
b)  ( x + 3)2 + ( y − 2)2 = 1 b)  x2 + y 2 − 10 x + 4 y + 23 = 0 équivaut à
c)  ( x + 1)2 + ( y + 4)2 = 6 ( x − 5)2 + ( y + 2)2 − 25 − 4 + 23 = 0
d)  x2 + ( y − 1)2 = 8 soit ( x − 5)2 + ( y + 2)2 = 6
# est le cercle de centre A(5 ; − 2) et de rayon 6.
70 Cercle bleu : ( x − 1)2 + ( y − 2)2 = 1.
Cercle noir : ( x − 3, 2)2 + ( y − 2)2 = 1. 76 a)  x2 − 4 x = ( x − 2)2 − 4
Cercle rouge : ( x − 5, 4)2 + ( y − 2)2 = 1. y 2 + 6 y = ( y + 3)2 − 9
Cercle jaune : ( x − 2,1)2 + ( y − 1)2 = 1. x2 + y 2 − 4 x + 6 y + 12 = 0 peut s’écrire
Cercle vert : ( x − 4 , 3)2 + ( y − 1)2 = 1. ( x − 2)2 + ( y + 3)2 = 1
^ est le cercle de centre Ω(2 ; − 3) et de rayon 1.
71 a)  AB = (5 − 1)2 + (0 − 1)2 = 17 b)  x2 + 10 x = ( x + 5)2 − 25
Une équation cartésienne du cercle de centre A qui
y 2 − 4 y = ( y − 2)2 − 4
passe par B est ( x − 1)2 + ( y − 1)2 = 17
x2 + y 2 + 10 x − 4 y + 29 = 0 peut s’écrire
b)  AB= (3 − 2)2 + (1 − 0)2 = 2 ( x + 5)2 + ( y − 2)2 = 0 ^ n’est pas un cercle.
Une équation cartésienne du cercle de centre A qui c)  x2 + 8 x = ( x + 4)2 − 16
passe par B est ( x − 2)2 + y 2 = 2. y 2 − 10 y = ( y − 5)2 − 25
c)  AB = 52 + (−5)2 = 5 2 x2 + y 2 + 8 x − 10 y + 42 = 0 peut s’écrire
Une équation cartésienne du cercle de centre A qui ( x + 4)2 + ( y − 5)2 = −1
passe par B est ( x + 1)2 + ( y − 3)2 = 50. ^ n’est pas un cercle.
d)  AB = 72 + 12 = 50 d)  x2 − 12 x = ( x − 6)2 − 36
Une équation cartésienne du cercle de centre A qui x2 + y 2 − 12 x = 0 peut s’écrire ( x − 6)2 + y 2 = 36
passe par B est x2 + ( y − 4)2 = 50. ^ est le cercle de centre Ω(6 ; 0) et de rayon 6.

 3 77 a)  x2 − 2 x = ( x − 1)2 − 1
72 I1 ;  et AB = 17
 2  y 2 − 4 y = ( y − 2)2 − 4
Une équation du cercle de diamètre [AB] est : x2 + y 2 − 2 x − 4 y + 3 = 0 peut s’écrire
2
 3 ( x − 1)2 + ( y − 2)2 = 2.
( x − 1)2 +  y −  = 17.
 2 # est le cercle de centre A(1 ; 2) et de rayon 2.
1 9 b)  (2 − 1)2 + (3 − 2)2 = 2. B appartient à #
73 I ; −  et AB = 2. 
 2 2 c) Un vecteur normal à d est n(1 ; 1).
 
Une équation du cercle de diamètre [AB] est : AB(2 − 1 ; 3 − 2) = n(1 ; 1)
 2  2
 x − 1  +  y + 9  = 2. d est bien tangente à # en B.
 2   2 
78 x2 + y 2 − 2 x − 6 y + 8 = 0
74 1. a)  I(1 ; 2) est le centre du cercle.
a)  x2 − 2 x = ( x − 1)2 − 1
Son rayon est IA = 32 + (−1)2 = 10.
y 2 − 6 y = ( y − 3)2 − 9.
b) Une équation de # est ( x − 1)2 + ( y − 2)2 = 10. x2 + y 2 − 2 x − 6 y + 8 = 0 peut s’écrire
 
2. a)  MA ⋅ MB = (4 − x)(−2 − x) + (3 − y)(1 − y) ( x − 1)2 + ( y − 3)2 = 2
= −8 + x 2 − 2 x + 3 + y 2 − 4 y # est le cercle de centre A(1 ; 3) et de rayon 2.
= x2 − 2 x + y 2 − 4 y − 5 b)  (0 − 1)2 + (4 − 3)2 = 2 donc B appartient à #.
  
b) M appartient à # si, et seulement si, MA ⋅ MB = 0 c)  AB(− 1 ; 1) est un vecteur normal à T, une équation
soit x2 − 2 x + y 2 − 4 y − 5 = 0. de T est de la forme −x + y + c = 0. B Î T donc
Par la méthode de complétion des carrés c = −4
x2 − 2 x = ( x − 1)2 − 1 y 2 − 4 y = ( y − 2)2 − 4 Une équation de T est −x + y − 4 = 0.

168

172909_Chap10_000-000.indd 168 30/07/2019 16:29:22


 84 1. a) Une équation cartésienne de d est de la
79 n(2 ; − 1) est un vecteur normal à d.
 forme 2 x + y + c = 0.
n′(1 ; 3) est un vecteur normal à d ¢.
  b) d passe par A(−3 ; 2) d’où −6 + 2 + c = 0 soit
n et n¢ ne sont pas colinéaires donc les droites d et
c = 4.
d ¢ non parallèles ne sont pas les tangentes à un
même cercle en des points opposés. c) Une équation de la droite d est 2 x + y + 4 = 0.
2. a) Une équation cartésienne de d ¢ est de la forme
80 1. C 2. B 3. A 4. D −x + 2 y + c = 0. B(1 ; − 4) appartient à d ¢ donc
−1 − 8 + c = 0 soit c = 9.
81 1. A × B 2. D 3. A × D Une équation cartésienne de d ¢ est −x + 2 y + 9 = 0.
b)  a x + b y + c = 0 est une équation cartésienne
82 1. Vrai. En effet, d’après la formule d’Al-Kashi 
de d ; u(−b ; a) est un vecteur directeur de d.
dans le triangle ABC, 
CD(−5 ; 3) est un vecteur directeur de (CD).
)
BC2 = AB2 + AC2 − 2 × AB × AC × cos(BAC Une équation cartésienne de (CD) est de la forme
)
202 = 102 + 122 − 2 × 10 × 12 cos(BAC 3x + 5 y + c = 0
) = 102 + 122 − 202 13 C(3 ; 2) appartient à (CD) d’où 9 + 10 + c = 0 soit
Ainsi, cos(BAC =−
2 × 10 × 12 20 c = −19. Une équation cartésienne de (CD) est
 ≈ 131°.
Avec la calculatrice, on obtient BAC 3 x + 5 y − 19 = 0.
2. Faux. En effet, l’équation de #1 s’écrit 85 a)  AB2 = (5 − 2)2 + (1 − (−3))2 = 25
2 2
( x − 2) + ( y + 1) = 11 et donc le centre de #1 est d’où AB = 5 le rayon du cercle # est égal à 5.
le point A(2 ; − 1). b) Une équation de # est ( x − 2)2 + ( y + 3)2 = 25
Or, le centre de # 2 est le point B(2 ; 1). c)  (2 − 2)2 + (2 + 3)2 = 25 ; E appartient à #.
3. Vrai. En effet, on note H le projeté orthogonal de O (4 − 2)2 + (3 + 3)2 = 85 ; F n’appartient pas à #.
sur la droite d.
(7 − 2)2 + (− 3 − 3)2 = 61 ; G n’appartient pas à #.
H a des coordonnées de la forme x ; − x + 2 . ( ) (6 − 2)2 + (0 + 3)2 = 25 ; H appartient à #.
2
OH2 = x2 + −x + 2 ( ) = 2 x2 − 2 2 x + 2
OH2 est minimum c’est-à-dire OH est minimum

S’entraîner
2
lorsque x = (c’est l’abscisse du sommet de la
2
parabole d’équation y = 2 x2 − 2 2 x + 2).
Donc H, projeté orthogonal de O sur d, a pour coor-
 2 2 
données  ; . 87 Voici la partie de l’algorithme concernée par les
 2 2 
adaptations.
1 1
OH2 = + = 1 et OH = 1, donc H appartient au Si p = 0 alors
2 2
cercle de centre O et de rayon 1. Donc la droite d est a ← ( xA + xB ) / 2
 2 2  b ← ( yA + yB ) / 2
tangente au cercle # en H ; .
 2 2  Afficher « G est réduit au point I(a ; b)  »
Sinon
)
83 a)  BC2 = AB2 + AC2 − 2AB × AC × cos(BAC Afficher «  G est le cercle de centre
= 9 + 12, 25 − 21cos(50°) I(a ; b) et de rayon p  »
Soit BC » 7, 75 » 2, 8 cm. Fin Si
 = 180 − 2 × 70 = 40°
b)  ACB 88  
)
AB2 = CA2 + CB2 − 2CA × CB × cos(ACB
d ← ( x − xA )2 + ( y − yA )2
= 9 + 9 − 18 cos(40°) Si d = r
soit AB2 » 4 , 211 soit AB » 2,1 cm. Afficher « M appartient au cercle »
c)  AC2 = BC2 + BA2 − 2BC × BAcos(β) Sinon
BC2 + BA2 − AC2 1 Afficher « M n’appartient pas au cercle »
soit cos(β) = = d’où β = 60°.
2BC × BA 2 Fin Si

Chapitre 10  ★  Applications du produit scalaire 169

172909_Chap10_000-000.indd 169 30/07/2019 16:30:32


90 a)  OM2 − R2 = (OM + R)(OM − R) 3 5
2 2
  > 1 donc d est extérieure à #.
OM + R > 0 donc OM - R , c’est-à-dire MA × MB 5
est du signe de OM - R. 94 Le cercle #¢ a pour centre B(2 ; − 2) et pour
b) 1er cas : M est à l’extérieur du disque de frontière rayon 3.
 
#, OM > R et MA ⋅ MB > 0. AB = (2 − 1)2 + (−2 + 1)2 = 2
2e cas : M appartient au cercle #, OM = R et et R + R ′ = 2 + 3 = 5.
 
MA ⋅ MB = 0. Or, AB < R + R ′ donc les cercles # et #¢ sont
3e cas : M appartient au disque de frontière # sécants.
 
(exclue) ; OM < R et MA ⋅ MB < 0.  
95 En décomposant AB et AD à l’aide de la rela-
91 La puissance du point M par rapport au cercle tion de Chasles on obtient
   
# est MO2 - r 2 soit MO2 - 16. AB2 + AD2 = (AC + CB)2 + (AC + CD)2
a)  MO2 − 16 = 20 équivaut à MO2 = 36 soit MO = 6   
= 2AC2 + 2AC(CB + CD) + CB2 + CD2
% a est le cercle de centre O et de rayon 6.
1
b)  MO2 − 16 = 16 soit MO2 = 32 et MO = 4 2 = 2AC2 + BD2
2
% b est le cercle de centre O et de rayon 4 2.   
(B milieu de [CD] donc CB + CD = 0 et
c)  MO2 − 16 = − 7 soit MO2 = 9 et MO = 3 1
% c est le cercle de centre O et de rayon 3. CB = CD = BD ).
2
d)  MO2 − 16 = −16 soit MO2 = 0 1
D’où 4 + AD2 = 2 × 4 2 + × 36
2
% d est réduit au point 0. 2
2
soit AD = 34 et AD = 34.
Ea
De même dans le triangle ACE
Eb 1
AC2 + AE2 = 2AD2 + CE2
2
Ec 36
soit 16 + AE2 = 68 + .
O 2
2
AE = 70 et AE = 70.

96 Dans le triangle MAC en décomposant MA et

MC à l’aide de la relation de Chasles on a
   
MA2 + MC2 = (MO + OA)2 + (MO + OC)
  
92 a)  AM2 = ( x − 2)2 + ( y − 1)2  = 2MO2 + OA2 + OC2 + 2MO(OA + OC)
 
b)  y = 1 − x OA + OC = 0 car O milieu de [AC ], de plus
c)  AM2 = ( x − 2)2 + (1 − x − 1)2 = 2 x2 − 4 x + 4 1
OC = OA = AC.
−(−4) 2
d)  AM2 est minimum pour x = =1 1
2×2 Donc MA + MC2 = 2MO2 + AC2 .
2

c) Les coordonnées de M projeté orthogonal de A sur 2


De même dans le triangle MDB on obtient :
d sont (1 ; 0).
1
 MD2 + MB2 = 2MO2 + BD2 .
93 a)  n(2 ; − 1) est un vecteur normal à d. 2
Or les diagonales d’un rectangle ont la même lon-
Une équation de la perpendiculaire à d qui passe par
gueur donc AC = BD et MA2 + MC2 = MB2 + MD2 .
O a pour équation −x − 2 y + c = 0 , avec
0 − 0 + c = 0. Les coordonnées du projeté orthogo- 
97 a) Dans le triangle ABC en décomposant BA et
nal de O sur d sont les solutions du système 
2 x − y = −3 6 3 BC à l’aide de la relation de Chasles on obtient
    
 soit x = − y = BA2 + BC2 = (BI + IA)2 + (BI + IC)2
 x + 2 y = 0 5 5
1
 6 2  3 2 = 2BI2 + AC2 .
La distance de O à d est OB = −  +   soit 2
 5   5  De même dans le triangle ADC on obtient
3 5
OB = ≈ 1, 34. 1
5 DA2 + DC2 = 2DI2 + AC2 .
2
170

172909_Chap10_000-000.indd 170 30/07/2019 16:31:34


b) On a  2
101 a)  AM = 4 équivaut à AM2 = 4 c’est-à-dire
AB2 + BC2 + CD2 + DA2 = 2BI2 + 2DI2 + AC2
  
  AM = 2. Donc l’ensemble cherché est le cercle # de
= 2(BJ + JI)2 + 2(DJ + JI)2 + AC2 centre A et rayon 2 cm.
  
     
= 2(BJ2 + JI2 + 2BJ ⋅ JI) + 2(DJ2 + JI2 + 2DJ ⋅ JI) + AC2 b)  BM ⋅ BC = 0 signifie que les vecteurs BM et BC
  
 
= 2BJ2 + 2DJ2 + 4BJ ⋅ JI + 4DI ⋅ JI + 4 IJ2 + AC2 sont orthogonaux. Donc l’ensemble cherché est la
BD2 DB2     droite d perpendiculaire à la droite (BC) en B, c’est
=2 +2 + 4 JI ⋅ (BJ + DJ) + 4 IJ2 + AC2 .
donc la droite (AB).
 4 
 4        
Or BJ + DJ = 0 donc 4 JI(BJ + DI) = 0 c)  CM ⋅ CD = 2 équivaut à CH ⋅ CD = 2 où H est le
On a donc projeté orthogonal de M sur la droite (CD).
 
AB2 + BC2 + AD2 + DC2 = BD2 + AC2 + 4 IJ2 . 2 > 0 donc les vecteurs colinéaires CH et CD sont
 
c) Donc comme IJ2 > 0 on a de même sens et CH ⋅ CD = 2 équivaut à
AB2 + BC2 + AD2 + DC2 > BD2 + AC2 . 2
CH × CD = 2 c’est-à-dire CH = = 1.
2. La somme AB2 + BC2 + AD2 + AC2 est minimale CD
Donc l’ensemble cherché est la droite d ¢ perpendicu-
quand IJ = 0 soit quand les diagonales du quadrila-
laire à la droite (CD) en H milieu de [CD].
tère ont même milieu.   1
ABCD est alors un parallélogramme. d)  MA ⋅ MC = 1 équivaut à MI2 − AC2 = 1, où I est
4
1 2

98  
( )
le milieu de [AC ], c’est-à-dire à MI2 − 2 2 = 1
4
soit MI2 = 3.
Ainsi MI = 3 et l’ensemble cherché est le cercle #¢
de centre I et de rayon 3.

102 a)  hA hauteur issue de A a pour vecteur normal


 
BC(−6 ; − 2). On choisira n(3 ; 1) comme vecteur
normal à hA , une équation de hA est 3 x + y + c = 0
99 AC = 2 2, AD = 4, AE = 4 2. d’où comme A Î hA  :

BAE = 3 × 45° = 135°. 3 − 2 + c = 0 , c = −1
La formule d’Al-Kashi appliquée dans le triangle ABE Une équation hA est 3 x + y − 1 = 0.
 
donne : b)  AC(−3 ; 3), on choisira n(−1 ; 1) comme vecteur
) normal à hB hauteur issue de B, B Î hB donc une
BE2 = AB2 + AE2 − 2AB × AE cos(BAE
équation de hB est −x + y + c = 0 et
BE2 = 4 + 32 − 16 2 cos(135°)
−4 + 3 + c = 0 , d’où c = 1.
BE2 = 52, BE = 2 13.
Une équation de hB est −x + y + 1 = 0.
c) Les coordonnées de H sont solutions du système :
100 a) Les coordonnées du point d’intersection de
3 x + y − 1 = 0
# et G sont solutions du système 
−x + y + 1 = 0
 y = x  y = x
 soit   1 1 
 x2 + y 2 = 1  x2 + x − 1 = 0 On trouve H ; .
   2 2 
La solution positive de x2 + x − 1 = 0 est
−1 + 5 103 a)  OA = 32 + 4 2 = 5, donc A appartient au
x= ≈ 0 , 62 cercle #.
2 
b)  OA(3 ; 4) est un vecteur normal à T. Une équation
−1 + 5
d’où y = ≈ 0 , 79 et donc A(0 , 62 ; 0 , 79). cartésienne de T est de la forme 3 x + 4 y + c = 0.
2 A Î T donc 9 + 16 + c = 0 d’où c = −25.
b) Le triangle IOA est isocèle en O
Une équation cartésienne de T est 3 x + 4 y − 25 = 0.
OI = OA = 1 et AI = (1 − 0 , 62)2 + (−0 , 79)2 ≈ 0 , 88 
d’après la formule d’Al-Kashi 104 T1 a pour vecteur normal AB(−2 ; 2, 5).

OA2 + OI2 − AI2 T2 a pour vecteur normal CD(3, 75 ; 3).
cos(I
OA) = ≈ 0 , 62  
2OA × OI AB ⋅ CD = −2 × 3, 75 + 2, 5 × 3 = 0.
 
 = 51, 7°.
d’où IOA AB ^ CD donc T1 et T2 sont perpendiculaires.

Chapitre 10  ★  Applications du produit scalaire 171

172909_Chap10_000-000.indd 171 30/07/2019 16:32:58


 109 On cherche les coordonnées de C projeté ortho-
105 a)  AA ¢ est un vecteur normal à d et I appar-
gonal de A sur d.
tient à d. 
 n(1 ; 1) est un vecteur normal à d, c’est un vecteur
b) Un vecteur directeur de d est u(1 ; 1).
directeur de la droite (CA).
Il est normal à (AA ′). Une équation cartésienne de
Une équation cartésienne de la droite (CA) est
(AA ′) est donc de la forme x + y + c = 0 et
x − y + c = 0. A ∈ (CA) donc 5 − 1 + c = 0 soit
A(2 ; 4) ∈ (AA ′) d’où 2 + 4 + c = 0.
c = −4.
Une équation cartésienne de (AA ′) est Une équation de (CA) est x − y − 4 = 0.
x + y − 6 = 0.  x − y = 4
Les coordonnées de I milieu de [AA ′ ] sont les solu- Les solutions du système  soit x = 4 et
 x + y = 4,
tions du système y = 0, sont les coordonnées de A ′(4 ; 0) le projeté
 x − y + 1 = 0 5 7 orthogonal de A sur d.
 soit I ; .
 x + y − 6 = 0  2 2 
AA ′ = 12 + 12 = 2 .
2 + xA′ 5 4 + yA′ 7 Une équation du cercle # est ( x − 5)2 + ( y − 1)2 = 2.
On a donc = soit xA′ = 3 et =
2 2 2 2

soit yA = 3 d’où A (3 ; 3). 110 x2 + y 2 − 2 x − 6 y + 8 = 0
 a)  x2 − 2 x = ( x − 1)2 − 1
106 1. a)  AB(3 ; − 2) est normal à d1. y 2 − 6 y = ( y − 3)2 − 9
1  x2 + y 2 − 2 x − 6 + 8 = 0 peut s‘écrire
I ; 2 milieu de [AB] appartient à d1 d’où d1 a
 2 
3 ( x − 1)2 + ( y − 3)2 = 2.
pour équation 3 x − 2 y + c = 0 et −4+c = 0
5 2 # est le cercle de centre A(1 ; 3) et de rayon 2.
soit c = . b)  (0 − 1)2 + (4 − 3)2 = 2 donc B Î #.
2 
5 c)  AB(− 1 ; 1) est un vecteur normal à T, une équation
Une équation cartésienne de d1 est 3 x − 2 y − = 0.
 2 de T est −x + y + c = 0.
b)  AB est normal à d2, une équation cartésienne de B Î T, donc 0 + 4 + c = 0.
d2 est 3 x − 2 y + c = 0. Une équation de T est −x + y − 4 = 0.
O(0 ; 0) ∈ d2 , donc c = 0.
Une équation cartésienne de d2 est 3 x − 2 y = 0. 111 a)  x2 − 2 x = ( x − 1)2 − 1

c)  AB est normal à d3 , une équation de d3 est de la y 2 − 4 y = ( y − 2)2 − 4
forme 3 x − 2 y + c = 0. Une équation de # est donc ( x − 1)2 + ( y − 2)2 = 2.
B(2 ; 1) ∈ d3 donc 6 − 2 + c = 0 et c = −4. # a pour centre A(1 ; 2) et rayon r = 2.
Une équation cartésienne de d3 est 3 x − 2 y − 4 = 0. b)  H( x ; mx + 5) ∈ d .
2. a) Les droites d1, d2 et d3 ont même vecteur nor- AH2 = ( x − 1)2 + (mx + 3)2

mal n(3 ; − 2), elles sont donc parallèles. = x 2 − 2 x + 1 + m2 x 2 + 6 m x + 9
b) Les droites d1, d2 et d3 sont perpendiculaires à la = (m2 + 1)x2 + (6m − 2)x + 10
droite (BC), elles sont donc parallèles. −(6m − 2)
AH2 est minimal pour x =
2(m2 + 1)
107 On note ( x ; y) les coordonnées de M. −3m + 1 −3m2 + m
  soit x = et y = +5
AM( x + 1 ; y − 1) AB(4 ; − 2) m2 + 1 m2 + 1
 
AM ⋅ AB = 0 équivaut à 4( x + 1) − 2( y − 1) = 0 −3m2 + m + 5m2 + 5 2m 2 + m + 5
soit y = =
c’est-à-dire 4 x − 2 y + 6 = 0 soit 2 x − y + 3 = 0. m2 + 1 m2 + 1
L’ensemble cherché est donc une droite (c’est la per- c) La position de d par rapport à # dépend du signe
pendiculaire en A à la droite (AB) ). de AH2 - 2 soit (m2 + 1)x2 + (6m − 2)x + 8.
 ∆ = (6m − 2)2 − 4(m2 + 1) × 8
108 BA(4 ; − 1) est un vecteur normal à la tangente = 36m2 − 24 m + 4 − 32m2 − 32
T en A au cercle de centre B. = 4 m2 − 24 m − 28
Une équation cartésienne de T est 4 x − y + c = 0 ; = 4(m2 − 6m − 7)
A(3 ; 2) ∈ T donc 12 − 2 + c = 0 soit c = −10. •  ∆ = 0 pour m1 = −1 et m2 = 7
Une équation de T est 4 x − y − 10 = 0. d est tangente à #.
172

172909_Chap10_000-000.indd 172 30/07/2019 16:35:01


•  ∆ > 0 m < −1 ou m > 7 2 4 
d’où A1(2 ; 0) et A2  ; .
d est sécante à #.  5 5 
•  ∆ < 0 −1 < m < 7   8 4 
2. a)  A1A2 − ;  est normal à d.
d ne coupe pas #.  5 5 
6 2
112 # a pour centre Ω(−5 ; 1) et rayon r = 2. I milieu de [A1A2 ], ainsi I ; .
 5 5 
#¢ a pour centre O′(0 ; − 3) et rayon r ′ = 3. Une équation cartésienne de d est de la forme
b)  ΩO′2 = (0 + 5)2 + (−3 − 1)2 = 25 + 16 = 41 8 4
− x + y + c = 0.
ΩΩ′ = 41 ≈ 6 , 4. 5 5
ΩΩ′ > r + r ′ les cercles sont disjoints. 48 8 8
I Î d donc − + + c = 0 soit c = .
25 25 5
Une équation de d est −8 x + 4 y + 8 = 0.
4
#¢ b)  I1(1 ; 0) donc −8 + 0 + 8 = 0 I1 Î d.
3
I2 (2 ; 2) donc −16 + 8 + 8 = 0 I2 Î d.
2
O¢ donc d est la droite (I1I2 ).
1 
I1I2 (1 ; 2) est un vecteur directeur de (I1I2 ).
- 8 - 7- 6 - 5 - 4 - 3 - 2 - 1 0 1 2 Une équation de (I1I2 ) est de la forme 2 x − y + c = 0.
-1
I1 ∈ (I1I2 ) donc 2 + c = 0 soit c = −2.
-2 Une équation de (I1I2 ) est 2 x − y − 2 = 0 soit
# O¢
-3 −8 x + 4 y + 8 = 0.
-4
-5 114 # a pour équation ( x − 2)2 + ( y − 2)2 = 1.
dm a pour équation y = mx + 2 − 2m donc
M( x ; y) ∈ dm équivaut à y − 2 = mx − 2m.
113 a)  x2 − 2 x = ( x − 1)2 − 1
a) Si x = 2, y − 2 = 2m − 2m donc y = 2
Une équation de #1 est ( x − 1)2 + y 2 = 1. I Î dm quelle que soit la valeur de m.
#1 est le cercle de centre I1(1 ; 0) et de rayon 1. b) En remplaçant y - 2 par mx - 2m dans l’équa-
b)  I1I2 = (2 − 1)2 + (2 − 0)2 = 5 . tion de #, on obtient ( x − 2)2 + (mx − 2m)2 = 1 soit
5 < 3 les cercles #1 et # 2 sont sécants. x2 − 4 x + 4 + m2 x2 − 4 m2 x + 4 m2 − 1 = 0 qui équi-
c) Une équation de # 2 est ( x − 2)2 + ( y − 2)2 = 4 vaut à (1 + m2 )x2 − (4 + 4 m2 )x + 4 m2 + 3 = 0
soit x2 + y 2 − 4 x − 4 y + 4 = 0. ∆ = (4 + 4 m)2 − 4 × (1 + m2 )(4 m2 + 3)
Les coordonnées des points d’intersection de #1 et ∆ = 16 + 32m2 + 16m4 − 16m2 − 16m4 − 12 − 12m
# 2 sont solutions du système. ∆ = 4 m2 + 4 = 4(m2 + 1) > 0 pour tout m Î 
 x2 + y 2 − 2 x = 0
 4 + 4 m2 − 4(m2 + 1)
 2 xm1 =
 x + y 2 − 4 x − 4 y + 4 = 0 2(1 + m2 )

 x2 + y 2 − 2 x = 0
 2m 2 + 2 − 1 + m 2
4 x + 4 y − 4 − 2 x = 0 donc xm1 =
 1 + m2
 x2 + y 2 − 2 x = 0
 2m 2 + 2 + 1 + m 2
2 x + 4 y − 4 = 0 et xm2 =
 1 + m2
 x2 + y 2 − 2 x = 0
 On obtient alors
 x = −2 y + 2
 2m 2 + 2 − 1 + m 2
4 y 2 − 8 y + 4 + y 2 + 4 y − 4 = 0 ym1 = m × + 2 − 2m
 1 + m2
 x = −2 y + 2
 2 m 3 + 2 m − m 1 + m 2 + 2 + 2m 2 − 2m − 2m 3
5 y 2 − 4 y = 0 ym1 =
 1 + m2
 x = −2 y + 2

 2m 2 − m 1 + m 2 + 2
 y = 4 ym1 =
 y = 0  5 1 + m2
 ou 
 x = 2  2 2m 2 + m 1 + m 2 + 2
 x = et ym2 =
 5 1 + m2
Chapitre 10  ★  Applications du produit scalaire 173

172909_Chap10_000-000.indd 173 30/07/2019 16:36:30


 
115 a)  AB(−1 ; 2) et AC(4 ; 2).
Organiser son raisonnement
   
AB ⋅ AC = −1× 4 + 2 × 2 = 0 donc AB ^ AC.
Le triangle ABC est rectangle en A.
b)  BC = 5
1 
I milieu de [BC] a pour coordonnées I ; 5 ,
 2 
c’est le centre du cercle circonscrit à ABC. 55   2. a) Une équation de # est
119
Une équation de ce cercle est ( x − 2)2 + ( y − 1)2 = 9
2 2
 2 soit x + y − 4 x − 2 y − 4 = 0.
 x − 1  + ( y − 5)2 = 25 .
 2  4 b) Les abscisses des points d’intersection de # et d1
sont solutions de x2 − 4 x − 1 = 0 (on remplace y
116 a) •  x2 + y 2 − 8 x + 6 y = −15 équivaut à par 3) soit x1 = 2 − 5 et x2 = 2 + 5 d’où
( x − 4)2 + ( y + 3)2 − 16 − 9 = −15 ( ) (
A1 2 − 5 ; 3 A2 2 + 5 ; 3 . )
c’est-à-dire ( x − 4)2 + ( y + 3)2 = 10. c) Les coordonnées des points d’intersection de # et
C’est donc le cercle de centre A(4 ; − 3) et de rayon d2 sont les solutions de y 2 − 2 y − 7 = 0 (on rem-
10. place x par 1) soit y1 = 1 − 2 2 et y2 = 1 + 2 2
•  x2 + y 2 − 10 x = 15 équivaut à ( ) (
d’où B1 1 ; 1 − 2 2 B2 1 ; 1 + 2 2 . )
( x − 5)2 + y 2 − 25 = 15
d2
c’est-à-dire ( x − 5)2 + y 2 = 40. 6
C’est donc le cercle de centre B(5 ; 0) et de rayon 5
2 10. 4
d1 #
b)  3
2
A
1

-2 0 1 2 3 4 5
-2

120 A et C ont même ordonnée donc d1 la média-


x + xC
trice de [AC ] a pour équation x = A soit
3 2
x= .
 4
AB(3 ; − 3) est normal à d2 , la médiatrice de [AB].
 3
d2 passe par J le milieu de [AB] soit J0 ; .
Il semble que ces cercles sont intérieurement tan-  2 
gents en I(3 ; − 6). Une équation de d2 est de la forme 3 x − 3 y + c = 0
9
et − + c = 0.
117 a) Si M Î ^ demi-cercle de centre O et de rayon 2 9
Une équation de d2 est 3 x − 3 y + = 0 soit
1 alors OM = 1, soit x2 + y 2 = 1. 2
3
b)  M(0 ; − 1) est tel que 02 + 12 = 1, M Ï ^. x − y + = 0.
2
La réciproque n’est pas vraie. Les coordonnées du centre W du cercle passant par
 A, B et C sont solutions du système
118 Un vecteur normal à d est n(m ; − 1).  
  x = 3  x = 3
Un vecteur normal à d ¢ est n′(m′ ; − 1).  4 4
   
d et d ¢ sont perpendiculaires si, et seulement si, n et  3  3 3 9
    x − y + = 0  y = + =
n¢ sont orthogonaux c’est-à-dire n ⋅ n′ = 0.  2  4 2 4
Ainsi, d et d ¢ perpendiculaires équivaut à  3 9 
Donc Ω ; .
mn′ + (−1) × (−1) = 0 c’est-à-dire mn′ = −1.  4 4 

174

172909_Chap10_000-000.indd 174 30/07/2019 16:39:35


On procède de façon différente avec d1 et d3 la  2  2  2
  3  3.  MA2 + MB2 + MC2 = MA + MB + MC
médiatrice de [BC] qui a BC ; 3 comme vecteur En utilisant la relation de Chasles on a :
 2     
 9 3 MA2 + MB2 + MC2 = (MG + GA)2 + (MG + GB)2
normal et passe par le milieu F ; .    
 4 2  +(MG + GC)2 = MG2 + GA2 + 2MG ⋅ GA + MG2
D’où une équation cartésienne de d3 de la forme    
+GB2 + 2MG ⋅ GB + MG2 + GC2 + 2MG ⋅ GC
3 3 9 3    
x + 3 y + c = 0 avec × + 3 × + c = 0 = 3MG2 + GA2 + GB2 + GC2 + 2MG ⋅ (GA + GB + GC)
2 2 4 2
63    
soit c = − . or GA + GB + GC = 0
8 D’où
Les coordonnées de W sont solutions du système
MA2 + MB2 + MC2 = 3MG2 + GA2 + GB2 + GC2
 3
 x = 4 4. a)  A

 3 63
 x + 3 y − =0
 2 8
2 2
 
 x = 3  x = 3 G

 4  4
soit 
 3 3 63  9
 × + 3 y − = 0 y =

 2 4 8  4 B I 2 C
 3 9 
d’où Ω ; . ABC est équilatéral donc la médiane [AI] est aussi la
 4 4 
hauteur issue de A.
121 C( x ; y) est le centre du cercle. D’où AI2 = AB2 − BI2 = 3 soit AI = 3
Il appartient à la droite d : 3 x + 4 y = 22 2 3
G est aux deux tiers [AI] donc AG=
A ∈ # donc AC2 = x2  2 2
3
 d’où AC = BC Il en est de même pour BG et CG.
B ∈ # donc BC2 = x2 
b)  MA2 + MB2 + MC2 = 25 équivaut à
soit ( x − 7)2 + ( y − 9)2 = ( x + 1)2 + ( y − 5)2 3MG2 + GA2 + GB2 + GC2 = 25
et x2 − 14 x + 49 + y2 − 18 y + 81
 2 3 2
= x2 + 2 x + 1 + y 2 − 10 y + 25 soit 3MG2 + 3  = 25 et 3MG2 = 25 − 4
 3 
On a donc −16 x − 8 y = −104 or 4 y = 22 − 3 x
21
On a alors −16 x − 44 + 6 x = −104 MG2 = = 7 soit MG = 7
3
soit −10 x = −60 et x = 6.
L’ensemble recherché est le cercle de centre G et de
En remplaçant dans 3 x + 4 y = 22 on obtient y = 1.
rayon 7 cm.
Ainsi C(6 ; 1).
122 1. a) En utilisant la relation de Chasles on 123 y 2 − 8 y = ( y − 4)2 − 16
        Une équation de #1 est donc x2 + ( y − 4)2 = 16
obtient GA + GB + GC = GA + GA + AB + GA + AC
     1   #1 est le cercle de centre A(0 ; 4) et de rayon 4
soit 3GA + AB + AC = 0 d’où AG = (AB + AC) x2 − 16 x = ( x − 8)2 − 64
3
b) Soit I le milieu de [BC]. Toujours grâce à la relation y 2 + 4 y = ( y + 2)2 − 4
         
de Chasles on obtient AB + AC = AI + IB + AI + IC Une équation de # 2 est donc

   
comme I est le milieu de [BC] ; IB + IC = 0. ( x − 8)2 + ( y + 2)2 = 36
  
 # 2 est le cercle de centre B(8 ; − 2) et de rayon 6.
On a AB + AC = 2AI
 2   Calcul de la distance AB :
On peut conclure AG = AI.
3 AB = (8 − 0)2 + (−2 − 4)2 = 10
2 La somme des longueurs des rayons est 10.
G est donc au de la médiane [AI] en partant de A.
3 Les cercles sont donc tangents extérieurement.
2. On montre de même que
 2 
BG = BJ (avec J milieu de [AC ] ) 124 •  x2 + y 2 − 4 y − 5 = 0
3
 2  équivaut à x2 + ( y − 2)2 − 4 − 5 = 0
CG = CK (avec K milieu de [AB] )
3 soit x2 + ( y − 2)2 = 9
G appartient aux trois médianes du triangle. # est donc le cercle de centre A(0 ; 2) et de rayon 3.

Chapitre 10  ★  Applications du produit scalaire 175

172909_Chap10_000-000.indd 175 30/07/2019 16:43:59


•  x2 + y 2 − 8 x − 2 y + 7 = 0 125 % est le cercle de diamètre [AB]. %¢ est la per-
équivaut à ( x − 4)2 − 16 + ( y − 1)2 + 1 + 7 = 0 pendiculaire à (AB) passant par C.
soit ( x − 4)2 + ( y − 1)2 = 8. Cette perpendiculaire coupe exactement deux fois %
#¢ est donc le cercle de centre B(4 ; 1) et de rayon si elle est incluse dans la bande de plan de frontières
8 = 2 2. d la droite perpendiculaire à (AB) passant par B et d ¢
la droite perpendiculaire à (AB) passant par A. (Fron-
tières non comprises). Le projeté orthogonal de C sur
 
la droite (AB) est C¢, d’où AB ⋅ AC = AB × AC′.
 
Si C = A, alors AB ⋅ AC = 0
   
Si C = B, alors AB ⋅ AC = AB2 soit 0 < AB ⋅ AC < AB2

b)  M( x ; y) est un point d’intersection de # et #¢. C¢


B
On a donc
x2 + y 2 − 4 y − 5 = x2 + y 2 − 8 x − 2 y + 7
soit −4 y − 5 = −8 x − 2 y + 7 en multipliant les
2 membres par -1 on obtient 4 y + 5 = 8 x + 2 y − 7
c)  2 y = 8 x − 12 et y = 4 x − 6 126 a) d, médiatrice de [AB] passe par C¢ le milieu
 3
il vient alors y 2 = 16 x2 − 48 x + 36 de [AB]. C′−3 ; .
 2
En reportant dans l’équation de # on a :
Les points A et B ont même abscisses, d est parallèle à
x2 + 16 x2 − 48 x + 36 − 4(4 x − 6) − 5 = 0
3
soit 17 x2 − 64 x + 55 = 0 l’axe des abscisses, elle a pour équation y = .
2
d)  ∆ = 356, les solutions de cette équation sont 3 3
d ¢ la médiatrice de [BC] passe par A ′ ; −  et a
32 − 89 32 − 89  2 2 
x1 = ≈ 1, 33 et x2 = ≈ 2, 44 
17 17 pour vecteur normal BC(9 ; 3). Une équation de d ¢
32 − 89 −4 89 + 26 est de la forme 9 x + 3 y + c = 0
On obtient y1 = 4 × −6 = 3 −3
17 17 A ¢ appartient à d ¢ d’où 9 × + 3 × + c = 0 soit
2 2
32 + 89 4 89 + 26
et y 2 = 4 × −6 = c = −9. Une équation de d ¢ est 9 x + 3 y − 9 = 0
17 17
soit 3 x + y − 3 = 0.
 32 − 89 −4 89 + 26 
Ainsi M  ;  et b) Les coordonnées de O sont solutions du système
 17 17 
 3
 32 + 89 4 89 + 26 
 y = 2  1 3
soit O ; 
N ;    2 2 
  
17 17 3 x + y − 3 = 0
 32 − 89 2  −4 89 + 26 2   2. a) m la médiane issue de A passe par A(−3 ; 6) et
e)    +   − 4  −4 89 + 26  − 5
17   17   17  3 3
A ′ ; −  le milieu de [BC]. Un vecteur directeur
1024 − 64 89 + 89 1424 − 208 89 + 676 16 89 − 104

2 2
= + + −5
289 289 17 de (m) est
1024 − 64 89 + 89 1424 − 208 89 + 676 16 89 − 104   3 3    9 15 
= + + −5 AA ′ + 3 ; − − 6 soit AA ′ ; − .
289 289 17  2 2  
2 2
1113 − 64 89 + 2100 − 208 89 + 272 89 − 1768 − 1445 Une équation de m est de la forme
=
289 15 9
=0
− x − y + c = 0 soit 15 x + 9 y + c ′ = 0
2 2
On démontre de même que M ∈ #′ et N appartient à A appartient à m, d’où −45 + 54 + c ′ = 0 soit
# et #¢. Conclusion # et #¢ sont sécants en M c ′ = −9. Une équation de m est 15 x + 9 y − 9 = 0
et N. soit 5 x + 3 y − 3 = 0
176

172909_Chap10_000-000.indd 176 30/07/2019 17:28:27


m¢ médiane issue de C a pour vecteur directeur Pour toute valeur de k #K est le cercle de centre
  3
CC′−9 ; . Une équation de m¢ est donc IK (k − 1 ; − k − 3) et de rayon 2k 2 + 4 k + 20 .
 2 3 3.  −k − 3 = −(k − 1) − 4 , l’ensemble des points IK
x + 9 y + c = 0.
2 est la droite d’équation y = −x − 4.
Comme A(6 ; 0) appartient à m¢ on obtient 4. Recherchons les coordonnées des points d’inter-
9 + c = 0 soit c = −9. section de # 0 et #1. Elles sont solutions de
3  x2 + y 2 + 2 x + 6 y = 10
Une équation de m¢ est x + 9 x − 9 = 0. 
2  2
 x + y 2 + 8 y = 10
b) Les coordonnées de G sont solutions du système 
5 x + 3 y = 3 2 y − 2 x = 0
 soit  2
3  x + y 2 + 8 y = 10
 x + 9 y = 9 
 2
On trouve G(1 ; 0).  x = y  x = y
 2    2
c) h la hauteur issue de C est l’axe des abscisses 2 x + 8 x − 10 = 0  x + 4 x + 5 = 0
 
d’équation y = 0. On trouve A(1 ; 1) et ainsi B(−5 ; − 5).
h¢ hauteur issue de A a pour vecteur normal On vérifie que A et B appartiennent aux cercles # k :

BC(9 ; 3). Une équation de h¢ est de la forme 12 + 12 − 2k + 2 + 2k + 6 − 10 = 0
9x + 3y + c = 0 52 + 52 − 10k − 10 + 10k − 30 = 10.
A appartient à h¢ donc −27 + 18 + c = 0 soit Les cercles # k passent par les points fixes A(1 ; 1) et
c = 9. Une équation de h¢ est 9 x + 3 y + 9 = 0 soit B(−5 ; − 5).
3 x + y + 3 = 0.
 
h¢ coupe l’axe des abscisses en H(−1 ; 0). 128 MA(1 − x ; 1 − y) MB(9 − x ; 1 − y)
  1 1   
d) Considérons les vecteurs GO ;  et MA ⋅ MB = (1 − x)(9 − x) + (1 − y)(1 − y)
   2 2 
1  = 9 − 10 x + x2 + 1 − 2 y + y 2
GH(−1 ; − 1). GO = − GH.
 2 = x2 − 10 x + y 2 − 2 y + 10
Les vecteurs GO et GH sont colinéaires, les points G, = ( x − 5)2 + ( y − 1)2 − 16
O et H sont donc alignés.  
MA ⋅ MB > −7 équivaut à ( x − 5)2 + ( y − 1)2 > 9
 
127 1.  # 0 a pour équation MA × MB < 0 équivaut à ( x − 5)2 + ( y − 1)2 < 16
x2 + y 2 + 2 x + 6 y − 10 = 0 L’ensemble des point M( x ; y) vérifiant
x2 + 2 x = ( x + 1)2 − 1 ( x − 5)2 + ( y − 1)2 > 9
y 2 + 6 y = ( y + 3)2 − 9 d’où une équation de # 0 
( x − 5)2 + ( y − 1)2 < 16
est ( x + 1)2 + ( y + 3)2 − 20 = 0 
# 0 est le cercle de centre I0 (−1 ; − 3) et de rayon  x < 7

20. est la partie de couronne comprise entre les cercles
#1 a pour équation x2 + y 2 + 8 y − 10 = 0 de centre Ω(5 ; 1) et de rayon 3 et 4 située à gauche
y 2 + 8 y = ( y + 4)2 − 16. D’où une équation de #1 de la droite d’équation x = 7
est x2 + ( y + 4)2 = 26.
#1 est le cercle de centre I1(0 ; − 4) de rayon 26 6
# 2 a pour équation x2 − 2 x + y 2 + 10 y − 10 = 0 5
soit ( x − 1)2 + ( y + 5)2 = 36. 4
# 2 est cercle de centre I2 (1 ; − 5) de rayon 6. 3
2. Une équation de #K est 2
x2 + y 2 + (2 − 2k )x + (6 + 2k ) y − 10 = 0 1
x2 + (2 − 2k )x = ( x + 1 − k )2 − (1 − k )2
0 1 2 3 4 5 6 7 8 9
y 2 + (6 + 2k ) y = ( y + 3 + k )2 − (3 + k )2 -1
Une équation de # K est donc -2
( x + 1 − k )2 + ( y + 3 + k )2 = 2k 2 + 4 k + 20 -3
Considérons 2k 2 + 4 k + 20 -4
∆ = −144 < 0 donc 2k 2 + 4 k + 20 > 0

Chapitre 10  ★  Applications du produit scalaire 177

172909_Chap10_000-000.indd 177 30/07/2019 16:51:06


129 a)  d0 a pour équation y = 2 Une équation de #¢ est donc ( x − 2)2 + ( y − 1)2 = 10
d1 a pour équation x = 1, elles sont sécantes en soit x2 − 4 x + y 2 − 2 y − 5 = 0
A(1 ; 2). c) Les coordonnées des points A1 et A2 sont les solu-
m × 1 + (1 − m) × 2 + m − 2 = 0.  x2 + y 2 − 2 x + 4 y + 1 = 0
A appartient à dm pour toute valeur de m. tions du système 
 x2 + y 2 − 4 x − 2 y − 5 = 0
b) Les droites passant par A ont pour équation 
a x + b y + c = 0 avec a + 2b + c = 0 y est donc solution de l’équation 5 y 2 + 14 y + 8 = 0
soit c = −a − 2b. 4 3
∆ = 36, donc y1 = − et y2 = −2 d’où x1 = −
Les droites d’équation a x + b y − a − 2b = 0 appar- 5 5
et x2 = 3
m 1− m
tiennent à ^ si, et seulement si, =  3 4
a b # et #¢ sont sécants en A1− ; − 
   5 5
= m − 2 vérifié automatiquement
 −a − 2b  et A2 (3 ; − 2).
m 1− m a
= si, et seulement si, m = 131 a) Pour construire M équidistant de l’axe des
a b a+b
Ce n’ est possible que si b ≠ −a. abscisses et de B on mène par A la perpendiculaire
L’équation de la droite passant par A qui n’appartient D à l’axe des abscisses. On trace la médiatrice d du
pas à ^ est a x − a y + a = 0 soit x − y + 1 = 0. segment [AB]. Ces droites sont sécantes en M équi-
c)  Hm est le projeté orthogonal de O sur la droite dm . distant de l’axe des abscisses et B.
Le triangle OAHm est rectangle en Hm  : il est inscrit b) 
5
dans le cercle # de diamètre [AO ]. B
4
Hm appartient à #. 3 M
d) Lorsque m décrit , Hm décrit # à l’exception du
2
deuxième point d’intersection de # avec la droite
1
d’équation x − y + 1 = 0. A
- 3- 2 0 1 2 3 4 5 6
130 1. a)  x2 − 2 x = ( x − 1)2 − 1
c)  M( x ; y) appartient à D donc x = a.
y 2 + 4 y = ( y + 2)2 − 4
M( x ; y) appartient à d de vecteur normal
Une équation de # est donc   a + 2 
( x − 1)2 − 1 + ( y + 2)2 − 4 + 1 = 0 AB(2 − a ; 4) qui passe par I ; 2 le milieu de
 2 
soit ( x − 1)2 + ( y + 2)2 = 4 [AB].
# est le cercle de centre Ω(1 ; − 2) et de rayon 2. Une équation de d est donc de la forme
(2 − a)x + 4 y + c = 0.
 2 + a 
I Î d donc (2 − a) +8+c =0
 2 
a2 − 20
soit c = .
2
La droite d a pour équation :
a2 − 20
(2 − a)x + 4 y + =0
2
Les coordonnées de M sont solutions de
 x = a
  x = a

 2    2
(2 − a)x + 4 y + a − 20 = 0  y = a − 4 a + 20
 2  8
2. a)  (TA1) la tangente à # issue de T est perpendi- x2 − 4 x + 20
M décrit la parabole d’équation y =
culaire à la droite (ΩA1). 8
quand a décrit .
Le point A1 appartient donc au cercle #¢ de dia-
mètre [ΩT ]. De même A2 appartient au cercle #¢.
b) Le cercle #¢ a pour centre I(2 ; 1) le milieu de
[ΩT ]. et pour rayon IT.
IT = (3 − 2)2 + (4 − 1)2 = 10 .
178

172909_Chap10_000-000.indd 178 30/07/2019 16:52:23


132 a)  133 On note I le milieu du segment [BC].
 
1 1
(AC2 − AB2 ) = (AC2 − AB2 )
2 2
1 2 2 1    
(AC − AB ) = (AC + AB)(AC − AB)
2 2
1 2 2 1 
           

(AC − AB ) = (AI + IC + AI + IB) ⋅ (AI + IC − AI − IB)
2 2 
   
Or I est le milieu de [BC] donc IC + IB = 0.
1 1 
     
 
Ainsi (AC − AB)2 = (2AI) ⋅ (IC − IB) = AI ⋅ BC.
2 2
  1
AM ⋅ BC = (AC − AB2 ) équivaut donc à
2

   2    


AM ⋅ BC = AI ⋅ BC , c’est-à-dire ⋅(AM − AI) ⋅ BC = 0,
 
soit encore à IM ⋅ BC = 0.
L’ensemble cherché est donc la médiatrice du seg-
ment [BC].
b) Il semble que l’orthocentre H appartienne toujours  
134 On choisit comme repère (O, i, j ) où O est le
à l’hyperbole, quelles que soient les positions sur -
centre du cercle central et d’unité le cm.
des points A, B d’abscisses strictement positives et du
point C d’abscisse strictement négative. 2
 1   1   1
2. On note A a ;  , Bb ;  , Cc ;  , H( x ; y) les A3 A2
 a   b   c 
coordonnées respectives des points A, B, C et H avec 1
a, b des nombres réels strictement positifs, c un
C O D
nombre réel strictement négatif et x, y des nombres -2 A4- 1 0 1 A1 2
réels. H est le point de concours des hauteurs du
triangle ABC, donc les droites (HC) et (AB) sont per- -1
pendiculaires ainsi que les droites (HA) et (BC). A5 A6
   
HC ⋅ AB = 0 et HA ⋅ BC = 0 conduisent a un système -2
d’inconnues x, y :
Équation de # le cercle central
 1   1 1
 2 2
(c − x) × (b − a) +  − y  ×  −  = 0 ( x − 0)2 + ( y − 0)2 =  
 c  b a  3 

 1   1 1 4
(a − x) × (c − b) +  − y  ×  −  = 0 2
soit x + y = 2
 a  c b 9
4 
Un logiciel de calcul formel permet d’obtenir la solu- Coordonnées de A1 ; 0.
 3 
tion :
Une équation de #1 le cercle de centre A1 :
 2
 x − 4  + y 2 = 4 soit x2 − 8 x + y 2 + 4 = 0
 3  9 3 3
Coordonnées de A2 .
A2

O A1
Le triangle OA1A2 est équilatéral. Les coordonnées
1  2 2 3 
D’où : y = , c’est-à-dire H appartient à l’hyperbole
x de A2 sont A2  ; .
-.  3 3 

Chapitre 10  ★  Applications du produit scalaire 179

172909_Chap10_000-000.indd 179 30/07/2019 16:53:00


Une équation de # 2 est : C
2
 2
  
 x − 2  +  y − 2 3  = 4 D G A
 3  
 3  9 I
E
Les points A3 ; A 4 ; A5 et A 6 sont les quatre der- O H F
B
niers sommets de l’hexagone régulier A1A2A3A 4 A5A 6
 2 2 3   
leurs coordonnées sont A3 − ;  , A 4 − 4 ; 0 ;
 3 3   3 

 2 2 3   2 2 3 
A5 − ; −  et A 6  ;
  136 Objectif BAC
 3 3   3 3 
1. a) En utilisant la relation de Chasles on obtient
On a donc # 3 de centre A3 qui a pour équation :        
MD ⋅ MA = (MI + ID) ⋅ (MI + IA)
 2  2      
  
 x + 2  +  y − 2 3  = 4 = MI2 + MI ⋅ (ID + IA) + ID ⋅ IA
 3   3  9

Comme I est le milieu de [AD]
# 4 de centre A 4 a pour équation : 
    
 

ID + IA = 0 et ID ⋅ IA = −IA2
 2  
 x + 4  + y 2 = 4 d’où MD ⋅ MA = MI2 − IA2 .
 3  9
b) L’ensemble % est l’ensemble des points M tels que
# 5 de centre A5 a pour équation :
IM = IA, c’est le cercle de centre I qui passe par A.
 2  2  
 x + 2  +  y + 2 3  = 4 2. a)  AC(−3 ; 2, 5) DB(5 ; 6)
 3   3  9  

AC ⋅ DB = −3 × 5 + 2, 5 × 6 = 0
et # 6 de centre A 6 a pour équation :  
AC et DB sont orthogonaux.
 2  2 
 x − 2  +  y − 2 3  = 4 AC est un vecteur normal à la droite (BD).
 3   3  9
 b) Une équation cartésienne de la droite (DB) est de
la forme −3 x + 2, 5 y + c = 0
135 A et B ont des ordonnées opposées donc I le
D(−5 ; 0) appartient à (DB), donc +15 + c = 0 soit
milieu de [AB] appartient à l’axe des abscisses.
c = −15
# est le cercle de centre I(2 ; 0) et de rayon
Une équation de (DB) est −3 x + 2, 5 y − 15 = 0 soit
IA = 4 2 + 12 = 17
−6 x + 5 y − 30 = 0.
Une équation de # est ( x − 2)2 + y 2 = 17  

 c) L’ensemble % des points M tels que MD ⋅ MA = 0
La droite (CI) a pour vecteur directeur CI(12 ; − 3).
est le cercle de diamètre [AD]. Son centre est
Une équation de (CI) est de la forme
I(−1 ; 0) le milieu de [AD]. Une équation de % est
−3 x − 12 y + c = 0
( x + 1)2 + y 2 = 16.
I appartient à (CI) donc −6 + c = 0 soit c = 6.
Une équation de (CI) est −3 x − 12 y + 6 = 0 soit
x + 4 y − 2 = 0.
Les coordonnées des points d’intersection de (CI) et

Exploiter ses compétences


( x − 2)2 + y 2 = 17
# sont solutions de 
 x = 2 − 4 y

 x = 2 − 4 y
soit 
17 y 2 = 17

# et (CI) se coupent en D(−2 ; 1) et F(6 ; − 1)
A et F ont la même abscisse, l’axe des abscisses est la 137 Les points A, B, C désignent respectivement les
droite perpendiculaire à (FA) qui passe par I. bouées 1, 2 et 3. Le cercle #1 de centre A(0 ; 0) et de
 1 rayon 4 a pour équation x2 + y 2 = 16
E 0 ;  est le milieu de [DI].
 2  Le cercle # 2 de centre B(4 ; 0) et de rayon 4 a pour
L’axe des ordonnées est la droite perpendiculaire à équation x2 + y 2 − 8 x = 0
l’axe des abscisses qui passe par E. Le cercle # 3 de centre C(0 ; 4) et de rayon 4 a pour
(AD) coupe l’axe des ordonnées en G(0 ; 1) équation x2 + y 2 − 8 y = 0
H le milieu de [OI] a pour coordonnées H(1 ; 0) Les coordonnées de I sont I(4 ; 4).
 
Le repère est (O ; OH ; OG). Les coordonnées de F sont solutions de
180

172909_Chap10_000-000.indd 180 30/07/2019 16:54:16


 x2 + y 2 = 16 Les cercles #1 et # 2 sont sécants en E(4 ; 4)

 2 1164 576 
 x + y 2 − 8 x = 0 et D ;
  121 121 
(
On trouve F 2 ; − 2 3 . )
 CE = (4 − 16)2 + (4 + 5)2 = 15.
( ) (
de même on obtient E −2 3 ; 2 . CE −2 3 ; − 2 ;

 ) E appartient au cercle # 3 .
CI(4 ; 0).
  1164 2  576 2
CE ⋅ CI = −8 3 CD =  − 16 +  + 5 ≈ 11, 66.
 121   121 
CE = 4 CI = 4
  D n’appartient pas au cercle # 3
CE ⋅ CI = 4 × 4 × cos EC I d’où 16 cosECI
 = −8 3 soit
Le téléphone de Victor est au point E(4 ; 4).
 = − 3 d’où ECI
cosECI  = 150°.
2 139 On réalise un dessin l’échelle 10/254. En effet
On obtient de même IBF  = FAE
 = 150°
10 inches sont représentés par 1  cm et
On en déduit que la longueur totale de la ligne de 10 inches = 25,4 cm.
 210 210 150   
délimitation est 8π + + On se place dans un repère orthonormé (O ; i ; j )
 360 360 360 
114π d’unité le cm et on trace les droites d’équations
soit ≈ 40 côtés de carreaux x = 1 ; x = −1 ; y = 6 et y = −6 et les cercles de
9
Comme indiqué dans l’énoncé : 1 côté de car- centre O et de rayons 4,5 et 3 cm.
reau = 100 m. Il faut environ 4 000 m de ligne de déli- Leurs équations sont
4 000 x2 + y 2 = 20 , 25 et x2 + y 2 = 9
mitation soit une dépense de × 40 = 64 000 €.
25
4

-6 -4 -2 0 2 4 6

-2

-4

Le rectangle bleu est la représentation de l’ensemble


−6 < x < 6
solution du système 
138 #1 est le cercle de centre A et de rayon 5 −1 < y < 1
Une équation de #1 est ( x − 1)2 + ( y − 0)2 = 25 soit La portion de couronne située au-dessus du rectangle
x2 + y 2 − 2 x = 24 est la représentation de l’ensemble solution du sys-
# 2 est le cercle de centre B et de rayon 10 est  x2 + y 2 > 9

( x − 12)2 + ( y − 10)2 = 100 tème  x2 + y 2 < 20 , 25
soit x2 + y 2 − 24 x − 20 y = −144 
 y > 1
# 3 est le cercle de centre C et de rayon 15. 
Les coordonnées des points d’intersection de #1 et Celle située en dessous est la représentation du sys-
# 2 sont les solutions du système tème
 x2 + y 2 − 2 x = 24  x2 + y 2 > 9
 
 2  x2 + y 2 < 20 , 25
 x + y 2 − 24 x − 20 y = −144 

 y < −1
 x2 + y 2 − 2 x = 24 
soit  Il reste ensuite à placer le texte UNDERGROUND
11x + 10 y = 84

On obtient les couples solutions (4 ; 4) et
1164 576 
 ; .
 121 121 

Chapitre 10  ★  Applications du produit scalaire 181

172909_Chap10_000-000.indd 181 30/07/2019 16:54:59


140   B C

65 O
x
2
16
56 E
A D
(OE) est une médiane du triangle AEC. En utilisant la
-2 relation de Chasles on a:
 2  2
EC2 + EA2 = EC + EA
   
= (EO + OC)2 + (EO + OA)2
  
= 2EO2 + 2EO ⋅ (OC + OA) + OC2 + OA2
Commandes Geogebra   
O milieu de [AC ] entraine OC + OA = 0
• a : −6 < x < 6 ∧ −1 < y < 1
1
• b : 9 < x2 + y 2 < 20 , 25 ∧ y > 1 et OC = OA = AC.
2
• e : 9 < x2 + y 2 < 20 , 25 ∧ y < −1 1
On a donc EC2 + EA2 = 2EO2 + AC2
• j : 9 < x2 + y 2 < 20 , 25 ∧ −1 < y < 1 2 1
soit x2 + 562 = 2 × (40 , 5 − 16)2 + × 812
2
d’où x2 = 1345. x = 1345 EC » 36 , 7 m.

UNDERGROUND

182

172909_Chap10_000-000.indd 182 30/07/2019 16:55:13


11
Probabilités
conditionnelles

•  B Ç C : « L’adhérent choisi pratique le badmin-

Découvrir ton et le canoë ». Cet ensemble est vide.


•  A È B È C : « L’adhérent choisi pratique l’athlé-
tisme ou le badminton ou le canoë ».
11 24
b) •  P(A ∩ F) =  • P(B ∩ F) =
115 115
15
1  Probabilités conditionnelles •  P(C ∩ F) =
115
c)  P(A ∩ F) + P(B ∩ F) + P(C ∩ F)
1   
Éthanol Non éthanol Total 11 + 24 + 15 50 10
= = = = P(F)
Électrique 39 26 65 115 115 23
Non électrique 46 389 435 La somme de ces trois probabilités est égale à la
Total 85 415 500 probabilité de F.

85
2  a) •  p1 = = 0 ,17. La probabilité que ce véhi-
500
cule fonctionne à l’éthanol est 0,17.
39
Acquérir des automatismes
p2 = = 0 , 078. La probabilité que ce véhi-
500
cule ait un fonctionnement hybride est 0,078.
39
b)  p3 = = 0 , 6. Parmi les véhicules qui fonc-
65
tionnent à l’électricité, la probabilité de choisir
un véhicule qui fonctionne à l’éthanol est 0,6. 3 « L’employé choisi est en CDI et a plus de
P2 30 ans » est l’événement D Ç J.
3  a)  P1 = P(B); P2 = P(A ∩ B). b)  P3 = . Sa probabilité est :
P1
P(D ∩ J) = PD × PD (J) = 0 , 2 × 0 , 4 = 0 , 08.
2  Formule des probabilités totales
4 Deux boules de l’urne portent des numéros
1  a)  F 2 1
11 24 impairs (3 et 5) donc P(B) = = .
15 10 5
Cinq boules de l’urne portent un numéro inférieur ou
44 6 15 5 1
égal à 5 (2, 2, 3, 4 et 5) donc P(C)= = .
10 2
A B C Univers L’événement B Ç C est réalisé par le tirage d’une
11 + 24 + 15 50 10 boule portant un numéro impair et inférieur ou égal à
b)  P(F) = = = . 2 1
115 115 23 5 (boule 3 ou boule 5) donc P(B ∩ C) = = .
10 5
2  a) •  A Ç B : « L’adhérent choisi pratique l’athlé- 1 1 1
Or, P(B) × P(C) = × = .
tisme et le badminton ». Cet ensemble est vide. 2 5 10
•  A Ç C : « L’adhérent choisi pratique l’athlétisme Ainsi P(B ∩ C) ≠ P(B) × P(C) donc les événements B
et le canoë ». Cet ensemble est vide. et C ne sont pas indépendants.

Chapitre 11  ★  Probabilités conditionnelles 183

172909_Chap11_000-000.indd 183 30/07/2019 15:41:58


7 a) On note P l’événement « La pièce est tombée 14 On note R l’événement « le joueur joue au Real
sur Pile » et F l’événement « La pièce est tombée sur Madrid F.C. » et F l’événement « le joueur est français ».
Face ». 8 4
a)  P(R) = = ≈ 0 , 267.
1er lancer 2e lancer Issue
30 15
2 1
b)  PR (F) = = = 0 , 25.
3 8 4
5 P ......... (P ; P) 5
c)  PF (R) = ≈ 0 , 714.
3 P 7
5 2 F ......... (P ; F)
5 15 a) • P(A) = 0 , 2  • P(A) = 0 , 8  • P(A ∩ B) = 0 , 05
3 0 , 05 1
2 P ......... (F ; P) •  P(A ∩ B) = 0 , 5  • PA (B) = = = 0 , 25
5 0, 2 4
5 F 0, 5 5
•  PA (B) = = = 0 , 625
2 F ......... (F ; F) 0, 8 8
5 b)  0,25 B

b) La probabilité p d’obtenir les deux côtés de la pièce A


0,2 0,75
est : B
3 2 2 3 12
p = P(P ; F) + P(F ; P) = × + × = .
5 5 5 5 25 0,375 B
0,8
A
8 Les événements L et L forment une partition de 0,625 B
l’univers, donc, d’après la formule des probabilités
totales : 16 a) •  P(A ∩ B) = PA (B) × P(A) = 0 , 4 × 0 ,1 = 0 , 04.
P(G) = P(L) × PL (G) + P(L) × PL (G) •  P(A ∩ B) = PA (B) × P(A) = 0 , 2 × 0 , 9 = 0 ,18.
P(G) = 0 , 4 × 0 ,12 + 0 , 6 × 0 ,15 •  P(A ∩ B) = PA (B) × P(A) = 0 , 6 × 0 ,1 = 0 , 06.
P(G) = 0 ,138 b)  A Total
A
On en déduit que : B 0,04 0,72 0,76
P(L ∩ G) P(L) × PL (G) 0 , 4 × 0 ,12 8 B 0,06 0,18 0,24
PG (L) = = = = .
P(G) P(G) 0 ,138 23 Total 0,1 0,9 1
La probabilité d’obtenir L sachant qu’on a obtenu G
8 17 a)  0,3 P
est égale à , soit environ 0,348.
23 T
0,8 0,7 P
9 a)  PA (B) : probabilité de choisir un élève de Pre-
mière sachant que l’on a choisi une fille. P
0,2 0,95
b)  PB (A) : probabilité de choisir une fille sachant
T
qu’on a choisi un élève de Première. 0,05 P

10 a) Branche verte : PA (B). b) •  P(T ∩ P) = PT (P) × P(T) = 0 , 7 × 0 , 8 = 0 , 56.


La probabilité qu’un chien choisi au hasard soit traité
b) Branche rouge : PA (B).
contre les puces et n’ait pas de puces est égale à 0,56.
c) Chemin bleu : P(A ∩ B).
•  P(T ∩ P) = PT (P) × P(T) = 0 , 95 × 0 , 2 = 0 ,19.
La probabilité qu’un chien choisi au hasard ne soit pas
11 a) (3) 0,6 ; b) (1) 0,9 ; c) (2) 0,18.
traité contre les puces et ait des puces est égale à 0,19.
12 a) Quentin a raison. 18 1. a)  1
8 A
P(A ∩ B) 0, 3 1
En effet, PA (B) = = = . 1 C
P(A) 0, 6 2
4 7 A
b) Nadia a tort.
P(A ∩ B) 0 , 35 7 7 5 8
En effet, PA (B) = = = et ¹ . 1
P(A) 0, 5 10 10 35 3 A
8
4 C
13 a)  P(A ∩ B) = P(A) × PA (B) = 0 , 2 × 0 , 5 = 0 ,1. 7 A
b)  P(A ∩ B) = P(A) × PA (B) = 0 ,1× 0 , 85 = 0 , 085. 8

184

172909_Chap11_000-000.indd 184 30/07/2019 15:42:23


b)  1 21 a)  0,2 A
4 C
M
1 A 0,8 A
8 3 C 0,5
4 0,15 A
1 0,4
7 C S
4
8 A 0,85 A
3 C 0,1
0,3 A
4
1 1 1 E
2. •  P(A ∩ C) = PA (C) × P(A) = × = . 0,7 A
4 8 32
La probabilité que la carte tirée soit l’As de cœur est b) •  P(M ∩ A) = PM (A) × P(M) = 0 , 2 × 0 , 5 = 0 ,1.
1 La probabilité que l’arbre choisi soit un mélèze et
égale à .
32 doive être abattu est égale à 0,1.
1 7 7 •  P(E ∩ A) = PE (A) × P(E) = 0 , 7 × 0 ,1 = 0 , 7.
P(A ∩ C) = PA (C) × P(A) = × = .
4 8 32 La probabilité que l’arbre choisi soit un épicéa et ne
La probabilité que la carte tirée soit un cœur mais pas
doive pas être abattu est égale à 0,07.
7
un As est égale à .
32
22 1. La probabilité que le scarabée se déplace vers
Total 1
19 a)  240 le point B sachant qu’il se trouve au point A est .
4
1 S
50 210 2. a) 
700 4 1
4 D
Transport B
en commun Deux roues 1 1 E
b) On note T l’événement « L’élève utilise les trans- 2 1 4
4 A
ports en commun » et R l’événement « L’élève utilise
un deux roues ». 1 S
50 1 1 4 1
PT (R) = = .
750 15 2 4 C
Sachant que l’élève utilise les transports en commun, A
la probabilité qu’il utilise aussi un deux roues est 1 E
1 1 4
égale à , soit environ 0,067. B
15 4
210 21 b) La probabilité que le scarabée soit au point B au
c)  PR (T) = = .
260 26 1 1 1
Sachant que l’élève utilise un deux roues, la probabi- bout de 2 minutes est × = .
2 4 8
lité qu’il n’utilise pas les transports en commun est
environ égale à 0,807. 23 Alicia a tort.
0,6 A En effet, P(A ∩ B) = P(A) × P(B) = 0 , 8 × 0 , 2 = 0 ,16 et
20 a)  F 0 ,16 ¹ 1 donc A Ç B n’est pas l’événement certain.
0,4 0,4 A
24 a)  P(A) × P(B) = 0 , 5 × 0 , 4 = 0 , 2 et 0 , 2 ¹ 0 , 3.
A Donc A et B ne sont pas indépendants.
0,6 0,5
b)  P(A) × P(B) = 0 , 7 × 0 , 5 = 0 , 35 = P(A ∩ B).
F
0,5 Donc A et B sont indépendants.
A
c)  P(A) × P(B) = 0 , 7 × 0 , 8 = 0 , 56 = P(A ∩ B).
b) •  P(F ∩ A) = PF (A) × P(F) = 0 , 5 × 0 , 6 = 0 , 3.
Donc A et B sont indépendants.
La probabilité que le CV reçu soit celui d’un garçon et
soit accepté est égale à 0,3. 25 a)  P(A ∩ B) = P(A) + P(B) − P(A ∪ B)
•  P(A ∩ F) = PF (A) × P(F) = 0 , 4 × 0 , 4 = 0 ,16. P(A ∩ B) = 0 , 5 + 0 , 4 − 0 , 7 = 0 , 2.
La probabilité que le CV reçu soit celui d’une fille et b) A et B ne sont pas incompatibles car P(A ∩ B) = 0 , 2
soit refusé est égale à 0,16. et 0 , 2 ¹ 0.

Chapitre 11  ★  Probabilités conditionnelles 185

172909_Chap11_000-000.indd 185 30/07/2019 15:42:41


c) A et B sont indépendants car 30 On note M1 (resp. M2) l’événement « Le premier
P(A) × P(B) = 0 , 5 × 0 , 4 = 0 , 2 = P(A ∩ B). (resp. le second) moteur de l’avion tombe en panne ».
M1 et M2 sont indépendants donc :
26 a)  P(A ∩ B) = 0 , 4 × 0 , 25 = 0 ,1.
P(M1 ∩ M2 ) = P(M1) × P(M2 ) = 0 , 000 1× 0 , 000 1 = 10−8.
P(A) = 0 , 4
p = P(M1 ∩ M2 ) = P(M1 ∪ M2 ) = 1 − P(M1 ∪ M2 )
P(B) = P(A ∪ B) + P(A ∩ B) − P(A)
p = 1 − (P(M1) + P(M2 ) − P(M1 ∩ M2 ))
= 0 , 55 + 0 ,1 − 0 , 4 = 0 , 25
p = 1 − (0 , 000 1 + 0 , 000 1 − 10−8 ) = 0 , 999 800 01
Or P(A) × P(B) = 0 , 4 × 0 , 25 = 0 ,1 = P(A ∩ B)
La probabilité que l’avion arrive à bon port est égale
donc A et B sont indépendants.
à 0 , 999 800 01.
b)  P(A ∩ B) = 0 , 7 × 0 ,1 = 0 , 07
P(A) = 0 , 7
31 1. On note A (resp. B) l’événement « La première
P(B) = P(A ∪ B) + P(A ∩ B) − P(A)
(resp. la seconde) salle est occupée ».
= 0 , 82 + 0 , 07 − 0 , 7 = 0 ,19
a)  P(A ∪ B) = 0 , 9 et P(A ∩ B) = 0 , 5
Or P(A) × P(B) = 0 , 7 × 0 ,19 = 0 ,133
De plus, P(A) = P(B).
donc P(A) × P(B) ≠ P(A ∩ B) et A et B ne sont pas
Donc 2P(A) = P(A ∪ B) + P(A ∩ B) = 0 , 9 + 0 , 5
indépendants.
2P(A) = 1, 4 et P(A) = 0 , 7.
27 a) Si l’on obtient 3 avec l’un des dés et 4 avec Ainsi, P(A) = 0 , 3 et la probabilité que la première
l’autre, alors les événements A et B sont réalisés. Ces salle soit libre est égale à 0,3.
événements ne sont donc pas incompatibles. b) On peut présenter les résultats avec un tableau
6 1 11 croisé.
b) D’une part ; P(A) = = et P(B) = ,
36 6 36 A A Total
11 B 0,5 0,2 0,7
donc P(A) × P(B) = .
216 B 0,2 0,1 0,3
2
D’autre part, P(A ∩ B) = . Total 0,7 0,3 1
36
P(A ∩ B) ≠ P(A) × P(B). p = P(A ∩ B) + P(A ∩ B) = 0 , 2 + 0 , 2 = 0 , 4.
Donc les événements A et B ne sont pas indépen- La probabilité qu’une seule salle soit libre est égale
dants. à 0,4.
2.  P(A ∩ B) = 0 , 5.
28 On note S1 (resp. S2 ) l’événement « Le premier P(A) × P(B) = 0 , 7 × 0 , 7 = 0 , 49.
(resp. le second) téléphone sonne ». Donc P(A ∩ B) ≠ P(A) × P(B) et les événements A et B
S1 et S2 sont indépendants donc ne sont pas indépendants.
P(S1 ∩ S2 ) = P(S1) × (PS2 ) = 0 , 6 × 0 , 7 = 0 , 42.
p = P(S1 ∩ S2 ) = P(S1 ∪ S2 ) = 1 − P(S1 ∪ S2 ) 32 •  P(D ∩ F) = P(D) × PD (F) = 0 , 2 × 0 , 5 = 0 ,1.
p = 1 − (P(S1) + P(S2 ) − P(S1 ∩ S2 )) •  P(D ∩ F) = P(D) × PD (F) = 0 , 8 × 0 ,1 = 0 , 08.
p = 1 − (0 , 6 + 0 , 7 − 0 , 42) = 0 ,12. •  P(F) = P(D ∩ F) + P(D ∩ F) = 0 ,1 + 0 , 08 = 0 ,18.
La probabilité que, dans l’heure qui vient, elle ne soit •  P(F) = 1 − P(F) = 1 − 0 ,18 = 0 , 82.
pas dérangée par un téléphone est égale à 0,12.
33 a) Laïla a raison.
29 a) Si A et B incompatibles, En effet, P(D) = P(D ∩ A) + P(D ∩ B) + P(D ∩ C)
1
P(A ∪ B) = P(A) + P(B), donc P(B) = . P(D) = 0 , 3 + 0 , 4 + 0 , 08 = 0 , 78.
12
b) Victor a tort.
b) A et B sont indépendants : P(A ∩ B) = P(A) × P(B)
En effet, P(D ∩ B) = P(D) − P(D ∩ A) − P(D ∩ C)
P(B) = P(A ∪ B) + P(A ∩ B) − P(A)
P(D ∩ B) = 0 , 85 − 0 ,1 − 0 , 02 = 0 , 73 et 0 , 73 ¹ 0 , 95.
= P(A ∪ B) + P(A)P(B) − P(A)
P(B)[1− P(A)] = P(A ∪ B) − P(A) donc :
34 a)  P(A ∩ D) = PA (D) × P(A) = 0 , 4 × 0 ,1 = 0 , 04.
1 1
− b) A, B, C forment une partition de l’univers, donc
P(A ∪ B) − P(A) 3 4
P(B) = = d’après la formule des probabilités totales :
1 − P(A) 1
1− P(D) = P(A ∩ D) + P(B ∩ D) + P(C ∩ D)
4
1 4 1 P(D) = 0 , 04 + 0 , 5 × 0 , 2 + 0 ,1× 0 , 7 = 0 , 21.
P(B) = × = P(A ∩ D) 0 , 04 4
12 3 9 c)  PD (A) = = = ≈ 0 ,19.
P(D) 0 , 21 21
186

172909_Chap11_000-000.indd 186 30/07/2019 15:43:35


3
35 a)  4 C 1er tirage 2e tirage
1
A
4 R2
0,8 1 C
2 R1
4
5 3 B2
0,9 C
0,2 4
A 1
3 R2
0,1 C 2
5 B1
b) Les événements A et A forment une partition de 1 B2
l’univers, donc d’après la formule des probabilités 2
totales : b) D’après la formule des probabilités totales :
P(C) = P(A ∩ C) + P(A ∩ C) P(B2 ) = PR1 (B2 ) × P(R1) + PB1 (B2 ) × P(B1)
3
P(C) = × 0 , 8 + 0 , 9 × 0 , 2 = 0 , 78. 3 2 1 3
4 P(B2 ) = × + × = 0 , 6.
3 4 5 2 5
× 0, 8 La probabilité que la deuxième boule tirée soit bleue
P(A ∩ C) 4 10
c)  PC (A) = = = ≈ 0 , 77.
P(C) 0 , 78 13 est égale à 0,6.
Sachant qu’elle comporte au moins 5 pièces, la pro-
babilité que l’habitation choisie au hasard soit un 38 a)  PA (T) = 0 , 4 signifie que, sachant que l’em-
appartement est environ égale à 0,77. ployé choisi fait partie du service A, la probabilité qu’il
1 réside à moins de 30 min de l’entreprise est égale à 0,4.
36 a)  4 E b)  T
0,4
F A
0,7 3 E 0,6 T
4 0,45
1 0,2 T
0,3 E
3 0,23 B
F
2 0,8 T
E
3 0,32
0,8 T
1
b)  P(F ∩ E) = PF (E) × P(F) = × 0 , 7 = 0 ,175 C
4
0,2 T
La probabilité que le passager soit un Français qui
parle couramment l’espagnol est égal à 0,175. c) Les évènements A, B, C forment une partition de
Les événements F et F forment une partition de l’uni- l’univers, donc d’après la formule des probabilités
vers, donc d’après la formule des probabilités totales : totales :
P(E) = P(F ∩ E) + P(F ∩ E) P(T) = P(A ∩ T) + P(B ∩ T) + P(C ∩ T)
1 P(T) = 0 , 4 × 0 , 45 + 0 , 2 × 0 , 23 + 0 , 8 × 0 , 32 = 0 , 482.
P(E) = 0 ,175 + × 0 , 3 = 0 , 275.
3 P(B ∩ T) 0 , 2 × 0 , 23
d)  PT (B) = = ≈ 0 , 095.
La probabilité que le passager parle couramment l’es- P(T) 0 , 482
pagnol est égale à 0,275. Sachant que l’employé réside à moins de 30 min de
P(F ∩ E) 0 ,175 7 l’entreprise, la probabilité qu’il fasse partie du service
c)  PE (F) = = = .
P(E) 0 , 275 11 B est égale à 0,095 environ.
Sachant que le passager choisi parle couramment
l’espagnol, la probabilité qu’il soit français est égale à 39 On note B l’événement « La pièce est bonne » et
7 A l’événement « La pièce est acceptée ».
, soit environ 0,64.
11 a)  A
0,96
37 a) On note R1 (resp. B1) l’événement « la boule
B
tirée est rouge (resp. bleue) au 1er tirage ». 0,98 0,04 A
On note R2 (resp. B2) l’événement « la boule tirée est
rouge (resp. bleue) au 2e tirage ».
0,02 A
0,02
B
0,98 A

Chapitre 11  ★  Probabilités conditionnelles 187

172909_Chap11_000-000.indd 187 30/07/2019 15:43:47


b) D’après la formule des probabilités totales : On note M l’événement « L’individu a pris le médica-
P(A) = P(B ∩ A) + P(B ∩ A) ment » et B l’événement « Le taux de glycémie a
P(A) = 0 , 98 × 0 , 04 + 0 , 02 × 0 , 98 = 0 , 058 8. baissé de façon significative ».
c) La probabilité qu’il y ait une erreur est : D’après l’énoncé ; P(M) = 0 , 5,
P(B ∩ A) + P(B ∩ A) PM (B) = 0 , 8 et PM (B) = 0 ,1.
= 0 , 98 × 0 , 04 + 0 , 02 × 0 , 02 = 0 , 039 6. La probabilité que le taux de glycémie de la personne
choisie au hasard ait baissé de façon significative,
40 a) La probabilité qu’il ait moins de 15 ans est :
c’est-à-dire P(B), est donnée par :
16 , 4 % × 5, 5 + 17, 8 % × 5, 3 + 17, 5 % × 10
p= P(B) = P(M ∩ B) + P(M ∩ B)
5, 5 + 5, 3 + 10
P(B) = 0 , 5 × 0 , 8 + 0 , 5 × 0 ,1
p ≈ 0 ,173.
P(B) = 0 , 45
b) Cet habitant a moins de 15 ans, la probabilité qu’il
soit suédois est :
17, 5 % × 10 43 a) F est l’évènement « Le candidat est une fille »
0 , 084 et A est l’évènement « Le candidat est admis ».
p ′ = 5, 5 + 5, 3 + 10 ≈ ≈ 0 , 486. D’après l’énoncé, P(F) = 0 , 52; P(F ∩ A) = 0 , 39 ;
p 0 ,173
On peut également représenter la situation par un PF (A) = 0 , 7.
arbre pondéré. P(F∩ A) 0 , 39
0,164 J Ainsi, PF (A) = = = 0 , 75.
P(F) 0 , 52
F La probabilité qu’une fille qui se présente soit admise
5,5 0,836 J est égale à 0,75.
20,8 5,3 J b) Les événements F et F forment une partition de
0,178
20,8
N
l’univers, donc, d’après la formule des probabilités
0,822 totales :
J
10 P(A) = P(F∩ A) + P(F ∩ A)
20,8 J
0,175 P(A) = 0 , 39 + PF (A) × P(F)
S P(A) = 0 , 39 + 0 , 7 × (1 − 0 , 52) = 0 , 726.
0,825 J La probabilité qu’un candidat soit admis est égale
à 0,726.
41 a)  1er déplacement 2e déplacement
1 1 2 1
2 C2 44 a) (2) ;   b) (1) ;   c) (3) .
36 36 6
1 B1
2 1 A2 45 a) Faux. Elle est égale à 0 ,1´ 0 ,1 soit 0,01.
A 2 b) Vrai. En effet, elle est égale à 0 , 9 ´ 0 , 9 soit 0,81.
1
1 B2
2
2 C1 46 a)  1er tirage 2e tirage Issue
1 A2
2
2
5 R ......... (R ; R)
b) D’après la formule des probabilités totales :
2 R
P(A2 ) = P(B1 ∩ A2 ) + P(C1 ∩ A2 ) 5 3 V ......... (R ; V)
1 1 1 1 1
P(A2 ) = × + × = . 5
2 2 2 2 2 2
R ......... (V ; R)
La probabilité qu’il se retrouve en A à l’issue de deux 3 5
1 5 V
déplacements est égale à .
2 3 V ......... (V ; V)
42   0,8 B 5
M b)  P(A) = P(R ; R) + P(V ; V)
0,5 0,2 B 2 2 3 3 13
P(A) = × + × = = 0 , 52.
5 5 5 5 25
0,1 B P(B) = P(V ; R) + P(R ; V) + P(V ; V)
0,5
M 3 2 2 3 3 3 21
0,9 B
P(B) = × + × + × = = 0 , 84.
5 5 5 5 5 5 25
188

172909_Chap11_000-000.indd 188 30/07/2019 15:44:12


47 a) On note Pi l’évènement « Le chiffre choisi par 49 La probabilité que les deux pièces proviennent
l’amie i est pair ». d’un pays différent est :
p = P(A ; E) + P(E ; A) + P(A ; F) + P(F ; A)
1re amie 2e amie
+ P(E ; F) + P(F ; E)
3 1 1 2
7 P2 p = 6× × = .
3 3 3
3 P1
50 La probabilité que l’automobiliste croise un feu
7 4 P2 vert et un orange est :
7
3 p = P(V ; O) + P(O ; V)
4 P2 35 5 5 35
7 p= × + ×
7 P1 60 60 60 60
4 P2 7
7 p= ≈ 0 , 097.
72
b) La probabilité qu’Alice ait raison est : 51 1. D ;    2. C ;    3. A ;    4. D.
3 3 9
P(P1 ∩ P2 ) = × = . 52 1. A, B, C, D ;    2. A, B, C ;    3. A, B, D.
7 7 49
55 1. Vrai. En effet, P(A) + P(B) + P(C) = 1
48 a)  1 1 donc P(B) = 1 − P(A) − P(C) = 1 − 0 , 3 − 0 , 4.
3 1
6 2
2. Faux. En effet, PC (D) = 1 − PC (D) = 1 − 0 , 5 = 0 , 5.
1 3. Vrai. En effet, P(B) = 0 , 3 et
1 3 PB (D) = 1 − PB (D) = 1 − 0 , 2 = 0 , 8
1 4 donc P(B ∩ D) = 0 , 3 × 0 , 8 = 0 , 24.
4 4 4. Vrai. En effet,
P(D) = 0 , 3 × 0 , 7 + 0 , 3 × 0 , 8 + 0 , 4 × 0 , 5 = 0 , 65.
1 1
1 3 1 5. Faux. En effet, P(D) = 1 − P(D) = 1 − 0 , 65 = 0 , 35.
3 6 2 P(D ∩ A) 0 , 3 × 0 , 7 21
6. Vrai. En effet, PD (A) = = = .
2 P(D) 0 , 65 65
1 1 3 P(D ∩ B) 0 , 3 × 0 , 2 6
7. Vrai. En effet, PD (B) = = = .
6 1 4 P(D) 0 , 35 35
4 4 8. Faux. En effet, P(D ∩ A) = 0 , 21 et
P(D) × P(A) = 0 , 65 × 0 , 3 = 0 ,195
1 1 donc P(D ∩ A) ≠ P(D) × P(A).
1 3 1
4 6 2 54 La phrase (1) signifie « La probabilité qu’un indi-
3 vidu choisi au hasard soit vacciné est 0,3 », soit
1 3 P(V) = 0 , 3.
1
1 4 La phrase (2) signifie « Sachant qu’un individu est vac-
4 4
4 1
ciné, la probabilité qu’il soit malade est  », soit
1 1 5
1 PV (M) = .
3 1 5
6 2 La phrase (3) signifie « Sachant qu’un individu est non
4 1
1 3
vacciné, la probabilité qu’il soit malade est  », soit
1 3
1 4 PV (M) = .
4 4 3
55 a)  0,05 A
b) La probabilité que les numéros obtenus soient
D
identiques est : 0,01 0,95 A
p = P(1 ; 1) + P(2 ; 2) + P(3 ; 3) + P(4 ; 4)
1 1 1 1 1 1 1 1
p= × + × + × + × 0,97 A
3 3 6 6 4 4 4 4 0,99
19 D
p= ≈ 0 , 264. 0,03 A
72

Chapitre 11  ★  Probabilités conditionnelles 189

172909_Chap11_000-000.indd 189 30/07/2019 15:44:37


b)  P(D ∩ A) = PD (A) × P(D) = 0 , 05 × 0 , 01 = 0 , 000 5. 59 a) 
La probabilité que la pièce choisie soit défectueuse et p¬0
acceptée au contrôle est 0 , 000 5. Pour i allant de 1 à 3

56 Les événements A et A forment une partition a ¬ un nombre aléatoire de [0 ; 1]


de l’univers, donc d’après la formule des probabilités Si p = 0
totales : 1
Si a < alors
P(B) = PA (B) × P(A) + PA (B) × P(A) 2
P(B) = 0 , 25 × 0 , 8 + 0 , 6 × 0 , 2 p ¬1
P(B) = 0 , 32. sinon
La probabilité que le touriste choisi parle anglais est p¬2
égale à 0,32. Fin Si
Fin Si
Si p = 1 alors
1
Si a < alors
2

S’entraîner
p¬0
sinon
p¬2
Fin Si
Fin Si
58 a)  Si p = 2 alors
p¬0 1
Si a < alors
Pour i allant de 1 à 2 2
a ¬ un nombre aléatoire de [0 ; 1] p¬0
Si a < 0 , 5, alors Sinon
p ← p+2 p ¬1
sinon Fin Si
Fin Si
p ← p +1
Fin Pour
Fin Si
Afficher p
Fin Pour
Afficher p b) On représente la situation par un arbre pondéré.
b) L’arbre des probabilités ci-dessous indique les 1
numéros des cases où le lapin peut se situer après 2 B
chaque déplacement. La probabilité que le lapin ter- 1
A
2
mine son parcours sur la case numéro 3 est donc, 1 C
d’après la formule des probabilités totales : B 1 2
1 1 1 1 1 1 A
× + × = . 1 2
2 2 2 2 2 2 C
2
1 B
1er déplacement 2e déplacement 2 1
1 2 B
1
2 4 1 A
2
1 2 2 1 C
2 1 C 1 2
3 A
2 1 2
1 B
1 3 2
2 1 C
2 1 2
1 2
2

190

172909_Chap11_000-000.indd 190 30/07/2019 15:45:00


D’après la formule des probabilités totales, la proba- P(A ∩ B) = P(A) − P(A ∩ B).
bilité que la fourmi termine son parcours au point A
c)  P(A ∩ B) = PA (B) × P(A)
1 1 1 1 1 1 1
est × × + × × = . Donc, PA (B) × P(A) = P(A) − P(A ∩ B)
2 2 2 2 2 2 4
PA (B) × P(A) = P(A) − PA (B) × P(A)
61 Dans la cellule G2, on saisit la formule =NB. P(A) − PA (B) × P(A)
d’où PA (B) = et PA (B) = 1 − PA (B).
SI(E2:E1001;1)/1000 P(A)
La probabilité de gagner une partie est estimée, grâce 2. On montre de même que PA (B) = 1 − PA (B).
au tableur, à environ 0,678 (on se rapproche de la pro-
64 1. a) On doit démontrer que
babilité obtenue par le calcul).
P(A ∩ B)=P(A) × P(B)
62 Dans la cellule E2, on saisit  : =SI(D2<=9; b)  P(A ∩ B)=P(B) − P(A ∩ B).
SI(ALEA()<0.1;0;1); SI(ALEA()<0.5;0,1)) c) Or A et B sont indépendants, donc
Pour simuler 500  parties, on recopie jusqu’à la P(A ∩ B) = P(B) − P(A) × P(B)
ligne 501. P(A ∩ B) = P(B) × (1 − P(A))
Dans la cellule G2, on saisit : =NB.SI(E2:E501;1)/500 P(A ∩ B) = P(B) × P(A)
Ce qui prouve que A et B sont indépendants.
2. On procède de même :
P(A ∩ B) = P(A) − P(A ∩ B)
P(A ∩ B) = P(A) − P(A) × P(B) (car A et B sont indé-
pendants)
P(A ∩ B) = P(A)(1 − P(B)) = P(A) × P(B) donc A et B
sont indépendants.
3. a) On a alors P(A ∩ B) = PA (B) × P(A) = P(A) × P(B)
Il semble que la probabilité de gagner une partie soit donc PA (B) = P(B)
environ 0,8. et P(A ∩ B) = PA (B) × P(A) = P(A) × P(B)
On joue une partie au hasard et on note S l’événe- donc PA (B) = P(B).
ment « La somme des deux dés est inférieure ou Ainsi PA (B) = PA (B) = P(B) lorsque A et B sont indé-
égale à 9 » et G l’évènement « La partie est gagnée ». pendants.
9
G On démontre de la même façon que
10
PB (A) = PB (A) = P(A).
30 S
36
b) A et B sont indépendants lorsque la réalisation ou
1 G
10
non de A n’a pas d’influence sur la réalisation de B (et
1 réciproquement).
6 G
2
36 S 65 A est indépendant avec lui-même lorsque
1 G P(A ∩ A) = P(A) × P(A) ce qui équivaut à
2 P(A) = P(A)2 ou encore P(A)(1 − P(A)) = 0
Avec un tableau croisé, on constate qu’il y a 30 façons soit P(A) = 0 ou P(A) = 1.
d’obtenir une somme inférieure ou égale à 9. A est donc soit l’événement impossible soit l’événe-
D’après la formule des probabilités totales : ment certain.
9 30 1 6 5
P(G) = P(G ∩ S) + P(G ∩ S) = × + × = 66 a)  P(A ∩ B) = PA (B) × P(A) = PB (A) × P(B).
10 36 2 36 6
soit P(G) ≈ 0 , 83. b) A et A forment une partition de l’univers donc
d’après la formule des probabilités totales :
63 1. a)  PA (B) désigne la probabilité que B ne se P(B) = P(A ∩ B) + P(A ∩ B)
réalise pas sachant que A s’est réalisé. c) D’après le a),
PA(B) désigne la probabilité que B se réalise sachant P (B) × P(A)
PB (A) = A
que A s’est réalisé. P(B)
b)  E
A B PA (B) × P(A)
PB (A) =
P(A ∩ B) + P(A ∩ B)
PA (B) × P(A)
PB (A) =
A ÇB PA (B) × P(A) + PA (B) × P(A)

Chapitre 11  ★  Probabilités conditionnelles 191

172909_Chap11_000-000.indd 191 30/07/2019 15:45:29


67 a)  1 70 a) On note T l’événement « L’élève a lu le
4 I 7e tome » et F l’événement « L’élève a vu le 7e film ».
A On représente la situation par un arbre pondéré.
0,4 3 I 0,9 F
4 T
1
0,6 I 0,1 0,1
3 F
B
2 I F
0,9 0,55
3
T
b) A et B forment une partition de l’univers donc 0,45 F
d’après la formule des probabilités totales :
D’après la formule des probabilités totales :
P(I) = PA (I) × P(A) + PB (I) × P(B)
1 1 P(F) = P(T ∩ F) + P(T ∩ F)
P(I) = × 0 , 4 + × 0 , 6 = 0 , 3. P(F) = 0 ,1× 0 , 9 + 0 , 9 × 0 , 55
4 3
1 P(F) = 0 , 585.
× 0, 4
P(A ∩ I) 4 1 b) On en déduit la probabilité demandée :
c)  PI (A) = = = .
P(I) 0, 3 3 P(F ∩ T)
1 PF (T) =
Donc PI (A) ≠ . Le fournisseur se trompe. P(F)
2 0 , 09
PF (T) =
0 , 585
68 a)  O
0,426 PF (T) ≈ 0 ,154.
0,464 A
Rh+ 71 On note A (resp. F) l’événement « Le touriste
0,076 B choisi est Anglais (resp. Français) » et P l’événement
0,821 0,034
« Le touriste choisi parle français ».
AB
On représente la situation par un arbre pondéré.
0,2 P
O
0,179 A
0,503 0,6
0,402 A 0,8 P
Rh–
0,067 B 0,4 1
0,028 F P
AB Les événements A et F forment une partition de l’uni-
Par exemple, le tableau indique : vers, donc d’après la formule des probabilités totales :
P(Rh + ∩A) = 0 , 381 et P(Rh+) = 0 , 821 P(P) = P(A ∩ P) + P(F ∩ P)
P(Rh + ∩A) 0 , 381 P(P) = 0 , 2 × 0 , 6 + 1× 0 , 4
donc PRh+ (A) = = ≈ 0 , 464.
P(Rh+) 0 , 821 P(P) = 0 , 52.
b) Les événement Rh + et Rh - forment une parti- La probabilité que le touriste choisi comprenne les
tion de l’univers, donc d’après la formule des probabi- explications du guide est égale à 0,52.
lités totales. 0,85 S
PO = P(Rh + ∩O) + P(Rh − ∩O) 72 a)  T
PO ≈ 0 , 426 × 0 , 821 + 0 , 503 × 0 ,179 0,12 0,15 S
PO » 0 , 440.
La probabilité qu’une personne choisie au hasard soit S
0,88 0,25
du groupe O est d’environ 0,44.
T
0,75 S
69  
A A Total b) Les événements T et T forment une partition de
B 0,075 0,45 0,525 l’univers, donc d’après la formules des probabilités
B 0,175 0,3 0,475 totales :
Total 0,25 0,75 1 P(S) = P(T ∩ S) + P(T ∩ S)
Par exemple, P(S) = 0 , 85 × 0 ,12 + 0 , 25 × 0 , 88
P(A ∩ B) = PA (B) × P(A) = 0 , 3 × 0 , 25 = 0 , 075. P(S) = 0 , 322.
192

172909_Chap11_000-000.indd 192 30/07/2019 15:45:50


P(T ∩ S) PT (S) × P(T) b)  A Ç F est l’événement « Le licencié est une femme
c)  PS (T) = = adulte ».
P(S) 1 − P(S)
0 ,15 × 0 ,12 La probabilité cherchée est :
PS (T) = ≈ 0 , 027.
1 − 0 , 322 p = P(A ∩ F ; A ∩ F) + P(A ∩ F ; A ∩ F) + P(A ∩ F ; A ∩ F)
Sachant que l’équipementier ne l’a pas sponsorisé, la 23 37 37 23 23 23
p= × + × + ×
probabilité qu’il ait fini dans les trois premiers est 60 60 60 60 60 60
p » 0 , 62.
environ égale à 0,027.
77 Situation 1
73 On note A l’événement « Le voyageur a choisi le
a)  1er tirage 2e tirage
train de 7 h 27 » et R l’événement « Le voyageur arrive
en retard ». D’après la formule des probabilités 1
B2
5 1
totales :
B1 5O
P(R) = P(A ∩ R) + P(A ∩ R) 2

P(A ∩ R) = P(R) − P(A ∩ R) 3


1 V2
5
P(A ∩ R) = 0 , 06 − 0 , 8 × 0 , 05 5 1 B2
P(A ∩ R) = 0 , 02 1 5 1
P(A) × PA (R) = 0 , 02 5 O1 5O
2
0 , 2 × PA (R) = 0 , 02 3
5 V2
PA (R) = 0 ,1. 3
5 1 B2
74 1. a)  A est l’événement « Les élèves choisies 5 1
V1 5O
sont toutes les deux des filles ». 2

18 18 81 3
P(A) = P(F ; F) = × = . 5 V2
32 32 256
175 b)  P(A) = P(B1 ∩ V2 ) + P(O1 ∩ V2 ) + P(V1)
b)  P(A) = 1 − P(A) = . 3 1 3 1 3
256 P(A) = × + × +
2. a)  B est l’événement « Aucun des deux élèves 5 5 5 5 5
choisis n’est une fille » autrement dit « Les deux élèves P(A) = 0 , 84.
choisis sont des garçons ». La probabilité qu’au moins l’une des boules tirées soit
14 14 49 verte est égale à 0,84.
P(B) = P(G ; G) = × = . 1 1
32 32 256 P(B) = 1 − P(B) = 1 − P(O1 ∩ O2 ) = 1 − × = 0 , 96.
207 5 5
b)  P(B) = 1 − P(B) = . La probabilité que les deux boules tirées ne soit pas
256
oranges est égale à 0,96.
75 a) •  P(A ∩ B) = P(A) × P(B) Situation 2
= 0 , 01× 0 , 95 = 0 , 009 5. a)  er e
1 tirage 2 tirage
•  P(A ∩ B) = P(A) × P(B) = 0 , 99 × 0 , 05 = 0 , 049 5. 2
1
•  P(A ∩ B) = P(A) × P(B) = 0 , 99 × 0 , 95 = 0 , 940 5. 5 2
•  P(A ∩ B) = P(A) × P(B) = 0 , 01× 0 , 05 = 0 , 000 5. 1 52
b) Présenter uniquement le défaut B se traduit par
1
A Ç B. 2 3
5
5
La probabilité que ces deux enceintes présentent uni- 2 1
quement le défaut B est : 2 5 2
5 2 52
0 , 049 5 × 0 , 049 5 ≈ 0 , 002 5.
1
5 3
138 160 23 1
76 a)  P(A) × P(F) = × = .
240 240 60 5 2 1
92 23 5 2
P(A ∩ F) = = . 52
240 60 3
Ainsi P(A ∩ F) = P(A) × P(F) et les événements A et F 1
5 3
sont indépendants.

Chapitre 11  ★  Probabilités conditionnelles 193

172909_Chap11_000-000.indd 193 30/07/2019 15:46:15


2 2 p = 0 , 02 × 0 , 03 × 0 , 05 + 0 , 98 × 0 , 03 × 0 , 05
b)  P(C) = P(1 ; 1) = × = 0 ,16.
5 5 + 0 , 02 × 0 , 97 × 0 , 05 + 0 , 02 × 0 , 03 × 0 , 95
P(D) = 1 − P(D) p = 0 , 003 04.
P(D) = 1 − (P(1 ; 2) + P(2 ; 1)) Ainsi, sur 50 000 animaux, 152 animaux présenteront
2 2 2 2 cette mutation (50 000 × 0 , 003 04 = 152).
P(D) = 1 − × − ×
5 5 5 5
P(D) = 0 , 68. 81 On note F l’événement « Recours à une FIV » et J
l’événement « Naissance de jumeaux ». On peut
78 a) L’implication et vraie car :
représenter la situation par un arbre pondéré.
P(A ∩ B) = P(A) × PA (B).
La réciproque est « Si P(A ∩ B) = P(A) × P(B), alors 0,25 J
PA (B) = P(B) ». F
P(A ∩ B) 0,025 0,75
Elle est aussi vraie car PA (B) = . J
P(A)
b) L’implication est vraie, c’est la formule des probabi-
0,0125 J
lités totales. 0,975
La réciproque est «  Si P(C) + P(D) = P(A), alors F
0,9875 J
C = A ∩ B et D = A ∩ B  ». Elle est fausse car C et D
peuvent être deux événements sans rapport avec A
D’après la formule des probabilités totales :
et B.
P(J) = P(F ∩ J) + P(F ∩ J)
c) L’implication est vraie car si A et B sont incompa-
P(J) = 0 , 025 × 0 , 25 + 0 , 975 × 0 , 012 5
tibles, P(A ∩ B) = 0, or P(A) × P(B) ≠ 0.
La réciproque est « Si A et B ne sont pas indépen- P(J) ≈ 0 , 018.
dants, alors ils sont incompatibles ». Elle est fausse car La probabilité demandée est donc :
P(J ∩ F)
le fait que P(A) × P(B) ≠ P(A ∩ B) n’implique pas que PJ(F) =
P(A ∩ B) = 0. P(J)
d) L’implication est vraie car si A ∩ B = ∅, alors 0 , 006 25
PJ(F) =
P(A ∩ B) = 0 et donc PA (B) = 0. 0 , 018
PJ(F) ≈ 0 , 347.
La réciproque est « Si PA (B) = 0, alors A ∩ B = ∅  ».
Elle est vraie car PA (B) = 0 implique P(A ∩ B) = 0 L’affirmation de Tom n’est donc pas correcte. Il est
donc A ∩ B = ∅. plus probable que ce ne soit pas suite à une FIV.

79 a) Pour tout événement A ; on a PA (A) = 1. 82 On note A l’événement « le père est asthma-
b) Il existe un événement C tel que P(A) + P(C) = 1. tique », B l’événement « la mère est asthmatique » et
c) Pour tous événements A et B, on a C l’événement « l’enfant est asthmatique ».
PA (B) × P(A) = PB (A) × P(B). On peut donc représenter la situation par un arbre
pondéré.

0,9 C
B

Organiser son raisonnement


0,05
0,1 C
A
0,3 C
0,04 0,95
B
0,7 C
80 La probabilité que la mutation ait lieu est : C
0,3
p = P(G1 ∩ G2 ∩ G3 ) + P(G1 ∩ G2 ∩ G3 )
0,96 B
+ P(G1 ∩ G2 ∩ G3 ) + P(G1 ∩ G2 ∩ G3 ) 0,05
0,7 C
Or les trois gènes sont indépendants et donc les trois A
mutations le sont, donc 0,1 C
0,95
p = P(G1) × P(G2 ) × P(G3 ) + P(G1) × P(G2 ) × P(G3 ) B
+ P(G1) × P(G2 ) × P(G3 ) + P(G1) × P(G2 ) × P(G3 ) 0,9 C

194

172909_Chap11_000-000.indd 194 30/07/2019 15:46:47


D’après la formule des probabilités totales : 1
2. a) D’après l’énoncé, P(A1) = .
P(C) = 0 , 04 × 0 , 05 × 0 , 9 + 0 , 05 × 0 , 95 × 0 , 3 3
+ 0 , 95 × 0 , 05 × 0 , 3 + 0 , 95 × 0 , 95 × 0 ,1 1 3
b)  PB1 (A2 ) = et PC1 (A2 ) = .
P(C) = 0 ,118 8 4 4
La probabilité qu’aucun des parents ne soient asth- c) Arbre de probabilités :
matiques sachant que l’enfant ne l’est pas est : 3
4 B2
P(A ∩ B ∩ C)
PC (A ∩ B) = A1
P(C)
0 , 95 × 0 , 95 × 0 , 9 1 1 C2
PC (A ∩ B) = 3 4
1 − 0 ,118 8 3
1 C2
4
PC (A ∩ B) ≈ 0 , 931 3 B1
La probabilité demandée est : 1 A2
1 − PC (A ∩ B) ≈ 0 , 069. 1 4 3
3 4 A2
83 1. a) La variable p correspond à la position de la
C1
fourmi. La variable a prend des valeurs aléatoires qui
1 B2
indiquent dans quelle direction la fourmi se déplace.
4
b) Il faut compléter l’algorithme par les lignes sui-
d) D’après la formule des probabilités totales,
vantes, avant Fin Si :
1 1 1 3 4 1
P(A2 ) = × + × = = .
Si p = 2 alors 3 4 3 4 12 3
a ¬ un nombre aléatoire de [0 ; 1] 1 1
e) De même, P(B2 ) = et P(C2 ) = .
3 3
Si a < 0 , 75 alors
p¬3 84 a)  1er tirage 2e tirage Produit Somme
Sinon 2
n+2 1 1 2
p ¬1
2 1
Fin Si n
n+2 n n n+1
Fin Si n+2
Si p = 3 alors 2
n
a ¬ un nombre aléatoire de [0 ; 1] n+2 1 n n+1
n+2
Si a < 0 , 75 alors n
n n n2 2n
p ¬1
n+2
sinon
2 n n 2 4n
p¬2 P(A) = × + × =
n + 2 n + 2 n + 2 n + 2 (n + 2)2
Fin Si
2 2 4
Fin Si P(B) = × =
n + 2 n + 2 (n + 2)2
c) Voici le programme en Ainsi, P(A) = 10P(B) équivaut à 4 n = 10 × 4 soit
langage Python. n = 10.
n n n2
b)  P(C) = × = .
n + 2 n + 2 (n + 2)2
2 n n 2 4n
P(D) = × + × = .
n + 2 n + 2 n + 2 n + 2 (n + 2)2
Ainsi, P(C) = 5P(D) équivaut n2 = 5 × 4 n soit n = 0
ou n = 20 (mais n ¹ 0 ) donc l’unique solution
est n = 20.

1 1 5
85 PA (B) = 1 − PA (C) − PA (D) = 1 − − = .
4 8 8
P(A ∩ B) PA (B) × P(A)
PB (A) = =
P(B) PA (B) × P(A) + PA (B) × P(A)

Chapitre 11  ★  Probabilités conditionnelles 195

172909_Chap11_000-000.indd 195 30/07/2019 15:47:22


1 b) aire (OABC) = 1× 1 = 1.
PA (B) ×
Ainsi, 0 , 4 = 2 c) • ligne 6 : on choisit une valeur de x au hasard de
1 5 1 l’intervalle [0 ; 1].
PA (B) × + ×
2 8 2 • ligne 7 : on choisit une valeur de y au hasard de l’in-
ce qui est successivement équivalent à :
 1 5 1 tervalle [0 ; 1].
0 , 4 PA (B) × +  = PA (B) × • ligne 10 : Le compteur s’incrémente lorsque le point
 2 16  2
0 , 2 × PA (B) + 0 ,125 = 0 , 5PA (B) M( x ; y) est situé dans l’aire bleue et A donne la fré-
0 ,125 = 0 , 3PA (B) quence obtenue pour la simulation de 10 000 expé-
5 riences.
PA (B) = .
12 d) Valeur approchée de l’aire bleue affichée par le
programme : A » 0 , 333 4.
86 a) On peut conjecturer que Lauriane a raison,
2. a) Voici le programme en langage Python.
sauf de n = 1 à n = 2.
b) On peut représenter la situation par un arbre pondéré,
où on indique la position du pion, de la gauche vers la
droite, par un nombre entier compris entre -2 et 2.
1
1--
n -2

1 -1
1--
n 1
- 0
n

1 1
1-- 0
-
n n b) Valeur approchée de l’aire bleue affichée par le
1 programme : 0,784 73
1
- 2
n (pour la simulation de 100 000 expériences).
La probabilité que le pion soit au centre après deux 1
c) Le disque bleu a pour rayon , donc :
déplacements est donc : 2
   1 2
1 − 1  × 1 × 2 = 2 − 2 = 2n − 2 Aire (bleue) = π ×   ≈ 0 , 784 73
 n  n n n2 n2  2 
On peut alors étudier la fonction f définie sur Ainsi, π ≈ 4 × 0 , 784 73 ≈ 3,138 92.
2x − 2
[0 ; + ∞[ par f( x) = .
x2 89 Test A 0,8 T1
−2 x + 4
f ′( x) = n donc f ′( x) est négatif pour 1 M
x3 200 0,2 T1
x > 2, donc la fonction f est décroissante pour
n > 2. Lauriane a donc raison, sauf de n = 1 à n = 2.
199 0,05 T1

87 On note A l’événement « Au moins l’une des 200 M


trois pommes choisies par Blanche-Neige est empoi- 0,95 T1
sonnée ». P(T1 ∩ M) P(T1 ∩ M)
PT1 (M) = =
Donc A est l’événement « Aucune des trois pommes P(T1) P(T1 ∩ M) + P(T1 ∩ M)
n’est empoisonnée ». 1
13 12 11 0, 8 ×
P(A) = 1 − P(A) = 1 − × × PT1 (M) = 200 ≈ 0 , 074.
16 15 14 1 199
0, 8 × + 0 , 05 ×
P(A) ≈ 0 , 49. 200 200
La probabilité que Blanche-Neige soit réveillée bien Test B
plus tard par son Prince Charmant est environ égale On procède de la même façon et on obtient :
à 0,49. 1
0 , 95 ×
PT2 (M) = 200 ≈ 0 , 046.
88 1. a) •  M( x ; y) est situé dans le carré OABC si, et 1 199
0 , 95 × + 0 ,1×
seulement si, 0 < x < 1 et 0 < y < 1. 200 200
•  M( x ; y) est situé dans le domaine bleu si et seule- La valeur prédictive positive du test B est donc infé-
ment si, 0 < x < 1 et 0 < y < x2 . rieure à celle du test A.
196

172909_Chap11_000-000.indd 196 30/07/2019 15:47:55


90 On schématise la situation par un arbre pondéré 0,03 E
où V est l’événement « La personne dit la vérité ». U
0,3 0,97
Karim Natacha Jérémy E
2
3 V 0,05 E
2 0,7
V V
3
1 V 0,95 E
V 2 3
V
Les événements U et V forment une partition de l’uni-
2
1 3 vers, donc d’après la formule des probabilités totales :
3 V
3
1
P(E) = P(U ∩ E) + P(V ∩ E)
V
3 2 P(E) = 0 , 03 × 0 , 3 + 0 , 05 × 0 , 7
3 V P(E) = 0 , 044.
2 La probabilité que le paquet prélevé porte le label
1 V
3 « extrafin » est égale à 0,044.
3 1 V
V 3 P(U ∩ E) 0 , 03 × 0 , 3
2 b)  PE (U) = = ≈ 0 , 205.
V P(E) 0 , 044
1 3
V Sachant qu’un paquet porte le label « extrafin », la
3
1 V probabilité que le sucre qu’il contient provienne de
3 l’exploitation U est environ égale à 0,205.
2. On note p la probabilité de U.
La probabilité qu’Assma ait réellement gagné au loto
P(U ∩ E) 0 , 03 × p
 2 3 8 PE (U) = = = 0, 3
est   = . P(E) 0 , 03 × p + 0 , 05(1 − p)
 3  27 ce qui est successivement équivalent à
0 , 03 p = 0 , 3(0 , 03 p + 0 , 05(1 − p))
91 Pour chaque question, la probabilité :
0 , 03 p = 0 , 009 p + 0 , 015 − 0 , 015 p
• qu’Enzo et Théo répondent correctement est :
0 , 036 p = 0 , 015
0 , 8 × 0 , 6 = 0 , 48 5
• qu’Enzo et Théo se trompent est : p= .
12
0 , 2 × 0 , 4 = 0 , 08 5
de son approvisionnement doit provenir de l’ex-
• que le jeu continue est donc : 12 7
0 , 48 + 0 , 08 = 0 , 56. ploitation U et de l’exploitation V.
12
• qu’Enzo gagne est :
0 , 8 × 0 , 4 = 0 , 32
• que Théo gagne est :

Exploiter ses compétences


0 , 2 × 0 , 6 = 0 ,12.

1re question 2e question 3e question 4e question

0,56 C
0,32
0,56 C E
93 Lorsque le joueur choisit, il a une chance sur 3
0,32 0,12 T
0,56 C E de désigner la bonne porte. Il avait donc 2 chances
0,32 sur 3 de désigner une mauvaise porte.
C E 0,12 T
0,56 1
S’il garde son choix, la probabilité de gagner est .
0,32 0,12 T
3
E
S’il modifie son choix, il gagne si son choix initial était
0,12 T
2
perdant, donc avec une probabilité de .
3
La probabilité qu’Enzo soit gagnant à la 4e question Il a donc intérêt à changer.
est 0 , 563 ´ 0 , 32 soit environ 0,056.
94 On note :
92 1. a) On représente la situation par un arbre M l’événement « Le voyageur passe le portique avec
pondéré. un objet métallique ».

Chapitre 11  ★  Probabilités conditionnelles 197

172909_Chap11_000-000.indd 197 30/07/2019 15:48:12


S l’événement « Le portique de sécurité sonne », D’après la formule des probabilités totales :
T l’événement « L’agent de sécurité trouve un objet P(C) = P(M) × PM (C) + P(D) × PD (C) + P(P) × PP (C)
métallique sur le voyageur ». P(C) = 0 , 3 × 0 , 5 + 0 , 4 × 0 , 2 + 0 , 3 × 0 , 05
On représente la situation par un arbre pondéré de P(C) = 0 , 245.
probabilités. On représente à nouveau la situation par un arbre
0,99 S
pondéré pour le modèle actuel.
1 M
0,01 0,8 C
20 S
M
S 0,2 C
19 0,02
0,3
20 M C
0,5
0,98 S 0,4
D
D’après la formule des probabilités totales : 0,5 C
P(S) = P(M) × PM (S) + P(M) × PM (S) 0,3
1 19 0,1 C
P(S) = × 0 , 99 + × 0 , 02
20 20 P
P(S) = 0 , 068 5. 0,9 C
On peut construire alors un autre arbre pondéré de
probabilités. D’après la formule des probabilités totales :
3
T P(C) = P(M) × PM (C) + P(D) × PD (C) + P(P) × PP (C)
4
S P(C) = 0 , 3 × 0 , 8 + 0 , 4 × 0 , 5 + 0 , 3 × 0 ,1
0,0685 1 P(C) = 0 , 47.
T
4 Finalement, la probabilité pour qu’une espèce ani-
male choisie au hasard ait disparu d’ici 50 ans est de
0,9315 0,245 selon le modèle écologique et de 0,47 selon le
S 1 T
modèle actuel.
3
Ainsi, P(T) = P(S) × PS (T) = 0 , 068 5 ×
4 96 On note :
P(T) = 0 , 051375.
A l’événement « La gardienne a arrêté le tir au but »
La probabilité que l’agent de sécurité trouve un objet
T l’événement « La gardienne a arrêté au moins l’un
métallique sur ce voyageur est 0 , 051375.
des trois premiers tirs au but ».
95 On note : Donc T est l’événement « La gardienne n’a arrêté
M l’événement « L’espèce animale choisie est mena- aucun des trois premiers tirs au but ».
cée », On représente la situation pour chaque gardienne par
D l’événement « L’espèce animale choisie est en danger », un arbre pondéré de probabilités.
P l’événement « L’espèce animale choisie est peu Gardienne de l’équipe rouge.
menacée »,
1er tir 2e tir 3e tir
C l’événement « L’espèce animale choisie va dispa- A
0,2
raître d’ici cinquante ans ».
A
On représente la situation par un arbre pondéré de 0,2
0,8 A
probabilités par le modèle écologique. A
0,5 C 0,2 A
0,2 0,8
M A
0,5 C 0,8 A
0,3
0,2 C 0,2 A
0,4 A
D 0,8 0,2
0,8 C 0,8 A
0,3 A
0,05 C 0,2 A
0,8
P A
0,95 C 0,8 A

198

172909_Chap11_000-000.indd 198 30/07/2019 15:48:22


T correspond au dernier chemin sur l’arbre, donc T correspond au dernier chemin sur l’arbre, donc
P(T) = 0 , 8 × 0 , 8 × 0 , 8 = 0 , 512. P(T) = 0 , 82 × 0 , 85 × 0 , 85 = 0 , 592 45.
Ainsi, P(T) = 1 − P(T) = 0 , 488. Ainsi, P(T) = 1 − P(T) = 0 , 407 55.
La probabilité que la gardienne de l’équipe rouge La probabilité que la gardienne de l’équipe bleue
arrête au moins l’un des trois premiers tirs au but est arrête l’un des trois premiers tirs au but est 0,407 55.
0,488.
Gardienne de l’équipe bleue.
1er tir 2e tir 3e tir
0,22 A
A
0,22
0,78 A
A
0,15 A
0,18 0,78
A
0,85 A

0,22 A

0,82 A
0,15
0,78 A
A
0,15 A
0,85
A
0,85 A

Chapitre 11  ★  Probabilités conditionnelles 199

172909_Chap11_000-000.indd 199 30/07/2019 15:48:26


172909_Chap11_000-000.indd 200 30/07/2019 15:48:26
12 Variables aléatoires

Découvrir Acquérir des automatismes

1 Définir une variable aléatoire 3 Le dé est équilibré donc on modélise cette expé-
rience aléatoire par une loi équirépartie.
1  a) 1re pièce Les valeurs prises par X sont 0, 2, 10 et 20.
2e pièce Issues Valeurs de X
Voici la loi de probabilité de X :
P PP 2
P a 0 2 10 20
F PF 1
1 1 1 1
P FP 1 P(X = a)
F 6 3 3 6
F FF 0
b) X prend les valeurs 0, 1 et 2.
4 Les valeurs prises par X sont 2, 5, 10 et 50.
2   a 0 1 2 Voici la loi de probabilité de X :
1 1 1 a 2 5 10 50
P(X = a)
4 2 4 25 10 5 1
P(X = a)
41 41 41 41

5 On utilise la loi de probabilité de X, qui a été


2  Calculer l'espérance déterminée à l’exercice 3 .
d'une variable aléatoire a)  P(X < 10) = P(X = 0) + P(X = 2)
1 1 1
1  a) G peut prendre les valeurs 1 000, 1 250 et donc P(X < 10) = + = .
6 3 2
1 500.
b)  P(X > 2) = P(X = 2) + P(X = 10) + P(X = 20)
b) a 1 000 1 250 1 500
1 1 1 5
3 1 1 donc P(X > 2) = + + = .
P(G = a) 3 3 6 6
5 5 5
c)  P(X > 5) = P(X = 10) + P(X = 20)
3 1 1 1 1 1
2  a)  E(G) = × 1 000 + × 1 250+ × 1 500 donc P(X > 5) = + = .
5 5 5 3 6 2
c’est-à-dire E(G) = 1 150. d)  P(X < 10) = P(X = 0) + P(X = 2) + P(X = 10)
b)  E(G) est exprimée en euros. 1 1 1 5
donc P(X < 10 ) = + + = .
c) Un candidat ne peut pas gagner la somme 6 3 3 6
E(G), car les seuls gains possibles sont 1 000 €, 6 On utilise la loi de probabilité de X, qui a été
1 250 € et 1 500 €. déterminée à l’exercice 4.
d) La candidate peut espérer gagner en moyenne a)  P(X < 10) = P(X = 2) + P(X = 5)
1 150 € par jour, soit un total de 115 000 € en 25 10 35
donc P(X < 10) = + = .
100 jours. 41 41 41

Chapitre 12  ★  Variables aléatoires 201

172909_Chap12_000-000.indd 201 26/07/2019 15:37:55


b)  P(X < 5) = P(X = 2) + P(X = 5) 12 On peut représenter la situation par un arbre :
25 10 35 1er jeton 2e jeton Valeurs de X
donc P(X < 5) = + = .
41 41 41 1 2
1
c)  P(X < 6) = P(X = 2) + P(X = 5) 2 3
35 1 3
donc P(X < 6) = . 2
41 2 4
d)  P(X > 2) = P(X = 10) + P(X = 50) Olmo a donc tort, X peut prendre les valeurs 2, 3 et 4.
5 1 6
donc P(X > 2) = + = . 13 {X = 2}
41 41 41
14 L’événement {X = 2} signifie que le chiffre des
9 On peut représenter la situation par un arbre :
dizaines de la date du jour est 2. Il est réalisé pour
1er jeton 2e jeton Valeurs de X tous les jours du 20 au 29. La réponse d’Alyah est
5 6 donc fausse.
1
10 11
5 7 15 a)  {X = 1} b)  {X > 2}
2
10 12
16 a)  {X = 1} b)  {X < 1, 50}
On peut alors établir la loi de probabilité de X :
a 6 7 11 12 17 a) « On obtient le numéro 6 deux fois. »
1 1 1 1 b) « On obtient le numéro 6 cinq fois. »
P(X = a)
4 4 4 4 c) « On obtient le numéro 6 une fois ou moins. »
L’espérance de X est donc : d) « On obtient le numéro 6 strictement plus de trois
1 1 1 1 36 fois. »
E(X) = × 6 + × 7 + × 11 + × 12 = = 9.
4 4 4 4 4
18 a) « Exactement un billet sort de l’appareil. »
10 a) L’espérance de X est : b) « Exactement trois billets sortent de l’appareil. »
E(X) = 0 , 2 × 10 + 0 , 3 × 20 + 0 , 4 × 40 + 0 ,1× 50 = 29. c) « Strictement moins de trois billets sortent de l’ap-
b) Pour calculer la variance de X, on complète le pareil. »
tableau ci-dessous. d) « Deux billets ou plus sortent de l’appareil. »

a 10 20 40 50 19 Voici la loi de probabilité de X :


a - 29 - 19 - 9 11 21
a - 2 10
(a - 29)² 361 81 121 441 3 1
P(X = a) 0,2 0,2 0,4 0,1 P(X = a)
4 4

La variance de X est donc :


V(X) = 0 , 2 × 361 + 0 , 3 × 81 + 0 , 4 × 121 + 0 ,1× 441 = 189.
20 a) Les valeurs prises par G sont : 98, 48, 8 et - 2.
L’écart-type de X est donc σ(X) = 189 , b) Il y a 111 billets gagnants, donc 889 billets per-
889
soit σ(X)  13, 7. dants. Par conséquent, P(G = −12) = = 0 , 889.
1 000
c) On vérifie ces résultats à l’aide de la calculatrice. c) Voici la loi de probabilité de G :
a - 2 8 48 98
P(X = a) 0,889 0,1 0,01 0,001

21 a) 1 2 3 4 5 6
1 1 2 3 4 5 6
2 2 4 6 8 10 12
3 3 6 9 12 15 18
4 4 8 12 16 20 24
11 X peut prendre les valeurs : 0,5 ; 2 ; 1,5 ; 2 ; 2,5 ; 5 5 10 15 20 25 30
3. L’affirmation d’Alicia est donc fausse. 6 6 12 18 24 30 36

202

172909_Chap12_000-000.indd 202 26/07/2019 15:38:11


b) Voici la loi de probabilité de X : 26 1re case 2e case 3e case
a 1 2 3 4 5 6 8 V B……… X = 3
1 2 2 3 2 4 2 R
P(X = a) B V……… X = 1
36 36 36 36 36 36 36
R B……… X = 1
V
a 9 10 12 15 16 18 20 B R……… X = 0
1 2 4 2 1 2 2 R V……… X = 0
P(X = a) 36 36 36 36 36 36 36 B
V R……… X = 1

a 24 25 30 36 a) Les valeurs prises par X sont 0, 1 et 3.


2 1 2 1 b) La loi de probabilité de X est donnée dans le tableau.
P(X = a)
36 36 36 36 a 0 1 3
1 1 1
P(X = a)
22 a) On peut représenter la situation par un arbre : 3 2 6
1er jeton 2e jeton Valeurs de X
c)  P(X > 2) = P(X = 3) =
1
2 0 6
0
1 - 1 27 a)  (X < 2) : « Le standard a reçu moins de
0 1 2 appels durant une minute ».
1
1 0 P(X < 2) = 0 ,1 + 0 ,15 + 0 , 25 = 0 , 5.
b) Voici la loi de probabilité de X : (1 < X < 4) : « Le standard a reçu entre 1 et 4 appels
a - 1 0 1 durant une minute ».
1 1 1 P(1 < X < 4) = 0 ,15 + 0 , 25 + 0 , 3 + 0 ,15 = 0 , 85.
P(X = a) b)  P(X > 3) = 0 , 3 + 0 ,15 + 0 , 05 = 0 , 5
4 2 4

23 a) X prend les valeurs 0, 1 et 4. 28 a) La somme des probabilités est égale à 1 donc la
b) Voici la loi de probabilité de X : valeur manquante est P(X = 3) = 1 − 0 , 3 − 0 , 5 = 0 , 2.
b)  P(X > 10) = P(X = 2) + P(X = 3)
a 0 1 4
= 0, 5 + 0, 2 = 0, 7
1 2 2
P(X = a)
5 5 5
29 a)  p1 + p2 + p3 + p4 = 1
donc p1 + p1 + p1 + 3 p1 = 1, ce qui donne 6 p1 = 1
24 a) Voici la loi de probabilité de X :
1
a 1 5 10 puis p1 = .
6
19 12 1 On en déduit la loi de probabilité de X :
P(X = a)
32 32 32
a 5 10 15 20
19 12 31
b)  P(X < 5) = P(X = 1) + P(X = 5) = + = 1 1 1 3
32 32 32 P(X = a)
6 6 6 6
La probabilité de gagner 5 points ou moins est égale
31 b)  P(X > 10) = P(X = 10) + P(X = 15) + P(X = 20)
à .
32 1 1 3 5
P(X > 10) = + + =
6 6 6 6
25 a) On peut utiliser un arbre pour déterminer les p
p p
4 chemins possibles. 30 a)  p1 = 2 = 3 = 4
2 3 4
2 1……… X = 5 donc p2 = 2 p1, p3 = 3 p1 et p4 = 4 p1.
1
1……… X = 4 Or p1 + p2 + p3 + p4 = 1
1 donc p1 + 2 p1 + 3 p1 + 4 p1 = 1
1
1____1……… X = 5 ce qui donne 10 p1 = 1 et donc p1 = 0 ,1.
Par conséquent, p2 = 0 , 2, p3 = 0 , 3 et p4 = 0 , 4.
3 1……… X = 5 b) Voici la loi de probabilité de X :
b) Voici la loi de probabilité de X : a 1 2 3 4
a 4 5 P(X = a) 0,1 0,2 0,3 0,4
1 3
P(X = a) c)  P(X < 3) = P(X = 1) + P(X = 2) + P(X = 3) = 0 , 6.
4 4

Chapitre 12  ★  Variables aléatoires 203

172909_Chap12_000-000.indd 203 26/07/2019 15:38:56


31 a) Voici les différentes façons de rendre les clés 38 a) Voici la loi de probabilité de X :
et les valeurs de X correspondantes. a 0 5 10
Locataire 1 Locataire 2 Locataire 3 Valeur de X P(X = a) 0,8 0,15 0,05
Clé 1 Clé 2 Clé 3 3
Clé 1 Clé 3 Clé 2 1 b) L'espérance de X est :
Clé 2 Clé 1 Clé 3 1 E(X) = 0 , 8 × 0 + 0 ,15 × 5 + 0 , 05 × 10
Clé 2 Clé 3 Clé 1 0
soit E(X) = 1, 25.
Clé 3 Clé 1 Clé 2 0
Clé 3 Clé 2 Clé 1 1
39 a) Dé 1
On en déduit la loi de probabilité de X :
Voici la loi de probabilité de X :
a 0 1 3
a 0,5 1 2 3
2 3 1
P(X = a) 1 1 1 1
6 6 6 P(X1 = a)
3 6 6 3
b)  P(X > 1) = P(X = 1) + P(X = 3)
Dé 2
3 1 4 2
P(X > 1) = + = = Voici la loi de probabilité de X :
6 6 6 3
a 1 2 3
32 Les points indiqués sur les lettres sont toujours 1 1 1
P(X2 = a)
positifs, donc l’espérance de X est un nombre positif. 2 3 6
L’affirmation d’Eddy est donc fausse. Dé 3
Voici la loi de probabilité de X :
1 3 7
33 E(X) = 1× + 2× = a 0 1 4
4 4 4
1 1 1
P(X3 = a)
34 E(X) = 0 × 0 , 3 + 2 × 0 , 4 + 3 × 0 ,1 + 5 × 0 , 2 = 2,1 3 3 3
L’affirmation de Marinella est vraie. 5
b)  E(X1) = ≈ 1,66
3
35 Voici la loi de probabilité de X : 5
E(X2 ) = ≈ 1,66
a 1 5 10 20 3
3 4 1 1 5
P(X = a) E(X3 ) = ≈ 1,66
9 9 9 9 3
3 4 1 1 c) Peu importe puisque ces trois dés correspondent à
E(X) = × 1 + × 5 + × 10 + × 20 la même espérance.
9 9 9 9
53
E(X) = , soit E(X) ≈ 5, 89. 40 On calcule l’espérance de chacun de ces jeux
9
grâce à la calculatrice.
36 a) Voici la loi de probabilité de X : Jeu 1 : E(X) = 0 donc ce jeu est équitable.
a 2 5 10 20 50 Jeu 2 : E(X) = −0 ,125 ≠ 0 donc ce jeu n’est pas équi-
1 1 7 1 1 table.
P(X = a) Olivier s’est donc trompé.
4 4 20 10 20
1 1
b)  E(X) = × 2 +  + × 50 = 9 , 75 41 X est la variable aléatoire qui donne le gain d’un
4 20
Sur un grand nombre de bons choisis au hasard, la joueur. L’espérance de X est :
réduction est, en moyenne, de 9,757 € par bon. E(X) = 0 , 97 × 0 + 0 , 01× 100 + 0 , 01× 200
+ 0 , 005 × 300 + 0 , 005 × 400
37 Voici la loi de probabilité de X : ce qui donne E(X) = 6 , 5.
Pour que le jeu soit équitable, l’opérateur doit donc
a 1 5 20 100
fixer la mise à 6,5 €.
1 1 1 1
P(X = a)
6 6 2 6
42 a) Voici la loi de probabilité de X :
1 1 1 1
E(X) = × 1 + × 5 + × 20 + × 100 ≈ 27, 66. a -4 -3 -2 1 m 4
6 6 2 6
Sur un grand nombre de parties, un joueur gagnera, 2 1 1 1 1 1
P(X = a)
en moyenne, environ 27,66 $ par partie. 9 3 9 9 9 9

204

172909_Chap12_000-000.indd 204 26/07/2019 15:39:26


2 1 1 1 45 Voici les résultats obtenus à la calculatrice :
b)  E(X) = × (−4) + × (−3) + × (−2) + × 1
9 3 9 9
1 1
+ ×m + ×4
9 9
14 1
E(X) = − + m
9 9
c) Le jeu est équitable lorsque E(X) = 0,
14 1
c’est-à-dire − + m = 0 , d’où m = 14.
9 9
43 a) On note X la variable aléatoire qui donne le
gain de ce jeu.
46 a) X prend les valeurs - 2, 0, 3, 8 et 48.
Voici la loi de probabilité de X :
Voici la loi de probabilité de X.
a - 1 4 9
xi -2 0 3 8 48
n-4 3 1
P(X = a) 14 3 1 1 1
n n n P(X = a)
20 20 20 20 20
n−4 3 3
E(X) = × (−1) + × 4 + × 9
n n n b) Voici les résultats obtenus à la calculatrice :
25 − n
E(X) =
n
Le jeu est équitable lorsque E(X) = 0, c'est-à-dire
pour n = 25.

b) Voici la loi de probabilité de X :


a 0-m 5-m 10 - m
4 3 1
P(X = a)
5 20 20
4 3 1 47 Voici les résultats obtenus à la calculatrice :
E(X) = × (−m) + × (5 − m) + × (10 − m)
5 20 20
25 − 20m
E(X) =
20
Le jeu est équitable lorsque E(X) = 0, c’est-à-dire
lorsque 25 − 20m = 0 , donc m = 1, 25.
La mise doit donc être de 1,25 €.

44 On note m le nombre de billes portant le


numéro 2 ajoutée(s) dans le sac.
X est la variable aléatoire qui indique le numéro de la 48 a) Le tableau ci-dessous indique la valeur de X
bille prélevée. pour chaque tirage possible.
Voici la loi de probabilité de X :
Dé 2 \ Dé1 1 2 3 4 5 6
a -2
1 2 4 1 0 1 2 3 4 5
5 2 m 1 2 - 1 0 1 2 3 4
P(X1 = a)
m+8 m+8 m+8 m+8
3 - 2 - 1 0 1 2 3
5 2 4 - 3 - 2 - 1 0 1 2
E(X) = × (−2) + ×1
m+8 m+8 5 0 1
- 4 - 3 - 2 - 1
m 1
+ ×2 + ×4 6 - 5 - 4 - 3 - 2 - 1 0
m+8 m+8
−4 + 2m On en déduit la loi de probabilité de X :
E(X) =
m+8
a - 5 - 4 - 3 - 2 - 1 0 1 2 3 4 5
Le jeu est équitable à condition que E(X) = 0, donc il
faut que −4 + 2m = 0 , c’est-à-dire m = 2. Il faut 1 2 3 4 5 6 5 4 3 2 1
P(X = a)
donc ajouter deux billes portant le numéro 2. 36 36 36 36 36 36 36 36 36 36 36

Chapitre 12  ★  Variables aléatoires 205

172909_Chap12_000-000.indd 205 26/07/2019 15:40:08


b) Voici les résultats obtenus à la calculatrice : L’affirmation de Jean-Baptiste est donc fausse. X et Y ont
des espérances différentes et des écarts-types différents.

50 1. B.     2. C.     3. C.     4. A.     5. B

51 1. C, D.     2. C.     3. B, D.     4. A, B.

52 1. Faux. En effet, les secteurs ne sont pas super-


posables, donc les probabilités des événements
{X = 10}, {X = 20} et {X = 30} ne sont pas égales.
c) Voici la loi de probabilité de Y :
2. Vrai. En effet, voici la loi de probabilité de X :
a 0 1 4 9 16 25
a 10 20 30
6 10 8 6 4 2
P(Y = a) 1 1 1
36 36 36 36 36 36 P(X = a)
6 3 2
d) Voici les résultats obtenus à la calculatrice :
Ainsi,
1 1 1
P(X = 10) + P(X = 20) = + = = P(X = 30).
6 3 2
3. Faux. En effet,
1 1 1 70
E(X) = × 10 + × 20 + × 30 = .
6 3 2 3
4. Vrai. En effet, on vérifie à la calculatrice que
500
V(X) =  55, 56.
9
500
5. Vrai. En effet, V(X) = et donc
49 Voici la loi de probabilité de X : 9
a 1 2 3 4 5 6 500 500 10
σ(X) = = = 5.
1 1 1 1 1 1 1 1 1 9 9 3
P(X = a)
10 10 10 10 10 10 10 10 10
53 a) L’issue qui réalise l’événement {X = 1} est PP.
On obtient alors les paramètres ci-dessous à la calcu- b) Les issues qui réalisent l’événement {X = 0} sont
latrice. PF et FP.
c) Les issues qui réalisent l’événement {X > 0} sont
PP, PF et FP.

54 a)  {X = 20}             b)  {X > 10}

55 a) X prend les valeurs 101 = 10 , 102 = 100


et 103 = 1 000.
b) L’événement {X = 10} est réalisé quand on tire
Voici la loi de probabilité de Y : une boule numérotée 1. Or, il y a trois boules numéro-
a 1 3 5 6 8 10 tées 1 sur les dix boules de l’urne. Par conséquent,
1 1 1 1 1 1 3
P(X = a) P(X = 10) = = 0,3.
10 10 10 10 10 10 10
On obtient alors les paramètres ci-dessous à la calcu- c) Voici la loi de probabilité de X :
latrice. a 10 100 1 000
3 2 5
P(X = a)
10 10 10

56 a)  E(X) = 0 , 71× 0 + 0 , 2 × 5 + 0 , 05 × 20


+ 0 , 03 × 100 + 0 , 01× 1 000
ce qui donne E(X) = 15. Cela signifie qu’en jouant un
grand nombre de parties, le joueur peut espérer
gagner 15 euros par partie.
206

172909_Chap12_000-000.indd 206 26/07/2019 15:40:35


b) Voici les résultats obtenus à la calculatrice :

61 Voici la feuille de calcul adaptée :

S'entraîner

58 a) La première partie de l’algorithme permet de


calculer l’espérance : E(X) = 15, 5. Voici le suivi des
valeurs lors de l’exécution de l’algorithme à partir de
l’instruction V ¬ 0. On a arrondi les valeurs déci-
males au centième.
n 4 4 4 4 4
i 0 1 2 3 62 Voici la feuille de calcul adaptée :
A[i] 5 10 25 50
A[i] - E -10,5 -5,5 9,5 34,5
(A[i] - E)² 110,25 30,25 90,25 1190,25
P[i] 0,3 0,4 0,2 0,1
((A[i] - E)² ´ P[i] 33,08 12,10 18,05 119,03
V 0 33,08 45,18 63,23 182,25
La valeur obtenue à la fin de l’algorithme est 182,25.
b) La valeur V obtenir à la fin de l’algorithme est la 63 1. a) 
5
variance de la variable aléatoire X.
c) On ajoute à la fin de l’algorithme l’instruction
∑ 10i = 10 ×1 + 10 × 2 + 10 × 3 + 10 × 4 + 10 × 5
(i =1)
s ¬ V. 5
donc ∑ 10i = 150.
59 a) On obtient 15,5, qui est bien l’espérance de la (i =1)
5
variable aléatoire X. b)  ∑ ki = k ×1 + k × 2 + k × 3 + k × 4 + k × 5
b) La commande len(A) donne la longueur de la liste (i =1)
A, c’est-à-dire son nombre de termes. 5
c) Voici le programme complet donc ∑ ki = 15k .
(i =1)

Chapitre 12  ★  Variables aléatoires 207

172909_Chap12_000-000.indd 207 26/07/2019 15:40:45


n 65 X peut prendre les valeurs : 85 + 235 = 320 ;
2. a)  ∑ kai = k × a1 + k × a2 + ... + k × an 170 + 1 340 = 1 510 ; 845 + 5 475 = 6 320.
i =1
a 320 1 510 6 320
n
P(X = a) 0,34 0,48 0,18
∑ kai = k × (a1 + a2 + ... + an )
i =1

n n 66 a) Le tableau ci-dessous indique la valeur de X


∑ kai = k × ∑ai pour chaque tirage possible.
i =1 i =1
n
Dé 2 \ Dé1 1 1 2 2 3 3
b)  ∑ k = k
+k + ... + k = n × k

1 1 1 1 1 1 1
i =1 n fois 2 2 2 1 1 2 2
n 3 3 3 3 3 1 1
c)  ∑ kai = k × a1 + k × a2 + ... + k × an 4 4 4 2 2 4 4
i =1
5 5 5 5 5 5 5
n n
6 6 6 3 3 2 2
∑ ai + ∑ bi = a1 + b1 + a2 + b2 + ... + an + bn
i =1 i =1 Voici la loi de probabilité de X :
n n n
a 1 2 3 4 5 6
∑ ai + ∑ bi = ∑(ai + bi ) 10 8 6 4 6 2
i =1 i =1 i =1
P(X = a)
36 36 36 36 36 36
64 a) Par définition de l’espérance et d’après les
propriétés de la notation S démontrée dans l’exercice  67 a) Les valeurs prises par X sont 0, 1 et 2.
n
1 1 n 3×2 6
P(X = 0) =  = = 0,3
63  , E(X) =
∑ n ai = n ∑ai 5 × 4 20
i =1 i =1
b) La loi de probabilité de X est donnée dans le tableau.
n
1 1 n
et V(X) = ∑ (ai − E(X))2 = ∑(ai − E(X))2 a 0 1 2
i =1 n n i =1 P(X = a) 0,3 0,6 0,1
2
b)  (ai − E(X)) = ai2 2
− 2aiE(X) + E(X) 3×2+2×3
P(X = 1) =  = 0,6
20
1 n 2 2
c)  V(X) = ∑(ai − 2aiE(X) + E(X)2 )
n i =1
P(X = 2) =  = 0,1
20

1 n n n  68 a) 1/2 A :X = 0
V(X) = ∑ai2 − ∑2aiE(X) + ∑E(X)2  1/2
B 1/2
n  i =1  C :X = 4
i =1 i =1 A 1/2 A :X = 0
1 n n  1/2 1/2 D 1/2
V(X) = ∑ai2 − 2E(X)∑ai + nE(X)2  C :X = 4
n  i =1 
i =1 B

n n
1 2E(X) 1/2
V(X) = ∑
n i =1
ai2 − ∑ai + E(X)2
n i =1
1/2
C :X = 2

2E(X) n 1 n
d)  ∑
n i =1
ai = 2E(X) × ∑ai or on sait que
n i =1
A
1/2 A :X = 0
B 1/2
1/2 1/2
1 n 2E(X) n
C :X = 4


n i =1
ai = E(X) donc ∑ai = 2E(X)2 .
n i =1 1/2
A
D
1/2 A :X = 0
1/2 1/2
C :X = 4

1 n D
Par conséquent, V(X) = ∑ai2 − 2E(X)2 + E(X)2 1/2
n i =1
C :X = 2
1 n La loi de probabilité de X est donc :
et donc V(X) = ∑ai2 − E(X)2 .
n i =1  1 4 1
P(X = 0) = 4 ´   = 
e) Avec la formule du cours, on doit effectuer n sous- 2 4
tractions, alors qu’avec la formule de König-Huygens,  1 2 1
P(X = 2) = 2 ´   = 
on n’en effectue qu’une. 2 2
208

172909_Chap12_000-000.indd 208 26/07/2019 15:41:26


 1 4 1 71 a) On sait que p1 + p2 + p3 + p4 = 1 donc
P(X = 4) = 4 ´   =  4 p4 + 4 p4 + p4 + p4 = 1 c’est-à-dire 10 p4 = 1 et
2 4
b) La probabilité que la fourmi ait traversé le carré donc p4 = 0 ,1.
pendant le temps imparti est : b) La variable aléatoire X donne le nombre de points
3 obtenus lors d’un lancer, donc elle prend les valeurs
P(X = 2) + P(X = 4) =  .
4 1, 3, 4, 6. Voici la loi de probabilité de X :
69 a) 3/5 B a 1 3 4 6
B P(X = a) 0,1 0,4 0,4 0,1
3/5
2/5 N
B c)  P(X > 4) = P(X = 4) + P(X = 6) = 0 , 4 + 0 ,1 = 0 , 5
3/5
2/5 N
B
3/5
2/5
72 a)
N
1er lancer 2e lancer 3e lancer 4e lancer Valeur de X
2/5 N P 1
b) La loi de probabilité de X est : P
2 F 1
P(X = 1) =  = 0,4. P
5
3 2 6 P 1
P(X = 2) =  ´ = = 0,24 F
5 5 25
F 1
 3 2 2 18 P
P(X = 3) =    ´ = = 0,144
5 5 125 P 1
3 3 P
2 54
P(X = 4) =    ´ = = 0,086 4 F 1
5 5 625
F
 3 4 81 P 1
P(X = 0) =    = = 0,129 6 F
5 625
F 1
c) On obtient E(X) = 1,657 6.
P 2
70 On numérote les cases de l’échiquier P
F 2
1 2 3 P
P 2
4 5 6 F
F 2
7 8 9 F
P 3
On peut représenter une issue de la façon suivante :
P
(2 ; 5 ; 7) signifie que le jeton rouge est sur la case 2, le F 3
vert sur la case 5 et le bleu sur la case 7. F
Il y a 9 ´ 8 ´ 7 = 504 triplets différents, donc 504 pla- P 4
cements distincts de trois jetons sur l’échiquier. F
F 0
X donne le nombre de jetons sur la diagonale en poin-
tillés. b) Voici la loi de probabilité de X :
(X = 3) est réalisé par les triplets (1 ; 5 ; 9), (1 ; 9 ; 5), a 0 1 2 3 4
(5 ; 1, 9), (5 ; 9 ; 1), (9 ; 1 ; 5) et (9 ; 5 ; 1) 1 8 4 2 1
P(X = a)
6 16 16 16 16 16
donc P(X = 3) =  .
504
On obtient par dénombrement des issues : 73 1. On note b le pourcentage de donneurs du
108
P(X = 2) =  groupe B et c le pourcentage de donneurs du groupe
504
270 AB. Alors, d’après l’énoncé, 43 + 45 + b + c = 100 et
P(X = 1) =  b = 3c . Par conséquent, 88 + 4 c = 100 , c’est-à-dire
504
120 4 c = 12 et donc c = 3. Il y a donc 3 % de donneurs
P(X = 0) = 
504 du type AB et 9 % de donneurs du type B.

Chapitre 12  ★  Variables aléatoires 209

172909_Chap12_000-000.indd 209 26/07/2019 15:41:46


2. On peut représenter la situation par un arbre. 9 1 156
er e E(X) = ×0 ++ × 10 = ≈ 2, 57
1 patient 2 patient Valeur de X 49 49 49
0,03 B 2 Jeu 2
B
a 0 1 2 4 5 10 25
0,03 0,97
Autre que B 1 33 4 4 1 4 2 1
P(Y = a)
B 1 49 49 49 49 49 49 49
0,03
0,97 33 1 81
Autre que B E(X) = ×0 ++ × 25 = ≈ 1, 65
0,97 Autre que B 0 49 49 49
E(X) > E(Y) donc le jeu 1 est plus intéressant pour le
On calcule, par exemple :
joueur.
P(X = 2) = 0 , 03 × 0 , 03 = 0 , 0009.
On obtient ainsi la loi de probabilité de X : 78 On note X la variable aléatoire qui donne le gain
a 0 1 2 de Monsieur X.
P(X = a) 0,000 9 0,058 2 0,940 9 Voici la loi de probabilité de X :
a -8 2
74 a) La loi de probabilité de X est donnée dans le
1 5
tableau. P(X = a)
6 6
a -3 -2 1 5 9 1 5 2 1
E(X) = × (−8) + × 2 = = ≈ 0 , 33
1 2 1 1 1 6 6 6 3
P(X = a)
6 6 6 6 6 Ainsi, sur un grand nombre de parties, Monsieur X
1 1 1 1 1 gagnera, en moyenne, environ 0,33 € par partie.
b) E(X) =  ´ (- 3) + ´ (- 2) + ´ 1 + ´ 5 + ´ 9
6 3 6 6 6 b) Voici la loi de probabilité de X :
4
E(X) =   » 1,33 a k - 10 k
3
Sur un grand nombre de parties, un joueur gagnera, 1 5
en moyenne, environ 1,33 € par partie. P(X = a)
6 6
75 X est la variable aléatoire qui donne la somme 1 5 5
E(X) = × (k − 10) + × k = k −
gagnée par le joueur, en euros, sans tenir compte du 6 6 3
prix d’une partie. Voici la loi de probabilité de X : Ainsi, sur un grand nombre de parties, Monsieur X
5
a 0 10 20 50 gagnera, en moyenne, k -  € par partie pour une
3
40 4 3 1 mise de k €.
P(X = a)
48 48 48 48
79 a) Voici la loi de probabilité de X :
L’espérance de X est :
40 4 3 1 a 4 7 8 10
E(X) = ×0 + × 10 + × 20 + × 50 P(X = a) 0,03 0,27 0,14 0,56
48 48 48 48
150 b)  E(X) = 8 , 73
ce qui donne E(X) = c’est-à-dire E(X) = 3,125.
48 Sur un grand nombre de spectateurs choisi au hasard,
L’espérance du gain est inférieure à la mise, donc le le prix moyen pour un spectateur est 8,73 €.
jeu n’est pas équitable. c)  3 000 × 8 , 73 = 26 190
Or 26 190 > 20 000, donc ce théâtre est rentable.
76 a)  E(G) = 0 , 5 × (−5) + 0 , 2 × 0 + 0 , 2 × 10
+0 , 05 × 20 + 0 , 05 × 50 80 a) Voici la probabilité de X :
ce qui donne E(G) = 3.
a 0 1 2
b) Pour que le jeu soit équitable, il faut soustraire 3 à 1 1 1
chaque valeur possible du gain. On obtient alors une P(X = a)
3 3 3
espérance égale à 0.
b) En calculant à la main ou avec la calculatrice, on
77 On note X (resp. Y) la variable aléatoire qui 2
trouve E(X) = 1 et V(X) = .
donne le gain, en euros, au jeu 1 (resp. au jeu 2). 3
Jeu 1 81 a) Voici la probabilité de X :
a 0 1 2 3 4 5 6 7 10 a 2 3 4
9 12 10 4 1 6 4 2 1 4 4 4
P(X = a) P(X = a)
49 49 49 49 49 49 49 49 49 12 12 12

210

172909_Chap12_000-000.indd 210 26/07/2019 15:42:13


4 4 4 b) Voici la loi de probabilité de X2 :
E(X) = × 2 + × 3 + × 4 = 3
12 12 12 a 1 4 25 100
Sur un grand nombre de déplacements, le temps 2
P(X  = a) 0,1 0,2 0,3 0,4
moyen de déplacement est de 3 s. 2
L’espérance de X est :
82 a) Les valeurs prises par X sont les nombres E(X2 ) = 0 ,1× 1 + 0 , 2 × 4 + 0 , 3 × 25 + 0 , 4 × 100
entiers de 1 à 12, alors que les valeurs prises par Y sont ce qui donne E(X2 ) = 48 , 4.
les nombres entiers de 2 à 12. L’affirmation de Selma est En utilisant la formule de la question 1.d), on trouve
donc fausse. À la calculatrice, on obtient que E(X) = 6 , 5 V(X) = E(X2 ) − (E(X2 )) = 48 , 4 − (6)2 = 12, 4.
et E(Y) = 7. L’affirmation d’Ari est donc fausse.
b) C'est Y qui a la plus grande espérance. 85 1. a) Faux. En effet, {X < 0} ) est réalisé lorsque
c)  V(X)  11,92 et V(Y)  5, 83 donc c'est X qui a la {X = −1} ou {X = 0}.
plus grande variance. b) Vrai. En effet, {X > 2} est réalisé par le seul événe-
ment {X = 3}.
83 a) L'espérance de X est :  1 
2. a) Faux. En effet, si X =  est réalisé, alors
E(X) = p1 × a1 + p2 × a2 + p3 × a3 + p4 × a4  n 
1
En ajoutant 10 à chaque valeur, on définit une nou- {X < 0} est réalisé .
velle variable aléatoire Y dont l’espérance est : n
b) Vrai. En effet, si {X = 3} est réalisée, alors {X > 2}
E(y) = p1 × (a1 + 10) + p2 × (a2 + 10) + p3 × (a3 + 10)
est réalisé.
+ p4 × (a4 + 10)
E(Y) = p1 × a1 + 10 p1 + p2 × a2 + 10 p2 + p3 × a3
86 1. a)  {X < 1} b)  {X > 3}
+10 p3 + p4 × a4 + 10 p4 c)  {X > 1} d)  {X < 3}
E(Y) = p1 × a1 + p2 × a2 + p3 × a3 + p4 × a4
2.  A : « Il pleuvra au moins un jour le mois prochain. »
+10( p1 + p2 + p3 + p4 ) B : « Au moins un des élèves ne réussira pas l’examen
E(Y) = p1 × a1 + p2 × a2 + p3 × a3 + p4 + 10 × 1 du code de la route avant d’avoir 18 ans. »
E(Y) = E(Y) + 10
b) En ajoutant un nombre k à chaque valeur, on défi-
nit une nouvelle variable aléatoire Z dont l’espérance
est :
E(y) = p1 × (a1 + k ) + p2 × (a2 + k ) + p3 × (a3 + k )

Organiser son raisonnement


+ p4 × (a4 + k )
E(Z) = p1 × a1 + kp1 + p2 × a2 + kp2 + p3 × a3 + kp3
+ p4 × a4 + kp4
E(Z) = p1 × a1 + p2 × a2 + p3 × a3 + p4 × a4
+k( p1 + p2 + p3 + p4 )
E(Z) = p1 × a1 + p2 × a2 + p3 + a3 + p4 × a4 + k × 1 87 a) Voici la loi de probabilité de X - x  :
E(Z) = E(X) + k Valeur a1 - x a2 - x … an - x
Probabilité p1 p2 … pn
84 1. a) À la calculatrice, on obtient E(X) = 1, 4 et 2
V(X) = 0 , 24. b) Voici la loi de probabilité de (X − x) :
b) Voici la loi de probabilité de X : Valeur (a1 − x)2 (a2 − x)2 … (an − x)3
a 1 4 Probabilité p1 p2 … pn
2
P(X  = a) 0,6 0,4 2. a) Pour tout nombre entier i compris entre 1 et n,
2
c)  E(X ) = 0 , 6 × 1 + 0 , 4 × 4 = 0 , 6 + 1, 6 = 2, 2 (ai − x)2 = ai2 − 2 ai x + x2 .
d)  E(X2 ) − (E(X))2 = 2, 2 − (1, 4)2 = 0 , 24 Par conséquent,
On retrouve la valeur de V(X). f ( x) = p1 × (a1 − x )2 + p2 × (a2 − x)2 + 
2. a) Voici la loi de probabilité de X : + pn × (an − x)2
a 1 2 5 10
f ( x) = p1 × (a12 − 2a1x + x2 ) + 
P(X = a) 0,1 0,2 0,3 0,4
+ pn × (an2 − 2an x + x2 )
L’espérance de X est :
E(X) = 0 ,1× 1 + 0 , 2 × 2 + 0 , 3 × 5 + 0 , 4 × 10 f ( x) = p1 × (a12 − 2a1x + x2 ) + p2 × (a22 − 2a2 x + x2 )
ce qui donne E(X) = 6. +  + pn × (an2 − 2an x + x2 )

Chapitre 12  ★  Variables aléatoires 211

172909_Chap12_000-000.indd 211 26/07/2019 15:43:27


f ( x) = ( p1 +  + pn )x2 − 2 x( p1a1 +  + pnan ) −5n2 + 12n + 9
E(X) =
+ ( p1a12 +  + pnan2 ) n2
Le discriminant du polynôme du second degré
f ( x) = ( p1 +  + pn )x2 − 2 x( p1a1 +  + pnan2 )
−5n2 + 12n + 9 est ∆ = 122 − 4 × (−5) × 9 = 324.
b)  f ′( x) = 2 x − 2E(X) = 2( x − E(X))
Il y a donc deux valeurs qui annulent E(X), n1 = 3 et
c) La fonction dérivée f ¢ s’annule pour x = E(X). Elle
n2 = −0,6 qui est impossible dans ce contexte.
est négative pour x < E(X) et positive pour x > E(X).
Le jeu est donc équitable pour n = 3.
La fonction f admet donc un minimum atteint pour
x = E(X). 90 1. a)  Sortie de Sortie de
la 1re fille la 2e fille
88 a) •  P(A) = P(3 ; 0) + P(4 ; 0) + P(5 ; 0) 1
— 1…X=1…Y=1
1 1 1 1 1 1 4
P(A) = × + × + × 2…X=1…Y=2
3 3 3 3 3 3 1 3…X=1…Y=3
1 4…X=1…Y=4
P(A) = 1

3 4 1…X=2…Y=1
•  P(B) = P(5 ; 1) + P(3 ; 2) + P(4 ; 2) + P(5 ; 2) 1 2…X=2…Y=2

4 2
1 1 1 1 1 1 1 1 3…X=2…Y= 3
P(B) = × + × + × + ×
3 3 3 3 3 3 3 3 4…X=2…Y=4
4 1 1…X=3…Y=1

P(B) = 4
9 2…X=3…Y=2
3
4 1 3…X=3…Y=3
Or ¹ donc A et B ne sont pas équiprobables. 1
— 4…X=3…Y=4
9 3 4
1…X=4…Y=1
b) On note X la variable aléatoire qui donne le gain,
2…X=4…Y=2
en £, du jeu. 4
3…X=4…Y=3
1
Voici la loi de probabilité de X : —
4
4…X=4…Y=4

a -1 2 3 4 5 7 9 1
b) •  P(X = 1) =
1 1 1 1 1 1 1 4
P(X = a) La probabilité que la première fille sorte au 1er étage
3 9 9 9 9 9 9
1 1 1
E(X) = × (−1) +  + × 9 = 3 est .
3 9 4
1
Ce jeu n’est pas équitable, il est favorable au joueur •  P(Y = 1) =
4
car E(X) > 0.
La probabilité que la seconde fille sorte au 1er étage
1
89 On note X la valeur du gain algébrique du est .
4
joueur. On représente la situation par un arbre. 1 1 1 1
  •  E(X) = × 1 + × 2 + × 3 + × 4 = 2, 5
1re jeton 1 2e jeton Valeurs de X 4 4 4 4
1 n rouge 16 Sur un grand nombre de répétitions, la première fille
n rouge
n -1 1 blanc 1 sera montée avec l’ascenseur de, en moyenne,
n n 2,5 étages par trajet.
n -1 rouge 1
blanc
n n -1 blanc - 5 1 1 1 1
•  E(Y) = × 1 + × 2 + × 3 + × 4 = 2, 5
n 4 4 4 4
On en déduit la loi de probabilité de X : Sur un grand nombre de répétitions, la deuxième fille
a 16 1 - 5 sera montée avec l’ascenseur de, en moyenne,
1 2(n − 1) (n −1)2 2,5 étages par trajet.
P(X = a) 2. a)  p = P(X > 2) × p(X > 2)
n2 n2 n2
L’espérance de X est donc, en fonction de n, p = P( X = 2) × P( Y = 2) + P(X = 2) × P( Y = 3)
1 2(n − 1) (n − 1) 2 + P(X = 2) × P(Y = 4) + P(X = 3) × P(Y = 2)
E(X) = × 16 + ×1 + × (−5)
n2
n 2
n2 + P(X = 3) × P(Y = 3) + P(X = 3) × P(Y = 4)

16 + 2n − 2 − 5n2 + 10n − 5 + P(X = 4) × P(Y = 2) + P(X = 4) × P(Y = 3)
E(X) =
n2 + P(X = 4) × P(Y = 4)

212

172909_Chap12_000-000.indd 212 26/07/2019 15:44:10


1 1 9 On peut alors conjecturer que si on lance n fois la
p = 9× × = n
4 4 16 pièce, l’espérance de X est .
b)  p ′ = P(X = 1) × P(Y > 2) + P(X > 2) × P(Y = 1) 2
p ′ = P(X = 1) × P(Y = 2) + P(X = 1) × P(Y = 3)
93 On note X la variable aléatoire qui donne le gain
+ P(X = 1) × P(Y = 4) + P(X = 2) × P(Y = 1) d’un joueur sur une partie (mise comprise).

+ P(X = 3) × P(Y = 1) + P(X = 4) × P(Y = 1) Voici la loi de probabilité de X :
1 1 3
p′ = 6 × × = a - 5 - 3 5 45 95 245 995
4 4 8
P(X = a) 0,703 0,243 0,027 0,012 0,008 0,006 0,001
1
c)  p ′′ = P(X = 1) × P(Y = 1) = E(X) = −0 , 344
16
Sur 5 années, 5 × 250 × 300 = 375 000  parties sont
91 On schématise la situation à l’aide d’un arbre. jouées, soit un gain pour le casino de
5 375 000 × 0,344 = 129 000  €. Ainsi, la rentabilité de
d
6 la machine sera de :
5
6 c 129 000 − 5 × 2 000 − 40 000 = 79 000  €.
1
5 6
b 6
6
1 6 94 On peut représenter la situation par un arbre.
5 a 6
6
1
C représente le fait de saisir le bon code.
6
6 1re essai 2e essai 3e essai Valeur de X
1
6 1
6 — C 1
1 000
a, b, c, d sont des nombres entiers naturels distincts 2
1
de 6. —
C
999 999 1
— —
La probabilité que la tortue gagne est : 1 000 C 998 C 3
 5 4 998 C
  ≈ 0 , 48. — C 4
 6  999 997

999
On peut calculer de deux façons la probabilité que le
On en déduit la loi de probabilité de X :
lièvre gagne :
a 1 2 3 4
 5 4
Soit : 1 −   ≈ 0 , 52. 1 1 1 997
6 P(X = a)
2 3 1 000 1 000 1 000 1 000
1 5 1 5 1 5 1
Soit : + × +   × +   × ≈ 0 , 52. L’espérance de X est donc :
6 6 6 6 6 6 6
La situation la plus enviable est celle du lièvre. 1 1 1 997
E(X) = ×1 + ×2 + ×3 + ×4
1 000 1 000 1 000 1 000
ce qui donne
92 Si on lance 1 fois la pièce, la loi de probabilité de 1 1 1 997
E(X) = ×1 + ×2 + ×3 + ×4
X est : 1 000 1 000 1 000 1 000
a 0 1
3 994
1 1 E(X) = = 3, 994
P(X = a) 1 000
2 2
Ainsi, le voleur peut espérer ne pas trouver le bon
1
Donc l’espérance de X est E(X) = . code au cours des trois essais.
2
Si on lance 2 fois la pièce, la loi de probabilité de X est :
95 Yasmine effectue n tirs.
a 0 1 2
X donne le nombre de tirs où elle touche la cible.
1 1 1
P(X = a)
4 2 4
P(X > 1) = 1 − P(X = 0) = 1 − (0,5)n .
1 − (0,5)n > 0 , 99 équivaut à (0 , 5)n < 0 , 01.
Donc l’espérance de X est E(X) = 1.
Avec la calculatrice, on obtient n > 7.
Si on lance 3 fois la pièce, la loi de probabilité de X est : Yasmine doit tirer au moins sept fois.
a 0 1 2 3
1 3 3 1
P(X = a) 96 X donne la somme des trois nombres obtenus.
8 8 8 8
(X = 9) est réalisé par les issues:
3
Donc l’espérance de X est E(X) = . (1, 2, 6) (1, 6, 2) (2, 1, 6) (2, 6, 1) (6, 1, 2) (6, 2, 1)
2
Chapitre 12  ★  Variables aléatoires 213

172909_Chap12_000-000.indd 213 26/07/2019 15:44:39


(1, 3, 5) (1, 5, 3) (3, 1, 5) (3, 5, 1) (5, 1, 3) (5, 3, 1) d) La probabilité que la seule boule noire soit tirée au
(2, 3, 4) (2, 4, 3) (3, 2, 4) (3, 4, 2) (4, 2, 3) (4, 3, 2) 2 4 4 16
premier tirage est : × × =
(2, 3, 4) (2, 4, 3) (3, 2, 4) 6 5 5 75
(2, 3, 4) (2, 4, 3) (3, 2, 4) On a vu que la probabilité qu’elle soit tirée au deu-
8
(2, 3, 4) xième tirage est .
25 45
P(X = 9) = 3 Enfin, la probabilité que la seule boule noire soit tirée
6 4 4 2 32 4
(X = 10) est réalisé par les issues: au troisième tirage est × × = = .
6 6 6 216 27
(1, 3, 6) (1, 6, 3) (3, 1, 6) (3, 6, 1) (6, 1, 3) (6, 3, 1)
16 8 4 89
(1, 4, 5) (1, 5, 4) (4, 1, 5) (4, 5, 1) (5, 1, 4) (5, 4, 1) On en déduit que P(X = 2) = + + = .
75 45 27 135
(2, 3, 5) (2, 5, 3) (3, 2, 5) (3, 5, 2) (5, 2, 3) (5, 3, 2) e) Voici la loi de probabilité de X :
(2, 2, 6) (2, 6, 2) (6, 2, 2)
a 0 1 2
(4, 4, 2) (4, 2, 4) (2, 4, 4)
8 364 37
(3, 3, 4) (3, 4, 3) (4, 3, 3) P(X = a)
27 675 225
27
P(X = 10) = 3 8 364 37 586
6 f)  E(X) = ×0 + ×1 + ×2 =
Donc P(X = 10) > P(X = 9). 27 675 225 675
c’est-à-dire E(X)  0 , 868.
97 On note x le prix de vente d’un billet de tombola Cela signifie que si l’on répète un grand nombre de
et X la variable aléatoire qui donne le gain algébrique fois cette expérience, on peut espérer tirer en
du joueur. moyenne 0,868 boules noires par expérience.
X prend les valeurs -x, 2 - x, 10 - x et 50 - x.
Voici la loi de probabilité de X :
a -x 2 - x 10 - x 50 - x

Exploiter ses compétences


P(X = a) 0,5 0,2 0,2 0,1
L’espérance de X est donc :
E(X) = 0 , 5(−x) + 0 , 2(2 − x) + 0 , 2(10 − x) + 0 ,1 (50 − x)
E(X) = −0 , 5 x + 0 , 4 − 0 , 2 x + 2 − 0 , 2 x + 5 − 0 ,1x
E(X) = −x + 7, 4
Pour que l’espérance soit comprise entre 4 et 6, il faut
que −x + 7, 4 > 4 et −x + 7, 4 < 6 , ce qui donne 99 Dans chaque cas, on note X le gain algébrique
x < 3, 4 et x > 1, 4. Il faut donc que le prix du billet du joueur.
soit compris entre 1,4 et 3,4 euros. Cas 1 : Si le joueur mise sur un unique secteur, il paye
1,5 € et il peut gagner un lot ou ne rien gagner. Voici
98 1. Voici l’arbre pondéré : alors la loi de probabilité de X.
1er tirage 2e tirage 3e tirage
4 a - 1,5 0,5 3,5 8,5

4 6 B 9 1 1 1
— B P(X = a)
6 12 12 12 12
B 2 N
4 —
4 — 6 1

6
2
— 5 B On obtient alors E(X) = − .
6 N 12
1
— 4 N Cas 2 : Si le joueur mise sur deux secteurs consécutifs,
5 —
2 4 5 B
— —
5 B il paye 3 € et il peut gagner un lot ou ne rien gagner.
6
N 1
— N Voici alors la loi de probabilité de X.
5
1

N B a - 3 - 1 2 7
5 1
3 1 1 1
2. a) X prend les valeurs 0, 1 et 2. P(X = a)
6 6 6 6
4 4 4 43 8 1 2
b)  P(X = 0) = × × = 3 = On obtient alors E(X) = − = − .
6 6 6 6 27 6 12
c) La probabilité demandée est égale à :
Cas 3 : Si le joueur mise sur trois secteurs consécutifs,
4 2 4 32 8 il paye 4,5 € et il peut gagner un lot ou ne rien gagner.
× × = = .
6 6 5 180 45 Voici alors la loi de probabilité de X.
214

172909_Chap12_000-000.indd 214 26/07/2019 15:45:08


a - 4,5 - 2,5 0,5 5,5 a - 5 0 3
1 1 1 1 1 1 1
P(X = a) P(X = a)
4 4 4 4 3 3 3
1 3 2 8
On obtient alors E(X) = − = − . On obtient alors E(X) = − = − .
4 12 3 12
Cas 4 : Si le joueur mise sur quatre secteurs consécu- Cas 9 : Si le joueur mise sur neuf secteurs consécutifs,
tifs, il paye 6 € et il gagne forcément l’un des lots. il paye 13,5 € et il peut gagner deux ou trois lots.
Voici alors la loi de probabilité de X. Voici alors la loi de probabilité de X.
a - 4 - 1 4 a - 6,5 - 1,5 1,5 3,5
1 1 1 1 1 1 1
P(X = a) P(X = a)
3 3 3 4 4 4 4
1 4 3 9
On obtient alors E(X) = − = − . On obtient alors E(X) = − = − .
3 12 4 12
Cas 5 : Si le joueur mise sur cinq secteurs consécutifs, Cas 10 : Si le joueur mise sur dix secteurs consécutifs,
il paye 7,5 € et il peut gagner un ou deux lots. il paye 15 € et il peut gagner deux ou trois lots.
Les valeurs possibles pour X sont donc : Voici alors la loi de probabilité de X.
2 − 7 , 5 = −5, 5 ; 5 − 7 , 5 = −2 , 5 ;
10 − 7, 5 = 2, 5 ; 2 + 5 − 7, 5 = −0 , 5 ; a - 8 - 3 0 2
2 + 10 − 7, 5 = 4 , 5 et 5 + 10 − 7, 5. 1 1 1 3
P(X = a)
Voici alors la loi de probabilité de X. 6 6 6 6

a - 5,5 - 2,5 - 0,5 2,5 4,5 7,5 5 10


On obtient alors E(X) = − = − .
3 3 1 3 1 1 6 12
P(X = a) Cas 11 : Si le joueur mise sur onze secteurs consécu-
12 12 12 12 12 12
tifs, il paye 16,5 € et il peut gagner deux ou trois lots.
5
On obtient alors E(X) = − . Voici alors la loi de probabilité de X.
12
Cas 6 : Si le joueur mise sur six secteurs consécutifs, il a - 9,5 - 4,5 - 1,5 0,5
paye 9 € et il peut gagner un ou deux lots. Les valeurs 1 1 1 9
P(X = a)
possibles pour X sont donc : 12 12 12 12
2 − 9 = −7 ; 5 − 9 = −4 ; 10 − 9 = 1 ; 11
2 + 5 − 9 = −2 ; 2 + 10 − 9 = 3 et 5 + 10 − 9 = 6. On obtient alors E(X) = − .
12
Voici alors la loi de probabilité de X. Cas 12 : Si le joueur mise sur les douze secteurs, il
a - 7 - 4 - 2 1 3 7,5 paye 18 € et gagne les trois lots, c’est-à-dire un total
2 2 2 2 2 2 de 17 €, soit une perte de 1 €.
P(X = a)
12 12 12 12 12 12 Conclusion : la stratégie qui permet d’espérer la
1 8 perte moyenne la moins élevée est de miser sur un
On obtient alors E(X) = − = − .
2 12 seul secteur.
Cas 7 : Si le joueur mise sur sept secteurs consécutifs,
il paye 10,5 € et il peut gagner un ou deux lots.
100 On note X le montant des frais de réparation
Les valeurs possibles pour X sont donc :
pour une location prise au hasard, en choisissant le
2 − 10 , 5 = −8 , 5 ; 5 − 10 , 5 = −5, 5; forfait 1.
10 − 10 , 5 = −0 , 5 ; 2 + 5 − 10 , 5 = −3, 5 ; Voici la loi de probabilité de X :
2 + 10 − 10 , 5 = 1, 5 et 5 + 10 − 10 , 5 = 4 , 5.
a 0 200 500 1 000 2 000 5 000
Voici alors la loi de probabilité de X :
P(X = a) 0,8 0,1 0,04 0,03 0,02 0,01
a - 8,5 - 5,5 - 3,5 - 0,5 1,5 4,5
1 1 1 1 1 1 L’espérance des frais de réparation est alors E(X) = 250.
P(X = a) En ajoutant le prix du forfait, on obtient un total de
12 12 12 12 12 12
7 389 €, qui est l’espérance du coût réel de location,
On obtient alors E(X) = − . incluant le forfait et les frais de réparation.
12
Cas 8 : Si le joueur mise sur huit secteurs consécutifs, On note Y le montant des frais de réparation pour une
il paye 12 € et il gagne forcément deux des lots. location prise au hasard, en choisissant le forfait 2.
Voici alors la loi de probabilité de X. Voici la loi de probabilité de Y :

Chapitre 12  ★  Variables aléatoires 215

172909_Chap12_000-000.indd 215 26/07/2019 15:45:40


a 0 200 500 1 000 102 Le tableau cl-dessous indique les bénéfices pos-
P(Y = a) 0,8 0,1 0,04 0,06 sibles et les probabilités correspondantes si la compa-
gnie met en vente 152 billets.
L’espérance des frais de réparation est alors
Nombre de
E(Y) ≈ 205,88. passagers
147 148 149 150 151 152
En ajoutant le prix du forfait, on obtient un total d’en- Recette 24 900 25 160 25 330 25 500 25 670 25 840
viron 384,88 €. Remboursement 300 600
On note Z le montant des frais de réparation pour une Bénéfice 24 990 25 160 25 330 25 500 25 370 25 240
location prise au hasard, en choisissant le forfait 3. Probabilité 0,07 0,15 0,25 0,35 0,12 0,06

Voici la loi de probabilité de Z : Dans ce cas, l’espérance du bénéfice de la compagnie


a 0 200 500 s’élève à 25 339,60 €.
P(Z = a) 0,8 0,1 0,1 Si la compagnie met en vente 151 billets, alors on
L’espérance des frais de réparation est alors doit exclure la possibilité que 152 personnes se pré-
E(Y) ≈ 183,82. sentent. On se limite alors à 94 % des cas, et on doit
En ajoutant le prix du forfait, on obtient un total d’en- donc diviser chaque probabilité par 0,94. On obtient
viron 402,82 €. le tableau ci-dessous.
Dans ces conditions, pour un grand nombre de loca- Nombre de
147 148 149 150 151
réservations
tions, le forfait le plus intéressant est le forfait 3. 24 990 25 160 25 330 25 500 25 670
Recette
Remboursement 300
101 Voici l’arbre pondéré avec les probabilités : 24 990 25 160 25 330 25 500 25 370
1 Bénéfice
— Gagné Probabilité 0,074 0,160 0,266 0,372 0,128
7
3 Voisin Perdu

8
6
— 1
Dans ce cas, l’espérance du bénéfice de la compagnie
3 7 —
— 7 Gagné s’élève à 25 346,96 €.
8
Coin Coin Si la compagnie met en vente 150 billets, alors on
4 6 Perdu
— 2 — doit exclure les possibilités que 151 ou 152 personnes
9 — 7
8 Autre 1 Perdu se présentent. On se limite alors à 82 % des cas, et on
1

7 Gagné doit donc diviser chaque probabilité par 0,82. On
Voisin obtient le tableau ci-dessous.
4

3
— 6

Perdu
9 8 1 7 Nombre de
1 — 147 148 149 150

Milieu 7 Gagné réservations
Milieu 8
1

opposé 6 Perdu Recette 24 990 25 160 25 330 25 500
9 4 — Probabilité 0,085 0,183 0,305 0,427
— 7
8 Autre Perdu Espérance gain 2 133,292 68 4 602,439 02 7 722,560 98 10 884,146 3
1
1

7 Gagné Dans ce cas, l’espérance du bénéfice de la compagnie
Centre 1
Autre s’élève à 25 342,44 €.
6

Perdu
7 Pour espérer obtenir une recette maximale, la compa-
On note X la variable aléatoire qui donne le gain algé- gnie doit donc mettre en vente 151 billets.
brique du joueur. X peut prendre les valeurs - 1 et 9.
19
À l’aide de l’arbre, on calcule que P(X = −1) = et
2 21
P(X = 9) = .
21
L’espérance de X est donc :
19 2 1
E(X) = × (−1) + × 9 = − .
21 21 21
Puisque l’espérance est négative, ce jeu n’est pas
équitable.

216

172909_Chap12_000-000.indd 216 26/07/2019 15:45:44


13 Simulation
d’échantillons

2  a) a est un nombre aléatoire égal à 0 ou 1.

Découvrir Si a = 0, on convient que le 1er lancer est Face et Pile


si a = 1.
De même, si b = 0, le 2e lancer est Face et Pile si
b = 1.
x représente alors le nombre de Pile obtenu.
1  Simuler une variable aléatoire b) On saisit et on teste cette fonction, par exemple :

1   x 1 5 10 20
1 4 1 1 3  a) On écrit :
P(X = x)
3 9 9 9 11     print(Nb_Pile())
2  a) a est un nombre entier aléatoire compris entre b) On saisit et on teste le programme complété.
1 et 9.
x est la valeur prise par la variable aléatoire X.
On convient que si a < 3, la boule tirée est verte

Acquérir des automatismes


ainsi x = 1.
Si 4 < a < 7, la boule tirée est rouge alors
x = 5.
Si a = 8, la boule tirée est bleue alors x = 10 et
enfin si a = 9, la boule tirée est noire et x = 20.
b) On saisit et on teste cette fonction, par 1 9
2 1. a)  P(X = 1) =
3 et P(X = −1) = .
exemple : 10 10
1 9
b)  E(X) = × 1 + × (−1) = −0 , 8.
10 10
2. a) Voici les fonctions X et Moyenne écrites dans le
langage Python.

2  Simuler un échantillon
d’une variable aléatoire

1 1er lancer 2e lancer Nombre de Pile


P ……… 2
P
F ……… 1
P ……… 1
F
F ……… 0

x 0 1 2
1 1 1
P(X = x)
4 2 4

Chapitre 13  ★  Simulation d’échantillons 217

172909_Chap13_000-000.indd 217 25/07/2019 19:19:40


b) On obtient par exemple : Le programme devient :

3. a) Voici ce programme :

b) Lorsque la taille de l’échantillon augmente, on


observe que la distance tend à se réduire.

1 2
3 1. a)  P(X = 10) = et P(X = −5) = .
3 3
1 2
b)  E(X) = × 10 + × (−5) = 0. 2. La fonction Distance renvoie pour résultat l’écart
3 3
entre la moyenne m de l’échantillon et l’espérance
2. a) Voici les fonctions X et Moyenne écrites dans le
µ = 0 de la variable aléatoire X.
langage Python.
3. La fonction Répétition renvoie la proportion
d’échantillons tels que l’écart entre m et e soit infé-
2s
rieur ou égal à .
n
4. b) On obtient des proportions proches de 95 %.

Pour environ 95 % d’échantillons, on a m − µ < .
n

1 2
6 1. a)  P(X = 0) = , P(X = 1) = ,
7 7
2 2
b) On obtient par exemple : P(X = 4) = et P(X = 9) = .
7 7
1 2
b)  E(X) = × 0 + × (1 + 4 + 9) = 4
7 7
1 2
V(X) = × (0 − 4)2 + × ((1 − 4)2 + (4 − 4)2 + (9 − 4)2 )
7 7
3. a) Voici ce programme : V(X) = 12
Donc µ = 4 et σ = 2 3.
Le programme devient :

b) Lorsque la taille de l’échantillon augmente, on


observe que la distance tend à se réduire.

5 1. a)  P(X = −6) = P(X = −3) = P(X = 0)


1
= P(X = 3) = P(X = 6) = .
5
1
b)  E(X) = ((−6) + (−3) + 0 + 3 + 6) = 0
5
1
V(X) = ((−6)2 + (−3)2 + 02 + 32 + 62 ) = 18
5
Donc µ = 0 et σ = 3 2.
218

172909_Chap13_000-000.indd 218 25/07/2019 19:20:25


2. La fonction Distance renvoie pour résultat l’écart 3. a) 
entre la moyenne m de l’échantillon et l’espérance
µ = 4 de la variable aléatoire X.
3. La fonction Répétition renvoie la proportion
d’échantillons tels que l’écart entre m et e soit infé-
2s
rieur ou égal à .
n
4. b) On obtient des proportions proches de 95 %.
Pour environ 95 % des échantillons, on a :
2σ b) On saisit et on teste ces programmes.
m−µ < .
n
10 1. et 2. 

7 a) x +3 +5 -2
1 1 3
P(X = x)
2 5 10
b) 

3. a) On saisit et on teste ces deux fonctions.


b) Par exemple :

c) On saisit et on teste cette fonction.

8 a) La variable a prend pour valeur un nombre Avec des valeurs assez grandes de n, le pourcentage
entier aléatoire de 1 à 32, elle représente la 1re carte de paniers réussis est proche de 80 %.
tirée.
La variable b prend pour valeur un nombre entier 11 1. 
aléatoire de 1 à 32, elle représente la 2e carte tirée.
b) On convient que si a < 4, la 1re carte tirée est un
as et que si b < 4, la 2e carte tirée est un as.
c) On saisit le programme et on exécute plusieurs fois
la fonction.
d) En effet, la probabilité de tirer deux as est :
1 1
× ≈ 0 , 016.
8 8 2. On complète la ligne 17 :
m = somme/n
9 1. Entre 1 et 100, on compte 14 multiples de 7
3. On saisit et on teste les fonctions obtenues.
donc P(X = 1) = 0 ,14 et P(X = 0) = 0 , 86.
2. Saisir n 3 2
12 1. a)  P(X = 0) = et P(X = 1) = .
Pour k allant de 1 à n 5 5
Afficher Aléa () 3 2
b)  E(X) = × 0 + × 1 = 0 , 4.
Fin Pour 5 5

Chapitre 13  ★  Simulation d’échantillons 219

172909_Chap13_000-000.indd 219 25/07/2019 19:21:08


2. et 3.  14 a) 
P(X = 2) = (0 , 75)2 = 0 , 562 5
P(X = 1) = 0 , 75 × 0 , 25 + 0 , 25 × 0 , 75 = 0 , 375
P(X = 0) = (0 , 25)2 = 0 , 062 5
b)  E(X) = 0 , 562 5 × 2 + 0 , 375 × 1 + 0 , 062 5 × 0 = 1, 5
c) et a)

4. a) On saisit et on teste ces fonctions.


b) Par exemple :

Avec de grandes valeurs de n, la fonction Moyenne


renvoie des valeurs proches de l’espérance de X.
On saisit et on teste ces fonctions.
13 1. a) 
0,6 R X=2 15 1. a)  P(X = 1) = 0 , 05 et P(X = 0) = 0 , 95.
0,6 R
0,4 V X =1 b)  E(X) = 0 , 05 × 1 + 0 , 95 × 0 = 0 , 05, µ = 0 , 05.
0,6 R X =1 2. La variable a prend pour valeur un nombre aléa-
0,4 V toire dans [0 ; 1[. On convient que la condition
0,4 V X=0
2
a < 0 , 05 correspond à la livraison d’un colis abîmé.
P(X = 2) = (0 , 6) = 0 , 36 3. Pour un échantillon de taille n, la fonction Somme
P(X = 1) = 0 , 6 × 0 , 4 + 0 , 4 × 0 , 6 = 0 , 48 renvoie le nombre de colis abîmés de l’échantillon.
P(X = 0) = (0 , 4)2 = 0 ,16 4. Pour un échantillon de taille n, la fonction Distance
b)  E(X) = 0 , 36 × 2 + 0 , 48 × 1 + 0 ,16 × 0 = 1, 2 renvoie à l’écart entre la moyenne de cet échantillon
2. La variable a prend pour valeur un nombre aléa- et l’espérance m de la variable aléatoire X.
toire dans [0 ; 1[. On convient que a < 0 , 6 corres- 5. a) On saisit et on teste ces fonctions.
pond au 1er tirage d’une boule rouge. De même, b b) Par exemple :
prend pour valeur un nombre aléatoire dans [0 ; 1[ et
b < 0 , 6 correspond au 2e tirage d’une boule rouge.
3. 

Lorsqu’on donne à n des valeurs de plus en plus


4. a) On saisit et on teste ces deux fonctions. grandes, la fonction Distance renvoie des valeurs de
b) Par exemple : plus en plus proches de 0.

16 1. a)  E(X) = 0 ,1× 1 + 0 , 4 × 3 + 0 , 4 × 4 + 0 ,1× 6


= 3, 5
µ = 3, 5.
b)  V(X) = 0 ,1× (1 − 3, 5)2 + 0 , 4 × (3 − 3, 5)2
Avec de grandes valeurs de n, la fonction Moyenne  + 0 , 4 × (4 − 3, 5)2 + 0 ,1× (6 − 3, 5)2 = 1, 45.
renvoie des valeurs proches de l’espérance de X. σ = 1, 45 ≈ 1, 2

220

172909_Chap13_000-000.indd 220 25/07/2019 19:22:04


2. a) b) et c) 21 a) y 10 5 1
P(Y = y) 0,2 0,3 0,5

b) La roue compte 0 , 2 × 20 = 4 secteurs rouges,


0 , 3 × 20 = 6 secteurs bleus et 0 , 5 × 20 = 10 sec-
teurs verts.

22 a) La variable s cumule les n valeurs obtenues


de la variable aléatoire X.
La variable m a pour valeur la moyenne des n valeurs
de X de l’échantillon.
b) La ligne 18 s’écrit : d=abs(m-e)

S'entraîner
3. a) Pour chacun des N échantillons de taille n, la
fonction calcule la distance d entre la moyenne de cet
échantillon et e. 24
3
Elle renvoie pour résultat la proportion d’échantillons

telle que d <
σ.
b) Par exemple :

Lorsqu’on exécute Répétition(100,1000), on constate


qu’on obtient des proportions proches de 95 %.

17 1. C            2. B            3. C

18 1. A, B, C, D            2. A, D
2. a) On saisit et on teste ces fonctions.
19 1. L’affirmation est fausse.
b) On exécute plusieurs fois Moyenne(10000).
En effet, dans la boucle l’affectation s=X() ne convient
pas. On devrait écrire s=s+X().
2. L’affirmation est vraie.
En effet, la moyenne d’un échantillon de taille n assez
grande est proche de l’espérance de X.

20 a) 
On propose alors 2,5 pour estimation de l’espérance
de X.

b) On saisit et on teste le programme obtenu.

Chapitre 13  ★  Simulation d’échantillons 221

172909_Chap13_000-000.indd 221 25/07/2019 19:22:20


25 1. 28 On adapte la feuille afin de simuler N = 100
échantillons de taille n = 500 de X.
On obtient une proportion proche de 0,95.

1 3
29 1. a) P(X = 10) = , P(X = 2) =
8 8
1
et P(X = −1) = .
2
1 3 1
b)  E(X) = × 10 + × 2 + × (−1) = 1, 5
8 8 2
2. a)

2. a) On saisit et on teste ces fonctions.


b) On exécute plusieurs fois Moyenne(10000).

b) On saisit et on teste cette fonction.


On propose alors 18 pour estimation de l’espérance 3. a)
de X.

27 1. P(X = 1) = 0 , 7 et P(X = 0) = 0 , 3.
E(X) = 0 , 3 × 0 + 0 , 7 × 1 = 0 , 7
V(X) = 0 , 3 × (−0 , 7)2 + 0 , 7 × (−0 , 3)2 = 0 , 21
σ(X) = 0 , 21 ≈ 0 , 46. b) On saisit et on teste ces fonctions.
2. Chaque cellule du domaine B2:CW2 contient la for- Par exemple  ; Moyenne(10000) renvoie une valeur
mule : proche de l’espérance E(X) = 1, 5.
= SI(ALEA()<= 0 ,7 ;1; 0)
On corrige le contenu de la cellule CZ2 :
= ABS(CY2−0 ,7) .
2σ(X)
≈ 0 , 092, on corrige la formule de la cellule
n
DA2 : = SI(CZ2<= 0 ,092;1; 0)
Le reste de la feuille convient.

222

172909_Chap13_000-000.indd 222 25/07/2019 19:22:54


30 1. b) On saisit et on teste cette fonction.
1er lancer 2e lancer X 3. a)
1 2
2 3
3 4 b) On saisit et on teste ce programme.
1
4 5
5 6 31 1. a)
6 7
1 3
2 4
3 5
2
4 6
5 7 b) On saisit et on teste cette fonction.
6 8 2. a) 
1 4
2 5
3 6 b) On saisit et on teste ce programme.
3
4 7
5 8 32 1. P(X = 1) = 0 , 4 et P(X = 0) = 0 , 6
6 9 2. a)
1 5
2 6
3 7
4
4 8
5 9
6 10 b) On saisit et on teste cette fonction.
1 6
3. a) 
2 7
3 8
5
4 9
5 10
6 11 b) On saisit et on teste cette fonction.
1 7
2 8 33 1. P(X = 5) = 0 , 2, P(X = 4) = 0 , 5,
3 9 et P(X = 3) = 0 , 3.
6 E(X) = 0 , 2 × 5 + 0 , 5 × 4 + 0 , 3 × 3 = 3, 9.
4 10
5 11 2. a)
6 12

x 2 3 4 5 6 7 8 9
1 2 3 4 5 6 5 4
P(X = x)
36 36 36 36 36 36 36 36

x 10 11 12
3 2 1
P(X = x) b) On saisit et on teste cette fonction.
36 36 36
3. a)
2. a)

b) On saisit et on teste cette fonction.

Chapitre 13  ★  Simulation d’échantillons 223

172909_Chap13_000-000.indd 223 25/07/2019 19:23:31


34 1. a) 36 1. a) 

b) On saisit et on teste cette fonction.


2. a)
b) On saisit et on teste cette fonction.
2. a) 

b) On saisit et on teste cette fonction.


b) On saisit et on teste cette fonction.
3. On obtient par exemple :
35 1. a) et b)

On propose 1,95 pour estimation de l’espérance de la


variable aléatoire X.
4. a)

b) On saisit et on teste cette fonction.


Pour de grandes valeurs de n, la fonction Distance
c) On saisit et on teste ces fonctions. renvoie des valeurs proches de 0.
2. a) et b)
37 1. a)

c) On saisit et on teste ces fonctions.


3. a) La fonction Proportion réalise une simulation de
b) On saisit et on teste cette fonction.
N échantillons de taille n.
2. On écrit une fonction Moyenne qui simule un
Elle renvoie la proportion d’échantillons tels que
échantillon de taille n de Y et renvoie la moyenne des
Distance(n)<=alpha.
n valeurs de Y obtenues.
b) On saisit et on exécute cette fonction.
4. a) On exécute plusieurs fois :
Proportion(100,1000,0.025), on obtient des propor-
tions proches de 0,95.
Par exemple :
On exécute Moyenne(n) avec de grandes valeurs de n
afin d’obtenir une estimation e de l’espérance de Y.
3. a) Par exemple, avec e = 5, 43.
On peut remarquer que E(X) = 0 , 8 , σ(X) = 0 , 4 et
2σ(X)
≈ 0 , 025
n
b) et c) On peut noter que les proportions sont très
sensibles aux variations de alpha. b) On saisit et on teste cette fonction.
224

172909_Chap13_000-000.indd 224 25/07/2019 19:23:40


38 a)  a = 1, b = 5, c = 7 b) Dans la cellule DA2, on saisit la formule :
a < b est vrai et b < c est vrai, donc le booléen = SI(CZ2<= 0,664 ;1;0)
a < b and b < c est vrai. On recopie la formule jusqu’à la ligne 101.
Alors X(1, 5, 7) renvoie 1. Enfin dans la cellule DA103, on saisit la formule :
b)  a = 1, b = 6, c = 4
=NB.SI(DA2:DA101;1)/100
a < b est vrai, b < c est faux donc a < b and b < c
est faux.
40
Alors X(1, 6, 4) renvoie 0.
c)  a = 8, b = 3, c = 5
a < b est faux, b < c est vrai donc a < b and b < c
est faux.
Alors X(8 , 3, 5) renvoie 0.
d)  a = 4, b = 6, c = 5
a < b est vrai, b < c est faux donc a < b or b < c
est vrai. 41 a)
Alors Y(4 , 6, 5) renvoie 1.
e)  a = 9, b = 3, c = 4
a < b est faux, b < c est vrai donc a < b or b < c
est vrai.
Alors Y(9 , 3, 4) renvoie 1.
f)  a = 9, b = 4, c = 2
a < b et b < c sont faux donc a < b or b < c est b) On saisit et on teste plusieurs fois cette fonction.
faux.
Alors Y(9 , 4, 2) renvoie 0. 42

Organiser son raisonnement

2 2
3 1. a)  P(X = 1) =
39 , P(X = 5) =
5 5
1
et P(X = 10) = .
5
2 2 1
b)  E(X) = × 1 + × 5 + × 10, µ = 4 , 4
5 5 5
2 2 1
V(X) = (1 − 4 , 4)2 + (5 − 4 , 4)2 + (10 − 4 , 4)2
5 5 5
V(X) = 11, 04 et σ = 11, 04 ≈ 3, 32
43 a) 
2. a) Dans chaque cellule du domaine B2:CW2, on sai-
sit la formule :
= SI(ALEA()<= 0,4 ;1; SI(ALEA()<= 2/3;5;10))
b) Dans la cellule CY2, on saisit la formule :
=MOYENNE(B2:CW2))
Dans la celluleCZ2, on saisit la formule :
= ABS(CY2− 4,4)
3. a) Afin de réaliser une simulation de N = 100 b) On écrit une fonction Moyenne qui simule un
échantillons de taille n = 100 de X, on recopie les échantillon de taille n de X et renvoie pour résultat la
formules jusqu’à la ligne 101. moyenne des n valeurs obtenues.

Chapitre 13  ★  Simulation d’échantillons 225

172909_Chap13_000-000.indd 225 25/07/2019 19:25:45


47

On exécute alors Moyenne(n) pour de grandes


valeurs de n. Les résultats obtenus nous donnent une
estimation de l’espérance de la variable aléatoire X.
48 1. a) La fonction X renvoie la valeur de N qui
44 a) L’aire du carré en cm2 est : 202 = 400.
représente le nombre de personnes vaccinées parmi
Celle du cercle est : π × 102 = 100π.
les 180 personnes interrogées.
100π π π
Donc P(X = 1) = = et P(X = 0) = 1 − . b) a est un nombre aléatoire dans [0 ; 1[.
400 4 4
On convient que si a < 0 , 4 , la personne interrogée
est vaccinée contre la grippe.
2. La ligne 14 s’écrit : som=som+X(), la ligne 15 s’écrit :
m=som/n.
3.

b) On saisit et on teste cette fonction.

45 Exploiter ses compétences

3 La fonction X simule les n réponses au hasard


49
du candidat et renvoie le nombre de réponses justes
qu’il obtient.
46

50 Dans le programme suivant, on note p = 0


lorsque la fourmi est en A, p = 1 lorsqu’elle est en B
ou D et p = 2 lorsqu’elle est en C.
La variable x a pour valeur la durée en min du par-
cours.
Pour de grandes valeurs de n, Moyenne(n) renvoie
des valeurs qui nous donnent une estimation de l’es-
pérance de la variable aléatoire X.

226

172909_Chap13_000-000.indd 226 25/07/2019 19:26:06


52

51

Chapitre 13  ★  Simulation d’échantillons 227

172909_Chap13_000-000.indd 227 25/07/2019 19:26:06


Édition : Julien Lionnet
Composition : DESK (www.desk53.com.fr)
Schémas : DESK
Adaptation graphique : Simon Géliot

172909_Chap13_000-000.indd 228 25/07/2019 19:29:35

Vous aimerez peut-être aussi